You are on page 1of 242

PHYSICS JEE SOLUTIONS

Part-2
Gravitation
Fluids
Simple Harmonic Motion
Wave Motion
Mechanical Properties of Matter
Thermal Expansion and Calorimetry
Heat Transfer
Kinetic Theory of Gases
Thermodynamics
Gravitation 10.1

Gravitation

INTEXT EXERCISE: 1
1. 2Gm #
2

F net = 2 cos 45c
a 2
d n
2
7.


By symmetry the direction of resultant force is as Gm Gm Gm
2 2 2

Net force = 2 + 2 + 2 + .....
shown in diagram. 1 2 4
4 G
2. Gravitational force doesn’t change. = 3

3. Resultant = 5G 8. Net force is


GMm = 4GMm

F' = 2 2
ca 3 m
3a
2
9.
4. Net force on any masses

Gm 2

F net = 2 2 cos 30c
a
F net Gm 3
acceleration = m = 2
a
5. For equilibrium forces must be equal. 2
Gm1
Finitial =
GMx m GM y m ^a 2 h
2
=
rx2
2
ry
2
Gm1 2Gm1 m2
Ffinal = cos 45c
^a 2 h
Mx r 2 2 +
e xo
2
= a
My ry
6.
3m 10. m = 34 πr3 ρ

2
= Gm 2
]2rg

F

INTEXT EXERCISE: 2

Gb 3 M l
1. a h = 6400 km = Radius of earth, 1
gsurface =
GM = gs = 9.8 m/s b 1 Rl
2 2

]2Rg

` gh = 2 4 4 3
= 2.45 m/s 2
]3 g ignoring the spin.



= GM
R2
2. On Z - 34,
3. Clearly (b)
10.2 Physics

gd = 1600 = gs b1 - 6400 l
4. From principle of superposition, Net field at P (as 1600

well as at any point inside the cavity.


= 34 Gπρ ]- OP g
= gs b1 - 14 l = gs b 34 l


Where OP = vector from center
gs b 25 l
16
gh = 1600
of sphere to center of Then gd == 1600 = = 16 # 34 = 64
gs b 4 l
cavity. 3 25 75

Force = 3 Gπd b 2 l M
4 R 8. Only the mass contained within a sphere of radius

` In this case,
'x' centred at the center of earth will b able to exert

= 2Gm3πdR a force on the mass m .

G b 3 πx3 ρ l m
4
5.
90 b 640 l
100 gs = gs 1 - 6400 .....(1)

` force =
M
& ρ= 4 3
x2

95 b d l 3 πR
100 gs = gs 1 - 6400 .....(2)
4 3 M
G f 3 π x 4 π R3 p m
1 90 = 6400 - 640 3
&
2 95 6400 - d
` force = 2 = GMmx
3
x R

& 95 ]6400g - 90 ]6400g
9. For the solid sphere case,
= 95 ]640g - 90d
G b 3 πR3 ρ l m
4

& 90d = 60800 - 32000 πGρRm
]2Rg2
F1 = = 3

` d = 320m
G b 3 πR3 ρ l m G b 34 π b R l ρ lm
3
4
6. According to the question. 2
]2Rg2
F2 = -
b 3R l
2

g poles b1 - 2Rh l = gequator without spin of earth - ω 2 R 2


πGρRm 3

But gequator without spin of earth = g poles , ignoring shape = 3 - G $ 34 π R8 $ ρ m $ 4 2
9R

of earth πGρRm 2
= 3 - πGρRm $ 27
2gh 2

` g- R = g-ω R 7πGρRm
= 27
ω2 R2 7

` h = 2g Now F2: F1 = 9

gh = 1600 = gs b1 + 6400 l
1600 -2
10. Outer shell will not exert a force on this particle &
7.
inner shell will act as a point mass placed at its non
= gs b1 + 14 l
-2
center.


Gm1 m 4Gm1 m
F =
r1 + r2 2 ^r1 + r2 h2
` =
= gs b 54 l = gs $ 16
-2

25 c m
2

INTEXT EXERCISE: 3

1. Wext = ∆ ^Energy Totalh - GMm


= U f - Ui = > b R H - : GMm
- D
R+ 4 l
3. ∆U R
- 3 = b 12 # 1 # 2 2 + 1 # V l - ]0 + 0g
= 45GMm GMm
-

R + R

` -3 = 2 + V

= GMm GM MR
5R = R 2 5
V =- 5 J/kg
mgR
2. a Potential inside a hollow sphere is a constant.
= 5
Gravitation 10.3
4. Conserving momentum,
Required work = b GMm l - b - GMm l
-
v 3r 2r
if speed of m = v , then speed of 2m = 2
= GMm
6r =W
3
KE = 2 mv 2 + 2 ]2mgb 2v l
1 1 2
`
8. Energy at P = Energy at O for the particle
2
= 12 mv 2 + mv 3mv 2
GMm
4 = 4 0- = 12 mv 2 - GMm
^3 + R 2h2
1 R
5. Energy at height R = Energy at Earth’s surface
- GMm 1 GMm GM v
2

& 2 m ]0 g2 - 2R = 2 mv - R 2R = 2 mv 2 - R
1 GMm 1 2 GMm & 2R = 2

1 GMm GM = GM GM
2
` 2 mv = 2R & v = $R
`v= R
R R2
9. Ui = U12 + U13 + U14 + U23 + U24 + U34

= gr

6. Vescape = 2gR ` KE needed to - GM 2 GM 2


= a - 2a
escape
2 2 2

- GM GM GM

= 12 m $ 2gR = mgR a - a - 2a
2

Given, initial
- GM
a
mgR
KE = 2 - 4GM 2 2 GM
2

` Ui = a - a

Then energy at surface = Energy at height h

Let ‘4’ be taken to 3 .
mgR GMm GMm
2 - R = 0-
Then U f = U12 + U13 + U23
R b1 + hr l
2
2 GM 2
= 2GM
mgR - mgR -

a - 2a

& 2 - mgR = b
1+ Rl
h 2
2GM 2 2 GM

` ∆U = U f - Ui = a + 2a = Wext
h

& ` 1+ R = 2 & h = R

Using M = 1, a = 1, Wext = 2G + G
7. Given Wext = W = ∆U 2
2 2 +1

= b GMm
- l - b - GMm l
=e oG
2r r 2
10. Can not be defined because no position with a

` W = GMm
2r specific potential has been specified.
INTEXT EXERCISE: 4

1.
Vescape, earth 2ge Re ge Re
= GMm GMm - 3GMm
4R - R = 4R
Vescape, moon = 2gm Rm
= gm $ Rm

= 6 # 10 3. Energy at surface = Energy at 3

2 m.9 ^2gRh - R = 2 mv3 + 0



= 7.75 1 GMm 1 2

&
2. PE at highest point = total energy at surface 9GMm - GMm

& R R = 12 mv32

( a KE at highest point = 0 )
16GM = 2 2
1 v
2
GMm
& R v3 & v3 = 16gR

` PE = 2 m 4e - R

= 8 # 2gR
2gR

= 12 m. 4 - GMm
R 2

& v3 = 8v e2

= 12 m $ 2 $ GM R GMm
$ - R
& v3 = 2 2 ve
R2 4
10.4 Physics
4. Energy at center = Energy at 3 (minimum) 2 # 6.67 # 10 11 # 6 # 10 24 =
-

` 16 R
9 # 10
1
mv
2
2
- 4GM m = 0 + 0 4 # 6.67 # -3
a
d n
&R= 3 10 m
2

c 4 # 2.22 mm c 8.88 mm = 9 mm
v 2 4 2 GM 8 2 GM

` 2 = a `v= a - GMm
8. At height h = Re , energy of the body is
2Re
5. For the object to escape when initially at a height R
above the surface of earth, ` Minimum amount of energy to be given so that it
+ GMm + mgRe
escape is
1 - GMm
2 =
mV
2 2R =0 2Re 2
9. Energy at height h b = 2 l = Energy at surface
2 GM GM R
V
& = R `v= R
GMm 1 2 GMm
6. V1 = 2gR 0 - 3R = 2 mv - R
b l
2
1 2 GMm = 2GM = 2
mV
2 2 - 3R 0 & V2 = 3R 3 gR GMm 1
& 3R = 2 mv 2

Then
2GM = 2gR V
V1 2gR v
` = 3R = e
= 3 & V1 = 3 V2 3 3
V2 = 2gR
e o 10. Ve =
2GM =
50 km/s
3 R

7. Vescape = 2gR =
2GM = 3 # 108 m/s. 2G ]4M g 2GM =
R V e' =
R =2 R 100 km/s

INTEXT EXERCISE: 5

T2 ? R3 GM ]a - bg ]a + bg]a - bg 2

1.
& ab = 12 2 vA
2 3
b
T1 R1 20 # 20 = r3

` 2 = 3 & 2bGM
a ]a + bg
T2 R2 T22
4 # 4 # 4r3
& vA =

` T2 = 20 # 8 = 160 days
2bGM
a ]a + bg

` angular momentum = m a
` ]d g


= m 2aGMab
+b
2.
3. Time between A & B along the marked route is
needed.


If v A & vB are the speeds of planet at A & B , then:

Equal areas in
m v A a = m vB b

& v A a = vB b .....(1)
equal times


Also, total energy will be conserved. π ab in a time = T .
- GMm 1
+ 2 mv A2 = GMm + 12 mvB2 πab 1
-

` a b Reqd. area = 2 - 2 # 2b # ae

GM : b - a D = 2 _vB - v A i
1 1 1 2
` Reqd. time = πab # ab : 2 - eD

2 T π

a2 2
= 12 d 2 v A - v A n = T: 2 - π D
1 e
2



b
Gravitation 10.5
L dA
4.2m = dt b -1l
GM 1 3

` va = 3R b
a+ 3l
L 1

or ∆A = 2m ∆t


= 12 # 4.4 # 1015 # ]365 # 24 # 3600g = GM 2 3 GM
3R # 3 # 4 = 6R

= 6.94 # 10 22 m 2 7. Using the condition of ‘Equal areas in equal times’,
5. Area travelled at A = Area travelled at B in since area SCD is double that of area SAB, hence
time ∆t time taken from C to D should be double that of A to
B.
1 # 1
2 OA # v A ∆t = 2 # OB # vB ∆t
` t1 = 2t2
vB OA
v A = OB = x 8. Conserving angular momentum about the center of
6. Here the sun,
m rmin v1 = m rmax v2
v1 rmax

` v2 = rmin
9. Since speed is maximum at P4 , KE is also max. at
P4 .
10. Angular momentum of satellite about the center of

Clearly 20 = 6 R earth.
JKits speed NO JKits distanceNO

And 2ae = 2R KK O K O
mass of KKalong the OOO KKKfrom the OOO
e =3
1 =e o # KK O#K O
KK tangentialOOO KKKcenter of OOO
` satellite
KK OO KK OO
Speed is min. at Aphelion Ldirection P Learth P
. . .
GM b 1 - e l
va a 1 + e here a = 3R m orbital speed R+h
& has mass of
INTEXT EXERCISE: 6
earth in its
GM 1 T = 24 hrs
expression
1. Orbital speed = r ? r
r = 36000 + 6400
v1 r2 16R 4
` v2 ? r = 9R = 3
1 = 42400 km
2. Since T 2 ? r 2 or T ? r3/2 , if r is changed by a
For the spy satellite, if TS & rS are the time period
factor of 4, T changes by a factor of ]4g3/2 = 8 & radius of orbit, then:
3. Since the satellite is moving in a circular path, it
24 # 24 = 42400 # 42400 # 42400
is accelerating (in this case, towards the center of 6800 # 6800 # 6800
TS2
earth).

Using height = 400 km
1
4. KE = 2 mv 2
2 24 # 24
]6.24g3
2πr
` TS =
v = T & r3 ? T 2 ` r ? T 2/3
2
-1
1
& TS = 1.54 hrs
v ? T3 & v ?T
-3
`
2
KE ? T
-3
`
Height
2 3 200 400 600 800 1000
5. T ?r (km)

For geostationary satellite, Time 1.48 1.54 1.61 1.68
10.6 Physics
6. Net force on particle A 8. Energy needed = Total energy - Total energy
towards the center in larger orbit in smaller orbit

= c ]3Rg m - c ]2Rg m = GMm


- GMm - GMm

= Gm2 cos 45c ]due to Bg
2
2 2 12R
γ AB
9. From the formula for 'ω' derived in the chapter,
+ Gm2 cos 45c ]due to Dg
2

γ AB G ^m1 + m2 h G ]m + 2mg 3Gm

ω= = =
2
r3 d3 d3
+ Gm2 (due to C )
γ AC 10. T 2 ? r3
Gm 2 1 # Gm 2 = mv 2
` Using TA = 1, TB = 8, rA = 104 km

` 2 $ 2+ r
2r 2 4r 2
3
1#1 = 1043
Gm e 4 + 2 o 8 # 8

& v = rB
r 4 2

` rB = 104 # 4 km = 416 km
LA mv A rA
7. LB = mvB rB 2π ]104g 2π ]416g
VA = 1 , VB = 8
GM ]2Rg
2R $ 2 ` VB w.r.t. A = 52 # 2π when A & B are closest.


= 2 = 1
GM ]4Rg 2
4R 52 # 2π = π
` ω B w.r.t. A = ]
416 - 104g 3
rad/s

EXERCISE - 1

G ]1g]1g 6.67 # 10 -11 5. Gravitation field may increaseor decrease based on


]2g2
1. F1 = F2 = = 0.04 = 1.67 # 10 -9 variation of density.

2 M i + 4Uj i = 6S
6. F = mg = 2 _3S i + 8Uj

F12
g = Gm
7. a2
M
g 1 = Gm 2
1 M F13 3
^a 2h
F net = F1 ^S
i h + F2 _Uj i = F _S
i + Uj i
= 1.67 # 10 -9 _S
i + Uj i

Net field = g 1 + 2 g cos 45c

8. Field in the cavity
Gm2
2. T sin i = ; T cos i = mg
= 34 πGρ b R l
gl2
2
; i = tan -1 d 2 n
Gm Gm

tan i =
gl2 gl
M
Use ρ= 4 3
GM G81M
3. At P ; g = 2 -
x ]D - xg2
=0 3 πR
P GM

& D–x = 9x; 10x = D M 81M 9. g = 2 &
x D–x r

D 9D
x = 10 from the Moon and 10 from the earth.
R is reduced to R/2 and the mass fo the mars
becomes 10 times
4. If we take complete spherical shell than gravitational
field intensity at P will be zero hence for the hemi 4 4

gmars = 10 gearth and Wmars = 10 = 80N
spherical shell shown the intensity at P will be
along c .
Gravitation 10.7

10. g' = g b1 - R l; g = R
2h Dg 2h 0 3
k
20. # dV =- # Ig .dx: # dV = # x3
dx
v r

1 = 2 R = 2 ; g' = g b1 - R l
h 1 d

0 - v = ;- E v =+ k 2 & V = k 2
3
1

2x2 r 2r 2x
D g' d h
g = R & R g decreases by 0.5% 21. Equilibrium position of the neutral point from mass
4 'm' is
GM gρ 3 πR3 4
11. g = 2 = &g= 3GρπR
m
R R2 = e od
g R m+ M
& g \ R & g' = 3R & g' = 3g
- G m1 - G m2
GM g mg 10
V1 = r1 ; V2 r2
]R + hg2 49
12. g' = = ; w' = 49 = 49 = 0.20 N
- Gm ^

Apparent weight of the rotating satellite is zero
V1 = M + m h;
md
because satellite is in free fall state.
M ^ M + mh
- GM

V2 =
13. g' = g - ~2 r cos 60 60°
Md
mω 2r
60°
2
g' = g - ~ R cos 60 2

V = d ^ M + mh
-G 2


g' = 0, g = ~ cos 60 2 2 r = R cos 60°

- 3GM
22. Vcentre = 2R
4g 2r R R
R = ~ , t = ~ = 2r 4g = r g
23. ∆V =- Eg .dr
14. There will be no buoyant force on the moon.

Because field is uniform
(Eventually balloon bursts)
` 2 =- Eg .20 & E = 10 ; ∆V =+ 10 54? = 5
GMm 1 1 2
15. 2
mrω = 2

r

& GM = r3 ω 2
work done in taking a 5 kg body to height 4m

r3 ω 2 m
= (change in gravitational potential)
g =
R2
= 5 b 52 l & 2 J

]4nRg3 ω 2
= 64 nRω 2
]nRg2

` g' =
24.
Gm 1m
16. g= 2
R 2m
4
M = 3 ωR 2 ρ 4m
8m
3g

So ρ = 4πRG

v = v1 + v2 + v3 + v4 + .....
17. 3/5mg = mg - mR~ 2
Gm Gm Gm Gm Gm Gm
=- 1 - 2 - 4 - 8 - 16 - 32 - .....
3 2 g

~2 = g - 5 g & ~ = 5 R =- Gm b1 + 12 + 14 + 18 + ..... l " G.P. of infinite
GM pGMe 4 4
18. g p = ge; ; πR d = πR d
=
series.
R P2 3 p p 3 e e
2
R P
dp 1 1

or 2Re d P = Re de & d = 2
e =- m f 1 - 1 p
2
GM GM
19. Ig =
v =- gm ]2g
, V =- R
R2

V = Ig R = 6 # 8 # 106 = 4.8 # 107 =- 2gm
10.8 Physics
25. Let the possible direction of gravitational field R h
GMm ] g = 2 GMm ]
at point B be shown by 1, 2, 3 and 4 (Figure 1). R 2R R R + hg
Rotate the figure upside down. It will be as shown in R2 = 3 R h
figure 2.
R
h = 3


Figure 1 1 B
29. By applying conservation of energy
2 4
3
KEi + PEi = KE f + PE f

1 2 GM m GM m
mv
2 - Re = 0 - 2Re
= B Figure 3
GM m
1
mv
2
2
= Re :- 12 + 1D
3
2 4
1 2 GM m GMe
mv
2 = 2Re & u = R

Figure 2 1 B

30. Ui + Ki = U f + K f


Now on placing upper half of figure 1 on the lower
- GMm 1 2 GMm
R + 2 mvi =- r + 0
half of figure 2 we get complete sphere. Gravitational
field at point B must be zero, which is only possible 1 2 = GM b Rl
b Rl
if the gravitational field is along direction 3. Hence 2 vi R 1 - r = gR 1 - r
gravitational field at all points on circular base of
hemisphere is normal to plane of circular base. R vi2 R

`
1 - r = 2gR &r =

` Circular base of hemisphere is an equipotential vi2
1 - 2gR
surface.
31. Let the mass fo the particle be m
26. Wexternal force + ^Wa " b hravitational force = ∆K
PE at a distance of R' = ]GMmg /R'

- 10 + ^Wa " b hgravitational force = 4 PE at a distance of Re =- ]GMmg /Re

^Wa " b hgravitational force = 14 J
Decrease in PE = Increase in KE

m ^VA - VB h = 14 & VA - VB = 2 = 7 Jkg 1


14 - GMm GMm 1 2

& - R' + R = 2 mv
e

VB - VA =- 7 Jkg 1
-

v 2 = 2GM ; R - R' E & v 2 = R ;1 - e E


1 1 2GM R
27. According to the problem, as the potential at 3 e e R'
increases by + 10 J kg 1 , hence potential will
-

& v = R c1 - e m
2 2GM R
increase by the same amount everywhere (potential
e R'
gradient will remain constant). Hence potential at
2GMRe
c1 - e m
point P = 10 - 5 =+ 5 J kg 1
-
R

&v= 2
Re R'
28. Work done in lifting a bod by a
2gRe c1 - m
height x Re

`v=
R'
=- ∆U
32. W = :- GMm D : GMm D
3R - 3R
= GMm GMm
R - R+x
GMm GMm
= 3R - 5R
= GMm c R ]R + xg m
x

= R : 3 - 5 D = 15 R
GMm 1 1 2 GMm

Given that
Gravitation 10.9
33. When they are r distance apart MS Tm 2 re 3 1 2

Me = d
Te
n d
rm
n = f 13 p ]392g3
1
39. From the conservation of angular momentum,

Conservation of linear momentum
mv1 r1 sin 90c = mv2 r2 sin 90c
m1 v1 = m2 v2

Conservation of energy
1 1 Gm1 m2
0 = 2 m1 v12 + 2 m2 v 22 - r


Relative velocity = v1 + v2
V r
34. By conservation of energy V1 = r2
2 1

r
& v2 = d r1 n v1 = d
1.5 # 10 7 n ^

- GMm 1 2 1 2 5 # 103h
R + 2 mv = 2 mv' + 0 2 0.5 # 10 7
or v2 = 1.5 # 10 4 ms
-1

use v = 20 km/s

1 C
R = 6400km 40. F ? ;F= m
rm r
2GM 2G2M = 2GM
This force will provide the required centripetal force
35. ve = R ; v' e = R/2 2 R
ve = 2 ]11.2 km/ secg = 22.4 km/ sec

Therefore

C 2= C
36. To escape from the earth total energy of the body mω 2 r =

rm mr
m+1
should be zero KE + PE = 0
2π ]m + 1g/2

T = ω &T \ r
mgRe
1
mv
2
2
- GMm
5R = 0 & KEmin = 5 1 GM1 M2
2
41. K.E. =+ 2 r
GMm = mv V
m
37. r
r2 r
r = 2r for the first and r = 8R for the II nd

M
GM K.E1

v= b 1 8R l
r2 K.E2 = 2R 1 = 4: 1
3 3
2πr 2πt 2 = 2πr 2 GM1 M2 P.E1

T= v =
Similarly P.E. is &- , P.E = 4: 1
GM 4 R
Gρ # 3 πr3 2

K.E
1
Put the ratio of P.E. = 2

T?
ρ
42. Relative angular velocity when the particle are
38. The time period Te of the earth around the sun of moving in same direction is
mass Ms is given by
ω1 + ω 2 & ^ω1 + ω 2 h t = 2π
T = GM ]reg3 , where re is the radius of the earth’s
4r2
e
2
S 2π π
orbit.
` ω 2 = 24 rad/ sec; ω1 = 6


Similarly, time period Tm of the moon around the
When the particles are moving in the same direction
earth is given by then angular velocity becomes.

4π 2 ^ω1 - ω 2 h & ^ω1 - ω 2 h t = 2π


T m2 = GM r m3
e

By substituting ω1 and ω 2 in equation we get
T e2 Me re 2

` 2 = d M nd r n
Tm S m
& 24 hrs.
10.10 Physics

43. Net force towards the centre & m~2 ]9Rg =


GMm 1
]9Rg2
T \ ω
GM 2r R T ω 1

& ~2 = & T = ~ & 27 # 2r g & T2 = ω1 = 2

729R3 1 2
44. By conservation of angular momentum
GM GM
2 2
48. VA = rA and VB = rB

mr min ω = mr max ω'

45. m
Given rB = 3rA
2
Fg 30º Fg mv M GM GMm
L
Now FA = r A = r r = 2
rA
2Fg cos30º L
A A A

GMm
m FB =
r B2
m

FB R A2 1
FA = r B2 = 9
MV 2

2 Fg cos 30 = R 49. Total energy,
2
2d 2 n 2 =
GM 3 MV 2
1 GmM
L L E = 2 mv 2 - 3r

3
GM GmM GMm mM
V= = ] g - 3r =- G 6r
L 2 3r
46. Net force on the package is zero hence it will revolve GMm - GMm

Required work = ∆U =- 6R - R
around the earth and never reach to earth surface.
5GMm
47. Angular speed of earth = angular speed of =
geostationary satellite 6R

EXERCISE - 2

1. In horizontal direction
Force due to this strip on 'm'


Net force =
G 3 mm
cos 30c -
Gm 2
cos 60c = dF = Gm dm
12 d 2
4d 2 x2
+ 2
= Gm a 2bx dx
2 2
= Gm2 - Gm2 = 0
x
8d 8d
α+,

in vertical direction a + bx 2

Total force F = # dF = # Gm
x
2 dx
G 3 m2 G 3 m2 α
= 2 cos 60 c + 2 α+,
12d 3d
c 2 + b m dx
a
= Gm # x
Gm 2 α
cos
+ 2 30c
= Gm & α - α + , + b, 0
4d a a

3 Gm 2 3 Gm 2 3 Gm 2

= Gm ' a b α - α + , l + b, 1
= 2 + 2 + 1 1
24d 3d 8d 2
3 Gm 2 1 + 8 + 3
= : D 3. Net force on M due to M and M, 2M and 2M , 4m,
d2 24
4M , 5M, 5M and 7M and 7M is zero. The force
3 Gm 2
= along SQ due to 3M is-
2d 2
2. Let’s take strip of length 'dx' at length x , from M3M

F =G
^0, 0h d2
3GM 2

Its mass = dm = ρdx = ^a + bx 2h dx =
d2
along X-axis
Gravitation 10.11
4. G ^ M1 + M2 h

So, FA = 2 m
p


For point 'B' :
GM
There will be no force by shell B
x ] L + xg

Field due to the left rod at a distance x is
GM1 m

Force on the small element dx
So, FB = 2
q

For point 'C' :
dF = dm c x ]L + xg m
GM


There will be no gravitational field.
Total force = # dF
So, FC = 0

2L 8.
= # L c x ]L + xg m dx
M GM


L
GM2 4
= 2 ln 3
L
5. When r < r1 , gravitational intensity is equal to 0
M2
GMm GMm # 3 = 3 GMm

F= cos θ =
^ 2R 2 h ^ 2R 2 h
M1
r1 2 8R
2

r2

9. At point P , Eg - Eg = Eg
1 2


when r > r1 , gravitational intensity is equal to

GM1

r2

When r > r2 , gravitational intensity is equal to
G ]M1 + M2g

r2
6. dEnet = 2dE sin θ


= 2 Gdm sin θ
r2 4 3
m1 = 3 πR ρ0

= 2G. λrd2 θ sin θ
m2 = 3 π b 2 l ρ0
r 4 R 3

2 G λ

= r sin θdθ

= 34 πr3 ρ0 18
Enet = # dE net
Gm
4
G πR3 ρ0
Eg1 = 2 1 = 3 2
π/2 x x
2gλ

= r # 4 1
G 3 πR3 ρ0 8
Gm2
0
Eg2 = 2 =
m , bx - R l bx - R l
λ = , and r = π 2 2
1 - 1
Eg = g 3 πR3 ρ0 > x 2 R 2H
4
Enet = 2Gm 2
π
8b x - 2 l
,

Along +y axis
10. The gravitational field intensity at a point inside the
7. For point 'A' spherical shell is zero.

For any point outside, the shells acts as point situated
Note: There is NO gravitational field inside a
at centre. spherical shell.
10.12 Physics
11. Flux through the surface
as Radius of the moon is one forth so g on moon is
also one fourth.
= # g .ds
Time period of a second pendulum on the earth
= g # ds
,

T = 2π gearth
= g # 4πR 2
= GM # 4πR 2 ,

at moon T = 2π gmoon
R2
g
, = , gmoon = , b 4 l
= 4πGm
dividing 1 1
earth
12. Gravitation field in side the cavity is uniform 1 99.2

, = 4 = 24.8 cm
4

g = 3 πGP GC 2 (vector joining the centres)
17. On the equator, ge = g - R~2
2
13. For r < R1 g=0
When ge = 0, g - Rω = 0


For R1 # r < R2 R

& ~ = g
2r R
g is only due to mass of shaded T = g

portion R

& T = 2r g
3 3
r - R1
g = 3 πGP e o
4 R


= 2r g
r2
GM # 3

For r $ R2 g = 2
r = 2π 640010 10 # 3600
1
h

14. 4r
OR T = 9

18. gh = g b1 - R l
2h

dwh dgh
w = mg & dh = m dh

` dhh = m dh cg - m =m
dw d 2gh 2g 2mg
, ,
Net torque = F2 . 2 - F1 . 2 R = R
R

= ] F2 - F1g 2
, dw 2mg


& dhh = R h 0

F2 = mgH2 = mg '1 - 1
2H2 dw

& dhh \ h 0
R
F1 = mgH1 = mg '1 - 1 1
2H
19. At height h above the surface fo the earth, therefore.
R
mg H1 - H2 h,
^
x = ^ F2 - F1 h 2 = g' = g b1 - R l

, 2h 2h
R & Dg1 = R g
15. we = 50 # 10 = 500 N
At depth d below the surface of the earth

w p = 50 # 5 = 250 N
g' = g b1 - R l
d d
& Dg2 = R g

Gravitational field become zero at one point during
journey..
Dg1 = Dg2 & d = 2h


Hence option (a) is correct 20. dV =- gdr

G ^ρhb 3 πR3 l 4
4 K
dV =+ r dr
GM
16. g = 2 = = 3 GπρR
R R2
Integrating, V = V0 + K log c r m
r

g\R 0
Gravitation 10.13
21. Both field and the potential inside the shell is non 1 R
2

zero 2 mv02 - mgR =- mg R + h

3GMm R
22. P.E. of the system is equal to Ui =- h =
2R 1 - k2

work done 30. Here, radius is not constant,. We assume that density
of the sphere is t
= DU = 6U f - Ui@ = Ui =-
3GMm
2R
M
Gm1
` t=
4
r1 rR 3
23. Gm = 34 3
r2
2

Consider the intermediate
situation.
m1 m2
2 =
4πr1 4πr22 dM = ^4πX 2 dxh ρ

m1 + m2 = m 4
M = ρ. 3 πX3
m = m1
Gm
4πR 2 4πr12 dW = x dM
Gm 5

or = Gm1 = 3
x bρ. 3 πx l_ 4πx dx $ ρ i
=G

4 3 2
R# r
1
2
24. Applying energy conservation from P to O
= 163π gρ 2 x 4 dx
16 2 2 R

Ki + Ui = K f + U f W = 3 π Gρ $ #
0
4
x dx

O + ^ M0 hd
= GM
n
5
2 R
= 16 2
R + ]2Rg2
2
3 π G ρ $ 5

= 12 m0 V 2 + ^m0 hb - GM l
JK M NO2
R
2
16r G R5 KK O
W = . . KK 4 3 OOO
3 5 K rR O

V=
2GM d1 - 1 n L3 P
R 5 3 GM 2

= 5$ R
r =- G : R + r D
25. V = R + b - Gm
GM l M m
3 GM 2

` Uself =- 5 $ R

r =- G : R + x D
26. V =- R + b - Gm
GM l M m
31. Gravitational potential at a point on the surface of
GM
27. F =- c dx i + dy tj m =- ait - bjt
dU dU earth is - R
If the earth is assumed to be a solid sphere, then the
28. Using conservation of energy
gravitational potential at the centre of the earth is
1 2 GMm - GMm - 3GM
2 mv0 - R = R + h 2R
1 - mgR 2 Decrease in gravitational potential is
2 mv02 - mgR = R + h
R # GM = Rg
2 2 2
h = max height R

Loss in potential energy is
R
h1 = ,h =R R # GM #
2 2 2 2 m
R
h3 = 2R
Now, gain in kinetic energy = loss in potential
29. Using conservation of energy energy.

Therefore,
1 2 GMm GMm
2 mv0 - R =- R + h 1
mv
2
2
= 12 mgR or v = gR
10.14 Physics
32. Net potential at mid point 36. According to Kepler’s law applying angular
momentum conservation m1 v1 r1 = m2 v2 r2
- Gm1 - Gm2


]d/2g
= + dA = L
d/2 37. dt 2m
- 2G

= ^ 1m + m2h ^ L = angular momentum h
d

If body is projected at escape velocity, it reaches mvmax rmin
= 2m
infinite distance with velocity just greater than zero.
2 16
12 # 4 # 10
^ h
=
1 - 2Gm 1 m + m 2 # 10
= ] P.E g f + ] K.E g f
2
mVe2 + = 4 # 10 4 ms -1
2 d
2G ^m1 + m2 h
2
Ve
= 40 kms 1 -

& =
2 d
2
4G ^m1 + m2 h 38.
4π 2
T 2 = GM a3, where a is semi-major axis.

& Ve = s
d 1 /3
T 2 GM
2
a =< F
33. = GMm
mg pole & mg' = R
mv
4π 2
R2
mg mv 2 2v 2
In case of ellipse, we have

& mg - 2 = R or g = R
2a = rmax + rmin

At the pole, total energy of a particle is
ET = U + K rmax = 2a - rmin
GMm 39. From the principle of conserving angular momentum,
ET =- R + 0
we have
GMm
` Binding energy = ET = R = mgR


mvR = mv'r .....(i)
1 2 2v2

` 2 mv e = mgR = m R R
[ v' = speed when spaceship is just touching the

& ve = 2v planet]

From conserving of energy, we have
34. Ve = 11.2 km/s. is escape velocity near
Earth’s surface. 1 2 1 2 GMm
2 mv = 2 mv' - r .....(ii)
2
mv GMm
Solving Eqs. (i) and (ii), we get
=
R R 2

R = v :v 2 + 2GM
r D
r 1 /2

GM
v' 2 = R
R2 40. Applying conservation of angular momentum at
2
v' = 10 # 64000000 = 8 km/s position A and B
2GM
mv A # OA = mvB # OB
35. Ve = R
GM vB OA
V0 =
r
Hence, v A = OB = x

Using conservation of energy
41. As the velocity of the particle is less than the orbital
- GMm -V 2
velocity of the satellite, the particle goes in the
r = 2 + 12 mv' 2 e
elliptical orbit of the semi-major axis less than r .
Canceling 'm'

Let r1 be the minimum distance and v1 be the
V2 velocity fo the particle at this position, then
-V0
2
= 2e + 12 mv' 2
2
V' = Ve2 - 2V02
m0 # 3 # v0 r = m0 v1 r1,
Gravitation 10.15

where m0 si the mass of the particle and v0 is the d d
d1 = M m2 and d2 = M m1
orbital speed equal to GM/r
2
Also,
Gm1 m2 d ^ M - m hD
= m1 ω 2 : M

v1 r1 = 3 v0 r d2
1


From energy conservation,
So, GM = ω 2 d3
2
GM = b 2Tπ l d3
2
m0 # v02 Gm0 m0 v12 GMm0
&
23 - r = 2 - r
1
1/3
GMT 2

Solving the equations, we get r1 = r/2 d =< F
4π 2
42. Net force is towards centre GMm 1
44. FG = R (as FG ? R given)
2F1

Fnet = F2 +
2 mv 2 GMm

So R = R & v 2 ? R0
= F2 + 2 F1 - GMm
45. Energy of each satellite in the orbit = 2r
_
Gm 2 = Gm 2 bbb
]2rg2
F2 = 2 bb
4r
Total energy of the system before collision,
2` " Fnet
Gm = Gm bbb
2
F1 =
^ 2 rh 2r bb
2 2
GMm - GMm
a
Ei = E1 + E2 = 2E =- 2 # 2r = r
gm
2
2 Gm 2 mv 2
As the satellites of equal mass are moving in the
= 2 + = r opposite directions and collide inelastically, the
4r 2r 2
velocity of the wreckage just after the collision is
Gm ^ +

& v= 4r 1 2 2 h Ans.
mv - mv = 2mV, i.e., V = 0
43. Let d = distance between stars and d1 and d2 be the
distances of stars from centre of mas of system.
the energy of the wreckage just after the collision
will be totally potential and will be

` d1 = c m + m m m2
d - GM ]2mg 2GMm
1 2
Ef = r = r
d2 = c m + m m m1
d
As after collision the wreckage comes to standstill in

and
1 2
the orbit, it will move along the radius towards the
m1 + m2 = M earth under gravity.

EXERCISE - 3

Numerical Type 2. Escape velocity is independent of direction of


- GMm projection.
1. Gravitational potential energy =
R
Total mechanical energy = 0
- GM # 3
- 54 = R GMm 1
R - 2 a + moe2 = 0

2
GM
Hence, = 18 GM
R
oe = 2 a
2GM
But oesc = = 36
R
10.16 Physics
3. Conserving angular momentum, 7. m is attracted by only horizontal component of
o2 gravitational force due to a part of ring, which
m (n1 cos60o) 4R = mn2R ⇒ =2
o1 changes by a factor of
Conserving energy of the system,
GMm 1 GMm 1 2r 4 2 4 10
2 2 3/2 = = 25
-
4R
+ mo12 =-
2 R
+ mo 22
2 ^ 4r + r h 5 5
1 2 1 2 3 GM 1 GM
o 2 - o1 = & o12 =
So, k=4
2 2 4 R 2 R GM
1 8000 8. g = 2 (if R increases, g decreases)
o1 = 64 # 106 = m/s ⇒ X = 8 R
2 2
Taking logarithms on both the sides, we get
2
dA rR
log g = log G + log M - 2 log R
4. =
dt T
Differentiating, we get
2
4r 3 dg
T2 =

GM
R g = 0 + 0 - 2dR
R
2r dA rR 2 dg
=- 2 b 100

T= R3/2 & = -2 l = 4
dT 2r g 100
GM R 3 /2
GM dg

%increase in g = g # 100
dA1
dt R1 R1 4 #
= =n& = n2
= 100 100 = 4%
dA 2 R2 R2
9. x 2 = 500y (Given), differentiating w.r.t. time t , we
dt dy
dx
get 2x dt = 500 dt
5. before collision after collision
2GM
Again differentiating w.r.t. time t , we get
v= m
r GM 2m GM 2
d 2 x + b dx l b dx lF d y
2 <x
m v= r 4r
GM 2 dt dt = 500 2
2r dt dt
dx 5
Given, dt = 360 # 18 ms 1 = 100ms 1

- -
and is
constant.

by angular momentum conservation,
d2 x =

So, 0
GM GMr 1 dt 2
2m e
o r = 2m (V1) R & V1 =
4r 4 R d y
2
2 b dt l = 500 2
dx 2

`

by energy conservation, dt
1
m <
GM GM
F-
GMm 1 GMm 2 ]100g2
2
d y
= mV12 - 500 = 40 = 4g = ay
+ & 2 =
2 4r 2r r 2 R dt
5GM 1 GMr GM
` Effective value of g is

&- = e o-
8r 2 4R 2 R geff = 4g + g = 5g
r
& - 8rR + 5R = 0
2 2
β =5
r = ^4 + 11 h R
10. We know that, gz = g - R~2 cos2 z

G # 103 = G # 105
At poles z = 90c , ` ge = g - R~2
]1 - xg2
6.
x2
= ^6.4 # 106h b 24 # 60 # 60 l
2r
g p - ge = R~2
1-x =

& x 10

= 34 # 10 -3 ms -2
1

& 11x = 11 km
= ]4 + 30g # 10 -3 ms -2


` ρ =1 p =4
Gravitation 10.17

= 8 b 3 rR3 tl, r = 2
GM 1 4 R
11. a =- 3 y

R
VA = R ; 3E
dv GM G rtR3

` v dy =- 3 y
`
R 3 - 2rtR
GM
G
5
=- 3 r tR2
vdv =- 3 ydy
R
RS V R V

during downward journey, u = 0, v = v and y runs SS Potential due WWW SSS Potential due WWW
from R to zero. Now VB = SSto the completeWW - SSto the sphericalWW

SS W S W
S sphere at B WW SS cavity at B WW
#0 v vdv GM # 0 T X T X

` =- ydy
=- 3 ^3R2 - r2h - :- 2 r D
R3 R GM 3 GM'


2R
v2 GM b R2 l

& 2 =- 3 0 - 2 11 GM 3GM'
R
or VB =- 8 R + R
GM
v = R 13. From the law of conservation of energy

After collision with the centre, velocity is v' (say) ]U + Kgsurface = ]U + Kgat 3
1 GMm 1 1

Given e =2
- R + 2 mu2 = 0 + 2 mv2
v' - 0 1 2GM

& v-0 = 2 - R + u2 = v2
1 GM - v e2 + u2 = v2

& v' = 2
v2 =- ]11.2g2 + ]15g2
R

In the upward journey, let the ball reaches to a height
=- 125 + 225 = 100
h
v = 10 kms -1 & a = 2
#v'
0 GM # h GM 4

Now, vdv =- ydy 14. We know that, g = = 3 rGRt
R3 0 R2
v' 2 GM h2 gp Rp t p

& 2 = $
` = R t also ve = 2GR
R3 2 ge e e


1 GM GM 2
4 $ R = R3 $ h
]vegp gp Rp gp gp te

]vege
` = ge Re = ge $ ge t p
R

& h = 2 gp te

= g
e tp

` Required distance is
6 3 3
R R
= 11 2 =1
R + 2 + 2 = 2R
]vegp 11
3 ]v g
e e =
3] g
11 11 = 3 km s
-1

` b =2
Z] Potential due b_ ]Z Potential due _b 15. Let v be the speed of projectile at the highest
]] bb ]] bb
] b ] b point. By conservation of angular momentum and
12. VA = ] [ to the complete ` [
bb - ]]to the spherical `b
]] b conservation of mechanical energy, we have.
] sphere at A b ] cavity at A bb
b ]
\ a \ a
muR sin a = mvrmax
3 GM b GM' l
=- 2 R - - r
1 G Mm 1 GMm

and 2 mu2 - R = 2 mv2 - rmax
4
M = 3 rR3 t 3R

Solving these two equations, we give rmax = 2
M' = 3 r b 2 l t
4 R 3
R

` hmax = rmax - R = 2 & } = 2
10.18 Physics
16. For a satellite in circular orbit, 19. 

mv02 GMm
r = r2
2 GM COM

or v0 = R + h B A
r2 r1
GM 1 2GM

& R+h = 2 R

GM GM
R + h = 2R & h = R
dA L mr2 ~ ~r2

When the satellite stops, k = 0
Rate of area swept dt = 2m = 2m = 2


` Ui - U f = K f
(r = distance of COM)

- GMm b- GMm l 1 dA

& = 2 mv2
& dt \ r2
2R - R
GM = b dA l
= 2 = b m2 l = 9
v = gR dt A r12 m 2
R
&
b dA l r2 1


` k =1 dt B

1
17. 2 mv esc
2 GM m
= Rm & vlaunch = 2
2GMm 20. w.r.t C.O.M frame of reference, total K.E and
m Rm
For flight of stone from moon to earth, energy is
angular momentum of system is conserved.
conserved. 1

w.r.t COM K.E. = 2 (red mass) v rel
2


` ]K + Uginitial = ]K + Ugfinal mr

w.r.t COM Angular momentum = 2 vrel
1 GmM 1 GM m
2 mv launch
2
- R E = 2 mv impact
2
- Rm
` Equating energy
mE mE
GmME 1 m 2 Gm2 1 m 2 Gm2

- R
E
2 2 v o - ro = 2 2 v rel - r

so, required elocity is
(Here vrel is relative velocity = to line as vrel along
the line joining is zero when separation is either min.
2G ; R m + R m + R E - R E E
3M M M M

vimpact = or max)
m mE E ME

q 3
Angular momemtum conservation

So, 2# p = 2# 2 = 3
mr mr
2 0 v0 = 2 vrel
18. The fastest possible rate of rotation of a planet is
rv
that for which the gravitational force on a body at v
` rel = 0r 0
the equator just barely provides the centripetal force

solving 3r2 - 4rr0 + r 02 = 0
needed for the rotation. Let M be the mass and R
be the radius of the planet and M is the mass of a r
r
` max = r0 rmin = 30
body on its surface. Then,

ratio = 3
GMm
mR~2max =
R2
GM

& ~ max =
R3
G 4

or ~ max = $ rR 3 t
R3 3
4Gtr rGt

= 3 =2 3
2r 3r
Tmin = ~ max = Gt


& b =3
Gravitation 10.19

EXERCISE - 4

1. Acceleration of small body w.r.t. earth = g - Enet = E1 - E2


^- 2g h = 3g
= GM & 1 - 1 0 = 7GM2
Now from second equation of motion R 2 4 18 36R
7 GMm
H = 2 ^3g h t 2 & t = 3g
1 2H Fnet = mEnet =
36R
2

2mx1 + m ^ x2 h 2mH + 0 2H 9. From modified Gauss’s theorem for gravitation


or X CM = 2m + m = 2m + m = 3 r=r
k

H =1 2 2H # E.ds = 4πG # 2
r 4πr dr
3 2 gt & t = 3g r=0
r=r
2. Gravitational field inside the shell is zero. But the k
force on the man due to the point mass at the centre 2
E 4π r = 4πG # 2
r 4πr dr
r=0
GM
get E = constant
is 2
3R 2
3. Center of gravity of the two particles 10. gequator = g - ω R

WX +W X ]0 g]0 g + ^mg h]Rg


At depth d g = g b1 - dR l
XCG = 1W1 + W2 2 =
0 + mg =R
1 2 11. Mass of the planet is M

The centre of mass of the two particle system is at
M = 3 rR3 t1 + 3 r "]2Rg3 - ]Rg3, t2
4 4
M ]Rg + m ]0 g R

XCM = 2M = 2
= 3 rR3 6t1 + 7t2@
4


4. VC = VC ]r' = 2Rg - VC ]r = Rg

` At the surface g1
- 3GM 3GM
= ] g1 + ] g2
2 2R 2 R $ rR3 ^t1 + 7t2h
GM G 4
]2Rg2 4R2 3

= =
M1 = ρ 3 π ]2Rg
4 3 4 3
M2 = ρ 3 πR
= 3 ^t1 + 7t2h
rGR


M
3 π ^]2Rg - R h
ρ = 4 3
Mass of the core is
6. Particle will perform oscillations about center of 4
m = 3 rR 3 t 1
earth. a =- kx
GM 2GM
` At a depth R1
7. v = R , Vescape = R Gm G 4 4rGR
]Rg2
g2 = = 2 $ 3 rR 3 t 1 = 3 t 1

Clearly Vescape = nv R

Given, g1 = g2
8. Gravatational field at m due to hollowed-out lead
sphere & 3 ^t1 + 7t2h = 3 t1
rGR 4rGR

field due to Field due to mass
=*
4-) 3 t1 + 7t2 = 4t1
solid sphere that was removed
GM'
& 3 t 1 = 7t 2

Field due to remoed mass = 2 = E2
x t1 7

& t2 = 3
M' = 4 3 # 3 π b 2 l = 8
M 4 R 3 M

3 πR
R

And x = d- 2
GM GM = GM

So E2 = 2 =
b
8 d- 2
R l 8 2 l
b 3R 2 18R 2
10.20 Physics
12. Ui + Ki = U f + K f dE ^ 2h

Also, dt =- F v =- kv v =- kv
# 3

b1 l
2 2
- 3Gm 3Gm
=- k b r l
2

& d + 0 =- 2R + 3 2 mv GM 3/2

dE =- k b r l dt
GM 3/2

or .....(ii)


From eqs. (i) and (ii), we get

2 dr =- k b r l
GMm GM 3/2
dt
2r

GMm b r l3/2 1
dt =- $ GM $ k dr
2r 2
3 2 = c Gm 2 Gm 2 m
2 mv 3 m 1 1
2R - d
= $ $ dr
2 GM k r

Gm b R - d l
1 2
v = m 1
#0 t dt #7R
R
=- dr
2k GM r
13. Let at a distance r from the centre of the earth,
m 6 @R
the body has a velocity v . From conservation of t =- r 7R
k GM
mechanical energy,
mR ^ - h ca g =
GM
m
t = 7 1
k GM R2
1 GMm 1 GMm
mv
2
2
- r = 2 mv e2 - R
2rR
15. T=
v2 = v e2 + R : r - 1D
2GM R v

1 GMm
E = 2 mv2 = n - 1
= 2gR + 2gR : r - 1D
R R


v = ; n-1E
2GM 1/2

or
ca ve = 2gR and g = m
GM R

R2
2r R 2r # ]n + 1g/2

` T = = R
2gR 2
dr 2g 2GM/R n - 1 2GM

Or v = r or dt = R r

& T ? R]n + 1g/2
1

Or dt = r dr 1/2
R 2g 16. Conservation of angular momentum of the body

& #0 t dt = 1 6]R + hg3/2 - R3/2@ about O yield the following:
R 2g
1 6]R + hg3/2
- R3/2@
2

Or t =3$
R 2g
2R ;b
g 1 + r l - 1E

1
3 h 3/2

14. Let M is mass of the planet.


Total energy of satellite at an orbital radius r is
GMm ]mv sin 30cg R = nV' ]R + hg

E =- 2r
V = V ]R + hg:
` V' = 4 D
V
GMm 2R 4

& dE =+ dr .....(i)
2r2
Therefore, h = R
Gravitation 10.21
17. Mass of sphere M \ R . If mass of
3 21. P = 0
complete sphere is M , then, the mass

This mass distribution can be taken as a solid sphere
of cavity will be M/8
of uniform mass density P0 .

Using the figure, F = force due to
whole sphere - force due to cavity.
GMm b R l - G ] M/8g m
]Rg2

F=
R3 2

GMm R 2 G ] M/8g m
2 2 -
R R2
Finding orbital speed at r < R
2 : 84 - 18 D = 8
GMm 3 GMm = 3
R R
2 8 mg GMmr mv2
3 = r
R
m 6GM GM m 2 GMe m
18. E = 2 # 5r e - 2re =- 3 r
V?r


which is the total energy of the earth-satelite system.
Orbital speed at r $ R
5r
1

So, semi-major axis fo the elliptical orbit is a = 4 V?
r

Speed of the satellite at the apogee position is
(Standard satellite orbital speed)
v #r 2 6GMe GMm
v A = 2aP - r = 3
5r 22. Total energy of satellite =- 2a

For orbit to change to a circle of radius 3r/2 - ]2a - rg



GMm 1
, the rocket has to be fired when the satellite is at the
Energy at P = - r + 2 mv
2

apogee position.
GM 2GMe
use conservation of energy
New orbital speed is v0 = 3r/2e =
3r

Required change in the orbital speed is 23.

GMe

∆v = v A - v0 = 0.085 R
19. As all the points on the periphery of either ring are
at the same distance from point P , the potential at
point P due to the whole ring can be calculated Wext = m ]VA - VBg

as V =- ]GMg / ^ R2 + x2h where x is the axial
= md n
GM GM
distance from the centre of the ring. This expression -
2R 5R
is independent of the fact whether the distribution of
mass is uniform or non-uniform. 25. By applying work energy theorem change in K.E. =
GM G # 2M work done by all the forces.

So, at P , V =- -
2R 5R

DK.E = Wg - Wfr; Wg > Wfr
=- R < F
GM 1 2

+
2 5

Therefore KE f increases due to the torque of the
20. Inside the spherical shell, V is constant, so from air resistance its angular momentum decreases
energy conservation, therefore (a), (c)
- GMm mv2 GMm
3R = 2 - R 26. Conditions for motion of geostationary satellite.
v2 GM :1 - 1 D GM # 2 4GM
2 = R 3 = R 3 or v = 3R
10.22 Physics
27. Pressing force by the particle on the wall of tunnel is 32. Gravitational potential due to hemisphere at the
and acceleration is mg sin i . centre is V because distance of each mass particle

Pressing force from the centre O is R . If the distance between the
point and mass is changed potential will also change.
GMx # R GM

= mg cos i & 2x & 2R2
R3 2GM

Pressing force is independent from 'x' thus it is 33. Escape velocity = R = Ve
constant acceleration. GM

Orbital velocity = R = V0
2 R2
GMx x - 4 GM

g sin i = = 4x 2 - R 2
Escape velocity = 2 # orbital velocity " (a)
R3 x2 2R3
θ 1 1
α
mV
2
2
e = 2#
2 mV0 " (c)
2

34. Kinetic energy decreases with increase in radius



R while the potential and total energy increases with
x is increases from 2 to R thus acceleration increase in radius.
increases.
35 PE =- G m1 m2 /r, ME =- G m1 m2 /2r
28. Motion of m
m CM
2m

On decreasing the radius of orbit PE and ME
2r/ 3 r/ 3 decreases

Gm ]2mg
m~ 2 b 3 l = 36. In case of earth the gravitational field is zero at
2r 2r r3
& T = ~ = 2r 3Gm
r2 infinity as well as the the centre and the potential is
T
` \ r3/2 and T \ m -1/2 minimum at the centre .

29. Due to symmetry the gravitational field at the origin 37. The angular velocity of the geostationary satellite
is zero. The equipotential line will take the shape of must be equal to angular velocity of earth in both
a circle in yz plane. direction and magnitude.

GMr 38. Inside a uniform spherical shell


30. Gravitational field intensity F =
R3

Inside the spheres
Ein = 0

^F ? r1, F2 ? r2h
Vin = constant = R
Gm

F1 r1 39. For a planetry motion Total mechanical Energy =


F2 = r2 of r1 < R & r2 < R
Constant

Gravitational field intensity
1
Angular momentum about the sun = constant

I? (Out side the sphere)
r2 dA
dt about the sun = constant
F r2

` F1 = 22 if r1 > R and r2 > R
2 r1 2r 2x
40. ~ S = 1.5 ~ E = 24
31. Gravitational potential
~ west to east = 2r : 1.5 - 24 D
1 1

GM

V =- R


Twest to east = ω = 1.6 hours

(b) Gravitational field at the point x from the centre west to east

of the coil is
Similarly

ω east to west = 2π :1.5 + 24 D



GMx
1 1
^R2 + x2h3/2
24

Teast to west = 17 hours
Gravitation 10.23
41. The reading of the beam balance will be independent 45. Minimum colatitude is given by
of effective g , so W0 = W' 0 but the reading of
R
the spring balance will proportional to geffective . At sin θ = R + h
equator due to centrifugal force of earth, geffective is

Curved area AB on earth
less so Reading of spring balance is less Ws < W' b
42. (a) If it is projected radially it will go up and than
= 2πR 2 ]1 - sin θg
move down in a straight line
Area on earth escaped from satellite
(b) If it is projected with a small velocity near the
earth’s surface, g will be almost constant. So
= 4πR 2 - 2πR 2 ]1 - sin θg
its path will be almost parabolic (Projectile
Motion).
= 2πR 2 ]1 + sin θg
(c) If the body is projected tangentially with orbital 46. Just before collision, the orbital velocityis
speed bV0 = GM r
l then it will revolve in
v0 =
GM
R
circular orbit.

Now, the total energy of satellite is negative. Even

(d) If the body is projected with a velocity after expelling Dm , if the satellite has to be in the
Vε ^V0, Ve h it may revolve in an elliptical orbit. gravitational field of planet, its’ total energy should
43. MP = 2Me be negative again.
1 GM ]m - Dmg
rP = 3re
` ]m - Dmg^v0 + vr h2 - #0
2 R
M

Now g ?
.....(i)
r2
2 ]v0 + vrg2 # 2GM

` gP = ge R
9
2GM
2GM
& v0 + vr # R
44. ve = R
GM 2GM
R + vr #
2G b 3 πR3 l ρ
4 R

= R 2GM

& vr # R
4Gρ

= 3 R
Maximum value of vr is

ve ? R
x = ^ 2 - 1h
GM

` R

A = 4πR12
In eq. (i), ]m - ∆mg gets cancelled out. So, no


and 4A = 4πR 22 modification is required.

& R2 = 2R1 47. By applying conservation of angular momentum

Mass of P3 is M3 = M1 + M2 mv0 R cos i = mv ]R + hg

v0 R cos i b R l
b 4 πR33 l ρ = b 4 πR13 l ρ + b 4 πR 23 l ρ

v= R + h < 1 & v0 cos i > v

3 3 3
R+h
48. By applying conservation of energy

R33 = R13 + R 23
1 GMe m 1 GM3 m
2 0 - r = 2 mv - ]R + hg
mv 2 2
3 3

& R 3 = 9R
1

v
Solving above equation
1

R3 = 91/3 and v1 = 2
2 v 02 sin2 i

h> 2g

` R3 > R2 > R1


So, v3 > v2 > v1
v3 1 /3 v1 1

Also, v1 = 9 and v2 = 2
10.24 Physics
G2Mm GM 55-57.
]2Rg2
49. F = &a=
2R2
Let the angular speed of
1 4hR 2
hR 2 revolution of both stars be

h = 2 at2 & GM = t2 & t = 2 GM w about the common centre
, that is, centre of mass of
GM GMh
50. v at surface = 2as = 2 h= R system.
2R 2

The centripetal force on
R R2 star of mass m is

If a = 0 , t1 = v = ;
GMh
2d Gm ]2mg
R2
m~ 2 3 = solving we get

but a > 0 ; t < d2
GMh
4r2
G ]2Mg m 1 GMm GMm
T = 3Gm d3
51. & 0 - ]
2R + hg
= 2 mv2 - R - 2R

The ratio of angular momentum is simply the ratio
GM
v
& , R of moment of inertia about center of mass of system.
b 2d l
2
52-54. m
Lm im ~ 3
LM = iM ~ = =2
2m b 3 l
d 2
T 2 = c 4π m R3
2

GM
GM 1/3
Similarly, The ratio of kinetic energy is simply the
R =c 2 m ratio of moment of inertia about center of mass of

GM system.
log R = 3 log T + 3 log c 2 m
2 1


va
y = mx + c 58. COAM : mva ra = mv p rp rP ra

2 (a) At perigee

(1) Slope = m = 3
vP
rp < ra ` v p > va (r)
GM
Intercept c = 13 log c 2 m = 6
(b) Distance from sun at the position of perigee

b 20 # 10 -11 l M decreases (q)
3

log 4 # 10 = 18 GMm

(c) Potential energy at perigee U p =- rp

(2) M = 6 # 10 29 Kg

(d) Angular momentum remains same (p)
2 3

(3) T ?R 59. (a) Potential at A is less than potential at B

(b) We cannot compare about gravitational field at
RA 3 R 2 ω 2
d n =d An =d Bn A and at B
RB RB ωA
ω 2 ω
(c) At C and D , gravitational field and potential
b R l =d Bn & d ωB n = b 8 l
1
3
4R ωA remains same.
A

(d) As one moves from D to A , field decreases.
ω rel = 8ω 0 - ω 0 = 7ω 0
θ rel = ^ω rel ht
2π = ^7ω 0 h t

t = 7ω
0
Gravitation 10.25
GMm 1 61. (a) At centre of thin spherical shell V ! 0, E = 0
60. P.E. =- r & K.E. = 2 mV2

(b) At centre of solid sphere V ! 0, E = 0
GMm 1

Total energy =- r + 2 mV
2


(c) At centre of spherical cavity inside solid sphere
GMm 1 V ! 0, E ! 0

T.E. = 0 if - r + 2 mV2

2GM
(d) At centre of two point masses V ! 0, E = 0

For V< r , T.E. is - ve

2GM

for v> r , T.E. is + ve

GM

If V is r i.e. equal to orbital velocity, path is
circular.

If T.E. is negative, path is elliptical.

If T.E. is zero, path is parabolic

if T.E. is positive, path is hyperbolic

Numerical Type Single Option Correct


3

3. T ' = T b R l = 3 3 T
1. Given that a particle is projected from the surface to 3R 2
a height of 10R.
Gm
4. g = 2

As the particle rises up, the work done by gravity r
reduces the kinetic energy and at maximum height it Gm
]3rg2
g' =
stops.
Gm

By the equation of conversation of energy, g' = 2
9r
- GMm 1 2 - GMm g
R + 2 mv = 11R + 0 g' = 9

20GM
v
& = 11R
2r

Escape velocity of body from earth surface is,
2GM

ve = R
10 r

So v can be written as v = 11 ve .
2. KEi + PEi = KE f + PE f
1 GMm 1 GMm
mu02 + d - n = mv 2 + d - n
2 10R 2 R
2GM 1 5. Given,
v 2 = u02 +
<1 - F The mass of satellite A = 200 kg and of satellite
R 10
B = 400 kg.
9 GM The height of satellite A above the surface of earth

v= u02 +
5 R = 600 Km and of satellite B = 1600 km.
9 (11.2)
2 & The radius of rotation of satellite
= 12 + d n
2
A = 6400 + 600 = 7000 Km
5 2
& The radius of rotation of satellite
= 144 + 0.9 (11.2) 2 = 256.896

B = 6400 + 1600 = 8000 Km.

= 16.028 km/s - 16 Time period of the satellite revolving in circular
10.26 Physics

r3 In potential energy of object m while in satellite



orbit is, T = 2r GM . 1
= –(2 × K.E.) = –2 × 2 mv2 = –mv2
73 Let K.E. given to mass m = x.
T
& A = 2r # 109 GM
So total energy = x – mv2 = 0
83 [Total energy at infinity is zero]
T
& B = 2r # 109 GM
x = mv2
2r109 6

& TB - TA = 8 8 - 7 7 @. Hence at the time of ejection, the kinetic energy of
GM 1 1
the object is 2 mv2 + 2 mv2 = mv2.
& TB - TA = 1.289 # 103 s. ]n + 1g
- GMe 1 - GMe m 1 2 10. T \ R 2
6. R + 2 mu =
2
2R + 2 mv Time period is given by
2πR
T = v ...(1)
R
> T \ v ...(2)
||

mv 2 1
and also we can write R = n
1 R
GM v2 \ n - 1
v=
u2 - R e R
1
v \ ]n - 1g ...(3)
R 2
From eq. (2) and (3)
R
T\ 1
]n - 1g
R 2

m 9m GMe
T \ Rb l
T \ Rb
n-1 n+1 l
10 VT = 10
2 +1 2
2R
Gm
m GMe 11. r g = ; r#R

10 Vr = m u - R
2
R3
Gm

Kinetic energy =
= 2 ; r>R
1 m 2 m b GMe GMe l r
2 10 (V T + V r ) = 20 81 2R + 100u - 100 R
2 2
g

= 20 b100u2 - 2R e l
m 119GM

= 5m b u2 - 200R e l
119GM

r
7. The area swept by the planet per unit time i.e.,
dA 1 2 di 1 2
dt = 2 r dt = 2 r ~ ...(i) GM =
12. V0 =
R gR
Also, angular momentum L = mr w ...(ii)
2

dA L
Ve = 2gR
Using (i) and (ii), dt = 2m .
Increase in velocity = Ve - V0 = gR ^ 2 - 1 h

8. E1 = Ef – Ei
13. VCenter = V due to complete sphere
E1 = c - ]R + hg m + R = 0
GMm GMm - V due to cavity

- GM b3R 2 - b R ll
2
GMm - 3GM'
f R p
E2 = ] + g
2
2b 2 l
2 R h = 2R -
As per question, E1 = E2 = GM
-
R
- GMm GMm GMm
]R + hg + R = 2 ]R + hg
14. W = 0 - b-
GMm l GMm
R = R
1 3 R
R = 2 ]R + hg or, h = 2 = 3.2 # 10 km
3
m
= gR2 # R = mgR
9. As the object of mass m is ejected from the satellite, = 1000 # 10 # 6400 # 103
1 = 64 # 109 J
which has same speed v so its total energy is − 2
mv2. = 6.4 # 1010
Gravitation 10.27

1. Let total mass included in a sphere of radius r be


M . For particle of mass m . 2GM s
` S2 =
v + ve 2
d + Re
GMm mv 2
= r
d + Re . d
r2
GMm 2Kr
2G _3 # 10 Me i
5

& r = 2K & M =
Gm v
` S
2
= 4 + ve2
2Kdr 2.5 # 10 Re

` dM =
Gm
M = 12ve2 + ve2 = 1 = 13ve2
]4rr dr g t =
2 K dr r

& 2
m
Gm
K
vS = 13 ve . 42 km/ sec .

& t =
2rr 2 Gm Rearth
t K 4. Given, R planet = and

` n = m = 2 2 10
2 rr m G
M M

Density, t = 4 earth = 4 Planet
GM 3 3
2. (p) v0 = R 3 rR earth 3 R Planet
v1 R2 2 & M Planet =
Mearth
v2 = R1 = 1 103

(q) L = mvR GM planet GMe .102

gsurface of planet = =
R 2planet 103 .R e2
L1 m1 v1 R1 2 2 1
L2 = m2 # v2 # R2 = 1 # 1 # 4 = 1 GMe gsurface of earth
= 2 = 10
10R e
GMm

(r) KE = R
= gsurface of planet b R l
x

gdepth of planet
K1 m1 R1 2 4
K2 = m2 # R2 = 1 # 1 = 8
where x = distance from centre of

T2 = d R2 n = 8
3/2
T1 R1 1

(s) planet
R

mdx g b R l = R ; x2 E
x mg 2 R
3.
d
T= # 4R / 5
= 108 N
4 R /5
Vs
Earth rA C rB B
Sun A
com
Ms = 3 # 105 Me
5. mA mB

Gm A mB 4r2 4r2
]rA + rBg
d = 2.5 # 10 4 Re

& 2 = m A rA 2 = m B rB
TA T B2
ve = 11.2 km/ sec . m A rA mB rB

` =

Applying COE (conservation of energy) T A2 T B2

As C com & m A rA = mB rB
- GM s m GM e m 1
hence TA = TB
- + 2 mv s 2 = 0
d + Re Re 2 3
6. T ? r
2GM s 2GMe 7. Gravitational force is a central force.
vs2 =
+
d + Re Re kr 2
8. U = 2 & F =- kr
2GM e

ve = mv 2 k
Re R =+ kR & v = mR
10.28 Physics
g
Angular momentum L = mvR = mk R 2
12. = 84 ; 1 + Rh = Re
h =
1
c1 + R m
h 2 e
GM.2m 1 e
9. - L + 2 mv2 = 0 + 0
h = Re
4GM M M
- GMe.m 1 - GMe m

& v= L m R + 2 Mv 2 =
L L
e 2Re
10. Given R A = RB = R 1 mυ 2 = GMe m ; υ = GMe

MB = 2M A 2 2Re Re
Calculation of escape velocity for A:

` ve = 2 v0
R
Radius of remaining star = 2A .
ve = 2gRe
4 R M
3
Mass of remaining star = t A 3 r 8A = 8 A N =2
13. Centre of mass of system lies at 6 R from lighter
2GM A (2) GM A
vA = 8R A = 2R mass.
Writing force equation for star 3MS along the radial
G ^18M S2 h

Calculation of escape velocity for B
7 direction, we get. 63MS ~2 # 6R@ =
Mass collected over B = 8 M A
81R2
GMS
&~ R=2

Let the radius of B become r. 81R2
` 3 r _r3 - R B3 i t A = 8 t A 3 rR 3A
4 7 4 81R3

&T'= GMS
7 ]15g1/3 R
& T ' = 9T
& r3 = 8 R 3A + R B3 =
2

& n = 09
v 2
23GM A 23GM A

` 2B = = 3 MS
1/3 R 4 # 151/3 R 6R 6 MS
8 # 15 2 C
23GM A
` vB =
9R
2 # 151/3 R
v 23 10 # 2.30

` vB = =
A 151/3 151/3

n = 2.30

11.


Writing the net force on system:
GMm GMm GMm Gm
+ -
^3l h2
^ 4l h
2
^3l h2
^ l h2
= m
2m
GM M = 2M - 2m

& +
9l 2 16l 2 9l 2 l2
M M

& 16 - 9 =- 2m
9M - 16M =-

& 144 2m
7M

& 144 =- 2m
7M

& m = 288

` k=7
Fluids 11.1

Fluids

INTEXT EXERCISE: 1
1. Pressure is independent of surface area of contact. 7. Liquids transmit pressure
2. Total pressure of base 8. Pressure in a liquid increases in the direction of
Due to oil & water = ^ρgh hoil + ^ρgh hwater
body forces. In this liquid there are two body forces:
3. For a hydraulic press,
F1 F2
A1 = A2 if the pistons are at the same height.

4. Force = Pressure # Area


= ]900 # 10 # 0.4g # 2 # 10 -3

` Maximum pressure is at B & minimum is at D
= 7.2 N
9. Since velocity is constant, the vessel is an inertial

Pressure at the base depends only on the height of
frame itself and so liquid level should be horizontal.
liquid column & not the shape of the vessel.
5. Since volumes poured are the same, the height of 10. Writing force equation for the liquid the horizontal
liquid in vessel C will be the greatest. tube at the base,

` Pressure at base in C will be the greatest. ^ρgh h A = ^ρAL ha
6. Equating liquid pressure to atmospheric pressure, aL

` h = g
3.4 # g # h = 13.6 # g # 70 cm

` h = 280 cm

INTEXT EXERCISE: 2

1. Weight = upthrust ρ w = density of water


V D g = v d g ρ e = density of liquid
v D 2

Then, Vρ B g = 3 Vρ w g .....(1)
` V = d
2. Since it is floating, 1
Vρ B g = 4 V ρ , g .....(2)

Weight of object = upthrust by liquid
Equating R.H.S. of (1) & (2),
= weight of liquid displaced. 8 8 3
ρ , = 3 ρ w = 3 g/cm
3. Weight of block = Spring force + upthrust
m 5. If ρ m = density of metal
& mg = kx + D d g
V = volume of metal
kx = mg b1 - Ddl
4 1
V ρm g = 5 V ρw g + 5 V ρ L g
mg
& x = k b1 - D dl
4 1

` ρ m = 5 # 1000 + 5 # 13500
4. Let V = the volume of body

= 800 + 2700 = 3500 kg/ m3
ρ B = density of body
11.2 Physics
6. In a freely falling vessel, gravitational effects will
Length of rod immersed = 2, - x
not be present.
,
7. For stability, G is lower than B and in the same
= 2, - 2 sin α
vertical line.
Upthrust = b 2, - 2 sin α l A ρ w g = U
,

8. If side of block = a , then

If A = area of cross section of rod
a3 ρblock g = ^a 2 # 2h 1 # g + ^a 2 # 8h

# ]0.6g g ρ w = density of water

]2 + 4.8g
This upthrust acts at the midpoint of the immersed
ρblock =
10 = 0.68 g/cm3
portion.
3

` mass of block = 10 # 0.68

Then balancing moments about the hinge,

= 680 g
9. From the vessel frame the body is
Anticlockwise moment = Clockwise moment
in equilibrium, with pseudo

of weight of upthrust
forces acting
` T = Vd ^ g + a h - Vρ ^ g + a h 6A ]2,g@b 3 ρ w l $ , cos α = ;b 2, -
2 sin α l Aρ w gE
,
4

= V ^ g + a h^d - ρh
; 12 b 2, - 2 sin α l + , E cos α
,

2 sin α
10.
1

Solving, sin α = 2 & α = 30c

INTEXT EXERCISE: 3
1. A1 v1 = A2 v2 5. In streamline motion, velocity of fluid is always 0 in
a direction perpendicular to the streamlines.
π ^4R 2h v = ^πR 2h vB
6.

` vB = 4v

2. Using Bernoulli between tap level and a level 0.15


m below it,

1 ]1 g2 + # # ]0.15g = 1 2
2ρ ρ g 2ρv
Speed of efflux = 2 g D

& v2 = 1 + 3 = 4

` v = 2 m/s.
For vertical motion,

Now using continuity equation,
1
H - D = 2 g t2
10 4 m 2 # 1 m/s = Area # 2 m/s.
-

2 ] H - Dg
A = 5 # 10 5 m 2 t =
-
` ` g
3. Clearly, (a)

Then horizontal distance
4. Using continuity equation, speed should be
maximum. 2 ] H - Dg

x = 2gD g = 4D ]H - Dg

Using Beinqulli’s equation then, pressure should be
minimum.
Fluids 11.3
7. Using Bernoulli equation, A dh

` dt = a
2gh
^0 02m of Hg h + 12 # 1000 # 1 2
. A
T = #
-1/2

And h dh
a 2g
= x + 12 # 1000 # 1.75


As per the question,
& x = 0.02 # 13600 # 10 + 2 # 1000 ]1 - 3.0625g
1
2A 6 1/2@H =
h 0 10 min
a 2g

= 1688.75 = pressure in S.I. units 2A 6 1/2@H/2 =
h 0 x min
1688.75 a 2g

` Height of mercury = 13600 # 10 = 0.0124 m
H

` = 10
x
8. Analogous to above, b Hl
2
10

or x = min c 7 min
0.02 # 13600 # 10 + 2 # 1000 ]0.16 - 0.36g
1 2

h = 13600 # 10 10. Let h = depth of water in cylinder.
= 0.019 m of Hg.
Then speed of efflux = 2gh
9. v = 2gh
for the water jet to hit the ground,

Volume of flowing out
1 2H
H = 2 gt 2 & t =
g

in time dt

= 2gh dt.a 2H

horizontal distance covered = 2gh g

If a = area of Orifice. = 4hH = R (given)

This volume leads to a decrease in 'h'

` 4hH = R 2

Such that:
2gh dt ]ag = A dh
2
R
h = 4H

EXERCISE - 1

1. tg ]H - hg 6. Pressure at point A = Pressure at point B


Thrust is equal to pressure at the point.
& ht oil g = 25 cm t water g
oil

2. F = 6tgh@ 5A?
h
25 # 1

& h = 0.8 = 31.25 cm
= ]1000g ]10g ]6g ]10g ]8g

& height difference
m1 g m2 g
3. A1 = A2
= 31.25 - 25 = 6.25 cm

Solving m2 = 3.75 kg
7. Barometer read atmospheric pressure.
4. WA > WB as mass of water in A is more than in B
= P2 V2 & ^Patm + htW gh 3 rr3
4
8. P1 V1

or PA Area A = PB AreaB
= 7HtW g & h = 7H

or FA = FB
2
9. h # 1000 # g = 100 # 13600 # g
5. Thrust is same for both containers
2 # 13.6
= ρgH
or h = 100 m = 27.2 cm
11.4 Physics
10. Force on the side walls is
(refer to figure in question to identify A, B, C )

F1 = Average pressure # curved surface area
Now, PB - PC = tgh
& PB = ^P1 + talh = tgh & PB - PA = htg + l ta

F1 = c m ]2rRhg = rRtgh2
tgh

or
2 14. Centripetal force on the element considered is

( t = density of liquid) ^dph A = ]dmg x~2

Force on the bottom is
= ^dxAth x~2
F2 = Pressure at the bottom # area of dp = t~2 xdx

the bottom p2 L


= ^tghh^rR2h = rtghR2 # dp = t~2 # xdx
p1 0

Given, F1 = F2
t~2 L2

` rRtgh2 = rtghR2 & h = R p2 - p1 = 2 ,
11. Consider an element
Also, p2 - p1 = tgh
of liquid of thickness t~2 L2
dh as shown in the tgh = 2
figure. Density at 2 2
~ L
this location is & h = 2g
t kh (where, k is
= m m
15. Dv = v f - vi = y - x
a constant).

Pressure due to this element is 16. mg = 60 .....(i)

dp = ]khg g dh
mg - t1 vg = 40 .....(ii)
h mg - t l vg 2 t0
mg =3 or tl = 3

& t = kg # hdh
0
Where t 0 = density of the block and t l = density
= 2 kg h2 = c m h2
1 kg of the liquid

2
17. ∆U = mgh
kg

t = 2 h2 is similar to y = kx2 (a parabola) ∆U = σ 0 Vgh
12. Let P1 and P2 be the pressures at the bottom of the 18. Wapp = mg - FB
left and right ends of the tube, respectively.

= tVg - tW Vg

= ^t - tWh Vg

= ^7tW - tWh Vg = 6tW Vg
4 1
19.103 # 5 + 13.5 # 103 # 5 = t # 1

or t = 3.5 # 103 kg/m3
Then F = ]P1 - P2gA = tghA
20. Let ρ S, ρ L be the density of silver and liquid. Also

Where A is the cross section of the tube. m and V be the mass and volume of silver block.

The mas of the liquid in the horizontal portion is
` Tension in string = mg - buoyant force.
m = tLA T = t S Vg - t L Vg = ^t S - t Lh Vg


Now, F = ma m

Also V = tS

& tghA = ρLAa
T =c
aL tS - tL m

` h = g
` mg
tS
13. The points A and C are in same horizontal level ]10 - 0.72g # 103

= # 4 # 10
10 # 103

hence PC - PA = tal
= 37.12 N
Fluids 11.5
21. Figure shows the points described in the equation. m g = a ]a - 2gtW g .....(ii)
2

& a2 ]a - 2g tW + 200 = a3 tW


& a2 = 100 & a = 10 cm

28. Stone is more denser than water. Initially, when ice


block with stone is floating more volume of water is
displaced to balance the weight of stone. Therefore,

Centre of gravity of submerged part of the object level of water in the container decreases when ice
will be at the centre of buoyancy if the object is completely melts. (Later volume of displaced fluid
uniform, is equal to volume of stone only).

Otherwise shifts from this point. From figure (d) is 29. Water displaced will be same in each case.
correct.
30. If stones in the boat are unloaded, volume of
22. Fb = vt liq g displaced fluid is less than volume of displaced fluid

'g' is different on moon and on the earth. initially. Therefore level of the fluid falls.

Hence only (iii) is a correct statement. 31. Balloon will stay at equilibrium at any position in

Hence (d) 32. Acceleration of ball in water
23. Velocity of ball when it reaches to surface of liquid.
net force Th - mg
m= = m

V ]d - Dg ]d - Dgg
= VD = D

Velocity at the surface
1000gV - 500gV ]d - Dg

a= ; where V is the
v = 2ah = 2 D gh
500 V

Volume of the ball 33. As the cork moves up, the force due to buoyancy

a = 10 m/ sec 2 remains constant. As its speed increases, retarding
force due to viscosity increase. The acceleration is

Apply v = u + at & 0 = 2gh - 10t variable, and hence the relation between velocity
& 2gh = 10 # ]2 g and time is not linear.


& 2 # 10 # h = 400 & h = 20 m 34. tA = 0.75, t p = 0.6

tB = 1.0, t Q = 0.9
24. Let mass of gold is m then mass of copper 210 - m

As relative density of P is lesser than B , so it

upthrust loss of weight
will float in liquid B and as relative density of Q
= 210g - 198g & Vin tW g is greater than liquid A so it will sink, because if

= 12g & Vin = 12 cm3 density of the object is greater than that of the liquid

Total volume i which it is immersed, then its weight is more than
the upthrust and vice versa.
m 210 - m m 210 - m

=t + t
gold cu
= 12 & 19.3 + 8.5 = 12 35. Let v be the volume of the solid block of density
t . Let t1 is the density of water. Weight of body

& m = 193 = vtg . When the body is immersed in water,

So weight of gold = 193 g
Tension in the string = Upward thrust - Weight of
25. In free fall geff = 0 the body
26. Weight = upthrust & mg = ^3 # 2 # 10 -2h # 103 # g & T = vt1 g - vtg = vg ^t1 - t2h


& m = 60 kg
When the lift is moving upwards with acceleration
27. Let mass of cube is m and side is a then a , the tension in the string is T = v ^t1 - th^gah

]m + 200gg = a3 tW g .....(i) From Eqs. (i) and (ii), T = T0 ^1 + a/gh



11.6 Physics
36. From Pascal’s law, pressure is changing at every 46. v A = vB (Since area is uniform)
point by the same amount. Hence, buoyancy

From Bernoulli’s principle
remains the same. So, the part of the block inside
water remains the same. VA2 PA vB2 PB
2 + gh + t = 2 + 0 + t & PA < PB
37. Apparent weight of the balloon = W2
Apparent weight = real weight - upthrust 47. For horizontal motion


or W2 = real weight - W 1 1

P1 + 2 tV12 = P2 + 2 tv22

Now, real weight = W1 + W
1
& 3 # 105 = 105 + 2 # 103 V22

` W2 = W1 + W - W = W1

& V22 = 4 # 102 & V2 = 20 m/s
38. As the piece touches the glass at small number of
1 1 1
points, water would be there at the bottom of the 48. tA + tVA2 = iB + tVB2 = tC + tVC2
2 2 2
piece also and hence buoyancy acts on it.

as rA = rC > rB

From equilibrium condition, mg = FB + N
rA = VC < VB
m
FB = V t water g = t metal # t water # g

` t A = tC > t B
0.16

= 8 # 10 = 0.2 N
` height of the liquid in the tubes A and C is same.


So,
160
N = mg - FB = 1000 # 10 - 0.2 49. V22 = V12 + 2gh = ]2g2 + 2 # 1000 # 5.1 # 10 -1

= 1.4 N = 1024
39. Work = DPV = ^3 # 10 - 1 # 10 h
5 5

V2 = 32 cm/s

# 50000 = 10 J 10
50. v A a A = vB # aB = v A # 4 = vB # 2 vB = 2v A
40. In case of non viscous, streamline flow, radial 1 1

Again, 2 tv 2A + tgh A + p A = 2 tv 2B + tghB + pB
velocity gradient is zero.
41. A1 V1 = Total area # ^velocity through each capillaryh
1 1
& 2 tv 2A + p A = 2 tv 2B + pB ]as h A = hBg
42. From equation of continuity & p A - pB = 2 t ^v 2B - v 2Ah = 2 # 1 # ^4v 2A - v 2Ah
1 1

]A # 3g = ]A # 1.5g + ]1.5A # Vg
1
& 2 # 1 # 1000 = 2 # 1 # 3v 2A

& V = 1 m/s2

( p A = pB = 2 cm of water column = 2 # 1 # 1000
43. Using A1 V1 = A2 V2 dyn/cm 2 )
2 # r ]2 # 10 -2 g2
4000
V2 = = 8 m/s. ` vA = 3 = 36.51 cm/s
r ]1 # 10 -2 g
&
2
So, rate of flow = Va a A = 36.51 # 4 = 146 cm3 /s
V
Maximum height = 2g = 3.2 m 2
51. The air through the horizontal tube will decrease
44. the pressure and more liquid will be pushed into the
V22 = V12 + 2gh capillary tube.
V2 = 2 m/s A
52. Using equation of continuity, we have v2 = A1 v1
A1 V1 = A2 V2 2

A = 5 # 10 5 m 2
- From Bernoulli’s theorem,
1
45. From continuity equation, velocity at cross-section
p1 + tgh1 + 2 tv12
(1) is more than that at cross-section (2).
p2 = tgh2 + 2 tv22 & g ]h1 - h2g = 2 ^v22 - v12h
1 1

& 60 = d - 1n v12 & A1 = 4


A12
A2 1
A22

Hence P1 < P2
Fluids 11.7
53. R = vt 59. Applying Bernoulli’s theorem at point 1 and 2,
difference in pressure energy between 1 and 2 =
2 ]H - Dg difference in kinetic energy between 1 and 2.
= 2gD g
= 2 D ]H - Dg
54. x = 2 h ]H - hg
dx H

For xmax, dh = 0 or h= 2

55. x = 2 H ]H - hg
mg 1

x1 = 2 70 # 20
Hence, thg + A = 2 tv2

2 dgh + n

x2 = 2 60 # 30 2mg mg

or v= 2gh + tA =
tA

x3 = 2 40 # 50
2g b 2 l = gh .....(i)
h
60. v1 =

x4 = 2 50 # 40 or x3 = x4 = maximum
56. Velocity of efflux
From Bernoulli’s theorem,

tgh + 2tg b 2 l = 2 ^2th v22 .....(ii)


= 2gh = 2 # 10 # 5 = 10 m/s. h 1


Rate of flow

& v2 = 2gh
= Av = ^1 # 10 -4h # 10 = 10 -3 m3 /s

v 1

` v12 =
57. Rate of flow = Av = rr2 # 2gh 2

= 3.14 # 1 # 2 # 1000 # 10 = 444 cm3 s 61. Rate of water coming out will decrease with time.

58. Total pressure at the bottom = 3 atm


Therefore t1 < t2 < t3

Pressure due to water in the tank 62. Height will become maximum when

= 3 atm - 1 atm = 2 atm
Rate of water = Rate of water

= 20m of water column
Flowing in flowing out.

Height of water in the tank is h = 20 m

So, velocity of efflux 70 cm3 /s = 2gh # 1cm 2
-2

= 2 gh = 2 # 10 # 20 = 400 m/s h = 2.5 # 10 m

EXERCISE - 2

1. Pressure exerted by fluid at closed end B is 3. The liquid will rise upto 80 feet height
only if the pressure at the point is less than
P = tgl
P0 by an amount t w .g.h. i.e.; Pressure at

` force exerted by fluid at closed end B is point A should be PA = P0 - tgh < 0 (As
tgh = 103 # 10 # 80 > 105 ]P0g ) and it is not
F = PA = ltg A0 possible for us to make the pressure negative.
2. To measure the atmospheric pressure, same length
of tubes containing mercury are required, no matter
how many tubes are used.

(Since Patm = t Hg g h ) Hence (d)
11.8 Physics
4.
At point B the pseudo force is maximum hence
pressure is maximum.
B 8. At point H the pseudo force is minimum hence
pressure is minimum.
O

9. For the given situation, liquid of


45°
45° 45°
θ density 2t should be behind that
A
G1 G2
of t
m1g m2g
From right limb
D C

G1 and G2 be the center of gravities of two liquids


PA = Patm + tgh
then l l

PB = PA + ta 2 = Patm + tgh + ta 2

+AOC = 90c = +COB
PC = PB + b2ta 2 l = Patm + tgh + 2 ta l
l 3

+AOG1 = 45c .....(1)

+G1 OD = 45c - i
But from left limb:

+COG2 = 45c PC = Patm + ^2th gh .....(2)


+G2 OD = 45 + i
From (1) and (2)

Net torque about point O is zero 3 3a

& rm1 g sin ]45c - ig = rm2 g sin ]45 + ig
Patm + tgh + 2 t a l = Patm + 2 t gh & h = 2g l

sv ]45 - ig = vv sin ]45 + ig
sin ~2 r2
10. y = 2y
s sin ]45 + ig
Put values and get y = 2cm
v = sin ]45 - ig


s sin 45 cos i + cos 45 sin i 11. N
v = sin 45 cos i - cos 45 sin i arel

s-v cos i + sin i - cos i + sin i ma


s + v =
cos i + sin i + cos i - sin i mgsinθ-macosθ
mgcosθ +masinθ
s-v
s + v = tan i mg


marel = mg sin i - ma cos i
i = tan -1 b s + v l
s-v


but for water surface tan i = a/g
5. a
& rel = 0

12.

ρ1 ]2cmg = ρ 2 g ]2cmg & ρ1 = ρ 2


g
Volume equality gives
6. Pressure at the bottom level of the block
1
mg 2 # 3 = 2 # h # 3 & h = 4 m
P = a + P0
4 a 4
` tan i = 3 = g & a = 3 g

The pressure at the same level remains the same.
13. Given R = 5cm, ~ = 2r ]4g rad/s
mg m

Hence, a + P0 = htg + P0 & h = at ~2 x2

Using h = 2g ,
7. a = a0 _S
i - Uj + U
ki
the required level difference is y . When x = R,
]8rg2 ^5 # 10 -2h

As there is no gravity; the pressure difference will
be only due to the acceleration.
y= 2 # 9.8 = 8 cm
Fluids 11.9
14.
cube is dipped is

& 625 = 10 # 10 # d & d = 6.25cm .

Now, 15 # 15 # h = 15 # 15 # 8 + 625

& h = 97/9
[ t c ^density of cubeh, t l ^density of liquid h

Vl ^volume of liquid displacement h ]

v = u + ax t, ax = vlt
a 22. Weight = Buoyant force Oil
v 0.5
tan i = gx = tg = 5 (in triangle ABC )
Mercury
10 # 20 V V

& t = 10 = 20 s
Vt m g = 2 t H g + 2 t oil g
g

15. 636 - t , Vl@g = 648 - t l V2@g


t H + t oil 13.6 + 0.8 14.4

tm = = = 2 = 7.2
g

;36 - t l b 36 lE g = ;48 - t l b 48 lE g
2 2
9 t0
23. Let h = height to of water column

Solving t0 = 3
then tW gh + t H g ]10 - hg = t Cu g10

16. In stable equilibrium the object comes to its original
g

state if disturbed. & h + 13.6 ]10 - hg = 73



17. Weight of left side will increase by the Buoyant
force = Vρ ω g = 40g
& 63 - 12.6h & h = 5 cm

18. F.B.D. of rod 24. In balanced condition


W = ]0.012g ]1g ^2 # 10 h ]10g
3
= 240 N V
Mg = Th & 6g = 3 tW g .....(i)
Fb = ]0.012g ]1g ^103h ]10g
and ]6 + mgg = V tW g .....(ii)
= 120 N


Torque about O

From equation (i) and (ii) 18 = 6 + m & m = 12 kg

(For equilibrium)
25. Reading of spring
]240 - 120gb sin2 a l = 45 ]cos ag
Mg
= - Th = Mg - Vin tW g
90 3
& tan a 120 = 4 & a = 37c
1000 # 10 -6 # 3 #

= 12 - 2 10 10 = 7N
19. Increasing the temperature of water from 2cC
to 3cC increases its density while decreases the 26. Due to extra water, extra upthrust act on the steel
density of iron. ball so ball move up.

Hence the bouyant force increases. 27. Let V1 volume of the ball in the lower liquid then
20. Buoyant force Fb = Vsub $ t l $ g V t g = V1 t2 g + ]V - V1g t1 g


where, Vsub, t l and g all are same w.r.t. O1 and O2 & Vg ^t - t1h = V1 g ^t2 - t1h


Hence (a) V t - t1 t1 - t

& V1 = t - t = t - t
21. Given: 2 1 1 2

28. When the ball is pushed down, the water gains


t c # ]10g3 # g = 5 # 105 dynes
potential energy, whereas the ball loses potential
& t c = 0.5; g = 1000 cm/s2 energy. Hence, gain in potential energy of water

Buoyant force should balance weight & = ^Vth rg - b 2 tl b 8 rl g
V 3

V
t l l g = 5 # 105 Vl = 625 cm3 , ` depth upto which

(When half ot he spherical ball is immersed in water,
11.10 Physics
3r
rise of c.g. of displaced water = 8 ) r ]7g1/3

& R = 2

= trg b1 - 16 l = 3 rr3 trg # 16 = 12 tr 4 tg


3 4 13 13 32. OP is the portion of the rod
v
immersed in water
4

Loss in PE of ball = Vt'rg = 3 rr 4 t' g
40
13 4
cos 60c = OP

Work done = 12 rr 4 tg - 3 rr 4 t' g
40 40

or OP = cos 60c cm = cm = 80 cm
= rr 4 tg ;13 - 4 t E
t' 1/2


12 3
The centre of buoyancy is at the centre of the
= rr 4 tg :12 - 3 # 0.5D = 12 rr 4 tg
13 4 5 immersed part of the rod. So, the required distance


is 40 cm .
29. In floating condition htg
33. Pressure on the wall = 2

weight = upthrust
Net horizontal force
& b 5 Ll Dg = b 5 4 l 2dg + b 5 4 l dg
A A L A 3L

= P # area = 2 # ]hvg = 2
hpg h 2 t gv
d 3d 5d
D & =2+ 4 = 4 34. At a depth y from the surface of the fluid, the net
30. W = weight of liquid force acting on the gate element of which dy is
fB = buoyant force on the ball dF = ^ p0 + ρgy - p0 h # 1dy = ρgydy
mg = weight of the ball
Torque of this force about the hinge is
N = normal reaction between the ball and
dτ = pgydy # b 12 - y l

the surface. The free body diagrams of

Net torque experienced by the gate is

the balls in each vessel are as follows.
τ net = # dτ + F # 12
1


= ρgydy b 12 - y l + F # 12 = 0 #
0

ρg

& F = 6

6tL@ = 2 tVL
2 dp dm
35. Fth = dt = 2 V dt = 2 V

At base, reaction force of buoyant force will act in
downward direction. 36. by A1 V1 = A2 V2

The forces acting at the base of each tank are
b r4D1 l V1 = b rD2 l V2
2 2

FA = W + fB = W + mg 4
V2 = 4V1
FB = W + fB = W + mg
37. ]rR 2 g v = n ]rr 2 g v'
FC = W + fB + N = W + mg
38. Rate of flow = Av

Thus, FA = FB = FC

Volume of water filled in tank in 15 s
31. Let t be the density of the material, t 0 be the
density of water. When the sphere has just started 15

A # 10 :1 - sin 30 tD dt
# π
sinking, the weight of the sphere = weight of water V =
0
displaced (approx.)
cos r/t 15
= 10A <t + F = 10A ;15 - E
30
& 3 r ^R3 - r3h tg = 3 rR3 t 0 g
4 4
r/t 0 r
^R3 - r3h
& ^R3 - r3h t = R3 t0 & Height of water level = 10A = :15 - 30 D
t0 V
= t
R3 r m
Fluids 11.11
gl v = 2gh .....(ii)
39. A1 V1 = A2 V2 or A # V1 = 2A 2 or V1 = 2gl
2 ρ 7V1 - V 2 A = ρgl sin θ
1 2 2
from equation (ii) put the value of v in equation (i)

& 2 ;2gl - E = gl sin θ


1 gl πR 2 dh/dt = πr 2 2gh

2
3
on solving sin θ = 4 2
R dh =

& # 2
r 2gh
# dt
40. As water moves downwards, speed increases.
Thereby area of cross section decreases as it moves 0 T
2
downwards to ensure constant rate of flow across the R dh = #
fluid flow.

r 2 2g
# h
dt
h 0

41.
R2 2h
T = 2 g
r

on solving
t = 46.26 second

ρ1 - ρ 2 = ρg d tan θ = ρ m g∆h 45.
42. Initially, the water flowing out will be less than
that flowing into it. Hence, the water level will go
on rising. When the water level is h , the velocity
of efflux = 2gh . When this becomes equal to the
velocity of inflow, the level will become steady as
2g b 2 l = gh
h
the area of cross section of the filling tube and area
Velocity of efflux of water =
of cross section of the hole are equal. This height is

force on ejected water = Rte of change of momentum
given Torricelli’s theorem.
of ejected water.


v2
v = 2gh or 2g = h = t ]avg]avg

= t av2

Thereafter the water level will not rise.

Torque of these forces about central line
43. = ^tav2h 2R.2
2
Fthrust = tav
Fnet = F1 - F2 = at 62g ]h1 - h2g@
= 4tav2 R = 4t agh R


= at ^2ghh 46. Pressure at (1):


or F ?h P1 = Patm + t g ]2hg


Applying Bernoulli’s theorem between points (1)
44. and (2)

6Patm + 2t gh@ + tg ]2hg + 12 ^2th ]0g2


= atm + ^2th g ]0g + 2 ^2th v2


1
P


& v = 2 gh Ans.
A1 v1 = A2 v2
h v
πR 2 dh/dt = πr 2 v .....(i) 47. v0 = 2gh , v = 2g = 4 0
2 2
11.12 Physics
48. The two streams strike at the same point on the
t1 = time of fall of water stream through Q is
ground.
2 ]H - h1g
= g

t2 time of fall of the water stream through
=
2h2

P= g

Putting these values is Eq. (i), we get

R1 = R2 = R ^ H - h1 hh1 = ^ H - h2 hh2

u1 t1 = u2 t2 .....(i) or 7H - ^h1 + h2 hA 6h1 - h2@ = 0

where u1 = velocity of efflux at Q = ^2gh1 h and



H = h1 + h2 is irrelevant because the holes are two

u2 = velocity of efflux at P = 72g ^ H - h2 hA different heights. Therefore, h1 = h2 or h1 /h2 = 1

EXERCISE - 3

Numerical Type 5. Acceleration of the ball when it reaches to liquid’s


1000gV - 500gV
1. surface, a =
500V
2 1
10 cm where, V is the volume of the ball.

15 cm
20 cm ⇒ a = 10 m/s2 V(1000)g
Applying n = u + at at t = 2 s,
10 cm we have n = 2 × 10 = 20 m/s a
ral = rgh Now, using n - u = 2ah, we have
2 2

gh 10 # 5
⇒ 2 × 10 × h = 400 ⇒ h = 20 m V(500)g
a= = = 5 m/s2
l 10
1 6.
2. 0.9P0 + rgh = P0 + rn2
2 x
& n = 5 m/s
x
3. 1
21
d-x d+x

mg = [2r1 × 103 + r1 × (203 - 103]g
vvg = # >t e1 +
0
h0
o (dv) g + t0 e1 +
h0
o (dv) gH

= 9r1 × 103g = tl × 8 × 103g v/2
d d

⇒ r1 =
8
g/cm3 vv = 2t0 e1 +
o # dv = t0 v e1 + o
h0 h0
9 0

d
4. x & v = t0 e1 +
o
dh h0
h 7. Buoyant force on soap, FB = A[xr0 + (L - x)rw]g

Weight, W = LArsg

For equilibrium, FB = W
dh
Q = an = px d -
n 2

⇒ LArsg = A[xr0 + (L - x)rw]g
dt x

a 2gh dt = -px2 dh ...(1)
⇒ Lrs = xr0 + (L - x)rw ⇒ rs = [r0 - rw] + rw
L

x2 = R2 - (R - h)2 ...(2) x tw - t s 1000 - 800 2

⇒ = = =

Substitute (2) in (1) and do integration L t w - t0 1000 - 300 7
Fluids 11.13
t 15

A # 10 <4 - r sin tF dt = 300A;


r
8. V = # Avdt = # 3
0 0
V V
13. Initially 2 t l g = Vt s g
H
` = = 5m
60A
l/4

t l = 2t s
9. Mg = # t o (1 + by) (dy) Ag
0

When the system accelerates upwards
1 bl
M = t0 Al < +
F
4 16
l


M’g = # t (1 + by) dyAg
o
l/4

3 15
M’ = t0 Al <
+ blF
4 32

3Mg
V't l c m = VPs c m
10. FH = (tgR) rR 2 = tgrR3 = 4g 4g
2 3 3

Fv = (M + m)g V
V' = 2
3Mg 2


Fnet = [(M + m) g] 2 + e o 14.
2
9 m 2
When it is depressed by x .

Fnet = Mg d1 + n
4 M
acc = a g b d = 1a l
dx
2h #
11. Initially g 2g # 10 = R
1
T = 2π g

After applying excess pressure

P + ρg ]10g
15.
2d
n = 2R
2h
g # ρ
P + ρg ]10g
ρg ]10g

Taking ratio =4
P = 3ρg ]10g

Apply Bernoulli theorem at point a & B,
= 3 atm

12. 1 1
PA + tVA2 = PB + tVB2
2 2

as A >> a, VA << VB
F 1

& + t0 t0 + tVB2
A 2
2 F

& VB2 =
tA
]2 g^ 4 # 103 h

=
^103 h]2 g

Assume area of cross section of rod: A

=4

Equating torque about P

& VB = 2 m/s.
(2LA) tgL sin i = ^xAt w gh b2L - 2 l sin i
x


x =L
r

This means i = 3
11.14 Physics
16. or T + ]0.8g ]250g g = ]250g t L g
.....(i)
T + BF = m2 g
2

or T + ]250g ^t Lh^gh = ]1.2g]250g^gh


1 2
.....(ii)


Time period of above system
l
g ]sin a1 + sin a2g

= 2r ,


when l is the length of liquid column.


From Eqs. (i) and (ii)

T = 0.5 N
R
k 1
R/2 18 = 2 & k = 9
30° 60°
20. Tmax = ^7 # 106h ^10 -6h = 7N

b R + rR l
Initial buoyant force is
3

In the given question, T = 2r
g ]sin 30° + sin 60°g BF1 = ]0.1g3 ]1000g]10g = 10N
2 ]3 + rgR
Weight of the block is

3g ^1 + 3 h
= 2r
w = ]0.1g3 ]1500g]10g = 15 N
17. Let H be the depth of the lake

` BFmin = 15 - 7 = 8N
As temperature is constant, ^pVhbottom = ^pVhsurface


(at the time of breaking of wire.)
6t + H ]1g^gh@ 34 rr3
gh = ^tghh 3 r ]2rg3
4


H = 7ht & b =7

18. t S < tW . So, block tends to move up


and thus the spring has elongation.

BF = mg + kx
VtW g = Vt S g + kx ` ^Vi h ]1000g ]10g = 8


0.5 # ]0.5g]10g + 50x
Volume of water to be removed
800 1000 10 =
#

= ]200 - 100g]2g = 200cm3
5
x = 0.025m = 2.5cm = 2 cm

` 2t = 200

& b =5
& t = 100s
19. T + m1 g = BF 1
& t =1


Fluids 11.15

EXERCISE - 4

1. No sliding & pure rolling 3


5. DP x = mv sin 60c = 2 mv

Therefore acceleration of the tube
= 2a (since COM of cylinders mv 3
DP y = 2 + mv = 2 mv
are moving at 'a' )
9 3
PA = Patm + ρ ]2ag L (From horizontal limb)
& DP net = DP x2 + DP y2 = d + n mv
4 4

Also PA = Patm + ρ gH (From vertical limb) DP net = 3 mv
gH

& a = 2L Since, dm = A ]v dt g ρ
f dm
p
2. As long as t # tW' pressure at the bottom of the & dt = A ρv
pan would be same everywhere, according to the
= 3 b dt l .v = 3 ρ Av 2
Pascal’s law. ∆F net dm

&
3. 6. Weight of sphere+chain = ]m + mhgg

Buoyant force = b3m + 7 l g


mh

for equilibrium, weight = Buoyant force or,


mh 7m
m + mh = 3m + 7 or h =
3m

Let height of liquid be x at any instant.
7. If sphere is displaced by x (say in upward direction)

Velocity of top layer
from is equilibrium position.
VA = 2gx a
a
Then increase in weight = mxg
V = A 2gx
mxg

Increase in buoyant force = 7
dv a 2g dx
acceleration = dt = A
2 x dt
& Net extra force = c mxg - m in downward
mxg

ga2 7

= 2
Direction.
A

( - ve because x is decreasing)
& ]m + mhga =
- 6mg
7 x
4. The four piston are initially in equilibrium. As
7 ]m + mhg
additional force F is applie to each piston, the
pressure in fluid at each point must be increased by Hence the motion is S.H.M. & T = 2r
6mg
F
A so that each piston retains state of equilibrium.
7m

Putting h = ;
3m

7.10m 2r 35m

T = 2r = 3
18mg mg



Thus the increment in pressure at each point is
F
DP = A (by Pascal’s law)
11.16 Physics
a h -h 4
8. tan i = g = h tan 45c2 + h1 tan 45c 13. T1 = T2, Mg + 2 T1 = 3 rr3 tW g T1 = 250 2 N
2 1

4 cm

= 20 cm


& a = 2 m/s 2
9. For spring balance A
Mg
= - Th = 2g - Th
14. Consider the condition of equilibrium for the mass

for balance B of water contained between cross section separated
mg
= + Th = 5g + Th by x and x + dx from rotation axis.
p2 r2
10. When the levels equalise then the height of the liquid
dp = t~2 xdx; # dp = # t~2 xdx
r1 r1
h1 + h2
in each arm =
2

Transferred length of liquid
h1 + h2 h1 - h2

= h1 - 2 = 2

Transferred mass

=c m Aρ
h1 - h2

2

Loss in gravitational potential energy
p2 - p1 = t~2 ; x2 E = t~2 b r2 r1 l
r2
2
2
- 2

= mgh = c m Aρg
h1 - h2 2
r 1 2
2
2 ^p2 - p1h

t ^r22 - r12h

Mass of the entire liquid ~=
= ^h1 + h2 + h hAρ 15. Absolute pressure in bulb = Gauge pressure +

If this liquid moves with a velocity v then its KE Atmospheric pressure = P + Pa

Using Bernoulli’s equation,
= 2 ^h1 + h2 + h h Aρv 2
1
t a v2

Pa + P = PBC + 2
&c m Aρg = 1 ^h1 + h2 + h h Aρv 2
h1 - h2 2
2
Where t a is the density of air
2
^h1 - h2 h
g
` PBC = Pa + P - b 2 l v2 = Pa - tgh
1.3
2 ^h1 + h2 + h h
v
& =

so equating these two values for PBC we get
11. P = 100 cm # 1 g cm -3 # 1000 cms -2
P + tgh
= 105 dyn cm -2
V=
0.65

F = 105 # 100 dyn = 1.99 N
12. When the oil is poured, the fraction of ice in the
water decreases, i.e., volume of ice melted into
water is greater than volume of water displaced by
ice. So water level rises. Overall volume of ice will
decrease as it melts. so the upper level of oil falls.
Fluids 11.17
16. Pressure at the location of element is 18. A snapshot of the system at an instant is shown in
p = tgh the figure. v = speed with which the water level

or p = tgy sin i
lowers (this has to be constant)


Force on the element is dF = p ^b dyh , where b is

the width of the wall.
A = rx2 = area of surface of water.
dF = tgy sin i ^bdyh = tgb sin i ydy


v' = 2gy = speed of

` Total force on the wall due to the liquid is
H/sin i

water through orifice.
F = tgb sin i # ydy
a = area of orifice (given)
0

H2 tgbH2
From equation of continuity, at this instant

or F = tgb sin i 2 =
2 sin i 2 sin i
tgH2 Av = av'
F

or b =
2 sin i
rx2 v = a 2gy
17. We know that
r2 x 4 v2
a 9 2gy =
tan i = g0 = 10 a2
r2 v2 4

&
y= .x
2ga2
r2 v2

If v has to be constant, = constant = k (say)
2ga2

then equation required is y = kx 4
h

Also, tan i = 3 (from figure) r2 v2

where, k=
h 9 2ga2

` 3 = 10 & h = 2.7 m
19. From the non-inertia frame of tube, force on the

So, when water level goes below C no water comes liquid element is zero.
out through C . Initial volume of water is

Vi = 3 # 3 # 2 = 18 m3

Final volume of water is
1

Vf = 2 # 3 # 2.7 # 3 + 3 # 0.3 # 3 = 14.85 m3

` Volume of water spilled

= Vi - Vf = 18 - 14.85 = 3.15 m3 ^dph A ^dx ath ~2 x = 0


= 3150 L
^dx At = mass of elementh
11.18 Physics

dp =- t~2 xdx ρQV Uj + ρQV S
i

#p p dp #L - h xdx
L
Moment of the force about 0 is
&
2
=- t~2
1
2
ρQ l

ρQV l = anticlockwise.
p2 - p1 =- 2 6L2 - ]L - hg2@
2
t~2 πr


23. In a static fluid, pressure remains same at the same
level, ie, pressure do not vary with x-coordinate.
=- 2 6L2 - L2 - h2 + 2 Lh@
t~2



Hence (c).
P = t ]2hgg
2
p1 - p2 = 2 62Lh - h2@ .....(i) 24.
t~


F ]2hgg

On applying Bernoulli’s theorem between points (1) A2 = t
and (s), we have Fbase = 2h t g A2
Fwall = h t g 6A2 - A1@, at the level x
p1 + 2 t ]0g2 = p2 + 2 tv2
1 1

25. Assuming that the block is completely submerged in
1 water, then
p1 - p2 = 2 tv2 .....(ii)
Fb = 1000 > mg ]920g So, not possible.

From Eqs. (i) and (ii), we get

Let complete in oil
: D
t~2 2 2L
Fb = ]0.6g ]4g ^1000 + ]1g]6g]100gh = 840
1 2
2 t v = 2 h h -1

v = ~h
2L Fb < mg So, not possible
h -1

So let 'x' part in oil and remaining water
20. PA = PB = t m g ]l - xg
920 = 6]1g]10 - xg + ]0.6g]xg@ 100
9.2 = 10 - x + 0.6 x
0.4 x = 0.8
x = 2 cm
21.
26. PV = constant

(Assumed isothermal process)

27. for FBD of flask



Condition for floating
ω1 > ω 2
4rR ]t g t s g = 3 rR3 t l g
4
2

22.


At the bend force exerted by water is
Fluids 11.19
28. Pressure at the base = tg 2h 32. The free-body diagram of

= 2tgh the entire liquid is as shown
in the figure, m is the mass

Force exerted by the liquid on the base of container. of the entire liquid in the
vessel. As the liquid is in

= 2tgh A2
equilibrium, net force on the
liquid is any direction has to
be zero. From the above statement, we can say that
option (a) is correct.

For liquid contained in section lettered A , vertical
component of forces acting on the inclined face is
equal to atmospheric pressure force and weight of
liquid. Force on the base is atmospheric force plus
weight in section B . From the above statement, we
~ + N1 = ^2tghA2 h
can say (directly or indirectly) that options (b), (c)

and (d) are also correct.

` ~ < 2tghA2
33. Pressure decreases in the

& N1 = ^ pressure at x h^ A2 - A2 h direcion of aceleration. If
we take two points 1 and 2,

= ^tgh h^ A2 - A1 h two points in the horizontal
1 1 line, then we can write
29. ρ1 + 2 ρV12 = ρ 2 + 2 ρV22 p1 - p2 = tal .
A1 V1 = A2 V2
As the inclination of liquid surface with tan θ = a/g.
The equipressure lines will be parallel to this line.

Also ρ1 - ρ 2 = ρgh

30. If one surface is pushed down by x the other surface 34. When balloon is filled with air, balloon experience
moves up by x buyoncy force due to surrounding air.
.. . . .

Net unbalanced force on ........................................ B  W
..............
............ ..........
...................................
.................................

..............
. .. .. ..
. .......................

.......................
.. ... .. ..

..............
................ ...........
.
. .

...............
. . .. .. ..
..............

The liquid column = 2xAtg .......................................
..............


Mass of the liquid column = lAt

& - 2x Atg = ^lAth a & a = c- mx


2g

W1 W
l
2g 2r l
Buyoncy force is equal to weight of displaces air,
a
a =- ~2 x & ~ = l & T = ~ = 2r 2g which is equal to ~ as thickness of balloon is
31. The maximum amount of negligible.
water that can be retained
` ~ 2 = ~1
is shown in the figure. If
35. Initially, the wooden block floats partially immersed
i is the angle made by the
in water. Initially, upthrust exerted by water is
water surface with the
exactly equal to weight of the block. But when
horizontal, then
it is exactly pressed down, then the more water
h a hg is displaced. Hence, upthrust exerted by water
tan i = c = g & a = c
increases. The force required to press down the

So the maximum volume that can be retained is block is F = upthrust - mg . Hence, work done by
^1/2h # h # c # b and the force F will be equal to work against (upthrust
- mg ). It means, work done by the external force
F = ;M + E
hcbt hg
c = work done against upthrust - loss of gravitational
2
potential energy of the block.
11.20 Physics
36. Applying Bernoulli’s equation at C and D , we P h ρg P
have h2 = ρg0 - ρ0 g = ρg0 - h0


KE of the water = Pressure energy of the wate at that
1 P
layer 2 mV 2 = m # ρ

V 2 = ρ = ρ 7P0 + ρg ^h1 - h0 hA
2P 2


V = ; ρ # P0 + ρg ^h1 - h0 h-E
1 /2
2
P0 + 0 + tg ]3.6g = P0 + 2 tv2 + 0
1


& v = 6 2 m/s We know 2P0 + ρg ^h1 - h0 h = h2 + h1


Volume blown per unit time
& X = ρg0 + ^h1 - h0 h = h2 + h1
P

= av = rr2 v = 96 2 # 10 -4 m3 /s

Similarly, at A and C
i.e. X is h1 metre below the top or X is - h1 above
the top.
PA + 2 tv2 + tg ]3.6 + 1.8g = P0 + 2 tv2 + 0
1 1

43. As long as water fills the tube (as shown in the figure)

& PA = 0.46 # 105 N/m2 and points A and B are open to the atmosphere, the
velocity at B will be given by Torricelli’s theorem.
37. If a0 is the horizontal acceleration of the vessel,
then the water surface makes an

angle i = tan -1 c g0 m with the


a

horizontal.

` i = tan -1 d
g 3n

g
= tan -1 ^ 3h = 60cs




Hence, vB = 2gh2 where h2 is the difference in
38. Let zmin is the minimum level of the dotted parabola.
the levels A and B .
r 2 ω2 ]0.1g2 ]10g2

` z1 = zmin + 12g = zmin + 2 # 10 44. We apply Bernoulli’s theorem for level A and for
the highest level, CD (lebelled by subscript 1), to
]0.2g2 ]10g2
2 2
r ω2 get

and z2 = zmin + 2g = zmin + 2 # 10
1

But z1 + z2 = 2 # 0.4 = 0.8 m PA = P1 + 2 ρv12 + ρgh1 .....(i)

Solving the above equations, we get
Since the tube has uniform cross section and water is
incompressible,
zmin = 0.275 m
v1 = vB = 2gh2 .....(ii)
39. Upto the instant, the block completely enters the
water, tension increases as BF increases. Therefore,
From Eqs. (i) and (ii)
T becomes constant. Hence, graph (b) is correct.
P1 = PA - 2 ρ 7 2gh2 A - ρgh1
1 -2

40-42.
= Patm - ρg ^h1 + h2 h ....(iii)
2P0 = ]h2 + h0gtg + p0




the minimum value of P1 = 0 ( P1 cannot be negative

(Since liquids at the same level have the same
become then no water will reach the level). Hence,
pressure)
putting P1 = 0

P0 = h2 ρg + h0 ρg
P

h2 ρg = P0 - h0 ρg ^h1 hmax = ρatm 1.0 # 105
g - h2 = 1 # 103 # 9.8 - 3.0

= 10.2 m - 3.0 m = 7.2 m
Fluids 11.21
45. Putting h1 = h2 = 3.0 m in eq. (iii) above. ^tl3h
= 2 ^l2h = 2 a = 2
lta ma

P1 = pressure at level CD

= Patm - ^tgh]3 + 3g = 1.0 # 105 - 6tg
in (b) tla part is zero while average force of tgx is


The gauge pressure at level ; E 6l @ = ^ h
0 + tgl 2 tg 3
2 l
5 2
CD = 6ρg = 6 # 10 - 6ρg
t ]l3g
= 2 ^gh = 2
mg

= 5.9 # 10 4 N/m2
46. Vertical force is buoyant force ]BFg upwards.
Similarly for other part

BF = Vi t L g = b r2R Ll tg =
2
2 rR Ltg

50. (a) On ABCD avg pressure = ; E
2 0 + t1 gh
2
47. p1 = tgR
So F = ; E5lh? =
t1 gh t1 gh2 l
p2 = tg $ 3R 2 2


(b) No contact of t2 and not any pressure on

ABCD due to t2


(c) On CDEF due to t1 at every point


pressure is t1 gh so average is also t , gh so


& pav =
p1 + p2
= 2tgR F = ^t1 ghh ]hlg = t1 gh2 l
2
Area = 2R $ L
(d) On CDEF force due to liquid of density t2

Horizontal force is t2gh2 l
is
FH = ^2tgRh $ 2RL = 4tgR2 L 2
51. When the plastic ball is in the container, it displaces
48. At every point Fnet is radial water whose weight is equal to the weight of plastic
ball. When the plastic ball is in water, it floats and
displaces water of equal weight. So, d remains
constant and h decreases.


When the stone is in the container, it displaces water
whose weight is equal to the weight of stone. When

` the stone is in water, it sinks and displaces water
whose volume is equal to the volume of stone. So,
49. Pressure varies with height & P = tgh d and h decrease.


and is horizontal with acceleration & P = tla
When ice cubes are in the container, before and
after melting, displace same amount of water. So,

so on (a) tgh part is zero while average force of d and h remain same.
tax is

When temperature rises, balloon expands. So, BF
; E 6l @
0 + tla 2 on it increases.

2

Thus, tension increases and d also increases.
11.22 Physics

Numerical Type 2mg


v
& = 2gH + At
1.

& v = 8 + 1.2

& v = 3.033 m/s
v
& - 3 m/s
Single Option Correct
4. P1 = t gd + P0 = 3 # 105 Pa
` t gd = 2 # 105 Pa
25 # 10 1 2
P0 +
0.5 + tgh = P0 + 2 tv P2 = 2t gd + P0
& 500 + ]10 4g 100 = 2 ]1000g v 2
40 1
= 4 # 105 + 105 = 5 # 105 Pa
& 4500 = 500 v 2 P -P
% increase = 2 P 1 # 100
1
& v = 3 m/s
5 # 105 - 3 # 105 # 200
A1 = 100 = 3 %
3 # 105
A2
5. t = t 0 c1 -
r2 m
V1 V2 01r#R
R2
P2

mg = B
2. P1
A # t (4rr2 dr) = t L 34 rR3;
A2 = 21
R
P1 - P2 = 4500 Pa
# c1 - Rr2 m4rr2 dr = t L 34 rR3
2

1 1 to
Now, P1 + 2 tV12 + tgh = P2 + 2 tV 22 + tgh 0

P1 - P2 = 2 t _V 2 - V1 i ..... (1)
1 R

# to4rc r2 - r 2 mdr = to4rc r3 - r 2 m = t L 34 rR3;


2 2 4 3 5 R

R 5R
And A1 V1 = A2 V2
0
0

2
& V2 = 2V1
..... (2)
& 5 to = t L
1
So, 4500 = 2 # 750 # 3V12
[From (1)]
6. using equation of continuity 40VA = 20VB
& V1 = 2 m/s


& 2VA = VB
Volume flow rate = A1 V1 = 24 # 10 -3 m3 s -1


Using Bernoullies equation
1 1
3. P0 PA + tV A2 = PB + tV B2

2 2
m 1
& PA - PB = t (V B2 - V A2)

2
1 V B2
& OP = 1000 fV B2 - p
A

v1 2 4
H
3V B2
P0
& OP = 500 #
4
a v
(OP) # 4 (700) # 4 28

m = 24 kg & VB = = m/s = m/s

A = 0.4 m2 1500 1500 15
a = 1 cm2
Volume flow rate = 20 × 100 × VB = 2732 cm3 /s


H = 40 cm dV dx
7. As, dt = A dt = A 2gh

Using Bernoulli’s equation
& c P0 + m + tgH + 1 tv12 = P0 + 0 + 1 tv2 ... (1)
mg 1 b dV l2 # 1
2 2
h=
A A2 dt 2g

Neglecting v1 , we get
Fluids 11.23
10. kx0 + FB = mg
2 # b 60 l # 2 # 9.8
1 0.74 2
1

=
b 22 l
7 # 4 # 10
-4
L

kx0 + σ 2 Ag = Mg
= 22 # 22 # 16 # b 60 l # 2 # 9.8 . 4.9m
49 # 108 # 10 -4 74 2 1

σLAg
Mg - 2

x0 = k
8. Since height of water in the given large tank remains
same so water inflow rate = water outflow rate Mg
= k b1 - σ2LA
M
l
Qi = Av = A 2gh
11. Diameter = 8 # 10 3 m
-

Q2i 10 -8 1

h= = -8 = m v = 0.4 m/s
A 2g 10 # 2 # 9.8 19.6
2

v = u 2 + 2gh
= ]0.4g2 + 2 # 10 # 0.2
= 0.051 m = 5.1 cm


9.
= 2 m/s
A1 v1 = A2 v2

π c 8 # 10 m # 0.4 = π # 4 # 2
-3
d2

4
3
d . 3.6 # 10 m .
-

12. For equilibrium, weight should be balanced by


R α d2 + R cos α d2 + R ]1 - cos αg d1
sin buoyant force.

Density of oil < density of water

= R ]1 - sin αgd1
And ball should be in between oil and water.
]sin α + cos αgd2 = d1 ]cos α - sin αg
13. Since solid ball floats in between the two liquids
hence ρ1 < ρ3 < ρ 2
d 1 + tan α
& d1 = 1 - tan α

2

1. P2 T 1-c
c

d 1n
P1 = T2
2
2

P2 = b 150 l
A2 = 0.4 m 300 -1
# 600
600
P2 = 4 = 150Pa

PM P
A1 = 0.1 m
2 P1 = 600 Pa
Now, t = RT & t \ T
T1 = 300 K
P T
t2 = d P1 nd T2 n = (4) b 2 l = 2
3
1 = 0.2 kg/m t 1
1 2 1

V1
t
dm t2 = 21 = 0.1 kg/m3

dt = t1 A1 v1 = 0.8kg/s
0.8 0.8
v1 = 0.2 # 0.1 = 40m/s
Now, t2 A2 v2 = 0.8 & v2 = 0.1 # 0.4 = 20m/s


g = 10 m/s2 Now, Won gas = T K + T U + T (Internal energy)


c =2 P1A1 T x1 – P2A2 T x2 =

Gas undergoes adiabatic expansion, p1 - c Tc =
^ P TV - P1 TV1 h
1 1 f
2 Tmv2 - 2 Tmv1 + (Tm) gh + 2 2 2
2 2
Constant
11.24 Physics
Now, A 1 T x1 = T V1 4.
And, A2 T x2 = T V2
O
1 ` P TV1 - P2 TV2 R
Rh
= 2 Tm _v22 - v12 i + ]Tmg gh + 2 ^ P2 TV2 - P1 TV1 h
1 2 A r
h

P P v2 - v2
& 2 d t1 - t2 n = 2 2 1 + gh
1 2

& 2 b 0.2 - 0.2 l =


600 150 202 - 402
2 + 10h

h = 360 m
In OOAB

R 2 = ] R - hg2 + r 2

2. h1 + h2 = 0.29 # 2 + 0.1
h1 + h2 = 0.68 R 2 = R 2 - 2hR + h 2 + r 2

.....(1)
& P0 + t k g ]0.1g + t w g ^h1 - 0.1 h - t w gh2 = P0 & 2hR = h 2 + r 2


6t k = density of kerosene & t w = density of water@
h2 + r2
R
& =
& t k g ]0.1g + t w gh1 - t w g # ]0.1g = t w gh2
2h

& 800 # 10 # 0.1 + 1000 # 10 # h1
Now considering equation of surface
- 1000 # 10 # 0.1 = 1000 # 10 # h2 ~2 r2

y = y0 +
& 1000 ^h1 - h2 h = 200 2g
& h1 - h2 = 0.02 ~2 r2

h=
.....(2) 2g
& h1 = 0.35 n 2 n1 n 2 - n1

Now using: v - u =
& h2 = 0.33 R
h1 35 1- 3
4

So, = 1 4
h2 33
& v +
3 ] H - hg
-
3. P1 = P2 - ta cos 45°d + t ^ g - a sin 45° h d -R
1 1 4
P -P 2a
& v = -

& 1tgd 2 = 1 - g 3R 3H
g 1 2h 4

& b = 0 for a =
& v = 2 -
2 3r 3H
2 -1 g 3H ~2 H
-1

b=
2
for a = 2 v
& = >1 + H
4 4g
dp
5. =- gt ] r g
dr
G m] r g 4
g] r g = ; m] r g = rtr3
r 2 3
4

& g ] r g = rtGr
d 3
2
d dp 4

& = rt 2 Gr
dr 3
1 a
R2 - r2
p ] r g - p ] Rg = rt 2 G d n
4
45°
&
3 2

put p ] Rg = 0
2
p ] r g = rt 2 G ^ R 2 - r 2 h
3
Fluids 11.25
9
P ]3R/4g
1-
16 9 7 63

p ]2R/3g
= = # =
4 16 5 80
1-
9
9
p ]3R/5g 1-
25 16

p ]2R/5g
= =
4 21
1-
25
1
p ] R/2g 1- 1 1
4 9 3 27 P0 - 2 ρ a v a2 = P0 - 2 ρ , v ,2 - ρ , gh


p ] R/3g
= = # =
1 4 8 32 ta
1-
v, = v considering h to be small.
9 t, a
6. 10. The buoyancy force is the force due to the pressure
of the liquid.

The forces on the bubble (except buoyancy force)
are – gravity and viscous.
11. The gas in the bubble is insulated. Hence the process
is adiabatic. For adiabatic process

γ-1

On small sphere p
` Tf = T1 # e f o
γ
4 3 ^ρh + = 4 2 ^2ρh T
γ - 1 = constant
3 πR g kx 3 πR g .....(i)
p γ
pi
P + ρ g ^ H - yh

on second sphere (large)

4 3 ^3ρh = 4 3 ^2ρh + Tf = T0 # < 0 P +, ρ gH F

3 πR g 3 πR kx .....(ii)
0 ,

P + ρ g ^ H - yh

by equation (i) and (ii) 5 /3 - 1

= T0 < 0 P +, ρ gH F
5/3
3
4πR ρg 0 ,

x= 3k
P + ρ g ^ H - yh 5
2

= T0 < 0 P +, ρ gH F
0 ,

7. For A, mg + T = B .....(1) BA

11
A

` d A < dF T
13 to 14.
mg

When bouyant force on (tube + air) system will

For B '
T + B = mg .....(2) become equal to weight of tube then tube will
B'
T start sinking. (Here we can neglect weight of air as

` dB > dF
compared to weight of tube)

(1) - (2)

net force enerted by gas on the test tube is zero.

& mg - B' = B - mg

Now, Let volume of air in this case = Vair

& mg + mg = B + B' mg

FB = mg
d A + dB = 2dF ( a B = B' )
So, d w ]Vtube + Vairgg = mg
&

8. A1 V1 = A2 V2 A1 = 400A2
& 1 b 2.5 cm3 + Vair l = 5
5

400 ^5 # 10 -3h = V2
& V2 = 2 m/s.

& 2 + Vair = 5
9. Pressure at A and B will be same
Vair = 3 cm3

As initial volume of air = 3.3 cm3

So, 3 V = 0.3 cc
11.26 Physics

So, X = 0.30 16.

As temperature of air is constant vg

So, PV = constant

P0 3.3 = Pf 3, Pf is final pressure of air

& Pf = 1.1P0 = P0 + 0.1P0
mg

So, 3 P = 10 4 Pa
480 # g = vt1 g

So, Y = 10 ]480 - N g g = vt 2 g

480 - N t2

So, X = 0.30 =
480 t1

Y = 10.00
- h2
N e
d1 - n=
FB h0

= e-
h1 - h2 50

=e h0 6000

480 e
-h1
h0

N 50 50 # 480

1- = 1- &N= =4
480 6000 6000

mg

15.
V
Vt

with respect to train



Applying Beroulli’s equation
1 1
P0 + tvt2 = P = tv 2

2 2

P0 - P = t ^v 2 - vt2 h
1
.....(i)
2
4

Also, 4St vt = v # 3St & v = vt .....(ii)
3

From (i) and (ii)
2
1 16 2 1 7vt
P0 - P = t d
v - vt n = t
2
2 9 t 2 9
N
` =9
Simple harmonic motion 12.1

Simple Harmonic Motion


INTEXT EXERCISE: 1

1. v and a are either parallel or antiparallel. 5. A = 2cm

rt rt ω ^ A 2 - x 2h = ω 2 x x = 1 cm
2. y = 0.4 d cos 2
- sin 2 n
2 2 6. 2
ω A = ωA
rt

= 0.4 cos 2 d n = 0.4 cos rt 8. V = ω 4A 2 - x 2 ω ]2Ag = 2V
2
3. 8 2 = ω 2 ^ A 2 - x 2h V' = ω 4A 2 - ] Ag2 = 3 ωA = 3 V

7 2 = ω 2 ^ A 2 - ] x + 1g2 h 9. T = 1 s & A = 2m
y = A sin ^ωt + φ h
6 2 = ω 2 ^ A 2 - ] x + 2g2 h
π

at t =0 Y =+ A & φ= 2
Vmax Aω = 65 2
25 x
2π 10.V2 = 4 - 4
4. Time period = ω = 2s . Total distance covered
= 5a
Compare with V2 = ω2 A2 - ω2 x2

INTEXT EXERCISE: 2

y2 = 2 sin b ωt + 4 l
1 π
1. Mech Energy = 2 mω 2 A 2
2. KE at mean position = 2 m ]ωAg2
1 π

Phase difference = 12
3. ω = 2πf = 50 rad/s 8.
Total energy = 2 mω 2 A 2 = ]0.4 + 0.5gJ
1


4. From diagram φ = 3


φ = ω/3
π
φ 1
Time taken = ω = 23π = 2 s
5.
2π 3
φ 1.2

t = ω = 23π = 3 = 0.4 s 9.
1.2
π φ

φ = 3 time = ω
6.

10. U = 2 ]1 - cos 3xg


From diagram φ = π/3 dU
F =- dx =- 6 sin 3x =- 18x
y1 = sin b ωt + 6 l
π
7. 2
a =- 9x & ω = 9
12.2 Physics

INTEXT EXERCISE: 3
1. ω1 A1 = ω 2 A2 ω 1 k
8.Frequency = 2π = 2π m' m' = 1.5 kg
k1 k2

Using conservation of momentum
& m A1 = m A2
m m v -1
2. T1 = 2π k T2 = 2π 0.5v = 1.5 v' & v' = 3 = 1 m/s
1 k2

The initial displacement from equillibrium position
T1 2 T 2
k = k + k = c π m +c 2 m
m m m is
eq 1 2 2 2 π
m 0.5 # g
T' = 2π k = T12 + T22
= k =x
eq

k1 + k2 v' = ω A 2 - x 2
3. ω = m
Solving we can get A
k
4. If m1 is removed ω' = m 9.
2

Initial displacement from equilibrium


mg
= k1
= Amplitude
5. Using conservation of energy
1 2 = 1 ] A'g2
2 kA 2k & A = A'
Net force on mass m = kx + 2kx sin 45c cos 45c
2π 2m = 2kx
6. T = ω = 2π k
m
Final velocity = 2 = ωA ]v = 3 m/sg
v
Time period = 2π
2k
7. Spring balance reads the contact force at lowest 10.
point
1 1 1 1
N - mg = mω 2 A Keq = 3K + 2K + K
N = mg + mω 2 A 6K
Keq = 11

at highest point
mg - N = mω 2 A
N = mg - mω 2 A

INTEXT EXERCISE: 4

1 16 = ,
1. T = 2π g T ' = 2π g 4T T = 2π geff

For the pendulums to be in phase again 4π 2
3. T2 = g ,
ωt + 2π = ω't 2

2 log T = log g + log ,
t 2π

2π T + 2π = 4T t 1

Required slope = 2
4T
t = 3 T T' ,/4 = T
4. Required time = 2 + 2 T' = 2π g 2
d
2. F = Vd1 g - Vd2 g = Vd1 g d1 - d2 n = T2 + T4
1

d
geff = g d1 - d2 n = 34T
1
Simple harmonic motion 12.3

5. R h l= g
g' b h = 2 l = g b1 - R
8.
2
, , eff = L2 - d 2
T' = 2π g' = 2 T = 2 2 s
, eff
g T = 2π
6. ω = , = π rad/s T = 2s g

θ = 2c sin ωt I m , 2 /3 = 2,
9. TA = 2π mgd = 2π mg ,/2 2π 3g
π 1

For θ = 1c ωt = 6 & t = 6 s
,
TB = 2π g
Total time period = 2 b T4 + 16 s l = 3 s
4

,
3
2mR 2
7. T = 2π
, , 10. 2π g = 2π mgR
geff = 2π g cos θ

INTEXT EXERCISE: 5
1. x2 n
y = A ^1 - 2 sin 2 ωt h = A d1 -
A2
5. F1 = mω12 x

F2 = mω 22 x

2 1 1
F1 + F2 = m _ω1 + ω 2 i x = m4π d t 2 + t 2 n x
2 2

Restoring force = wt of 2x length of liquid 1 2

R
= ]2xg APg
6. T1 = T2 = 2π g

]2xg APg 7. Resultant amplitude = A12 + A22 + 2A1 A2 cos φ


a = m
8. Path will be elliptal with major axis inclined to
x
a = hg coordinate axes.
h 9. Equation of motion y = y1 + y2
T = 2π g
2. For no slipping at the max displacement
= 2 A sin b ωt + π
4
l

^ fs hmax = µmg = mω 2 A
Total mechanical energy = 2 mω 2 ^ 2 A h
1 2

3. For small θ

= mω 2 A 2

restoring torque = mgR sin θ
I
10. T = 2π 2Mgd
= mgRθ

I A = 3 + d M, + M c 5 , m n
2
M, 2 2
mgRθ g 12

α = = Rθ 2
mR 2
R ,2 + ,2 = 5 ,

T = 2π g d = 4 16 4

4. x = A sin ωt

y = A sin b 2ωt + 2 l = A cos 2ωt


π
12.4 Physics

EXERCISE - 1

1. Uniform circular motion get repeated at regular 12. V = ω A2 - x2


intervals of time. Therefore it is a periodic motion;
but it doesn’t satisfy the conditions of S.H.M. ]50πg2 = ]10πg2 ^10 2 - x 2h
^RFnet =- k x h , so it is not S.H.M.
& x = ! 75 = ! 5 3
2. Comparing F =- kx
So, separation between points is

with F =- cx1/3
` ∆x = 2 # 5 3 = 10 3 = 17.32 cm
kx
& = cx1/3
13. Consider SHM as projection
c
& = kx2/3 of uniform circular motion.

As x increases c also increases

From figure the phase
3.
d2 x
a= 2 = mx
k difference between two
dt particles is 120c
or a \ x
4. A particle returns back to its original position in one 14. Velocity v = ~ A2 - x2 & v2 = ~2 A2 - ~2 x2
time period.
.....(1)
5. The total distance moved by particle in one time
Acceleration a =- ~2 x & a2 = ~ 4 x2
period is four times the amplitude.

.....(2)
6. Position where we see the particle once in a time
period that is only extreme position. twice through
From (1) and (2):
every other position
v2 = ~2 A2 - a2 /~2 & v2 + a2 /~2 = ~2 A2
~ r 1
7. ~2 = r2 & r & f = 2r = 2r = 2 Hz
v2 a2


8. y = a sin ]~t - kxg
& 2 2 + =1
~ A ~ 4 A2

y = b cos ]~t - kxg y = b sin b~t - kx + 2 l


r
& v2 =- a2 c
& 1 m
+ 1 its straight line with -ve slope
~2

So phase difference is r/2
and +ve intercept.
9. y = a cos ~t
15. Acceleration ]ag = ~2 x
& amax = b T l A
a 2r 2
2 = a cos ~t
r
When amax = g then block and piston will be
~t = 3
separated
2r r
24 g = 3
When amax = g then block and piston will be
t = 4 sec . separated

; ^g = r2h
2
10. a =- ω x
amax = g = b T l A
2r 2
2

=- ω A sin ωt
T 4r2 1
-ω A 2
# ]sin ωtgdt g = 1 A & A = 4 = 0.25 m
0
< a > = T
A/2 1 r -A A

# dt
O
16. cos i = A = 2 & i = 3 -A A
O
0

- ω 2A : ω D
- cos ωt T
Total phase difference A cosθ =A


= 0
=0 between them θ θ
T-0
φ
11. X = A + B sin ωt

x - A = B sin ωt 5r A/2 A/2

z = 2i + r = 3

Hence, Amplitude = B
Simple harmonic motion 12.5

17. = 3 b 2 l & a = 2r
ar 1 3
10
5
21. a b
0


x = A cos ~t & a = A cos ~ and a + b = A cos 2~

& a + b = A 62 cos2 ~ - 1@ = A ;2. - 1E


a2

From figure
A2

Maximum amplitude A = 10
& A - A = a + b & A2 + ]a + bg A - 2a2 = 0
2a2


Position of particle at t = 0 ,

x=5 - ]a + bg + a2 + b2 + 2ab + 8a2
A
& = 2

let equation of SHM is
22. Maximum possible average velocity will be around
x = A sin ^wt + fh At t = 0, x = 5 mean position
2r 2r
5 = 10 sin z & z = r/6 and ~ = T = 2 = r
Average velocity in time
Thus, equation of SHM x = 10 sin brt + 6 l
r T 2 ^A/ 2h 4 2 A
4 = T/4 = T
A r 5r
18. x = A sin ~t = 2 & ~t = 6 or 6 M.P.
T/ 8 T/ 8
-A A
5r r 2r
& Phase difference = 6 - 6 = 3 or 120c
x x
A/2

OR 3T/ 4 φ
T/ 4

θ

OR From phaser θ
T/ 2

2r T r

z = ~t = T 8 = 4
A/2 1 x r x A

cos i = A = 2 & i = 60c
cos z = A & cos 4 = A & x =
2

Phase difference 2i = 120c
Average velocity
19. x = a sin ~t = a sin b T l
2r t
total displacement

= total time
2r b 8 l p
T
At t = 8 , x = a sin f = a sin b 4 l =
T r a
2x 2A/ 2 4 2A
T 2
= = = T
T/4 T/4
t=0
3T
t=T t=T/ 8 23. For (a): at t = 4 , particle at extreme position.
φ a a =- ~2 x ` F ! 0
θ

OR t=3T/ 4 t=T/ 4
For (b): at t = T/2, particle at mean position

v = ~A ]maximumg
t=T/ 2
For (c): at t = T , particle at mean position
2r T r r
& a =- ~2 x = 0

~ = ~t; z = T 8 = 4 & i = 4

For (d): at t = T/2 , particle at mean position
x a

As cos i = a so x = 1
2
so x = 0U = x kx2 = 0
20. x = a sin ^~t + zh
24. sin b 6 l = sin b2r + 6 l = sin 6 = 2
13r r r 1
At t = 1s, x = 0 = a sin ^~t + zh & z =- ~

Now x = a sin ]~tg = 2
a
& 4 = a b 6 l cos ]~g = 3 cos b 3 l
1 2r ar r

12.6 Physics

DT Dz 2 Dz r
Dividing Equation (1) and (2), we have
25. T = 2r & 8 = 2r & Dz = 2
3 M
` 5 = M+m
26. At equilibrium position K.E. is maximum
9 M
25 = M + m & 9M + 9m = 25M
27. 2 kx2 = 2 k ^A2 - x2h
1 1
m 16
A & 16M = 9m M = 9

or x =
2 1 1
38. 2 mv2 = 2 K1 x12
28. Total Energy of S.H.M. remains constant so average
energy = Total energy 1 1
mv2
2
= 2 K2 x22
29. X = A sin 2ryt, K1 x12 = K2 x22

= 2 K ^A2 - x2h = 2 K ^A2 - A2 sin2 2ryth


1 1 x1 K2

K.E. x2 = K1

= 4 A2 ]1 + cos 4rytg 39.


K


Hence, the frequency of K.E. is 2y

Rubber ribbon can exert only tension not
1 compression so at a time only one is effective.
30. E = 2 m~2 A2
m
1 k
= 2 m # m # A2
T = 2r K
1 1 K
= 2 kA2 40. f1 = 2r m1
E is independent of mass 1 K
f2 = 2r
PE ]ming = 15 J
m2
31.
f1
KE ]maxg = 2 ^Avg KE h = 10 J f2 = 2 or m2 = 4m1 or m2 - m1 = 3 kg


Total energy = 25 J F 6.4
41. K = x = 0.1 = 64
32. PEay2 m r m
T = 2r k & 4 = 2r 64 & m = 1kg.
& PE is zero at mean position.
42. Frequency depends only o K and m
33. 2 mω 2 ^ A 2 - x 2h = 2 mω 2 x 2
1 1
m T2
34. Potential energy associated with restoring force is 43. T = 2r K m = k
4r2
zero at equilibrium position. mg
mg = Kx x = K
35. As the body is undergoing S.H.M., total energy of
system remains constant. T2 g
T2 K g
x = x =
36. Keq = K1 + K2 4r 2 K 4r2
1 1 1 44. kx = mg sin 30c
K1 = 2K + 2K
& K1 = K k g sin 30c 5 # 100

~= m = x = 2.5 = 14.14

K2 = K + 2K = 3K
2k # 2k
Meq m m 45. keq = 2k + k + 2k + 2k = 4k

T = 2r Keq = 2r 4K \ 4K
1 Keq 1 4K
M so, frequency f = 2r M = 2r
37. T = 2r k .....(1) M
46. In spring mass sytem time period depends only on k
M+m
T' = 2r k and m , not on g


5T
& 3 = 2r
M+m
.....(2) 47. Frequency depends only on K and m
k
Simple harmonic motion 12.7
48. Both the spring are in series f1 k/2 3
f2 = = 2
K ]2Kg
2k/3
2K

` Keq = K + 2K = 3
T T T
52. 4 - 12 = 6
n

Time period T = 2r Keq gEarth
53. gMoon = 6 ` TMoon = 6 TEarth
m m2 m

where = m 1+ m Here n = 2
1 2 54. Given time for both are same
m 3 3m

` T = 2r 2 . 2 K = 2r 4K
9T1 = 7T2

OR l1 l2

9 # 2r g = 7 # 2r g
k 2k
m m l1 49

& 9 l1 = 7 l2 & l2 = 81
v
keq
m m l l 2
55. T = 2r geff = 2r = T
g + g/4 5
x x
l

Total extension = 2x 56. T = 2r g , As it does not depend on amplitude

By energy conservation ` % change in time period of 0% Hence option (d)
is correct.
E = 2 Keq ]2xg2 + 2 mv2 + 2 mv2
1 1 1

v2
1 2k 1 1 4 57. Tmax = mg + m~2 l = mg + m l

E = 2 3 4x2 + 2 mv2 + 2 mv2 = 3 kx2 + mv2

= mg + m l # l ca ~ = m
g A2 g
dE 4 ]2xg dx ]2vg dv


dt = 3 k dt + m dt
l
or Tmax = mg + mg b l l = mg ;1 + b l l E
There is no loss of energy A 2 A 2

dE 8 8kxv
dt = 0 & 3 kxv + 2mva = 0 & 3 =- 2mva l
58. T = 2r g , At high altitude value of g decreases
4kx 4kx 4k

a =- 3m & - ~2 x = 3m & ~ = 3m
` length of pendulum must be decreased to keep
2r 3m correct time.

T = ~ & 2r 4k
l
49. 59. T0 = 2r geff .....(i)

T0 l

Now 2 = 2r g + a .....(ii)
1 k f1 m2
f= m m = f2 = m1
a0
& f =b M l = b1 + M l T
f1 M+m 1/2
m 1/2

2

m
50. Time period of spring block system is T = 2r k
mg

a constant force doesn’t affect T ..
g+a
51. For 1st condition
k
keff = 2
& g = 4 & a = 3g (upwards)

]2kg]kg 2 1 l 1 g

For 2nd condition keff = 2k + k = 3 k 60. T = f = 2r g & f = 2r l
f1 l2 b n l2 l2 l1 b n + 1 l2
T
m 1 1 k f2 = l1 & n + 1 = l1 & l2 =
` = 2r k ` f = T = 2r m n
12.8 Physics
61. In an artificial satellite L
1 m.g. 2
f0 = 2π

geff = 0 & T = 3 ^mL2 /3h
62. KE at centre 3g

= 21π # L = 2 f0
= 2 m~2 ]A2g = 2 m4r2 f2 A2
1 1


when bottom half of the stick is cut of

KE at distance x
m L
= 2 m4 r2 f2 ^A2 - x2h

1 1 2 .g. 4
m ] L/2g2
f0 ' = 2π


1
Difference = 2 m # 4r2 f2 x2 = 2r2 f2 x2 m 2 3

3g
63. The center of mass of the sphere and water system
= 21π # L = 2 f0
first go bellow the centre of mass of sphere till lowest
position and then again rises to initial value.hence 67. Time period of compound pendulum
the effective lenght of pendulum first increases to
maximum then decreases to the original.hence time Ta I
period of oscillation first decreases to minimum then
and Iring > I
again increases and become equal to the original.
disc


` T1 > T2
64. The net downward force on the changed sphere is
I
^mg - qEh making the acceleration bg - l 68. T = 2π
qE
Mgl
m
Fe = qE
1 2 2


= 2π 2 MR + MR 2π 3R
MgR 2g

69. y = a sin ^~t + zh and x = A sin ^~t + zh

W = mg
Then y = x so path is straight line.

So, the restoring force will be m bg - m l sin e



qE 70. x = C sin ~t + D sin ^~t + r/2h


And the time period will be
A=
r
C2 + D2 + 2CD cos 2
L l

T = 2r anet = T = 2r qE
Ar = C 2 + D 2
g- m

So, choice (b) is correct and the rest are wrong. anet 71. y = 5 ^sin 3rt + 3 cos 3rt h
qE
cannot be g + m , so the choice (d) is wrong. 3
= ]5 g]2 g= sin ]3rt g + cos ]3rt gG
1


65. I = 5 mR2 = 5 # 25 # ]0.2g2 = 5
2 2 2 2 2

= 10 sin d 3rt + n
r
x 0.1


x = Ci c = = 1 = 0.1 3
i
I 2
` Amplitudes = 10 m

T = 2r C = 2r 5 # 0.1 = 2r 2 = 4r secs.
#

= 100 cm
1 mgl
66. f0 = 2r I 72. x = 2 sin ~t &

Where, l is distance between point of suspension
y = 2 sin b~t + 4 l = 2 sin ~t + 2 cos ~t
r
and centre of mass of the body.


Thus, for the stick of length L and mass m : x
& 2 + y2 - 2 xy = 2

Which represent oblique ellipse
Simple harmonic motion 12.9

EXERCISE - 2

1. v max only dx
7. Average velocity V = dt =- Aω sin ω t

If initial velocity is v max t t


then after one time period particle acquires same # v dt - Aω # sin ω dt
speed vmax in same direction means same velocity <V> = 0
t = 0
t
v max
# dt # dt
2. Non-positive means negative 0 0

Aω b

scalar product of two vectors will be negative if - cos ωt lt
ω 0
they are antiparallel as F and a are always in same <V> =
[t - 0] t0
direction and opposite r
- A [cos ωt - cos 0]
3. F.a
= ]t - 0g

= ma .a (It is always non-negative) A 51 - cos ω t?
<V> =
4. The equation a =- 100x + 50 =- 100 ^x - 1/2h t
ba t = 6~ l = ]π/6ωg :1 - cos ω 6ω D
1 1 r A π
[ a =- kx from where x = x - 2 and x = 2 is the
mean position]
3 E 3Aω 6

itself shows that the particle performs SHM.
= 6Aπω ;1 - = π 2 - 3@
2

Hence (d)
Since particle does not change it’s direction in the

given interval, average speed v = π ^2 - 3 h


T/2
3Aω
# - ω 2 A sin ωt dt
0
5. aavg = d2 v
T/2 8. =- kv
# dt dt2
0
This equation has standard solution
- ω A: ω D
cos ωt T/2 2 v = v0 sin ^ k t + ih where ~ = k
0

& T/2
Hence the particle executes SHM with angular
- ω A 5- 1 - 1?
2 frequency ~ = k
a avg = T/2
Hence the particle executes SHM with angular
2
4ωA 2ω A frequency ~ = k

& 2π = π
ω k

or frequency f = 2r
6. x1 = A sin ^ωt + φ1 h
9. Oscillations represented by curve 2 lags in phase
x2 = A sin ^ωt + φ 2 h by r/2 and the periods are same. amplitude
of curve 2 is double that of 1. Put t = 0 then
& x1 - x2 = 2A sin c 2ωt + 1 m cos c 1 m x = 2A sin ^0 - r/2h =- 2A .
φ + φ2 φ - φ2

2 2

To maximize x1 - x2 :
Hence (a)

10. a = - d
r2 n r2
sin c 2ωt + m should be 1.
φ1 + φ 2 r
&~=
2
x 64 = 8
2r
a 2 = 2a # 1 # cos c 1 m
φ - φ2
& T = ~ = 16 sec .
2
T

There is a time difference of 2 between t = 2 sec .
cos c 1 m
1 = φ - φ2

&
2 2 to t = 10 sec . Hence particle is again passing
through the mean position of SHM where its speed
π φ -φ
& 4 = 12 2
is maximum.
π 4 2 32 2

& φ1 - φ 2 = 2
i.e. Vmax = a~ = 4 2 & a= = r m
r/8
12.10 Physics

dv
OR -3 4
11. v dx =- 100x + 50 -5 5


& # vdv = # ]50 - 100xgdx 2r 2r r
3 4


~ = T = 20 = 10 5 5
θ2 θ1
v2 100x2

& 2 = 50x - 2 + c

At x = 2 v = 0
r
from figure i1 + i2 = 53c + 37c = 90c or 2

& 0 = 100 - 200 + c & c = 100
r r
i = ~t & 2 = 10 t & t = 5 sec.
& v2 = 2 ;50x - 100 E
2
x
2 + 100 15. a x = A sin ~t

At x = 0
` x1 = A sin ~ & x1 + x2 = A sin ]2~g

v = 2 ]100g
2r 2r r A

~ = T = 8 = 4 & x1 = & x1 + x2 = A
2
v = 10 2 m/s. A x 1

& x2 = a - , Therefore x12 =
π/6 π/3 2 2 -1
12. T1 = ω T2 = ω T1 < T2
16. For weightlessness
Free body diagram of bob of the pendulum with
mg = m~2 a & g = ^2rfh ]0.5g
2
respect to the accelerating frame of reference is as
follows: 2g
& 2 r f = 2 g & f = 2r

` Net tension in the string is T = mg cos a
17. x = 3 sin 2t + 4 cos 2t = 5 sin ^2t + zh
a = 5, vmax = a~ = ]5g]2g = 10
18. From question
1
m 2 2
# -3
2 ~ A = 8 10
& 2 # 0.1 # ~2 # ]0.1g2 = 8 # 10 -3
1


mg cos a
& ~=4
T

So, geff = m = m = g cos a

So, question of SHM is

x = A sin ^~t + zh = 0.1 sin b4t + 4 l


L L r
T = 2r geff or T = 2r g cos a

13. x1 = a sin ~t, x2 = a sin ^~t + zh


1 1
19. PAV = 4 KA2 and K AV = 4 KA2


Greatest distance 20. x = A cos ~t

K.E. = 2 k ^A2 - x2h


1
z 3a z 3

= x2 - x1 max = 2a sin 2 = 2 & sin 2 = 4
1
= 2 kA2 sin2 ~t

Now according to question x1 = x2
]1 - cos 2~tg
& a sin ~t = a sin ^~t + zh
1
= 2 kA2 2
r-z kA2 ]1 - cos 2~tg

& r - ~t = ~t + z & ~t = = 4
2

Frequency of K.E. is double of acceleration.
& x1 = a sin c m = a cos =
r-z z a 7

2 2 4 1
21. E1 = 2 Kx 2
14. Minimum phase difference between two position
1
E2 = 2 Ky 2
53
= c + 37c = 90c 4 5 1
53° 4 E = 2 Ky 2
37°

E = 2 K ^ x + yh
T 20 3

& Time taken = 4 = 4 = 5s 3 1 2

Simple harmonic motion 12.11

E = 2 K _ x 2 + y 2 + 2xy i
1 2k 3

& Vmax = mA

= E1 + E2 + K xy
dU =

= E1 + E2 + K xy 28. U = U0 sin ax & dx U0 a cos ax


= E1 + E2 + 2 E1 E2
At equilibrium, - dU
dx = 0
22. Due to impulse force, the total energy of the particle
& U0 a cos ax = 0
becomes: π π

& ax = 2 or x = 2a
1 1
m 2 2 2 2 2 2
2 ~ A + 2 m ~ A = m~ A d 2 U =- 2
U0 a sin ax

Let, A' be the new amplitude. (Apply energy dx 2
conservation law) d2 U
keff =
dx 2 x = 2πa
` 2 m~2 ]A'g2 = m~2 A2
1


= U0 a 2 sin b a $ 22π
a
l
& A' = 2 A

= U0 a 2
23. Restoring force F = dx = dx ^u0 ]1 - cos axgh
- du - d
m
F ]xg =- u0 a sin ax
29. Time period = t = 2r
K


For small angle sin ax . ax
Spring divided into two equal parts so length is
reduced to half.

F =- u0 a2 x
1

We know K \ l
=- ~2 x = b T l # x
- u0 a2 x 2r 2

& acc. = m
` K become twice
m

so, time period T = 2r
u0 a2 m m
Tnew = 2r Knew = 2r 2K
24. At t = 4 x=0
1 b2r m l T

i.e. particle must pass through the mean position su
= K = 2
2
curve (3) is not correct.
30.

The magnitude of slope of x - t graph is the speed

Among curves (1) and (2), slope of (1) greater at
t=4
so (a) is correct
1 1
25. Here 2 mv2 = 2 kx2 &
1 2
^ 2 2
h 1 2 2
2 m~ a - x = 2 m ~ x
a 4
x & = = =2 2 cm
2 2

Velocity before collision = 2gH
26. TE = 2 mω 2 A 2 b ω = 2π l
1
T

Pan is massless so velocity after collision
27. U ^ y h = k y
3


= 2gH

As the particle is oscillating under the influence
of conservative force, total mechanical energy is
by energy conservation
conserved throughout the journey of the body.
mg ]xg + 2 m ^ 2gH h = 2 kx2
1 2 1

At extreme position,

K $ E = 0, P $ E = kA3
kx2 - 2mgx - 2mgH = 0
Total energy = kA3
mg mg 2Hk

x= k + k 1 + mg

At mean position, kinetic energy is maximum

at equilibrium kx0 = mg & x0 = mg/k
m ^Vmax h + 0 = kA3
1 2

`
Amplitude = x - x0
2
12.12 Physics
31. Let displacement of block is x1 and of cart is x2 as
+ 2 0 + 4 = 0 bwhere a = dt l
Kx Kx dv

shown.
4M1 + M2 - Kx
Hence, 4 a= 4

a =- c 4M + M m; ω 2 =
K K
^4M1 + M2 h
`
1 2

K 2π
^4M1 + M2 h

ω = ; T= ω


By linear momentum conservation 4M1 + M2
` T2 = 2π K
mv mx

mvl1 = Mv2 & v2 = M1 so x2 = M1
34. By work energy theorem;

For block force equation can be written as

wext + wspring = k f - ki
F = 2k ]x1 + x2g = m~ x1
2

Let x1, x2 be the initial and final extensions and
& 2k bx1 + M x1l = m~2 x1 & ~2 = 2k b Mm l
m M+m v, v' are initial and final velocities.

1 1

Initially, Fx1 - 2 kx12 = 2 mv2 .....(1)
Mm
2k ]M + mg
So T = 2r 1 1

and finally, Fx2 - 2 k'x22 = 2 mv'2 .....(2)
n
32. Time period T = 2r k
In both cases: force applied is same, and velocity
becomes maximum when F = kx 9at equilibrium)
M M2 M#M M

where n = M 1+ M = M+M = 2
1 2
(After which the mass will deaccelerate)

So, time period T = 2r
M ]a k = ag ` F = kx1 = ]4kg x2

2a
& x2 = 41 ]k' = 4kg
x
33.

Substituting in (2):
1 1
fx2 - 2 k' x22 = 2 mv'2
Fx 1 1
4 1 - 2 k'x22 = 2 mv'2

& 4 :Fx1 - 2 kx12D = 2 mv2 .....(3)


1 1 1


Dividing (3)/(1); we get

(i) Equilibrium position determination 1 v'2 v
4 = v2 & v' = 2 Hence (c)
M1 g = T " ]from FBD of M1g
35. Let x is extension in the spring, when block is in
2T = M2 g + kx0 ]from FBD of M2g equilibrium

i.e. 2 k x = mg
` 2M1 g = M2 g + kx0
` kx0 = 2M1 g - M2 g mg
x =
2k

(ii) Displace block M1 by small disp. x by

Block is at rest 9i.e. top extreme position) when
At nes displacement position extension is x0 .
- Mgx + 2 M1 V 2 + 2 M2 b 2v l
1 1 2

` Maximum displacement of block
x - x0
+ M2 g b 2x l + 2 K b x0 + 2x l = C
1 = 2e o
2


2
M a M2 g
(From top extreme to bottom extreme)
&- M1 g + M1 a + 42 + 2

= ^ x - x0 h
+ 2 b x0 + 2x l = 0
k

mg
Mg Ma
=e - x0 o
&- M1 + 22 + M1 a + 42 2k
Simple harmonic motion 12.13
36. Block is oscillating between positions A and B. m 2 ^2gh h m2 g2

& + = A2
m+M k2
1 k
39. f = 2r m+M
m 40. Time taken to collide on left wall and get back to the
2L
mass attached with spring is t1 = v
A
1
B

Time to et compressed once and back is,
cm
T 2r m m
2 t2 = 2 = 2 K =r K

In absence of wall, block would have performed
Remember the colliding block will come to rest on
S.H.M. with an amplitude of 2 cm ^OA = 2 cm h exchanging momentum and starts back when the
mas connected to spring hits it, on its way back.


A B O C

So,
1
v = t1 + t2
Phase covered
Time taken ] A " Bg =
And the frequency of collision will be double the
time taken
free of oscillation.
r/3

= ~

So frequency of collision.
r m 2

=
2v = 2v = 2L
3 k : v +r mD
K
2r 0.9
t =
3 k 41. The blocks will continue to move together till the
2
r 0.9 frictional force on mass m 5B? compensates the
-2
10 = maximum force in oscillation
9 k

& k = 10r 2
` nmg = amax = m~2 a
~ For the arrangement

= 100 N/m
K
k
37. ~1 = m , ~2 = 4m = 21
k ~ = ]M + mg

Frictional force max
]~1 - ~2g t = 0, 2r4r, ..... t = 0
mka kam ka
= nmg = ]
M + mg
= 2m = 2
& = ~ - ~ = 4 & 2r = 4 ]~1 - ~2g
2r
t
1 2
5as M = m?

r ]~ - ~ g

& 2 = 1 2
2 & k = r N/m

So, choice (b) is correct and the rest are incorrect.
38. Let v is velocity of (pan+body) system after 42. Time period of spring
collision.

]m + M g v = m 2gh T = 2r b k l
m

m 2gh
k , being the force constant of spring
v =
t1 = 2r b k l
m+M m

For first spring. .....(1)

Immediately after the collision, system is at a 1
mg
For second spring
distance of from new equilibrium position.
k
t2 = 2r b k l .....(2)
m

In S.H.M.,
2

v 2 = ~ 2 ]n 2 - x 2 g
The effective force constant in their series
m ^2gh h
2 2 2 kk
k m g combination is k = k 1+ 2k

& = f A2 - p 1 2
]m + M g2 m+M k2
Therefore, time period of combination
12.14 Physics

]k1 ++ k2gE
T - 2r ;
k1 k2
4 ]k1 + k2g
m
` keq =
k1 k2
4 r 2
m ]k 1 + k2g keq k1 k2
4m ]k1 + k2g
& T2 = .....(2) ~ =
k1 k2 m =

the effective force constant in their series
kk 44. From the figure it is clear that, if the displacement of
combination is k = k 1+ 2k
1 2 block is x0 , extension in spring will be 4x0 .

Therefore, time period of combination
]k1 + k2gE
T = 2r ;
m

k1 k2
4r m ]k1 + k2g
2

& T2 = k1 k2 .....(3)

From equations (1) and (2), we obtain

t12 + t22 = 4r2 b k + k l


m m
1 2


` t12 + t22 = T2 [from equation (3)]

43. Imagine pulley 2 is fixed and cannot change its
position. Now if pulley 1 moves down by x1 ,
Force on the block will be
block will move down by 2x1 . similarly, if pulley
]8kx0g2 = 16kx0
1 is assumed to be fixed and pulley 2 moves up by
x2 . then block will move down by 2x2 . But in the
` 16kx0 = keq x0
problem both pulleys are not fixed. So, if block m

& keq = 16k
moves down by x0 then
keq 16k k
~ = m = m =4 m
~ 2 k

` f = 2r = r m

45. Let x1 = A1 sin ω1 t and x2 = A2 sin ω 2 t Two


pendulums will vibrate in same phase again when
there phase difference ]~2 - ~1gt = 2r


& b T - T l t = 2r
x0 = 2x1 + 2x2 2r 2r

2 1
2T = k2 x2
&c m n # T1 = 2r (where n is number
g g

and 2T = k1 x1
1 - 1.44
2T of vibrations completed by longer pendulum)

` x1 = k
1
2T

&c m n # 2r
and x2 = k g g 1.44
g = 2r
1 -
2
1.44

If keq is equivalent spring constant of the system,
then
& n=5
T
Thus after 5 vibrations of longer pendulum they will
x0 = k
eq again start swinging in same phase.
T

& 2x1 + 2x2 = k 46. For simple pendulum ~ = g/l and maximum
eq

2T 2T T linear displacement x0 = li 0 and equation of S.H.M.


2 $ k + 2 $ k = k
1 2 eq
x = x0 cos ~t


&
1 :1 1D
keq = 4 k1 + k2
g
x = li 0 cos l t
Simple harmonic motion 12.15

47. Point O is moving as shown


g where l' = d 2 n so T = 2r
l' l l

T = 2r

So, acc. of particle w.r.t. O 2g

i + ^a2 - ghiUj
= _- aS
L
51. As b < a so T = 2r g

So geff . = a12 + ^g - a2h 52. Restoring torque


2


So time period
x = ky L = KL2 i (Since y , Li from figure)

l mL2

= 2r
_a + ^g - a2h2i
2
1
2 & kL2 i = 3 .a
1

48. For simple pendulum


3k
& a = m $ i = ~2 i
l - const
g =g - changeG
l

T = 2r
& ~=
3k
m
I 53. Time period of a rod oscillating about a horizontal

For physical pendulum T = 2r mgl
axis with center of mass at a distance x is
^g - changeh I, m, l " const

I I + Mx2 dT

For Torsional pendulum T = 2r
T = 2r Mgx For least T, dx = 0
C
b I = Constl for any planet
mgx. 2 ^1 + Mx2h2 2Mx - ^I + Mx2hH
1
>
1
C dT
dx = 2r
m Mgx

for spring mass system T = 2r K
1 Mgx

c m = gl = l = Const for all planetm
` 2 2Mx = I + Mx2
mg
` = kl I + Mx2
K g'
mgxMx = ^I + Mx2h2
3

Both the spring-mass system & torsional pendulum
have no dependence on gravitational acceleration

Solve to get x and substitute back in T to
for their time periods.
get
49. Free body diagram of bob of the pendulum with l
xleast =
respect to the accelerating frame of reference is as 12
follows:
54.

` Net tension in the string is T = mg cos a

force = ]P - P0gA = Ma



by P1 V1 = P2 V2
T = mg cos α =

So, geff = m m g cos α
P0 hA = P ]h - xgA
P0 h
& P = h- x
L L
T = 2π geff or T = 2π g cos α
= bh - lA
P0 h

So force x - P0 M = a
50.
P0 Ax P0 Ax
M ]h - xg

a= . Mh ; (For small x )

Mh

T = 2r P0 A
12.16 Physics

& , - a 4 k, + k = 0
55. For small angular displacement ( i ) 2 T2 2

I ,2 + k2
58. T = 2π mg, = 2π g,
59. Initially finally

k(x+y)
kx FB FB

Net torque on body = Ia

(/ 2+y)
/ 2

= ^k1 a sin θ h a + ^k2 b sin θ h b = c mL2 + ML m α


2

3 mg mg

2 2
k1 a + k2 b
kx + FB = mg

For small i &a= i
ML2
mL2 + 3

kx = mg - ρVg
56. Center of mass 2m of a system is at a distance from ρA,g

kx = mg - 2 .....(i)
,
peg P is and moment of inertia of the system
2 2
Let cylinder be displaced through y then restoring
force,
2m, 2
fnet =- 7k ^ x + y h + FB - mgA
is 3
ω
fnet =- :kx + ky + ρA b 2 + y l g - mgD
,
P
θ

fnet =- 6ky + ρAgy@ & ma =- 6ky + ρAgy@


/ √ 2

θ
C.M. mgsinθ
mg mgcosθ k + ρAg 1 k + ρAg

& ω= m & f = 2π m
,

τ = r # F =- mg sin θ 60. Let small angular displacement of cylinder be θ
2 2
, ^ then restoring torque

& Iα =- mg sin θ sin θ - θ for small θh
2 2
Iα = - k ]Rθg R where I = 2 MR 2
3


2m, 2 , 3gθ

& 3 α =- mgθ & α =- =- ω 2 θ
2 2 2 2, 2
d θ kR
2
d2 θ 2k

& 2 + 3 θ = 0 & 2 + 3M θ = 0
3g 2 2, dt 2 dt

&ω=

& T = ω = 2π 2 MR
2 2, 3g 2k

& ω 2 = 3m
57.
61. If block is in equilibrium mg = kd ,

where d is extension in the spring.
Now, let the block is pulled down
θ and released. It performs
oscillations. The figure shows the
mgsinθ mg mgcosθ snapshot of the system in which x
is the displacement (downwards)
τ =- mg sin θ, & Iα =- mg, sin θ from the mean position and v is
& m ^, 2 + k 2h α =- mg,θ ]sin + θg
the velocity (downwards), at that

instant. Angular speed of the pulley
v
g, θ g, at this instant ω = R

& α =- =- ω 2 θ & ω =
^, 2 + k 2 h , + k2

At this position total energy of oscillating system is
, + k 6 = 2@
2 2 2 2
2π , +k =
ET =- mgx + 2 mv 2 + 2 /ω 2 + 2 k ] x + d g2

& T = ω = 2π g, 2 , g π 1 1 1
Simple harmonic motion 12.17

2 va + Mva + 2k ^ x0 + 2x h v = 0
dE l
dtT =- mg b dt l + 2 m ]2vg dt + 2 2 ]2vg dt
dx 1 dv 1 l dv
` - Mgv +
R R
a c M + 2 m =- 4kx
l

+ 12 k2 ] x + d g dx
dt

R

As total energy of the system remains constant, [a Mg = 2k x0]
dET 4k
dt = 0 a =- f M + l p x, comparing with

R2
` - mgv + mva + 2 va + k ]s + d gv = 0
l
2
R
a =- ω x, we have
- mg + ma + l 2 a + kx + kd = 0 4k
R ω = l
M+ 2
a ;m + 2 E =- kx 6a mg = kd @
l R

R
l
k M+
a =- > m + l H x
2

& T = 2π R
4k
R2

Comparing with standard equation of SHM i.e. 63. x = A sin ~t, y = a cos ~t

2
a =- ω x , we have
or x2 + y2 = A2

m + ^l/R 2h
Thus the motion of the particle is on a circle.
T = 2π k
64. y = 4 cos2 b 2 l sin ]1000 tg
62. In the equilibrium position, let the tension in the t
string is T and extension in spring is x0 . Then,
T = k x0
y = 2 cos b 2 l ;sin b 2 l + sin b 2 lE
t 2001t 1999t

2k x0 = Mg
y = sin ]1001tg + cos ]1000tg
`


Now, let the pulley is slightly + sin ]1000tg + cos ]1999tg
displaced from its equilibrium
position ]Og and released. It starts y = sin ]1001tg + 2 sin ^1000t + r/4h + cos ]999tg

oscillating about the equilibrium
position ]Og . The following figure

So the given expression is composed by three
equation of S.H.M.
shows the snapshot of the system
3 sin a - sin ]3Ag l
during its oscillations. 65. x = 10 sin3 ]rtg ; bsin3 A = 4

String does not slip over pulley’s surface
3 sin ]rtg - sin ]3rtgD
v & x = 10 :
& ω = R 4
[Particle of the string has velocity 2v ^ . h for no
30 10

& x = 4 sin rt - 4 sin 3rt
slipping]
30 10

So amplitude = 4, 4

Total energy of the system at this instant is

Frequency = 1/2, 3/2
ET =- Mgx + 2 /ω 2 + 2 Mv 2 + 2 k ^ x0 + 2x h
1 1 1 2

66. Resultant displacement will be the vector sum of


=- Mgx + 12 / v 2 + 12 Mv 2 + 12 k ^ x0 + 2x h
2
2 two displacements.


R
S = S12 + S22 + 2S1 S2 cos 37c
dE dx 1 l dv 1 dv
dtT =- Mg dt + 2 2 2v dt + 2 M2v dt
R

4
= a2 sin2 ~t + b2 sin2 ~t + 2ab sin2 ~t # 5

+ 12 k 2 ^ x0 + 2x h $ s dx
dt 8ab
S = a2 + b2 + 5 sin ~t
dET
dt = 0
Which shows that the particle will perform SHM
12.18 Physics
67. The resultant equation is 4

= 3+5# 5 = 7
/ A4 = 5 sin 37c - 15

= 3 - 15 =- 12
A = 144 + 49 = 189
m 21.2
70. x = b = 2 = 10.6 s
E = E0 e -t/x
e t/10.6 = 10

t

& 10.6 = ln 10
x = A sin ]~t + fg

t = ]10.6g]2.302g = 24.4 s
/ Ax = 2 + 4 cos 30c + 6 cos 60c = 8.46
k 20500
71. ~0 = m = 10.6 b 44 s
-1
/ Ay = 4 sin 30c + 6 sin 60c = 7.2

A = ^/ Axh + ^/ Ayh
b 3

`
2 2

2m = 2 # 10.6 = 0.1415 s
-1


= ]8.46g2 + ]7.2g2 = 11.25

b
2m << ω 0
/ ay o
f = tan -1 e
/ ax = tan b 8.46 l
-1 7.2
` Frequency will be approximately same as that of

undamped oscillation.

= tan -1 ]0.85g
1 k 44

` f = 2r m = 2 # 3.14 = 7Hz

so, the resultant equation assumes the form

x = 11.25 sin 6~t + tan -1 ]0.85g@


1 1
72. T = f = 7 s
b
68. x = A sin ~t, y = a cos 2~t A = A0 e - 2m $ t
6a Phase difference = r/2@ A0 b

& A = e + 2m $ t

Let us eliminate t from the above equations to find
A0 0.1415
the path = e+ 7
A

so A0

& A = 1.0204
x

` sin ~t = A

or A0 = 1.0204 A

and y = A 61 - 2 sin2 ~t@
A0 - A
%decrease = A0 # 100
y = A ;1 - 2 x E
2 2

`
A 0.0204

This is the equation of parabola
= 1.0204 # 100 b 2%

69. x1 = 3 sin ωt, x2 = 5 sin ]ωt + 37cg 73. T = 2s , Time elapsed,


t = 10T = 20 s
b
A = A0 e - 2m $ t
b

& 1.5 = 2e - 2m $ 20
b
e 2m $ 20 = 1.333
]1.333g
b

& 2m $ 20 = ln
x3 = 15 sin ]~t + 270cg b 0.2877
2m = 20 = 0.0144 s
-1

/ Ax = 3 + 5 cos 37c
Simple harmonic motion 12.19
74. In case of driven oscillations
= 3N
F0 /m 3/2
A = A =
4r - ^ 10h + 0
`
_ω 2d - ω 02 i + c
2
m
2 bω d 2 2

m


b = 0 ^a no dampingh
= 0.0509 = 5.09 cm

Here,
75. The motion happens with the angular frequency of
~ 0 = angular frequency of driving force

= 2r rads -1 driving force.

k 20
` ~ = ~ d = 2r
~0 = m = 2 = 10 rads
-1

2r
F0 = Amplitude of driving force T = ~ = 1s

EXERCISE - 3

Numerical Type r di r
4. i = Sin (wt + f) ⇒ = w cos (wt + f)
72 dt 72
1. a = g, geff = g 2
where w = 2p/T = p,
ma
T l
Therefore at the instant when
3
o
45 r r
g 2 45o
i= Sin (wt + f) = #
72 72 5
m~ 2 l di r r~ 4
Equilibrium
; ;= w cos (wt + f) = #
2 position dt 72 72 5
mg 2 or geff mg
MR 2
5. (kdq) d = e oa
m~ 2 l 2

T cos 45° = .....(i)
2kd 2 1 mR 2 2
2 a =- e
o i; K.E. max = .~ 0
MR 2 2 2

T = sin 45° = mg 2 .....(ii)
(i) ' (ii), we get, mR2 2Kd2 2

= 4 . .i 0
mR2
m~ 2 l 2g 2g
1 = & ~2 = &~= 6. For small angular displacement of cylinder.
2mg l l
k
M
l 2l

T = 2r =r g M
k
2g

P
2. In the given situation, sphere undergoes pure
The energy of system angular displacement q is
translatory motion
1 1 1 1
l E = k (2Ri) 2 + k (Ri) 2 + Mv 2 + I~ 2

t = - d mg + mgl n sin q 2 2 2 2
2
where n is the velocity of centre of mass and w is the
ml 2 angular velocity of cylinder. Since E is constant.
I= + ml 2
3 dE

` =0
x - 3mgli - 9 gi 2 9g dt
`a= = = ;~ = =9 di di dv d~
I 4ml 2 8 l 8ll
2= G & 4kR 2 i.

dt
+ kR 2 i
dt
+ Mv
dt
+ I~
dt
=0
3

⇒ 5kR2q + MR2a + Ia = 0
3. Since the equation is a sin2 function, it
5kR 2 MR 2

~ of S.H.M. = 40r
⇒ a =- i where I =
(MR 2 + 1) 2
2r
Compare it with a = - w2q

T = 40r
10k 3M

= 50 ms Thus ~ 2 =
or T = 2r
3M 10k
12.20 Physics

7. x 10. When block is displaced down by y, then level of


liquid in beaker also rises.
K K
A A
 y = d A - n y'
4 4
y A
Massless rod So, up thrust = e y + o 3dg
3 4
A 4y A

Torque about ‘O’ d n dha = - e o (3d)g
L L 4 3 4
d K i n = K (x - Li) L
4g
2 2 a = -e oy
5Li h
= x
4 h h

Torque on sphere about bottom most point T = 2p =r g
4g
7
K x - Li) R = mR 2 a
( 11. Just after impact, at t = 0 the velocities of A and B
5
3v0
x 7 are and 0 respectively and the CoM (of A and

K R = mRa 2
5 5
K 3v0

a= x B) is . Since the spring starts from relaxed state
7m 4
at t = 0, the blocks A and B are in SHM (wrt their
7m K M

T = 2r ,~= CoM) with time period T = 2r where M/2 is
K 7m 2k
the “reduced mass” of the system.
8. Effective torque constant = KR2; Also effective
So w.r.t CoM both A and B have a speed (max speed)
m2 R2 m1 l 2 3v0

MOI = + of (towards CoM) initially, and therefore
2 3 4
3v0
m2 R2 m1 l 2 will have a speed of (half of max speed) at
+ 8
2 3 T r M

TP = 2p 2 t= = . That is when (w.r.t ground) A
kR 6 3 2k

m2 = 2 kg 9v0 3v0
and B will have individual speeds of and

m1 = 3 kg 8 8
respectively.

l =1m

R = 1m ML2
12. I = + mx 2
3

K = 1 N/m
1+1 L x

TP = 2p = 2 2r
XCM = +
1 4 2
2m I m (L2 /3 + x 2)
9. TP = 2p
T = 2p = 2r
3k mgl L x
2mg d + n
Torque constt 4 2

~=
moment of inertia dT (L2 /3 + x 2) 2x
for minima, = 0 &- + =0
kb 2 3k dx L L
d + xn
2

= = d + xn
2mb 2 2m 2 2
e o
3 L2 L2

& + x 2 = xL + 2x 2; x 2 + xL =
2r 2m 3 3

Time period = ~ = 2r 3k
L 7 L

x= e - 1o .
2 3 4
Simple harmonic motion 12.21

13. U = 2 - 20x + 5x
2
As it is restoring

dU 3
dx =- 20 + 10x
in nature, FR =- 4 kx

F =- dx = 20 - 10x =- 10 ]x - 2g
dU

` a
3 k
=- 4 $ m x

` The particle is executing SHM about mean
3 k

&~ = 4$M

position x - 2 = 0 or x = 2
3 4M
` k = 10
= 4M $ 3

& m~2 = 10
= 1 rad s -1
10 10
~2 = m = 0.1 = 100 16. Velocity of the particle at the time of collision is

& ~ = 10 rad s -1 u = 2gh
vmax = A~ = 5 ]10g = 50ms -1
4.5mg m

u = k = 3g k
2g $

` b =2

Let velocity of combined mass after collision is v .
14. In SHM, U - x graph is a parabola

Then, mu = ]2m + mgv

` U = C1 x2 , where C1 is a constant.
u m

& v = 3 =g k

From figure

Now, the time period of SHM is

when, x = 0.2m, U = 2J
3m
2 = C1 ]0.2g2
T = 2r k
2

& C1 = 0.04 = 50

` U = 50x2

Also,
1

in SHM U = 2 m~ 2 x 2
100
Mean position from the point of collision is at a

& ~2 = 4 = 25
mg
distance k as shown in figure.

& ~ = 5s -1
2r 2r y = A sin ]~t + dg

T = ~ = 5
mg
2r
At t = 0 , y = k
T = 5 s
mg
15. If the sphere moves down along the plane by a
` k = A sin d .....(i)
distance x ,
Extension in the spring = x cos 30c
Now, for amplitude v = ~ A2 - y2

` Restoring force = F cos 30c m2 g2
m k

g k = 3m A2 -
k2

&c 3 m
mg 2 m2 g2
= A2 - k
k
2mg

A = k .....(ii)

= ]kx cos 30cg cos 30c

From eqs. (i) and (ii), y = 0 to y = A

= kx c
3m
2
3

= 4 kx
& t'
T
= 4
2
12.22 Physics

T T T
Force along the direction of displacement on each

Total Time = 12 + 4 = 3
ball is
1 3m F = k 3 x cos 30c + k 3 x cos 30c = 3kx

= 3 2r k
2r 3 30 # 10 -3
# F =- 3kx

= 3 100

[negative because restoring in nature]

-2
= 2r # 10 = 20r ms
F 3k
` b =2
` a = m =- m x
F
17. h = 3k
Ai
& ~=
x m

Here, A = L2 and i = l
1 3k 1 6k
f = 2r m = 2r 2m

` Restoring force is F =- hAi =- hLx

hL
& b=6

Acceleration is a =- m x
mg
19. In equilibrium, kx0 = mg sin 30c = 2

&a \- x

hL

` Oscillations are simple harmonic with ~ = m
2r m

So, T = ~ = 2r hL
4m

= r hL
ET = 2 k ^ x0 + x h - mgh + 2 mv 2
1 2 1

& a =4
+ 12 b mR l ω
2

18. In the figure. i = 30c



2
= 12 k ^ x0 + x h - mg x sin 30c + 12 mv 2
2

1 mR 2 v 2

+2 2 $ 2
R
dtT = 2 k $ 2 ^ x0 + x hv - mgv $ 2
dE 1 1


When displacement is each ball x , the extension in
+ 12 m2v $ a + 14 m2va = 0
each spring is mg m

kx0 + kx - 2 + ma + 2 a = 0
x' = x cos i
; E
3 mg
3 2 a $ m =- kx a kx0 = 2

=x 2
2k - 2 ]300g

each spring is connected to two balls. a =- 3m x = ] g x =- 400x
3 0.5

So, net extension of each spring = 2x' = 3 x ω = 400 = 20 rads 1
-


& p =4
Simple harmonic motion 12.23
20. In the equilibrium of two blocks,
In the light of Eqs. (1) and (2), the above equation
5
reduces to 4 ma =- kx
T = kx0 + mg sin i .....(1)
2T = mg .....(2) 4k

& a =- 5m x

During oscillations if displacement of block 1 is x 4k
and its velocity v , then the respective values for
& ~ = 5m
x v
block 2 will be 2 and 2 .

` b =5
ET = 2 k ]x0 + xg2 + 2 m1 v2
1 1

x1 = 10 sin d 4rt + n
r
21.
2
+ 2 m2 b 2 l + m1 gh1 - m2 gh2
2
1 v



& A1 = 10 units
ET = 2 k ]x0 + xg2 - 2 v = 0
1 mg
x2 = 5 sin ]2rt g + 5 8 cos ]2rt g
5

+ 8 mv2 + mgx sin i
= 5 sin ]2rt g + 5 8 sin d 2rt + n
r


2
dtT = 2 k2 ]x0 + xg $ v + 8 m2va
dE 1 5

Phase difference between SHM’s of second equation
mg r

+ mgv sin i - 2 v = 0 is 2

` Resultant amplitude is
dtT = 2 k2 ]x0 + xg $ v + 8 m2va
dE 1 5
]5g2 + ^5 8h + 2 $ 5 $ 5 8 $ cos r
2
A2 = 2
mg

+ mgv sin i - 2 v = 0

= 225 = 15 units
5 mg

kx0 + kx + 4 ma + mg sin i - 2 = 0 A1 10 2

` A2 = 15 = 3 & b = 3
EXERCISE - 4

1. vdv v2 bx2
f = dx = a - bx & 2 = ax - 2 + c


At x = 0, v = 0 &c=0
a

Substituting; x = b ; gives
a

vmax =
b

Let upper block is pushed down by x . at equilibrium
Also, the velocity of the car should become zero at
mg = ky , i.e. weight of upper block is balanced by station B .
spring when it is deformed by y . Upper block will
perform SHM with amplitude x about equilibrium
x = 0; x = b b l
bx2 2a
position. Lower block will leave surface when
i.e. ax - 2 = 0 &
spring is extended by y , means upper block is at 2a
distance 2y from its mean position. That should be
` Distance between the two stations is b
upper extreme position of upper block. So amplitude
x = 2y . di
3.
dt = 2 ` # di = 2 # dt & i = 2t
2. f = a - bx

Let BP = a ` x = OM = a sin i = a sin ]2tg

For maximum velocity, acceleration should be zero.

Hence M executes SHM within the given time
a

i.e. a - bx = 0 & x= b period and its acceleration is opposite to 'x' that
means towards left
a

` At x = b , the particle has its maximum velocity.
12.24 Physics

4. Let the line joining AB represents axis 'r' . By the 8. a = 8r2 - 4r2 x =- 4r2 ]x - 2g & ~ = 2r
conditions given 'r' coordinate of the particle at
time t is
Here a = 0 so mean position at x = 2
Let x = A sin ^~t + zh


As particle is at rest at x =- 2 (extreme position)

and amplitude = 4 as particle start from extreme
position. Therefore

x - 2 =- 4 cos 2rt & x = 2 - 4 cos 2rt

9. x = A0 ^1 + cos 2πγ 2 t h sin ^2πγ1 t h


= A0 6sin 2πγ1 t + cos 2πγ 2 t sin 2πγ1 t@

Since particle starts SHM from point A (Extreme
position) amplitude = 2 2 = A0 :sin 2πγ1 t + 12 sin 2π ^γ1 + γ 2ht - 12 sin 2π ^γ1 - γ 2htD
10. y = sin ~t + 3 cos ~t = 2 sin b~t + 3 l
r = 2 2 cos ~t r
2r 2r To breaks off mg
~= T = 2 =r


` r = 2 2 cos rt g

mg = m~2 min A & g = 2~2min & ~ = 2
r
x = r cos 45c = = 2 cos rt
2
moment it occurs first after t = 0

` 2 2
ax =- ~ x =- r 2 cos rt
2 = 2 sin b~t1 + 3 l & ~t1 = 6 & t1
r r


` 2
Fx = max =- 4r cos rt
r r 3
= 6~ = 6 g
5. Torque about hinge
11. The small block oscillates along the inclined plane

2.5 g 40 cos i - 1g.100 cos i = 0 with an amplitude A . As a result the centre of mass
of the system undergoes SHM along the horizontal

So rod remains stationary after the release
direction:
6. Amplitude phasor diagram: mA sin ~t 1 m

xcm = m + M cos 60 = 2 m + M A sin ~t


The acceleration of the C.M. is acm =- ~2 xcm , along
/ / the horizontal.

While the net horizontal force is = ]M + mgacm ,



` resultant amplitude = 6 2
which is equal to the force of friction acting on it.

7. U ]xg = ax2 + bx 4 12. If the string is displaced slightly downward by x ,


we cn write, the net (restoring) force
dU

F= =- 2ax - 4bx3 c - 2ax for small x
dx = ^nx - 2nxh 2g =- 2nxg ` ^5nlh $ xp =- 2nxg

2a

So m~2 = 2a & ~ = m 2g

or xp =- 5l $ x

peg
µ
A'
x
B'


Simple harmonic motion 12.25

π = δ 2 = sin 1 b 6 l
13. If this instant is taken as t = 0 . Then, 1

-
`
π
Initial phase for particle (1) is δ1 = 2
δ 2 = π - sin b 6 l
-1 1


` Phase difference is

δ 2 - δ1 = π - sin 1 b 16 l - sin 1 b 3 l

- 1 - 1

and initial phase for particle (2) is δ 2 = π
15. The left half of the graph is usual SHM.
x1 = A sin b ωt + 2 l
π m
`
` t1 = π
k

and x2 = A sin ]ωt + πg
For the right part, U = mgx


When both of them have same displacement,
At the origin, U = 0

x1 = x2
` K = E0

As we move right the potential energy inreases
A sin b ωt + 2 l = A sin ]ωt + πg
π
` linearly with x .

& cos ωt =- sin ωt
This is similar to a body that is thrown vertically

& tan ωt =- 1 upwards with initial velocity.

3π = 2π = 3π 1 2 2E0

& 4 ωt or T t 4
` 2 mu = E0 & u = m
3T
Time of flight till it reaches point of projection (i.e.

& t = 8
origin) is
14. Let particle (1) is moving towards right and particle 2u 2 2E0 2E0
(2) is moving towards left at this instant, t = 0 t2 = g = g m =2 mg 2

Total time of oscillation is

m + 2E0
T1 = t1 + t2 = π k 2 2
mg

x1 = A sin ^ωt + δ1 h
16. In the absence of wall W2 , the time period of block

m
would have been 2π k . But due to the presence

And x2 = 2A sin ^ωt + δ 2 h
of W2 , it alters. But for the left part of oscillation

So, v1 = Aω cos ^ωt + δ1 h (from the mean position shown, the period will be
m

and v2 = 2Aω cos ^ωt + δ 2 h T1 = π k
A
For the right side part,

At t = 0, x1 = 3 and v1 is positive
d = A sin ωt [from x = A sin ωt ]
A sin 7ω ]0 g + δ1A & sin δ1 = 3
A = 1

` 3
5d
δ1 = sin b 3 l
-1 1
` d = 3 sin ωt

`
3
A
& sin ωt = 5

At t = 0, x2 = 3 and v2 is negative.
ωt = sin b 53 l
-1

2a sin 7ω ]0 g + δ 2A
A =

` 3 3
1
& sin ωt = 5

& sin δ 2 = 6
ωt = sin b 53 l
-1

As v2 is negative at this instant, sin ^π - δ 2 h = 6
1

t = 2π sin 1 b 53 l
T

-
&
12.26 Physics

This is the time taken by the block to reach W2 from Restoring force is F =- ρg ^sin θ1 + sin θ 2 h Ax

mean position. Collision with W2 is perfectly elastic
(given) ρg ^sin θ1 + sin θ 2 h

Acceleration is a =- m Ax

Time taken for right side part of oscillation will be

Comparing this with
T2 = 2t = 2 : 2π sin b 5 lD
T -1 3
2
a =- ω x, we have
m

=
2 $ 2π k
sin 1 b 53 l
- ρg ^sin θ1 + sin θ 2 hA
2π ω = m

=2 m b l
-1 3

` Time period of oscillation of water column is
k sin 5

` Total time period is m
ρgA ^sin θ1 + sin θ 2 h
T = 2π
T = T1 + T2
20. Let particle is displaced by “y” upwards from

=π m m b l
-1 3
k + 2 k sin 5 equilibrium position. Change in length of each upper

: b lD
y

= m -1 3
k π + 2 sin 5 spring is y cos 60c = . Lower springs undergo
2
17. Half of oscillation (from mean position), the block elongation by y cos 30c each.
performs SHM with spring of stiffness k and the
remaining half with spring of stiffness 4k. ky

Force exerted by each upper spring is and by
T M 2
t1 = 21 = π k 2k 3 y
T M π M each lower spring is = 3 ky

and t2 = 22 = π 2
4k = 2 k
ky ky
3 M 120

` T = t1 + t2 = 2 π k 2 2
FL
18. For the wire Y = A∆L0 60
0

F YA

& ∆L0 = L0
ky
Fnet = 2 ^ 3 ky h cos 30c . + 2
YA

` For the wire k' = L cos 60c .
0 2

Spring and wire are in series ky

= 3ky +
kk' kYA 2
keq = k + k' =
L0 c k + L m
YA 7ky

=
0
2
kYA 7k

or keq = YA + kL
0 ~2 =
2m
m m ^YA + kL0 h
T 2π 2m
keq = 2π kYA T = 2r
7k
19. An intermittent snap-shot of oscillating water 21. Let initial length of spring is L. After charging its
column is shown below. length becomes 2L . For equilibrium of sphere -2,
2
1 Q
4πε0 ]2Lg2 = k.L
2 2
1 Q = Q

& k = 4πε 3 .....(i)
0 4L 16πε0 L3

Pressure acting is
dp = ρg ^sin θ1 + sin θ 2 h Ax
Simple harmonic motion 12.27


+ 12 m2v ]ag + mgR sin ^θ + φ h ddtθ = 0
dθ = = v
dt ω R

- Mg + Ma + 2 a + ma + mg sin ^θ + φ h = 0
l

Now, if he sphere-2 is slightly displaced away from R
sphere-1, from the equilibrium position such that the
- Mg + a ;M + m + 2 E + mg 6sin θ cos φ + cos θ sin φ@ = 0
l
separation between two spheres becomes x , then R
we have restoring force on sphere-2 as
θ is very small
2 x
F = k ] x - Lg - 4πε
1 Q
` sin θ - θ = R and cos θ = 1
2
0 x
- Mg + a ;M + m + E + mg R
l x

[ a net extension = x - L ]] cos φ + mg $ sin φ = 0
R2
mg cos φ
a ;M + m + E =-
m l
f' ]2Lg

Now, T = 2π x
R2 R

[ a here, equilibrium position is mg cos φ
a =- l x

at x = 2L ] Rc M + m + 2 m
R
2
f' ]2Lg = dx 1 Q
= =k + 4πε 3 $ 2G
dF mg cos φ
0 x

& ω =
Rc M + m + m
x = 2L x = 2L l
1 Q
2 R2

= k + 2πε
Rc M + m + 2m
0 8L
3
l
2 2π R
Q
` T = ω = 2π mg cos φ

= k+
16πε0 L3

Using Eq. (i) in this, we have 23. Figure (i). When the plank is situated symmetrically
on the drums, the reaction on the plank from the
f' ]2Lg = k + k = 2k drums will be equal and so the force of friction will
m be equal to magnitude but opposite in direction and

` T = 2π 2k hence, the plank will be equilibrium along vertical
22. In equilibrium, net torque on the pulley must be as well as horizontal direction.
zero.

` MgR = mgR sin φ

& M = m sin φ

Snapshot during oscillations is shown in the figure.



Figure (ii). Now, if the plank is slightly displaced
horizontally, the reactions ill not be equal. Consider
a moment at which the displacement of COM is x
rightwards.

In the situation shown, total energy is
For vertical equilibrium of the plan,
N A + NB = mg .....(i)
ET =- Mgx + 2 Mv 2 + 2 l b R
1 1 v l2 + 1 2
2 mv N A ] L + xg = NB ] L - xg .....(ii)

+ mgR 7cos φ - cos ^θ + φ hA

From Eqs. (i) and (ii), we get
dET 1 ] g 1 l ] g] g
dt =- Mgv + 2 M 2v a + 2 R 2 2v a N A = mg b L2L x l
-

12.28 Physics

NB = mg b L2L x l
+
and 2T = k4 x4


Note NB > N A 2T 2T 2T

` x1 = k , x2 = k , x3 = k
1 2 3

f = µN. So, friction at B will be more and tries to 2T



and x4 = k
bring the plank back, i.e. restoring force here is 4

F =- ^ fB - fA h =- µ ^ NB - N A h
`
2T 2T 2T 2T
x0 = 2 $ k + 2 $ k + 2 $ k + 2 $ k
=- µ L x =- b lx
1 2 3 4
mg µmg


= 4T ; k + k + k + k D
L 1 1 1 1



& Acceleration is 1 2 3 4

F µg T
a = m =- L x
Also x0 = k
eq

µg T ;1 1 1 1D

` ω =
` keq = 4T k1 + k2 + k3 + k4
L
2 π L 1

& T = ω = 2π µg
` keq =
4 ; k + k2 + k + k D
1 1 1 1
1 3 4
24. A snapshot during
oscillations of the keq 1
~ = m =
4m b k + k + k + k l
cylinder is shown in 1 1 1 1
the figure. Its 1 2 3 4

displacement is x
T = ~ = 2r 4m b k + k + k + k l
v 2r 1 1 1 1
and velocity v . Its angular velocity is ~ = R (due
1 2 3 4
to pure rolling)
26. Let the horizontal displacement

As the centre of the cylinder is at a distance x from
of the plank, at an instant during
the initial position, the springs which are connected
its oscillations is x and its
at a point of its rim must be compressed and stretched
velocity is v .
by a distance 2x . Thus, at this intermediate position
total energy of the oscillating system can be given as
For pure rolling at A ,

ET = 2 mv2 + 2 b mR l 2 :1 k ]2xg2D2
1 1 2 v = vCOM + ~R
2 ~ + 2 v v
1 1
` vCOM = 2 , ω = 2R

= 2 mv2 + 4 mv2 + 4kx2
ET = 2 k1 x 2 + 2 k2 x 2 + b 12 /ω 2 l + 2 b 12 m0 vCOM l
1 1 2
2
3

= 4 mv2 + 4kx2
dET 3
& is MI of sphere along horizontal diameter.
dt = 4 m2v $ a + 4k $ 2x $ v = 0

6a ET is constant@
1 1 1 2
ET = 2 k1 x2 + 2 k2 x2 + 2 $ 5 m0 R2 $
v2
$2
4R2
3
2 ma =- 8k x 1

+ 2 m $ 2v $ a = 0
a =- b 3m l x
16

& m m

k1 x + k2 x + 50 a + 20 $ a + ma =0

a b 10 m0 + m l =- ^k1 + k2 hx
16k 7
~ = 3m
2r 3m r 3m k1 + k2

& T = ~ = 2r 16k = 2 k
& a =- f m + 7 m p x
10 0
25. Let x1, x2, x3 and x4 are extensions of respective
springs. If x0 is the downward displacement of k1 + k2

` ~ = 7
block, then m + 10 m0
x0 = 2x1 + 2x2 + 2x3 + 2x4 1 k1 + k2

` f = 2r 7

Also, 2T = k1 x1, 2T = k2 x2 2T = k3 x3 m + 10 m0
Simple harmonic motion 12.29

ET = 2 /~2 + 2 k ^y0 + yh
v 1 1
27. ~' = R - r
2


For pure rolling. v = r~ 4

- mg $ 3 y
r
= 2 $ 36 mL2 ~2 + 2 k ^y0 + Lih

` ~' = R - r ~ 1 81 1 2


d~' r d~
dt = R - r dt 4

- mg $ 3 Li
r

& a' = R - r a dE

` dtT = 0

For pure rolling,
` 2 $ 36 mL2 2~ $ a + 2 k $ 2 ^y0 + Lih $ L~
1 81 1

a
a = r
4
- mg $ 3 L~ = 0
g sin θ g sin i 2

where, a = I = 1 = 3 g sin i
Using eq. (i), we get
1+ 2 1+ 2
mr
1 81
2 2
2 36 mL 2a~ + k L i~ = 0
2g

` a = 3r sin i
81 36k
36 ma =- ki & a =- 81m i
r 2g
a' =- R - r $ 3r sin i
36k 6 k 2 k

` ~ = 81m = 9 m =3 m

[negative, since restoring torque]
3 m m
2g
` T = 2r 2 k = 3r k
a' =- ]
3 R - rg
i
30. Let us locate the COM of the body. It is given that

5a i is small? mass of the body is m . mass fo the cylinder (if
m
present) that fills the cavity = 15
2g
3 ]R - rg

` Angular frequency =

2r

` T = Angular frequency

3 ]R - rg

= 2r 2g
6 ]R - rg

=r g

Mass of the cylinder without cavity
28. Moment of inertia of T-bar about O is
m = 16

= m + 15 15 m
16 ]0 g - m b R l
15 m 15 2 R
YCOM = 16m - m =- 30
15 15

2m ]2Lg2
MI of cylinder (without cavity) about contact point
l = 3 $ 3 is
16 R2 16
L2 m ]2Lg2E
+ ;m
l1 = 15 m 2 + 15 m

3 $ 12 + 3
16 3 8 8
81 R2 = 15 m 2 R2 = 5 mR2 = 5 mR2
l = 36 mL2


In equilibrium,
MI of cylinder (if present) that fills the cavity about
O' is
4
ky0 $ L = mg $ 3 L .....(i)
12.30 Physics

m ^R/4h m b 3R l2 ∆x2 =+ x2
2
l2 = 15 2 + 15 2
∆x3 =- x1
= mR2 b 15 $ 32 + 15 $ 4 l
1 1 1 9 M∆x1 + m∆x2 + M∆x3 = 0


& M ^- x1 h + mx2 + M ^- x1 h = 0

l2 = mR2 :15 $ 32 D = mR2 b 480 l
1 + 72 73
m
x1 = 2M x2

MI of remaining body about O' is l = l1 + l2
The stretch in the left spring = x1 + x2

The compression in the right spring = x2 + x1
l = b 5 - 480 l mR2
8 73

` Net force on carbon atom is F =- k ^2x1 + 2x2 h

= b 480 l mR2 a =- m $ ^ x1 + x2 h
768 - 73 k


695 139
l = 480 mR2 = 96 mR2
=- 2mk b 2m
M x2 + x2
l

d 2 x2

In the second figure, restoring torque is x = mgy sin i =- 2mk b m 2+M2m l x2
dt 2
k b m + 2m l

=- m M x2
R

or x =- mg 30 i
mb M l
k m + 2m

` ω2 =
139 ω1

Also, x = /a = 96 mR2 a m

` ω 2 = m + 2M
139 mgR k

` 96 mR2 a =- 30 i
= M

mb M l
k m + 2M

a =-c mi
g

R 0.023 32. The time period of a physical pendulum is

]0.023g r
g l

& ~ = T = 2π
Mgl

Where, l = MI of the system about point of
2r suspension.
x = ~
M = mass of the system and
R
= 2r ]
0.023g g


l = distance betweem COM of the system and point
suspension

= 41.4 R/g
Now, m = m + m = 2m
31. Case (i) is similar to m ]Rg + m ]3Rg
l = m+m = 2R

l = b mR l 6 2 ]3Rg2@
2
2
2 + mR + mR + m
`
= mR2 :2 + 1 + 1 + 9D = 2 mR2
1 23


In case (ii), let us assume the system is at rest 23 2
initially. 2 mR = r 23R

Now, T = 2r ]2mg g ]2Rg 2g


∆x1 =- x1
Simple harmonic motion 12.31

33. x = A1 sin ]- 2~tg + A2 sin2 ~t 2r 2r 1


37. From given equation T = ~ = 100r = 50 = 0.02

x =- A1 sin 2~t + 22 ]1 - cos 2~tg and


A A = 0.01

38. Given y = ]sin ~t + cos ~tg
=- b A1 sin 2~t + 22 cos 2~tl + 22
A A

So A = 2 m and at t = 0 y = 1m
= A1 sin ]2~t + 180cg
x0 ]1 - cos 2~tg
39. x = x0 sin2 ~t =
+ 22 sin ]2~t + 270cg + 2
A A 2

x0 2r 2r

So A = 2 Time period = ~1 = 2~ = r/~
= A1 sin ]2~t + 180cg
v = 108 - 9x 2
2

+ 22 sin ]2~t + 270cg + 22
A A
2vdv =-
40. dx 18x & acc. A =- 9x (non-uniform)

Phase difference = 90c

at x = 3cm
A2

` AR A12 + 42
= a =- 27 or a = 27cm/s 2
A

Mean position is 22
also a =- 9x is a SHM equation so particle perform
SHM about the given fixed point.

` The resultant motion is SHM with amplitude
A2 V is maximum at x = 0
A12 + 42
and V is zero at x = 12
A

Mean position is 22
So Amplitude = 2 3 cm

` The resultant motion is SHM with amplitude
41. For S.H.M.
A2
A12 + 42
y = A sin ~t = A sin b T l
2 rt

34. When two SHM’s (which are perpendicular
to each other) are given by x = A1 sin ~t and
v = ~ A2 - y2 vmax = ~A
y = A2 sin ]~t + dg and when they are combined
the trajectory of the particle is given by
(a) At y = A/2
x2 y2 2xy
2 + - cos d = sin2 d 3 ~A ~A
A1 A22 A1 A2 v = ~ A2 - y2 /2 = 2 > 2

(This is equation of elipse) V

r
In the given equations, phase difference is d = 6
(b) For v = max
2
and A1 = A2 = A ~A
& ~ A2 - y2 = 2
2
x y2 2xy 3 1

` + - 2 $ 2 =4 3A
A2 A2 A A2 A
& A2 - y2 = 4 &y= 2 > 2
1
x2 + y2 - 3 xy = 4 A2 and this is an elipse.

(c) for t = T/8
35. Vectors in all the pairs are either parallel or anti
y = A sin b 8.T l =
parallel so angle is either 0c or 180c hence cross 2r.T A A
> 2
product is zero. 2

(d) for y = A/2
36. ~ = 3r # 5 = 15r A = 0.05
A/2 = A sin b T l
2rt
2r 2r 1
T = ~ = 15r = 7.5 sec. Ans: (b)
2r t
amax = ω 2 A & T = r/6

= ]15rg2 0.05 T
& t = 12 < T/8

= 225 # 0.05

= 11.25 r2 Ans: (c)
12.32 Physics
42. In S.H.M. direction of v and a may be same or 46. Given A = 0.4m , and a = g
opposite and also direction of y and a may be same 10

so A = g & ~2 0.4 = 25
or opposite and also direction of y and v may be
same but direction of a and y never same because 2r
& ~ = 5 T = ~ = 2r/5 sec .
a =- ~2 y (always opposite to y ) At lowest position acceleration
43. Equation of S.H.M. x - x0 = a sin ^~t + zh = ~2 A + g = g + g + 2g
& x = x0 + a sin ^~t + zh So weight = m ^2gh = 2mg


(a) x = sin 2~t
at half distance a = g/2
So weight at upper half distance = m ^g - g/2h = mg/2

(b) x = sin2 ~t = b
1 - cos 2~t l 1 1
2 = 2 - 2 cos 2~t

and weight at lower half distance
1 1
& x - 2 =- 2 cos 2~t
= m ^g + g/2h = 2
3mg

(c) x = sin ~t + 2 cos ~t
= sin ^~t + zh "z = tan -1 2,

actual weight at equilibrium position (maximum v )

47. r = A cos ~t S
i + 2A cos ~t Uj & x = A cos ~t ,

So represent equation of S.H.M.

y = 2A cos ~t so y = 2x

(d) But sin ~t + cos 2~t cannot write as
sin ^~t + zh r = A cos ~t. 5 = 5 A cos ~t

so it is not S.H.M. equation.


so motion is at straight line, periodic and S.H.M.

2πt 48. Acceleration a = ~2 x


44. V = V0 cos ωt = V0 cos T
Maximum acceleration amax = ~2 A = b T l 100
2r 2 2.5

T

at t = 6
When block and platform are separated.
2π T V
amax = g = 10
V = V0 cos T . 6 = 20 (a)
4r2 1 r2
a = a0 sin ωt = a0 sin b 2Tπt l 2 . 40 = 10 & T2 = 100 T = r/10 sec .
T
T 49. From superposition principle
t = 6

y = y1 + y 2 + y 3
a = a0 sin b T T 6 l

= a sin ~t + a sin ]~t + 45cg + a sin ]~t + 90cg



3a
= 2 0 = 0.86 a0 (c) = a "sin ~t + sin ]~t + 90cg, + a sin ]~t + 45cg

= 2a sin ]~t + 45cg cos 45c + a sin ]~t + 45cg


F = dx =- 510x - 20? = 20 - 10x
- du
45.
= ^ 2 + 1h a sin ]~t + 45cg
= A sin ]~t + 45cg

Acceleration

100 ]2 - xg
20 - 10x =
a =0= m
Therefore, resultant motion is simple harmonic of
=- ω 2 ] - 2g
amplitude

x
A = ^ 2 + 1h a

a at x = 2 So V is maximum at x = 2

and which differ in phase by 45c relative to the first

This is equation of S.H.M. so particle exectures
Energy in SHM \ (amplitude) 2 :E = 2 m A2 ~2D
S.H.M. 1

Also ω 2 = 100 & ω = 10

` Eresultant = b a l = ^ 2 + 1h = ^3 + 2 2h
E A 2 2

T = ω = π/5 sec . single

` Eresultant = ^3 + 2 2h Esingle

Simple harmonic motion 12.33

52. x = 3 sin ]100rtg, y = 4 sin ]100rtg & y = 3 x


50. When cylindrical block is partly immersed 4
FB = mg & 3ρAyg
& Motion of particle will be on a straight line with

= ρA ^60 # 10 -2h g
slope 4/3 .
y = 20cm
As r = x2 + y2 = 5 sin ]100rtg


& Maximum amplitude = 20cm

so motion of particle will be SHM with amplitude 5.
40cm
53. At maximum compression v A = v B & kinetic
20cm
energy of A-B system will be minimum
v 1

v A = v B = 2 & K AB = 4 mv2

Restoring force when it is slightly depressed by an
amount of x .
From energy conservation
1 b v l2 1 b v l2 1 2
F =- ^∆Vσg h =- ^ Aσg h x
1
mv
2
2
= 2 m 2 + 2 m 2 + 2 kx m
m m
T = 2π Aσg
& x
& m=v 2k
ρAh h 54. y = 10 sin ^~t + zh

= 2π A3ρg = 2t 3g
1

Maximum KE = 2 m~2 A2
60 # 10 2r -2

= 2r 3 # 9.8 = 7 s
& 100 # 2 m~2 A2 = 2 m~2 ^A2 - x2h
64 1 1

51. The position of momentary rest in S.H.M. is extreme
position where velocity of particle is zero. & 64A2 = ^A2 - x2h 100 & x = 0.6A

& 2 m~2 ^A2 - x2h = 2 m~2 A2 # x


1 1
a= g N
A a = g/ 2
Motion A/ 2
a= 0
& A2 - 0.25A2 = A2 x
Position
A/ 2
A a = g/ 2
& x = 0.75
a= g mg

& x = 0.75 means 75% of energy

As the block contact with the plank at this position 55. Maximum speed = v0 & A = ~00
v
i.e. normal force becomes zero, it has to be the upper
So equation of motion x = ~00 sin ]~ 0 tg
extreme where acceleration of the block will be g v

downwards.
56. x = 3 sin 100t + 8 cos2 50t
10
2 2
` ~ A = g & ~ = 0.4 = 25 & ~ = 5 rad/s
= 3 sin ]100tg + 4 ]1 + cos 100tg
2r 2r

Therefore period t = ~ = 5 s
= 4 + 5 sin ]100t + 37cg

Acceleration in S.H.m. is given by a = ~2 x

Amplitude = 5

From the figure we can see that, at lower extreme,
acceleration is g upwards.
Maximum displacement

` N - mg = ma & N = m ^a + gh = 2mg
= 4 + 5 = 9


At halfway up, acceleration is g/2 downwards

` - mg = ma & N = m cg + m = 2
g mg
N
2
12.34 Physics
57. Velocity of 3kg block just before collision. K
60-62. ~= m = 200 rad/s

= ~ a2 - x2 = b k l^a2 = x2h
m
Maximum extension the spring from natural position
is x .
= b 3 l]22 - 12g = 30 m/s.
900


Velocity of combined masses immediately after the
Then mg + ma = kx
collision 2 ]10 + 5g

& x= 400 = 7.5 cm
]3g]30g Eq. line

= 3 + 6 = 10 m/s. ΣF = 0

Extension of the spring when A it

New angular frequency is stretched to equilibrium V=0
line is 'x'
k 900

~' = m = 9 = 10
mg = kx'

Therefore v' = ~' a - x '2 2
2 # 10
& x' = 400 = 5 cm

& 10 = 10 a'2 - 12 & a' = 2m
therefore amplitude A = x - x' = 2.5 cm
58. From energy conservation
If upward direction is taken as positive at
t = 0, x =- A

Using x = A sin ^~t + zh
- A = A sin φ

φ = 2

63. Max. acceleration of 1kg



]0.6g]1g]10g

]d + x g 1 2
1 = 1 = 6ms -2
k 2
2 d1 = mg 1 1 + 2 kx1

Max. acceleration of 2kg .
15m2 g2
]0.4g]3g]10g
2mg
& d12 - k d1 - =0
k2
= 3 = 4ms -2
5mg 4mg
& d1 = k & d = k
therefore maximum acceleration of system can be
4 m /s 2
4mg
& if d $ k the lower disk will bounce up. 4 4 4 4

~2 ^k/mh 54/6 9
& ~2 A = 4 & A = = = = m
2mg

Now If d = k then maximum normal reaction
1
2mg 64. ~2 A = constant & a \ k
from ground on lower disk d = k
N = 3mg + k ]x0 + dg = 6mg
65. As A is at its negative extreme at t = 0

59. a a =- ~2 y and at t = T , y is maximum so x - 3 = 2 sin ^2rt + 3r/2h & x = 3 - 2 cos ]2rtg

y
acceleration is maximum at t = T . B (0,4)

3T 3T

also y = 0 at t = 4 , so force is zero at t = 4
T

At t = 2 , v = 0 & PE = oscillation energy x
A (3,0)
Simple harmonic motion 12.35

Am : 1m2 + m2 1 D = nm2 g
66. As B is at its equilibrium position and moving m k -m k

towards negative extreme at t = 0 1 2

so y - 4 = 0 2 sin ]2rt + rg & y = 4 - 2 sin ]2rtg


n ]m1 + m2gm2 g

Am = m1 k2 - m2 k1
67. Distance between A & B
71.

= x2 + y2
= ]3 - 2 cos 2rtg2 + ]4 - 2 sin 2rtg2


= 9 + 4 cos 2 2πt - 12 cos 2πt + 16 + 4 sin 2 2πt - 16 sin 2πt

= 29 - 20 b 5 cos 2rt + 5 sin 2rtl


3 4

b 3A l + b 3 A l = 3 A
2 2
1 1
AR = 8 4 64 + 16
= 29 - 20 sin ]2rt + 37cg

1+4 3 5

= 3A 64 = 8 A

Maximum distance
72. d = tan -1 d n = tan -1 b 1 l
3A/8

= 29 + 20 = 49 = 7cm 3A/4 2


Minimum distance 73. Vmax = Aω
Vmax 2π

= 29 - 20 = 9 = 3cm
& A = ω = 2π # ]0.2g = 0.20m

1 keff 1 k1 + k2 m T2 k
68. f = 2r &m= T = 2r
Total Mass = 2r m1 + m2 k 4r 2

= 0.2 kg
69. Suppose the system is displaced towards left by a
distance x , The restoring force on m2 is
At t = 0.1 , acc. is maximum

F = m2 ~ x 2
(towards right)
amax =- ω A =- b 0.2 l # 0.2
2 2π 2
&

F = m2 b m1 + m2 l # :a ~ = m1 + m2 D
k +k k1 + k2

=- 200 m/s2
1 2


Friction f on m2 will be towards left 1

Maximum energy = 2 mv max
2
=4J

v2
74. The velocity-displacement relation is x2 + = A2
~2

x2 v2

& 2 + =1

If k2 x > F A ~2 A2

which may be a circle if ~ = 1 . and elipse if ~ ! 1
k2 x > m2 b m1 + m2 l x
k +k

1 2

The acceleration - velocity relation is
k1 + k2 m1 + m2

& k2 < m2 v2 a2
2 2 + 4 2 = 1 , which may be a circle if ~ = 1
~ A ~ A
k1 m1
k2 < m2
and elipse of ~ ! 1 .

k1 Am + nm2 g = m1 b m1 + m2 l Am
Acceleration - displacement graph is straight and
k +k
70.
1 2 acceleration time graph is sinusoidal.

Am : 1m1 + m1 2 - k1D = nm2 g



m k +m k (since a =- ~2 x =- ~2 A sin ^~t + zh )

1 2
12.36 Physics
75. (a) In frame of lift effective acceleration due to
ml 2
g 3g i 3 2l
gravity is g + 2 = 2 downwards.
(c) T = 2r = 2r = 2r
mgd l 3g
2l 2
` T = 2r 3g
k g m ^ρ/2h Al l

(b) kl = mg ` m = L
(d) T = 2π ρAg = 2π ρAg = 2π 2g
constant acceleration of lift has no effect in time
period of oscillation.

m l
` T = 2r k = 2r g

Numerical Type
The gravitational force acting on the particle
r r r GMmr ]- rtg
Tz 2 -6 3 = 1 sec = .
1. t = ~ = 2r = r R3
6 3 & Force component along the tunnel = restoring
force, FR =-c m cos i
GMmr
2. V = ~ A - x2 Vmax = A~ R3
A~ gm
& FR =- R x c a 2 = g, r cos i = x m
2 = ~ A2 - x2 GM

R
A2 & Acceleration of the particle along the tunnel is,
4 = A -x
2 2

g
3A2 a =- R x.
x2 = 4

i.e. a \ x.
3
By comparing the acceleration of particle with the
x= 2 A
standard form of acceleration of S.H.M, we can say
Single Option Correct g
~2 = R .
3. Let x = A sin ~t g
A
&~= R.
& 2 = A sin 3~ 2r R

& Time period of oscillation is, T = ~ = 2r g.
1
& sin 3~ = 2
r 5. (A) F = ma a =- ~ 2 x
& 3~ = 6
3T
r 2r at 4 displacement zero (x = 0), so a = 0 F = 0
& ~ = 18 = T
(B) at t = T displacement (x) = A
& T = 36 s

x maximum, So acceleration is maximum.
4. Given, the radius of the earth = R.
(C) V = ~ A 2 - x 2
R

The tunnel is drawn at a perpendicular distance 2 Vmax at x = 0

from the center of the earth. Vmax = A~

Consider the particle is displaced by a distance x T
at t = 4 , x = 0, So Vmax.
from the mean position of the particle
(D) KE = PE
A
is at x = , which is possible between
2
T T
r t = 4 to t = 2 .
 x
R ml 2
2 6. xCM = 3m = 3 l
2
K
Angular frequency, ~ = I

Simple harmonic motion 12.37
m T
m # 2 2 ml2 3K 10. K.E. is mas at t = 0 , then it decreases to zero t = 4

I = nl 2 = 3m l = 3 `~=
ml2
2 2 4A 2
11. v = A - 9
l
Required tension in the rod = m~2 i20 3
5A 2
v =
= c 2 m i20 b 3 l = l 0
3K l ki 2 9
m
ml 2
3v = A' 2 - 49A
7. Given, Potential energy = Kinetic energy
5A 2 = 4A 2
1 1 3 9 A' 2 - 9
2 mω x = 2 mω2 (A − x )
2 2 2 2

2
A 5A
or, 2x2 = A2 ; x = . 5A 2 + 9 = A' 2
2
8. For torsional pendulum, oscillation frequency 7A
A' = 3
1 C
,' , a1 + AY k
υ = 2r Mg
I 12.
1
For given C, υ ∝
I ,
T = 2π g .....(1)
y1 I2
y2 = I1 ...(i) ,'
Tm = 2π g .....(2)
1 1
I1 = 12 M(2L)2 = 3 ML2 ,
2
Tm
I2 = 3 ML2 + 2m b 2 l
1 L 2 = ,,'
T2
1 mL2 Tm
2
Mg

= 3 ML2 + 2 2 1 + AY
T
4
y2 = 80% of y1 = 5 y1
Y = :a Tm k - 1D Mg
T 2 A

Using these values in eqn. (i),
1 mL2 13. x = A cos ~t
5 3 ML 2
+ 2
4 = 1
displacement in t time = A - A cos ~t
3 ML
2


for T = x A 51 - cos ~x? = a
b 5 l = b1 + 3m l or, 25 = 1 + 3m
2

A 51 - cos 2~x? = 3a
4 2M 16 2M
for t = 2x
3m 9 m 3 1 - cos ~x 1
2M = 16 or, M = 8 = 0.37 =3
2 sin2 ~x
9. Frequency
1 1
& ]
2 1 + xg 3
=
1 k 2 1 k 2 2
f = 2π m f = 2#m k = mF # 4π
4π 1

& 3 = 2 + 2x & x = 2 = cos ~x

6.023 # 1023 atoms present in " 108 gm
r 2r r

A = 2a, ~x = 3 & T x = 3

1 atom will be present in
T
& = 6x
108 # 10 -3

"
6.023 # 10 23 14. A = A0 e - 2m
bt

108 # ]1012g2 # 4 # 9.86 # 10 -3


after 5 second
k=
b]5g
6.023 # 10 23
0.9A0 = A0 e - 2m .....(i)
707.20 # 10 24 # 10 -3
After 10 more second

= = 7.07 c 7.1 N/m b]15g
10 23

A = A0 e - 2m .....(ii)
12.38 Physics

From (i) & (ii)
A = 2A sin c m
z1 - z2


A = 0.729 A0 2
z1 - z2 r
15. 2 = 6
x r

z1 = 3
x0

19.

Mg
A = P0 P0 V0c = Pv'c


mg = P0 A .....(1)
P0 Ax0c = PA ]x0 - xgc

At equilibrium
c
Px
]x0 - xgc

Let piston is displaced by x P = 0 0

Fb = mg


tAl0 g = dAl g .....(i)
Mg - d n A = Frestoring
p0 x0c
]x0 - xgc

P0 A da -
x0y n 6x0 - x . x0@
]x0 - xgc
= Frestoring

cP0 Ax

F =- x0

cP0 Ax

F = x0

Restoring force,
d2 x dx dx
17. m 2 + b dt + kx - b dt
dt
F = mg - Fb'

d2 x dx

m 2 + b dt + kx = 0
F = mg - ρA ^l0 + x h g
dt


here b is demping coefficient

This has solution of type
dAla = dAl g - ρAl0 g - ρgAx

x = e mt substituting this

mm2 + bm + k = 0
tg
a = dl x
- b ! b2 = 4mk

m= 2m tg

~= dl

on solving for x we get
ld
a cos ^~1, t - ah .....(i)
b

x = e 2m t T = 2r tg
k

~1 = ~20 - m2 where ~ 0 = m
b

m =+ 2
2

So, average life = b

18. x1 = A sin ^ωt + φ1 h

x2 = A sin ^ωt + φ 2 h

x1 - x2 = A ;2 sin ;ωt + E sin ; 1 EE


φ1 + φ 2 φ - φ2

2 2
Simple harmonic motion 12.39

1. 14M 8Mg 4Mg


(iii):
3 a1 = 3 - 3
2g
a1 = 7
x
At 20 , speed of the block (2M) = w × amplitude
3k 8Mg
= 14M # 3k

∴ Option (a) is correct


In the frame of pulley B,
The hanging masses have accelerations : 2.
M → (a2 – a1), 2M → (a3 – a1) : downward.
∴ (a2 – a1) = –(a3 – a1) [constant]
V
Assuming that the extension of the spring is x m
45
We consider the FBD of A :

d2 x 10 m

2M $ = 2T - kx
dt2
At t = 0, distance between block and pebble
d2 x
where a1 / 2 …(i)
= 10 - ]4.9 + 0.2g
dt

= 4.9 m

And the FBD of the rest of the system in the frame 2r
T = = 6 sec .
of pulley B : r/3
A

at t = 1, block travells a distance of = 0.1 m
2
towards “O”

` Range of pebble = 5 m
]V cos 45cg T = 5


& V = 5 2 m/s.

= 50 m/s.

Upward acceleration of block M w.r.t. the pulley B = 3.
Downward acceleration of block 2M w.r.t the pulley
T - M ^g - a1 h 2M ^g - a1 h - T
M = 2m
& = 3 ^g - a1 h …(ii)
4M
Torque about
T
P = ]kxg 2 + ]kxg 2 = kxL = k 2 i
L L L2
Substituting in equation (i),we get

8M ba x = L sin i - L il

2M. a1 = 3 (g - a1) - kx 2 2
8Mg
x=Ia
14M
or 3 a1 = 3 - kx …(iii)
KL2 ML2 - 6ki
This is the equation of SHM
& - 2 i = 12 a & M =a
Maximum extension = 2 × Amplitude 6K

& a =- M i =- ~2 i
8Mg
i.e. x0 = 2× 3k
6K ~ 1 6K
x
& ~= M and f = 2r = 2r M
At 40 , acceleration is easily found from equation
12.40 Physics

4. mω 2 A2 = R & mω 2 R = R & ω 2 = 1
m
ω1 1 1
ω 2 = mn 2 # m = n 2

Extensions in springs are x1 and x2 then E1 E2
ω1 = ω 2

k1 x1 = k2 x2
7. x = A sin ~t

and x1 + x2 = A ~A

v = A~ cos ~t = 2
kx k2 A

& x1 + k1 1 = A & x1 = k + k2 1
2 1

& cos ~t = 2
5. (a) m r 2r r m

~t = 3 & t = 3 = 3 k
M 2r m r m

for (c) time = 3 k +2 k
MωA = ]m + M g ω'A'


5r
= 6
m
k

M M A = ^m + M h
K k
2r
for (d) time = 3
m m

M + m A' k +r k
A' = A' = M 5r m

= 3 k
A A m+M
8. Torque is same for both the cases
^ M + mh

(b) T = 2π k I

T = 2r mgd
Time period remain same in both case. After m
mass is placed on both block. I A > IB

~A < ~B
6. v = ω A2 - x2


or v 2 = ω 2 ^ A 2 - x 2h 9.


or v2 = ω2 A2 - ω2 x2
x=0 x=A

or v2 + ω2 x2 = ω2 A2

v2 x2 =

or 2 2 + 1
ω A A2

as A =a ` mω1 a = b
10. In 1st case amplitude of SHM is a
a 1 2
b = mω1 = n (given)
In 2nd case amplitude of SHM is 2a
Total energy = 2 k ^amplitudeh
a 1 2
R = n (given)

E1 = 2 k ]2ag2
1
1
E1 = 2 m ω 22 A22
E2 = 2 k ]ag2
1

1 2 2
E2 = 2 m ω 2 R
E1 = 4 E2
b 11. Linear momentum
mω1 A1 = b & mω1 a = b & ω1 = am

P = mv

= 12
mn = m~ A2 - x2
Simple harmonic motion 12.41

& P2 + m2 ~2 x2 = m2 ~2 A2 vvB, A =- 5rit - tj


represents a circle of P - x diagram with radius of
vBA = 25r2 + 1

circle R = A ^a m2 ~2 = 1h
(iii) xA = sin t

~ of spring mass system remains constant and
1
equal to m
k vA =cos t = 2 m/s
xB = cos t

Amplitude of oscillation inside liquid will decrease
3
due to viscous force vB = - sin t = - 2 m/s

So radius of circular arcs will decrease as position 3 1
vBA = - 2 - 2
change
(iv) vvA & vvB are always perpendicular

Correctly shown in option (b)
So, | vvBA | = v 2A + v 2B = 10 m/s


12. When 0 < E < V0 there will be acting a restoring
force to perform oscillation because in this case
particle will be in the region x # x0
13. V = ax 4
1

T.E. = 2 m~2 A2 = aA 4 (not strictly applicable just

for dimension matching it is used)


2aA2 1 m

~ m & T? A a
dU
14. F =- dx
As for x > x0 V = V0 = constant
dU
& dx = 0 & F=0
15. (i) v 2BA = v 2A + v 2B - 2v A vB cos i
As ~ A = ~ B, i = 90° remains constant.
Also, vA = vB = 1 m/s

So, vBA = 2 m/s


5r 5r
(ii) uvA = 2 it + 2 tj

vvA = 2 i + b 2 - 10. 3 ltj


5r 5r r

5r 5r
= 2 it - 6 tj

5r 5r
uvB =- 2 it + 2 tj

vvB =- 2 it + ; 2 - 10 b 3 - 10 lEtj
5r 5r r 1

` vvB =- 2 it + b1 - 6 ltj
5r 5r

Wave Motion 13.1

Wave Motion

INTEXT EXERCISE: 1
1. Ist equation can’t be written in the form of f ]ax ! bt g
`
T
∆t = 4 & T = 0.17 # 4 s
2. From comparing the given equation with standard
f = T = 0.17 # 4 . 1.5 Hz ]1.47 Hzg
1 1

`
wave equation,

` (a)
K = 1.57, ω = 314
ω 314 -1
10. Amplitude can be read on the y-axis and is 0.2 cm

` v = K = 1.57 = 200 ms

Wavelength can be read on the x-axis & is 2 cm.

` (b)

The wave is transverse.
3. Since any direction in x-y plane is perpendicular to
the wave.
Hence (d)

4. Only the initial phase is different. 11. Maximum speed of of particle = Aω .


5. a The terms kx & ωt have similar signs, this wave
` Vmax = ]0.1 cmg # ]10π/ secg
travels along - ve x-axis.

= π cm/s
ω 15π

Further, v = K = 10π = 1.5 m/s.
` (c)
2π 2π

Also, λ = K = 10π = 0.2 m 12. Vmax = Aω

` only (b) 13. If we assume that the two particles vibrating in
6. Since the question is specific about speed (& not the same phase are closest two such particles, then
λ = 10m and the disturbance takes 0.1 & to travel
λ this distance.
velocity), two such particles will be 2 distance
apart. 10m

` v = 0.1 - 100 m/s.

` (c)

` (a)
7. Intensity of sound wave is:
2π 2 B 14.
I = v S02 f 2


where S0 = amplitude & f-frequency
2 2

` I' ? ^2S0 h c
f 2 S0 f

2
m ? I
4 4 = 4
Mass of the wire = ρA, = µ,

` (c)

` µ = ρA
8. Here, ω = 15π, k = 15π
T
ω 15π
` v = ρA ` (c)

` v = k = 15π = 1 m/s.
15. Given f = 250 Hz

a ωt & kx have the same sign, the wave travels
along - ve x-axis.
From the diagram, measuring along x-axis,
2π 2π 2 λ = 0.4 cm

Also λ = K = 15π = 15 m
4
9. Given time ∆t = 0.17 s is from maximum to mean
` v = 250 # 1000 m = 1 m/s.
position.

` (a)
13.2 Physics

INTEXT EXERCISE: 2

= 0.1 # 0.54 = 0.054 m = 5.4 cm

` (b)
7. The other wave should travel along - ve axis & should
1. have the same amplitude. Also, the displacement
produced by the wave. y = a cos ]kx - ωt g at x = 0
should be exactly cancelled by the other wave
]at x = 0g .
` y =- a cos ]kx + ωt g ` (c)

This can also be obtained by considering a phase
change of π when y = a cos ]kx - ωt g undergoes
reflection.

` yreflected = a cos ]kx + ωt + πg

=- a cos ]kx + ωt g ` (c)

At t = 2s, the two pulses will interfere to give a flat 2π
8. Here, k = 3 ; ω = 120π
string, implying no extension of string and hence no
PE. ω 120π =

` v = k = 180 m/s.

` All energy is kinetic. ` (b) b 2π l
3
2. Here, A1 = 5, A2 = 10 T T

from v = µ , 180 =
c 10 2 m
-
3 #

` Amax = 15 Amin = 5 1.5
Imax 15 2
3 # 10 12 -2

` Imin = 5 2 = 9 ` (d)
` T = 180 # 180 #
105
3. Reflection at a fixed boundary leads to a phase
= 648 N
change of 180c .

` (a)

The velocity is reversed too, since the wave starts
moving in the opposite direction. λ
9. From the problem statement, 2 = 10 cm

` (d) `
20
v = 100 # 100 = 20 m/s.
4. For zero PE, string should be flat,
` (b)
π

& 40π t = ]odd g 2 20 2
10. v = -4 = 2 # 10 m/s.
odd 1 3 5 10
#

& t = 80 s ` 80 s & 80 nλ

For a fixed-fixed string, L = 2

` (c)
2L 2 # 1m

` λ = n = n ( in this case)
λ
5. Distance between adjacent nodes = 2 2
2 # 10 =

` f = 100 n Hz
= b 2Kπ l 12 = K
π
b2l
n
` (d)
The plucked point should have maximum
6. The resultant wave will be displacement.

ynet = 2 ]0.05g sin ]3πt g cos ]2xg



` Antinode

` The string vibrates in

` for the particle at x = 0.5, amplitude its second harmonic

= 2 # 0.05 cos ]2 # 0.5g ` f = 200 Hz
14444244443
This is in radians
= 2 # 0.05 # cos ]57.3cg

` (c)
Wave Motion 13.3

490 - v v
11. v =
Similarly, 2x = 2, & , 2 = 4x
0.1 70 m/s. 2


If 400 Hz is the nth mode of oscillation, v v
3x = 2, & ,3 = 6x
3
70 # n

then 400 = 2, .....(1)

As is seen,
70 # ]n + 1g

Similarly 450 = 2, .....(2)
1 1 y y
:
,1 , 2 : ,3 = 1 : 2 : 3 = y : 2 : 3
1 8 = n
& 9 n+1 & n = 8
,1 + , 2 + ,3 = 100 cm
2

using n = 8 in 1 , y y 11y

` y + 2 + 3 = 100 & 6
70 # 8
, = 400 # 2 m = 0.7 m
= 100 & y = 600
11

` (b) 600 300 200

` the lengths are 11 , 11 & 11
T
µ
12. Given, f = 2, also µ = ρA if A =
area of cross section.

1 T

Then f = 2,
πr 2
1 2T
f ' = ] g
π ]2rg2

Also,
2 2,
T
; 2, E
2 1

= 12
4 πr2
15. Frequency of both the wires is the same.

= 1 f
2 2 T T =1 T

vthin = 2 v
ρ.π ^4r 2h 2 ρπr 2
=

` (d) ρ.πr thick

` λThick = 12 λthin since vthick = 12 vthin
14. f1 : f2 : f3 = 1 : 2 : 3 = x : 2x : 3x
` 100ps formed in the thicker wire will be double
the loops formed in the thinner wire.

Then

` 1: 2
` (b)
v v
x = 2, & ,1 = 2x
1

INTEXT EXERCISE: 3

γRT γRT 3RT


1. a v = M , 3. v = M vrms = M

vNitrogen b l
γRT 7
M Nitrogen 5 # 28 γ

` v = =
v b 3 l1 = 1
Helium
b γRT l 5 ` vrms = 3 = 2 3 2
M Helium 3#4
3
` (b)
= 5 ` (c)
2. According to the problem

γRT γR ]27 + 273g


M = 2 M

` T = 4 # 300 K = 1200 K = 927 cC
13.4 Physics

γRT I
4. a Speed in hydrogen = v = 8. Here, 56 = 10 log I
2 , 0


if I = sound intensity t the windows
γRT γRT v

` Speed in oxygen = 32 = 16 # 2 = 4 . I 5. 6
& I = 10 6.4
-
& -12 = 10
10

Average molar mass of the mixture

This is intensity at a distance of 84 m from the
speaker. If the total power output is symmetrically
= 3x
# 2 + 2x # 32
5x distributed, then

= 70 x P
5x = 14 10
-6.4
= 2
4 # π # 84
γRT 1 γRT v
` P = 10 6.4 # 4 # π # 84 2
-
v
` mixture = 14 = 7 2 = 7

= 0.035299...
γ 2 # 1011
5. Speed of sound waves = ρ = 8000

- 0.035 W
= 12 # 10 4 m/s.

9. If I represents intensity, then
1m = 2 # 10 -4

` Required time = ∆t = 1 P
c 2m
4
2 # 10 m/s. IP 4π # 9

` (b) IQ = P
c 2m
4π # 25
6. Power output = 63µW

if p = output power of point secure.

Intensity at a distance of 210 m

-6 IP 25 2

= 63 10 2
# ` IQ =
4 # π # ]210g
92

-6 amplitude at P 5
63 # 10
# 1
& amplitude at θ = 3
4 # π # ]210g2 10 -12

And then, required ratio =

. 110
` (a)


` (a) 10. β = 10 log ^ I/I0 h

7. Acoustic power = Intensity # Area β - 20 = 10 log ^ I'/I0 h

Area of windows = 2m 2 20 = 10 7log ^ I/I0 h - log ^ I'/I0 hA

I
& 2 = log I/I'
Also, 79 = 10 log
10 12
-

I = 107.9 ` I = 10 -4.1
& 100 = I/I'

` -12
10
Power = 10 4.1 # 2 W
-
`
& I' = I/100


= 158.86 # 10 6 -


. 160 µW


` (a)
Wave Motion 13.5

INTEXT EXERCISE: 4
1. Educated guess: The phase difference should be π . 5.

A + 4A + 2 ] Ag]2Ag cos π
2 2

` Anet =

=A

` Intensity = I0 ` (a)
2. ∆x = 1m

For destructive interference, ∆x = odd b A


2
l


For minimum frequency, λ should be max.
Initially,

λ ∆x = ]SB + BDg - SD


` ∆x = 1m = 2 & λ = 2m
= 2 c d 2 + d m - dy
2 V2
340 m/s

` f = 2m = 170 Hz 64
1

` (d) = 2d b1 + 64

1 l2 - d
4
= 2d b1 + 128 l - y = 74d + 64
1 d d =
nλ .....(1)

Finally
3.
∆x = ]SA + SDg - SD

1

= 2 c1.01 2 d 2 + d m - dy
2 2

64

S1 P = 144 + 2.25 = 146.25 . 12.1 m 1

= 2d b1.01 2 + 64
1 l2 - d
4

S2 P = 144 + 12.25 - 156.25 = 12.5
= 7 .1608
4
α + 64 = ]n + 1g λ .....(2)
d
λ

` ∆x = 0.4 m = 2 a initial path difference was 0

from (2) - (1) operation,
343 0.1608d =

` λ = 0.8 m & f = 0.8 = 428.75 Hz 4 λ


` (c)
` λ = 0.0402 d

` (a)
4.

(Highly calculation intensive- Student is doomed if
he/she doesn’t know how to use binomial theorem.)
6. Here, ∆x 4.4m - 3.6m = 0.8m

for constructive interference, ∆x = nλ

S1 P = 144 + 1 = 145 = 12.04 m v nv
& 0.8 = nλ = n f ` f = 0.8

S2 P = 144 + 49 = 193 = 13.9 m
Here v = 344 m/s.

a ∆x = 13.9 - 12.04 = 1.86 m n # 344 = #

` f= 0.8 n 430

` This is the second destructive interference,

` f n = 5, f = 2150 Hz (Not possible)


` 1.86 = 2 & λ = 1.24 m n = 6, f = 2580 Hz (Possible)
343 n = 7, f = 3010 Hz (Not possible)

` f = 1.24 Hz = 276.6 Hz

` (a)

Hence (a)
13.6 Physics
344 of sonometer would have been 470 Hz to start with.
7. Here, λ = 875 m
v v
9. fwire = 2 # 95 cm in one case and 2 # 100 cm in
Exactly midway, ∆x = 0

If the person walks a distance the other. The difference between these numbers is
'd' then ∆x = 2d . 8.

λ 100v 100v

for a minima, ∆x = 2 using n = 1
` 190 - 200 = 8 [converting length to S.I.]

λ λ 1 344 86
` v = 19 # 8 # 2 m/s.

` 2d = 2 & 4 = 4 # 875 = 875
c 0.1 m
` fwire = 100 # 10 v = 8 # 190 # 200

In fact, slightly less than 0.1 m .
` Frequency of fort = 156 Hz

` (a)

` (a)
8. a Fork frequency = 480 Hz ,
10. On the lines of question II,

` chronometer frequency is either 470 Hz or 490 100v - 100v =
Hz. 96 100 8
4v
T v
& 96 = 8 & v = 192 m/s.

v= µ and fsonometer = λ ? T
100 # 192 =

Then fwire = 96 200 Hz
` if T increases, frequency of sonometer also
increases.
` ffork = 196 Hz

If then, the number of beats decrease, the frequency
` (a)

INTEXT EXERCISE: 5
λ =
a This pipe is 20 cm long, no resonance will take
1. Given 4 20 cm
place.

Resonances will also be obtained at lengths of air

` (d)
column odd b λ
4
l
340
4. Here λ = 1700 m = 20 cm

& 3 # 20cm = 60cm or 5 # 20cm = 100cm etc.

` (d)
If this wavelength has to excite an open pipe then
λ
the length of pipe should be (integer) 2 , i.e.;
2. Here for closed pipe 10 cm, 20 cm, 30 cm.....
v
f = 4L if LC = length of closed pipe.
A 20 cm pipe will oscillate in its second harmoic,
since 20 cm = ]2g 2 .
C
20 cm

Similarly, if L0 is the length of open pipe, then

3v
` (b)
f = 2L
0 v
5. Given, 512 = 4L if L = length of the closed tube.
v 3v LC 2 1 a

` 4LC = 2L0 & L0 = 4 # 3 = 6 v
` L = 4 # 512
340
3. For 850 Hz source, λ = 850 m = 40 cm
if the same length is used as an open pipe,
v
fundamental frequency = 2L

If 40 cm wave is to excite a closed pipe, then length v
of pipe should be odd b λ l
= = 1024 Hz
2 b 4 # 512 l
4 v


= 10cm, 30cm, 50cm etc.
` (a)
Wave Motion 13.7
6. If L0 = length of open pipe, then: Now, fundamental frequency of open pipe

300 = 2L
v = 2vL = 100 Hz
0

3v V 3V

Also, 600 = 4L 8. 4,1 = 2, 2
C
,1 1
3 # 330 3 # 165 & ,2 = 6

` LC = 4 # 600 m = 4 # 6 cm
Ans (c)

= 41.25 cm
5V - V

` (c) 9.4, 2, = 100
V
7. Frequency of 3rd harmonic of closed pipe 4, = 100
V
= v = 43L v 2, = 200 Hz
b 4 L l
3
10.
V
4, = ^ f0 hinitial

Assuming that 3rd harmonic of closed pipe is higher
than fundamental of lcosed pipe by 100 Hz.
V
2,' = ^ f0 h final
3v v 2v ,
4L - 4L = 100 & 4L = 100 ,' = 2
v

& 2L = 100
Minimum length of water column = 60 cm

INTEXT EXERCISE: 6
v f v + vS
f ' = f f v + 10 p = f b 11v l 1 11 4. Given = 1.2 &
1. 10 v = 10 f v = 1.2
f cv+v m
v
v
S
11f
f '-f -f vS
` # 100 = 10 f # 100
f & v = 0.2

= 10%

For the second case,

` (c) f
= 1 v = 01.8 = 1.25
fc m
2. Since there is no relative motion between the source v - vS
1 - vS
& the observer, doppler effect will not come into v
picture.
` (b)

Frequency heard = frequency emitte = 200 Hz.
5. Since the velocity will have no component along

` (d) the line joining source & observes, hence, Doppler
fapproach - fRecession effect will not be observed.
3. Given factual
# 100 = 2

` n1 = 0 ` (b)
v + vS - v + vS
v= G = 100
2

&
v 2 - vS2 6. ‘A’ hears a frequency V due to the source held in his
30000vS _90000 - vS i
2 hands.

&


& vS2 + 30000vS - 90000 = 0 Frequency received by a from B = V b v +
v
ul

Assuming vS << 300 m/s. & hence neglecting vS2,




` Beat frequency = V b1 + uv l - V
30000vS = 90000


& vS = 3 m/s.
= Vu
v ` (a)


` (b)
13.8 Physics
9. From vehicle going away
f ' = 500 b 34034010 l
+
7.

= 240 b 320 + 4 l
320

= 500 # 35 b 1 l
34 = 500 1 + 34

From vehicle coming towards

= 500 + 14.7
= 240 b 320 - 4 l
320


c 515 Hz

` No. of beats per sec

Wavelength = Shortest distance between two
wavefronts in the same phase. = 240 b 320 l b 320 l
316 - 240 324

= 240 # 320 b 324 # 316 l


m/s 324 - 316
= 340
500 /s = 0.68 m


[The observer will feel the same distance between
= 240
# 320 # 8
324 # 316 c 6
the wavefronts but will encounter them footer as a
result of running towards them]
` (a)


` (d) 10. Let the observer detect a frequency f ' . The wall
8. Since both, the man on the platform & the train, are also detects the same frequency f ' , & reflects it
stationary there is no relative motion between them. too.


Hence, no Doppler effect in spite of the wind.
There is no Doppler effect between the observer 2
the wall 9no relative motion).

` fheard = foriginal = 500 Hz

Hence, observer receives f ' from the wall
λ required = λ original = 0.68 m

` No beats.

` (a)
` (a)

EXERCISE - 1

1. VPmax = Aω = Y0 2πf = 4Vω ` y = 0.5 cos ]2rx + 4rtg



2πf
Y0 2πf = 4 c
m
4. VAB =
6.4g m
2π/λ = 6400 = 80 sec
10 # 10 -3
πY
` λ = 20 3.2g m

VCD = = 20 10 sec
8 # 10 -3
2. Put α, β, A, x and t in the equation
2π 1.6g
= 0.56 cm 1
-
VDE = = 1600 = 40 m/s.
λ 10 # 10 -3
r 12.56 # 180 54

ax + bt + 6 = + 30 = 750c 5. v = fm = 60 # 10
3.14

y = 7.5 cm sin 750c = 3.75 = 9 m/ sec .

v = dt = ab cos bax + bt + 6 l
dy r 6. Path difference is m between B and G

7. v is same for all the waves
3

= 7.5 # 12 # 2 = 77.94 cm/ sec .

From the figure m1 = m3 < m2
3. ~ = 2rf = 4r sec -1

So ~1 = ~3 > ~2
2r

K= = 2r m -1
m
Wave Motion 13.9
ω 420 100
8.
Vω = k = 21 = 20 # 4 # 10 -2 -2
4 # 10
T
& f = 30Hz

` V = µ = 20


& T = ]20g2 µ = ]20g2 # 0.2 = 80N P 42
19. P a A2 0.40 = 2 & P = 1.6 watt
2
9. Satisfy the standard equation of wave.
20. By definition
log x - t = log ] x + t g
2 2
x -t

& I ? bT l
2r 2 t A 2 v A 2
21. I =
y = 0.1 sin 2π ]0.1 # - 2t g
2
T
10.
& I1 = b A1 T2 l = b 2 # 1 l = 1: 1
I A T 2 1 2 2

] - g
dy 2 2 1
dt = 0.4π cos 2π 0.1x 2t 22. Amplitude depends upon phase angle.
vmax = ]0.4π m/sg
23. Amplitude varies between A1 + A2 to A1 - A2
2 waves
24. A2 net = ^ym + ymh
11. f = 1 sec . = 2Hz; λ = 5m 2
for z=0
` v = fm = 10 m/s.
A
` net = 2ym & Inet aA2 net a 4ym2 ? 4I
12. y1 = a sin ωt
y2 = a cos ωt = a sin ]ωt + π/2g

where i & Intensity of either wave.

π
y1 lags y2 behind by phase 2
25.
13. y1 propagartes in + x -axis and y2 along -ve x-axis.

3 ; y2 = ^sin 3πt + 3 cos 3πt h


14. y1 = 10 sin b3πt + π l
Resultant Amplitude = 32 + 42 = 5nm
26. The pulses will pass through each other.
y2 = 10 c 1 sin 3πt +

3 m = 10 sin b3πt + π l
2 2 cos 3πt 3 27. Second string is denser so speed will decrease.

` A1 /A2 = 10/10 = 1: 1 28. Reflected pulse will be inverted as it is reflected by a
15. y1 = 0.25 cos ]2πt - 2πxg
denser medium. The wall exerts force in downward
fλ = const.
direction.

f = f/2 & λ = 2λ
29. As x = 0 is node & standing wave should
y2 = 2 # 0.25 cos b 2 + 2 l = 0.5 cos ]πt + πxg
2πt 2πx

be y = 2a sin kx sin ~t
16. p0 = A02 ω 02 µv
reflected wave =- A cos ]kx + ωt g
p0 2 2
2 = A ω µv n T n+1 T
30. 2l n = 350 and 2l n = 420
2 2
A ω
0 0

`2= T T T
2
A ω 2
31. v = n = At = rr2 t

` As, ω = ω 0 (frequency remains the same)
f1 T1 t2 r22
#1 #1 1
A0 f2 = T2 $ t1 $ r12 = 2 2 4 = 2
A
` =
2 4V v L
32. f = 2L & m = f = 2 = 40 cm
17. By definition. < P >= 2 ] Mvg A 2 ω 2
1
33. y = a sin ~t cos Kx
18. As < P >= 2π 2 f 2 A 2 µv
y = 2 ]2a sin ~t cos Kxg
1

Put values

90 = 2 # 10 # f 2 # 25 # 10
-4

` Amplitude of component wave is a/2
13.10 Physics
34. If T = mg = vtg 42. Frequency depends on source not on medium.
43. f1 m1 = f2 m1
]3000g]1g = ^ f2h ]1.5g
1 T

` f = 2l n = 300 .....(1)
v
200 Hz = f2

Now T ' = mg - B = vρg - 2 g
544
44. time to reach sound wave = 340
T ' = vg b l
2ρ - 1


2 544 544
time to reach bullet = ]
340 - 20g
= 320
1 vg ^2p - 1h

` f' = 2l .....(2)
Dt = 544 :320 - 340 D = 544 # 320 # 340 0.1 sec .
n 2 1 1 20

1 1

c m f' = 300 c 2ρ m
f' 2ρ - 1 2 2ρ - 1
2

` f = 2ρ 45. V a T V increases m increases
ω c1
35. K = Vω for either component waves v
46. v12 =
M2
=
5 # 18 # 3
m1 # c2 3 # 40 4 = 0.75
YA 1 YA 4
36. As T = 20 4 & v0 = 2 # 24 20 $ µ .....(1) c RT
47. the speed of sound in air is v = M
YA 1 YA 6 c

and T' = 20 6 & v' = 2 # 26 20 $ µ .....(2) M of H2 is greatest in the given gases, hence
v' 24 6 speed of sound in H2 shall be maximum.

` v0 = 26 4
48. Speed of sound in a gas is given by:

` v' > v0
γRT
2π 2cm v = M
37. K = 0.025 π = λ = 0.025
λ
1
λ 1 v ?

Required length = 2 = 0.025 = 80 cm M
v1 M2 m2
n T = n-1 T =
38. 2l µ 384 2l µ 288
` v2 = M1 = m1
n Here = C = 3 for both the gases bc monoatomic = 3 l
4 Cp 5 5

` n-1 = 3
v

n = 4 Now; 4 b 2L l = 384
V T1
` V1
49. V2 = T2

Put L = 75 cm
V1 Kx
K ]1.5xg
V2 =

` V = 144 m/ sec .
m m V1 1
39. L = 5 2 & 10 = 5 2 & m = 4m V2 = 1.5
v 20
f
` = = 4 = 5 Hz
V
m V2 = 1.5
40. Distance between position having 3 nodes and 2 V
antinodes = wavelength = 1.21 Ac V2 = 1.5
41. For sonometer wire V2 = 1.5 V
n # 100 = ]n + 1g # 95
V2 = 1.22V
n = no of harmonics
I I
50. b1 = 10 log I b2 = 10 log I2

& n = 19 0 0

f = 19 b 2 l + 4 = 20 b 2 l - 4 & L = 16
L L b1 - b2 = 3.0103

`
I

f = 20 b 2 l - 4 = 156 Hz
L 10 log I = 3.0103

& 2

Longitudinal waves (sound waves)


& I = 2 # 10 -4
Wave Motion 13.11

51. P0 = B k S0 , B1 k1 S1 = B2 k2 S2 & ^262 - fh = ! ^256 - fh # 2



here B1 = B2 a of same medium.
& f = 250, 258Hz
S1 k2 m1 59. First maxima after O will appear when path
S2 = k1 = m2 = 2 difference DS = m
P2

Power = 2t0v
2.42 + 12 - 2.4 = m m = 0.2

P0 is same for both waves

sound velocity = fλ = 1800 # 0.2 = 360 m/s.

60. Path difference ]DSg between direct and reflected



t is same for both waves


v is same for both waves wave = 130 - 120 = 10m

I 4I
52. B0 = 10 log I B1 = 10 log I
0 0

I

= 10 log 4 + 10 log I
0


= 20 log 2 + B0 = B0 + 6

for so constructive interference DS = nm

Hence when intensity is increased four times, level
becomes ]B0 + 6g decibels.
DS 10
m= n = n ^n = 1, 2, 3.....h

P 10 10 10
53. = I for an isotropic point sound source.
m = 10, 2 , 3 , 4 .....
4rr2


& P = 4πr
2
Ans (a)

= ^0.008 w/m2h^4.r.102h = 10.048


61. Frequency of two source n1 = 50 n2 = 51

, 10 watt.
so beat frequency = 1 / sec .
I 400
54. dB = 10 log I2 = 10 log 20 = 10 log 20
Now intensity ratio of maximum & minimum value
1

= 10 ]1 + 0.03g = 13dB ]a1 + a2g2


2 =b 8 l = 1
I 16 2 4
]a1 - a2g
= Imax =
min
55. In the interference the energy is redistributed and the

So, Ans (d)
distribution remains constant in time
56. Ist minima Dx = nm = 12 62. To get beat frequency 1, 2, 3, 5, 7, 8, it is possible
when other three tuning fork have frequencies 551,

IInd minima Dx = nm + m = 36 553, 558, etc.

& m = 24 cm
63. I1 = b A1 l = 1 & A1 = 1
I A 2 9 A 3
2 2 2
v 330
f= = = 1375 Hz JK A1 N
m 0.24
KK A + 1 OOO
= b A - A l = KK A
2
I A 1 + A2 O
57. Path difference = rr - 2 r
` Imax 2
min 1 2 KK 1 - 1 OOO
DS = r ]r - 2g
A2
= b 3 - 1 l = b 2 l = 4: 1
3+1 2 4L 2 P



for constructive interference
4 ]π - 2g
64. If fB > fA; fB = 260 Hz;
nλ = r ]π - 2g
λ= n

If fB < fA; fB = 252Hz
V Vn
f = = ]π - 2g
λ r

after waxing, fB further decreases and beat increases
58. 262 - f = 256 - f # 2 to 6, if fB = 252
13.12 Physics

65. 2f = f + 15 # 8 & f = 120Hz v


71. For open pipe first overtone frequency = 2l # 2
0
66. Given that a closed organ pipe and an open organ v #
pipe of same length L are vibrating simultaneously for closed pipe 1st overtone frequency = 4l 3
C
in their fundamental mode, and produce 2 beats. the two frequencies are equal
Since both pipes are in fundamental mode, open l 3
pipe has more frequency.
& lC = 4
0
Given, f0 - fc = 2
m
72. 2 = l ` m = 2l
V V
& 2L - 4L = 2, where V is velocity of sound and
L is length. V

n1 = 2l
V
& L =8
3m 4l
V
& = 8L 4 = l m = 3

When lengths of both pipes are changed, pipes n2 - n1 = 100
produce 7 beats. The new length of open organ pipe
is L0 and new length of closed organ pipe is LC . 3V 2V
V V 4l - 4l = 100

So 2L - 4L = 7
O C
V
1 1 7 4l = 100

Using V = 8L, L - 2L = 4L
0 C
L fundamental frequency of open organ pipe

From this above condition, LO = 2 & LC = 2L is V
the only satisfying value. 2l = 200 Hz

i.e. the length of the open pipe is halved and the
73. v = f1 m1 = f2 m2 & 512 # 4L = f2 # 2L
length of the closed pipe is doubled.
f
& 2 = 1024 Hz
67. m1 = 2L; m2 = 2 ^L - yh
74. v = f1 m1 = f2 m2 & f # 2L = f' # 2L & f' = f
= ;
v v v 1 - 1E

Df = f2 - f1 = -
m 2 m1 2 L - y L 75. Doppler effect in Frequency depends upon relative
vy vy velocity between source and observer
2 ^L - yh L

& - 2
2L
68. When a sound wave gets reflected from a rigid
boundary, the particles at the boundary are unable 76.
to vibrate. Thus, a reflected wave is generated
which interferes with the incoming wave to produce

At A observer is moving towards source so v1 > v .
zero displacement at the rigid boundary. At these
points (zero displacement), the pressure variation is
At B observer is moving away from source so
maximum. Thus, a reflected pressure wave has the v2 < v
same phase as the incident wave.
At C observer is moving = to line joining source
69. n1: n2: n3 = 1: 2: 3 and observer so v3 = v
V V V =
& v1 > v3 > v2
: :
λ1 λ 2 λ3 1: 2: 3
v 1
1 1 77. f1 = f0 v sound = 1.2 f0
:
λ1 λ 2: λ3 = 1: 2 : 3 sound + vtrain

vsound
70.
m1 V
& 1.2vsound = vsound + v &v=
2 = l + .6d y1 =
m1
5
vsound - vman
f2 = f0 vsound = 0.8

f0

& f = 1.25
2

m2 V
4 = l + .3d y2 =
m2
y2 2 ]l + .6dg ]l + .6dg
y1 = 4 ]l + .3dg =
2 ]l + .3dg
Wave Motion 13.13

f2 = f b V - V l = b 340 - 17 l = 323 f
78. Let original frequency is f V 340 340


by the concept of Doppler effect s


frequency of reflected wave f1 323 19
f2 = 306 = 18
V+u 332 + 12 v 330
f' = V - u f = 332 - 12 # f f' - f = 6 80. f = f0 v sound = 450. 330 - 33 = 500Hz
sound - vsource


344 81. Apparent frequencies of both the whistles are the
320 f - f = 6
same as heard by the stationmaster because both
320 # 5

& f = 24 = 80Hz trains are moving away from station.

79. f1 = f b V - V l
V 82. 7m = 0.14; m = 0.02
s
v 3 # 108
f1 = f b 340 - 34 l = f b 306 l
340 340 f= = 0.02 = 1.5 # 1010 Hz
m

EXERCISE - 2

5m m
1. As 2 = 20 & m = 8cm 6. Distance between boat = 2 = 10m

2r 314
& m = 20m

K= = 4
m
time period T = 4 sec .
2r #
~ = KV 350 = 27475
` V = m/T = 20 m/4 sec .
8 # 10 -2
` y = 0.05 sin b 4 # - 27475tl
314 = 5 m/s.

V V
1 ]t = 0g 7. R A = V , RB = V
2. y = A B
1 + x2
as VA > VB, R A < RB
1 ]t = 2g
1 + ]x - 2vg2
y= 8.
Now comparing

x - 2v = x - 1
m
Dotted shape shows pulse position after a short

v = 0.5 sec
time interval. Direction of the velocities are decided
according to direction of displacements of the
3. V \ T
particles. at x = 1.5 slope is +ve
V1 T1 2T

V2 = T2 = T = 2
at x = 2.5s slope is -ve

4. λ = 2, A = 0.25m 9. For the pulse


nxg dx

T = 0.4 V= n = xg = dt
x x
dx dx
dt = xg & # x = g # dt
0 0
x

t = 2 g .....(1)


for the particle
mg
5. 50 = n .....(1) 1
L - x = 2 gt2
m ^g2 + a2h 2 ]L - xg
1
2


52 = n .....(2)
t= .....(2)
g

Solve (1) and (2)

from equation (1) & (2)
m

` a = 4.1 L
sec2 & ` x = 3 from the bottom
13.14 Physics

2r 17. At t = 2 second, the position of both pulses


10. Vmax = A~ = 5 & A 4 =5
are separately given by fig. (a) and fig. (b); the
10 superposition of both pulses is given by fig. (c)

& A = r cm

measure AB m
11. measure CD = T = V

1
12. 2f = 6, f = 3, T = 3 sec . v = 3 m/ sec .

m

3 = T & m = 1m

x = vt

3 = 3t &t=1
3T

Total time = t + 4

= 1 + 4 b 3 l = 4 = 1.25 sec .
3 1 5

13. The equation of the given transverse wave is given 18.


by the displacement of the particle y .
y = A sin2 ]~t - kxg


Velocity of the particle
=
]~t - kxg cos ]~t - kxg
dy
dt = 2a~ sin

Maximum velocity = A~
Area = 5 mm


2~
But the velocity of the wave = 2k =
2ry 19. As y = A sin ]Kx - ~t + 30c + 180cg for incident
2r/m wave.
~ 1 m m

If A~ = k , A = k = r & A= r Now for reflected wave: Energy aAmp2

` Y = 0.8A sin ]- Kx - ~t + 30 + 180g


T nxg
14. v = 2

n = n = gx & v = gx

(symmetrical about x ) Y = 0.8A sin ]- Kx - ~t + 210g

15. y = sin ^~t - kx + zh Y =- 0.8A sin ]kx + ~t - 210g


v = ~ cos ^~t - kx + zh
Y =- 0.8A sin 5Kx + ~t - 30 - 180?

r

at
Y = 0.8A sin 6180 - ]Kx + ~t - 30g@
t = 0, x = 0, y =- 0.5, v > 0 & z = 6

y = sin b~t - kx - 6 l
r

Y = 0.8A sin ]Kx + ~t - 30g
`

16. y = 2mm sin b2rx - 100rt + 3 l


r
20. the right end will shoot up on the wire

& 0 = 2 sin b2rx - 100rt + 3 l


r
21. The equation represents a progressive wave moving
along x-axis of single frequency wave reflects back
& nx = 8r - 100rt + 3 ^n = 0, 1, 2, 3, ...h
r
in the same medium.
25r
- nx r 1
& tmin = 3100r = 3 /100r = 300 s

Wave Motion 13.15
π 3π 1 mg
22. At x1 = 3K and x2 = 2K 27. f = 2 # 40 µ = 256 .....(1)
Nodes are not formed because neither x1 nor x2 1 mg - fb
gives sin kx = 0 f = 2 # 22 µ = 256 .....(2)
mg - f
mg b = b 22 l
7π 2

` ∆x = x2 - x1 = 6k . 40

m
As this Dx is between m and 2 v ]1 g g 40 2 - 22 2
vσg =
40 2

` φ1 = π ; for 1st wave. 40 2
σ= 2
7π 40 - 22 2

and φ 2 = K∆x = 6
4m
28. For third overtone 2 =l
φ1 6

` φ2 = 7 l
& 2λ = l & λ = 2
m
23. n = l = tA

As x = 0 is a node

` m1 = trr 2
2r

` x As.ω. = A sin
m
m2 = t4rr 2
2π ,

= a sin f , 3 p
T/n1
v1 2

` v2 =
T/n2 3

Let P loops and q loops are formed
=a 2


respectively 1st & 2nd wire. 29. v =
T 4.5 # 107
n = 0.05
p q p 1

` 2l V1 = 2l V2 & q =2 n #
4.5 10 7
2l 0.05 = 420 .....(1)
1 Tl1 1 Tl2
24. f1 = 2l m f2 = 2l m n+1 4.5 # 107
1 2
2l 0.05 = 490 .....(2)
f1 l2 l1 ]1 - 20ag

` f = l1 = l1 n 6
2
From equation (1) & (2) & n+1 = 7 & n = 6

f = ]1 - 10ag
f1
6
2
Put n in (1) ` 2l 3 # 102 = 420

(By binomial theorem, assume α to be small)
30000 1500

l = 140 l = 7 = 214 cm
f2 - f1

a = 10f = 10 -4 cC -1
2
YA
1
25. n = 2l n
T 1
n = 2l
YA a DT 1
30. f = 2l
T 1 l .Dl
n n = 2#1 tA
Put values
1 9 # 1010 # 5 # 10 -14
n
` = 11 Hz
=2 = 35 Hz
9 # 103
26. 31. In sonometer
v v
Va T

n1 = 2l n2 = 2l
1 2

v V T1 T

and n = 2l (for complete length of wire)
` V1 = 2 = T2 & T2 = 41
2


`%

As l = l1 + l2 + ...
T
V V V T1 - T2 # T1 - 41
2n = 2n1 + 2n2 + ... T 100 = T # 100 = 75%
1 1
1 1 1
n = n1 + n2 + ...
13.16 Physics

32. It obvious that particle at 0.2 L will have larger 40. sin 2rnt + sin 2r ]n - 1g t + sin 2r ]n + 1g t
amplitude than particle at 0.45 L, 0.5 L being the
62rt ]n - 1 + n + 1g@
node and 0.25 being antinode.
= sin 2rnt + 2 sin 2
62rt ]n + 1 - n + 1g@

cos 2

sin 2rnt + 2 sin 2rnt cos 2r t

sin 2rnt 51 + 2 cos 2r t?



f
& beat = 1
33. during clapping multiple wave are produced with
different wave parameters so wave is resultant of all 41.
of these wave.

34. Speed of sound in an ideal gas is given by


cRT

V= M
At B path difference is O and at A path difference

c is 4m . From nm formula there are 3 maxima position
V\ M [ T is same for both the gases] between A & B . So total maxima in ellipse = 16
^7/5h 4
b l = 3 /5
VN cN MH

VHe = c He # MN = ^5/3h 28 42. Let frequency of tuning fork = f
2 2 e


c N = 7/5 (Diatomic) v
2
Frequency of wave in wire = 2l

c He = 5/3 (Monoatomic)
When l = 1m
35. The equation of pressure variation due to sound is
v
f1 = 2
p =- B dx =- B dx 7s0 sin ]ωt - kxgA
ds d 2

when l = 1.05 m
= B ks0 sin ]2~t - 2kxg v
2 # ]1.05g
f2 =
I I
36. b = 10 log I , 60 = 10 log I f1 - f = 5 and f - f2 = 5 & f1 - f2 = 10
0 0

8I
& 2 b1 - 1.05 l = 10

b = 10 - log I v 1
0
I
= 10 log 8 + 10 log I = 30 log 2 + 60 = 69dB 20 # 1.05
0
or v = 0.05 = 420 m/s.
1
37. I ? A2 and I ? 2 rRL v
f1 = 2 = 210Hz
I1 R2 A12 A R2 25
I2 = R1 = & A1 = R1 = 9 = 5: 3 f = f1 - 5 = 205 Hz
A22 2

P02 43. The amplitude of vibration at any point changes


38. I = 2PV
simply harmonically with a frequency equal to the
difference in the frequencies of the two waves.
0.8 P02
2 ]1.29g]340g 0
; P = 4.98 N/m 2
4π ]3/2g

& 2 =
44. Wall will be a pressure antinode
39. n=3

m v 330

Minimum distance = 2 = 2f = 2 # 165 = 1m
Wave Motion 13.17

500r 506r Z] v
45. f1 = 2r = 250; f2 = 2r = 253 ]] for open pipe
] 2,
51. ffun = []

` Df = 3 s -1 = 3 # 60 min -1 = 180 min -1 ]] v
] 4, for closed pipe
\
46, In an open pipe the ends are points of displacement f \ T , but f does not depend on pressure.
antonodes and hence pressure node. The midpoint
(for fundamental mode) is a point of displacement
for closed pipe ftest overtone = 3ffundamental
node and hence pressure antinode. (variation of
pressure is maximum at pressure antinode and zero 52. y1 = A cos ]ax + btg
at pressure-node)
so here k = a
47. Closed Open 2r 2r
= a & m= a
m
V V
4l1 = l2 b

~=b & n = 2r

c aincident m
2
Iincident I
=
Ireflected 0.64I = areflected

For reflected wave displacement equation
y2 = 0.8A cos ]- ax + bt + rg

l2 = 4l1
=- 0.8 A cos ]ax - btg
l 50
l1 = 42 = 4 = 12.5 cm

by super position law resultant wave
48. Now the tube becomes a closed pipe with length l/2
y = y1 + y2


Fundamental frequency of = A cos ]ax + btg + 6- 0.8A cos ]ax - btg@

vsound v
so incorrect option is (d)
= sound
4 ^l/2h
B= 2l
53. For pipe A , second resonant frequency is third

which is fundamental 3V
harmonic thus f = 4L
A

Frequency of A

For pipe B , second resonant frequency is second
3V 2V
49. Second overtone of open pipe = 2l harmonic thus f = 2L
1 B

5V 3V 2V

Second overtone of closed pipe = 4l
Equating 4L = 2L
2 A B


Since, these frequency are same = 3 . ]1.5g
4 4

& LB = 3 L A = 2m
3V 5V l1 4 # 3 6
2l1 = 4l2 & l2 = 2 # 5 = 5 54. Let e be the end correction.

Now, the ratio of fundamental frequencies: m 3m

0.1 + e = 4 0.35 + e = 4
V
f1 2l1 2l
Solving this equation we get
f2 = V & l2
1
4l2
e = 0.025 m = 2.5 cm

= 10: 6 = 5: 3
V
55. n = 4L initially
m
50. 4 = ,1 + e .....(1)
As the length changes by l , and velocity changes by

3m v , the new frequency.
4 = ,2 + e .....(2)
V+v
n' = ]
4 L + lg
From (1) and (2) e = 2 cm


So, choice (c) is correct and the rest are incorrect.
13.18 Physics

& 2n'n'' = ]n' + n''g n & n = ]


m 2n'n''
n' + n" g
56. 4 = L + 0.6 r1
(closed organ pipe) 63. frequency heard by listner

m = L + 1.2 r1
V-u V

n1 =

(open organ pipe) V m
& 4 ]L + 0.6 r1g = ]L + 1.2r2g & r2 - 2r1 = 2.5L

V+u V
n2 = V
m
cRT k k 2u
57. f = = , fA =
beat frequency = n2 - n1 =
2L M L M L 2 m

fB =
2k
, fC =
3k 64. Using the formula
L 32 2L 28
f' = f b
v + v0 l
3k 11
fD = , ` fC /fD = 28 v
L 44
5.5 = 5 b
v + vA l

we get .....(1)
58. n' = b V s l nr, nr = b V - V l n
V+v V v
6.0 = 5 b v B l .....(2)
s
v+v

and
n' = b V - Vs l n = 350 - 50 # 1.2
V+V 350 + 50

here v = speed of sound
s


400
= 300 # 1.2 = 1.6 KHz
v A = speed of train A

and vB = speed of train B
59.

solving eqn. (1) and (2) we get:
vB
vA = 2
65. The motorcyclist observes no beats. So the apparent
frequency observed by him from the two sources
must be equal.
n' = b V - V0 cos a l n
V - V sin a
f1 = frequency recorded by motorcyclist from police
s
car.
1 f2 = Frequency recorded by motorcyclist from

tan a = 2 is constant and n’ remains constant and
stationary siren

n' < n so, graph, must be a straight line.

For no beats & f1 = f2
60. There is no relative motion between source and
V 176
` b 330 - y l = 165 b 330 + y l
observer so frequency remain constant n = 330 - 22 330
m0
when wind start blowing in the direction of wave
Solving this equation we get, y = 22 m/s.
motion then velocity of sound = V + uw
v+v

V+u V+u
so apparent wave length m' = n w = V w m 0 66. v' = v d v - v0 n
s

v-v
T' = T d v + v s n
61. frequency of sound for approaching observer
c+v
fa = c f 0

T' = 10 b 330 + 60 l = 10 # 390


c-v 330 - 30 300

For receding observer fr =
c f
fr + fa = 7.692 sec . = 7.7 sec .
fr + fa = 2f f = 2
67. fmin = f0 b v + ~R l = 385 b 340 + 20 # 0.5 l = 374Hz
v 340
& ]n' - ngvsound = n'vtran
Vsound
62. n' = n. V
sound - Vtrain

+ vtran & ]n - n''g vsound = n'vtrain 68. f = f0 c m& b l


v v + gt df g

n'' = n. vsoundsound
v dt = f0 0 + v
]n' - ng n' 1000 1000 # 10

& n - n'' = n'' & n'n'' - nn'' = n'n - n'n''
& 30 = & v = 300 m/s.
v
Wave Motion 13.19

EXERCISE - 3

Numerical Type nv
9. f = 2l .
1. IIIrd overtone node is formed at 3m from end A, in 5th, 10th, 15th ...
4V 2V 2 200 harmonics
f= = = = 2(200); f =400 Hz
2L L 1 (5/1000) 5 144
& = ] g
f 1 = 6 Hz
2 5
P p02
2. I = = 5
4rr 2 2tv 10. y(at t = 1) = 2
x + 6x + 9
2tvP 2 # 1 # 330 # 50r
(by assuming wave is moving in +ive x direction)
p02 =
= = 5 N/m2
2 rr 2 4r # 330 5
y (x, t) =
3. The given equation can be represented as 2
((x - v (t - 1)) + 6 (x - v (t - 1)) + 9)
2r x 2r
y = 2 × 10 cos d n sin d 5000t n y (at t = 2) =
5
50 50 2
(x - v) + 6 (x - v) + 9
Comparing it with standard equation of stationary 5
2r 2r 2
wave = x + x (6 - 2v) + v 2 - 6v + 9
m 50

by comparing with given equation at t = 2
l = 50 units

6 - 2v = 12 ⇒ v = -3 m/s ⇒-ive ×-direction

So loop length l/2 = 25 units.

⇒ speed = 3m/s
4. When the two positions of the resonance are obtained

⇒ Ans. V + C = 3 + (-1) = 2
at distances l1 and l2 respectively then the velocity
11. Tension at the top = (6 + 2) × 10 = 80 N
V of sound is given by V = 2f ( l2 - l1 )
Tension at the bottom = 2 × 10 = 20N
V = 2 × 1600 × 10 × l0-2 = 320 m/s
Ttop 80
5. For x = 5/2,
\ Velocity of wave at top = =
5/2r n n
y = 4 sin d n cos (96pt) = 2 cos (96pt) Tbottom 20
15
Velocity of wave at bottom = =
n n
So, y will be maximum when cos (96pt) = 1
Since source is same ⇒ frequency same.
vtop m top 80

ymax = 2 cm at x = 5/2 cm.

` v = = =2
bottom m bottom 20
v - v1
6. Direct frequency received by O(f1) = d nf
` m top = (2 # 0.06) = 0.12 m
v

frequency received by wall = f (No relative motion 1 m
12. Total energy = tA 2 ~ 2 #
between S and wall) 2 4
v - v1 13. Let velocity of train A = VA & of train B is VB
frequency received by O from wall (f2) = d
nf
v
velocity of sound = VS

beat frequency = ; f1 - f2 ; = 0 VS
VS + VA
7. For train B, v = 25 m/s \ given, 5.5 = f
p 5 & VA =
VS 10
350 A VS + VB VS. VB 10

fA =
25
f0 Also,6= f
p 5 & VB = ` = =2
350 - VS 5 VA 5
2 14. Phase difference between S1, and S2

For train C, v = 25 m/s 2m 2r
350 = # Ox = #1 = r

f lA = f B C m 340
25 0 170
350 +
2 IS = IS = IS = I
1 2 3


\ Frequency of beat = fA - f lA . 5 Hz
intensity due to all three source at point P is = I
13.20 Physics

40 m D m
L1
So, (n + 1) = 240
2
80
9m
⇒ (n + 1) = 240
15. 41 m 2
L2
or n + 1 = 6

OPC
n=5

Dx = 1 m So string is vibrating in fifth overtone.
nl = 1 19. Acceleration of system a = 5 m/s2
C
In case of backward direction wave transmission:
f= = C # n; n = 1; f = 330 Hz Tension at point from r distance from C in the string
m
BC is T = 15 - 5r
16. For direct sound from the source,
VS - V0 T
f1 = f pf
Wave velocity, v =
n
VS + Vsource 0
VS = velolcity of sound 15 - 5r

⇒ v=
V0 = Velocity of observer 1
350 dr
Vsource = Velocity of source f1 = # 180
⇒ = 15 - 5r
360 dt
For sound heard after reflection, 1
dr
t1

VS - V0 VS
⇒ # = dt #
f2 = f
pf l & f l = f p f0 15 - 5r 0
0
VS VS - Vsource 2
⇒ t1 = (151/2 - 101/2)


\ f2 = f0 ; 5
350 2
\ No. of beats = f2 - f1 = d180 -
# 180 n
⇒ t1 = ( 3 - 2) s
360 5
180 # 10

= = 5 beats/s
In cace of forward direction wave transmission:
360

Tension at point from r distance from B in the string
17. Path difference = Dx = d2 - d1 = 0.5 BC is T = 10 + 5r
2r 2r f 100 1 r
= Ox = v Ox = 2r # . = 10 + 5r dr
m 300 2 3
v= & = 10 + 5r
1 dt
intensity at C
t2
P1 P2 2
1

P1 P2 cos d n = 6W/m 2
r dr
=
4rd1 2
+ 2
4rd 2
-
4 r d d 3
# = # dt
1 2
0 10 + 5r 0

18. 2A sin kx = 3 2 2

& t2 = (151/2 - 101/2)

⇒ 2 × 3 sin kx = 3 2 5
1 2

⇒ sin kx = & t2 = ( 3 - 2) s
2 5
2r r 3r & t1 = t2

⇒ x= ;
4 4 3l2 - 5l1 3 # 74 - 5 # 44
20. e e = o; e =
m

Distance between consecutive points 2 2
3m m m

= - =
8 8 4
m
= 20 cm & m = 80 cm
4
Wave Motion 13.21

EXERCISE - 4
4. Draw a sketch graph showing the two terms of
f = b l at lowest point
v
1. use of principal of superposition to find the final
m
1 mg waveform.
f= n
m
v1
f1 = at highest point
m1
1 ] M + mg g
f1 = n
m1

f1 = f: no any change in sourc frq
5.
1 mg 1 ] M + mg g
= ,
m n m n

& <m1 = m
M+m F

m di

2T sin 2 = dm.~2 R
Ey 4
2. tan a = E = 3 di m m~ 2 R 2
x 2T 2 = l Rdi.~2 R T= l
tan a = E = c m
Ey Ey
1 1

T T ~L
x 2
1
V ` = n = = ~R ` V = 2r
m

But tan a = tan a1 l
~L
6. VP/R = 2r
Ey 4 1

2 =3 ~L ~L ~L
VP./G - VR/G = 2r & VP/G - 2r = 2r
Ey = b 3 l units
8

VP/G = 2 b 2r l
1
~L

3. Mass per unit length of the string,
7. Due to Doppler effect apparent frequency of S1 will
10 -2
m = 0.4 continuously decreases. But apparent frequency of

= 2.5 # 10 -2 kg/m S2 changes to lower value when it crosses o so best
represented graph is.

` Velocity of wave in the string,

Ans (c)
1.6
V = T/m = 8. vsource
2.5 # 10 -2
V = 8 m/s (2r, 0)


For constructive interference between successive S

pulses: vobserver

2, ]2 g]0.4g
Dtmin = V = 8 = 0.10 s
Velocity of approach of observer to source decreases,

(After two reflections, the wave pulse is in same becomes zero and finally becomes velocity of
phase as it was produced since in one reflection separation. Hence apparent frequency continuously
its phase changes by r , and if a this moment next
identical pulse is produced, then constructive decreases. ^ fapp = f when vapp = 0 h
interference will be obtained.)
13.22 Physics
9.
= x 2 + 2 ] x + 1g

Solving v =- 5 m/s.
15. Given wave is produced by the superposition of the
following two waves
z1 = A sin ]kx - ~t g and z2 = A sin ^ ky - ~t h

Hence the direction of propagation is along 45º with
vsound V/ofu positive X-axis.
f = f0 v
sound - vsource V/ofu - V/ofu
16.
330 330
= 600. = 600. 330 - 20 cos 37
330 - v cos i

= 600. b 314 l - 630.5 Hz.


330

y = A sin ]kx - ωt g
10. Let the velocity of source at mean position is u
observer hear maximum frequency when source at 'P' , ]kx - ωt g
= π+ π3
approaching line from mean positions.
v = dt =- Aω cos ]kx - ωt g
dy
c

Lmax = c - u v

=- ]10mmg 2Tπ cos b π + π
3
l

and minimum frequency when source reseeding π ^ tj h
c
= 20
from mean position Lmax = c + u v
17.
velocity at mean position u = 2gd ]1 - cos ig

11. After a time t , velocity of observer V0 = at

` f' = b
V + V0 l
f =b V l f,
V + at
V which is a

straight line graph of positive slope. - δy


Vp =
δx V

12. Pressure amplitude = P0 cos 2 x + 20 3 =- ]tan 60cg V

& Vwave =- 20 cm/s.
max ^= ! P0 h at x = 0, 3 , 3 , 2 etc.
2 4
λ = 4cm = 1
T = 2.0 cm/s 5 sec .
V


So pipe is closed at x = 0 and also closed at other ω = T = 10π/ sec .

Wave is traveling along - ve x-axis.
end if length is 2m.
y = 4mm sin b10πt + 2 x + 4 l
π π

13. y ^ x, t = 2h = 10 cos b 5x l

Total energy carried per cycle.
y ^ x, t h = y ^ x - v ]t - 2g, t = 2h

= 12 m A 2 ω 2 λ
]t - 2g l
= 10 cos b x - 405
= 12 ^50 # 10 -3h^4 # 10 -3h
2


= 10 cos b 5x - 8t + 16 l
6]2πg2 5 2@^4 # 10 -2h
1
14. Here y =
= 1.6 # 10 5 J -

1 + (x - vt) 2
At t = 0.2 s, 1 + ] x - 0.2vg2
Wave Motion 13.23

T ^ yh = T1 + c my
T2 - T1
2π ]1 - cos θg = 2π d1 -
n
1
18.
h 2

'y' is the height from earth’s surface
power dissipated across total solid angle 4π is
0.10 ω .
Vsound ? T
2 -1
s1 = 4π 2π e o
0.1
Vground
power through
T1 2

& Vat y =
T1 c my
T2 - T1
= ]0.1ge
2 -1
h
o
2 2
T1 c my
T2 - T1
= 0.0146 ω .
]c g
h

& Vat y = T1
Pthrough s3 = Total power - Ps1 - Ps2

= 0.1 - 0.029
T1 + c my
T2 - T1
- dy
]c g
h
= 0.7 ω
dt = T1
P02
y=0
dy
t 21. I = 2PV
-c #
# dt
P02 = ]2.6g # ]330g
=
T1 + c 2 1 m y
y=x
T -T T1 0
&
h
P0 - 29.29 N/m 2 ^30 N/m 2h
19. 2πf
B = PV 2 ; K = V
P0 B A K & A = 1.1 # 10 5 m
-

µ alu = PA Sound level = 10 log I/I0


3 -6

= 2.6 # 10 # 10
= 120 dB
-3

= 2.6 # 10 22. ∆ = 2λ cos θ
4 -6
µ cop = 1.04 # 10 # 10
& λ = 2λ cos θ

= 1.04 # 10 2 -
cos θ = 1/2
n1 v1 n2 v2 x = D tan θ
2,1 = 2, 2

=D 3
T T
n1 n2
2.6 # 10
-3
1.04 # 10
-2
5 7
2 ]0.6g

& = 2 3 RT 5 5 RT
2, 2 23. =
2, MA 4, mB
n1 b 2 l =
3 2.6

& 10.4 n2 24. If tension increases, frequency increases.


& 3n1 = n2
` f - 280 = 10

& f = 290 Hz

` n1 = 1 & n2 = 3 (minimum possible values)
25.
]1 g 100
2.6 # 109 = 161.8 Hz
2 ]0.6g
f =

20. θ = 45c
V - V sin θ
f' = V 0+ 2V cos θ ; tan θ = 1 2
0

f' is constant and less than f0 .



Solid angle made by s1 at ‘P’ is
` ]b g is correct.

13.24 Physics

A ] xg = 4 sin b 15 l
26. πx
33.

A]x = 5cmg = 4 sin b 3 l = 2 3 cm


π
a

t1 = V - V
S 2 πx

at nodes, 15 = nπ
27.
x = 15n ^n = 1, 2, .....40h

v = 4 sin b 15 l - sin ]96πt g]96πg


πx


Let waves emitted from source at t1 sec reaches
observer at t = 10 sec. v ^ x = 7.5, t = 0.25h = 0

340 ^10 - t1 h = 2 ]10g t12


1 34. Use the principles of super position.

& t12 + 68t1 - 680 = 0
35. V α T

fopp = c V + 10t m f0
V f α T

1

T 1/2

f 21T p = 10

= 1349 Hz f
=
y = f ] x - Vt g
f + 15 T + 100 11
28.
2y 2y f
11 = 10f + 150
2t =- v 2x f = 150 Hz
22 y 22 y
2 = f" ] x - Vt g v 2 = v 2 2 36. From properties of standing waves.
2t 2x
37. In case of sound wave, y can represent presence and
29. y = 10 4 sin ]60t + 2xg
-
displacement, while in cae of an electromagnetic
k = 2 ; λ = πm wave it represent electric and magnetic fields.

30 38. y1 = A sin ^2πf + t - kx h


ω = 60; T = r/30 sec; f = r Hz
f
2π = ω
v = 30 m/ sec along - ve x-axis.
y2 = 3a sin ^2πf + t + kxh

30. Wave equation has to be in the form of f ]kx ! ωt g .
due to supeposition ^y = y1 + y2 h


only options (a) & (c) can be written in this form.
& y = ^ A sin ]ωt - kxg + 3A sin ]ωt + kxgh

31. At node, cos ]10πxg = 0
y = ]4A cos kx sin ωt - 2A sin kx cos ωt g
10πx = ]2n + 1gb 2 l

i.e. π

so at the point on the path average displacement is
1 3 5 zero.
x = 20 , 20 , 20 , ...
A1 sin φ = 2A sin kx

At antinode, cos ]10πxg = ! 1

A1 cos φ = 4A cos kx

x = 0.1, 0.2, 0.3, .....
A1 = 4A 2 sin 2 kx + 16A 2 cos 2 kx

λ = 0.2m ;
ω 50π
so A1 change with x (position)
V = k = 10π = 5 m/s.
39.
32. ^Vmax hparticle = 10
V

Aω = 1 m/s
1
ω = 103 ;
3
f = 2π # 10 Hz
A and B point will move in opposite direction

λ = 2π # 10 m
-2 displacement level of A and B will be equal.
Wave Motion 13.25
40. Since the edges are clamped, displacement of the 45. For constructive interference path difference ]∆S g
edges u ^ x, yh = 0
∆S = nλ ( n = 1, 2, 3, 4, ..... )
y

here ∆S = 2 m
(L,L)
(0, L) C B

So option (a) & (b) are satisfied for constructive
interference similarly for destructive interference

∆S = ]2n + 1g 2
λ

A
O x
so (d) option satisfied condition
(0,0) (L,0)


Line OA i.e. y = 0, 0#x#L 46.

AB i.e. x = L, 0#y#L
BC i.e. y = L, 0#x#L
OC i.e. x = 0, 0#y#L


The above conditions are satisfied only in alternatives
] Bg and ]C g .

Wave emitted from Q y = A sin _ωt - kxQ i
Note that u ^ x, yh = 0 , for all four values e.g. in
y = A sin b ωt - kx p + 2 l
π
alternative ] Dg, u ^ x, yh = 0 for y = 0, y = L but
Wave emitted from P
is not zero for x = 0 or x = L . similarly in option
∆φ = φ P - φ Q = ωt - kxP + π _ωt - kxQ i
2-
(a), u ^ x, yh = 0 at x = L , y = L but it is not zero
= k _ xQ - xP i + π
2
for x = 0 or y = 0 , while in option (b) and (c),
u ^ x, yh = 0 for x = 0, y = 0, x = L and y = L = 2 π _ x - xP i + π
λ Q 2
41. , = 1m f = 10 sin ]80 r t - 4rxg
r ^x - x h r
= 10 Q p +
2

For superimposed second wave is
For A xQ - xP =- 5
f2 = 10 sin ]80r t + 4rxg

& ∆φ = 10 ]- 5g + 2 = 0
π π


Amplitude of stationary wave = 2a = 2 # 10 = 20m
2r
For B xQ - xP = 5
K = 4r =
m
2r 1
& ∆φ = π
m = = = 0.5 m
4m 2

For C xQ - xP = 0
, = 1m
π

& ∆φ = 2
], = 2mg

I A IB: IC = ] I + I + 2I cos 0g : ] I + I + 2I cos πg
:

: b I + I + 2I cos π
2
l

Total ] N = 5g
47. When sound wave is reflected from rigid end
displacement wave get extra phase at π and pressure
42. For monochromatic plane wave polarization does wave get no extra phase So option (a), (b), (c) are
not require: correct
43. At closed end displacement node and pressure 48. Energy per unit area associated with progressive
antinode are formed
sound wave I = 2π 2 a 2 n 2 sV if we increase
44. For stable interference, phase difference should not amplitude to 2 times or frequency to 2 times
vary with time. Hence waves should have same
I will be doubled. So increment 41% or either
frequency and a constant phase difference, same
wavelength. amplitude of frequency
13.26 Physics
49. Fundamental frequency of V - Vs V - V/5 4V
54. For observer O1 , λ1 = f = f = 5f
V V

open pipe, n0 = 2, closed pipe, nC = 4,
1 2
For O2, there is change of medium hence at the
V V surface of water, keeping frequency unchanged
2,1 - 4, 2 = 5 .....(1)
V = 4V

V 3V λa λw

For first overtone n0 = ,
nC = 4,
1 2 16V λW 4V

& λ w = 4λ a = 5f
V - 3V =
,1 4, 2 5 .....(2)
velocity of wave relative to observer
, 1 f'' =
on solving (1) and (2) ,1 = , 2 & ,1 = 1
λw
2
21V 5f 21f
, = 5 . 16V = 16
50. Making hole at 3 length from close end. Pipe start
,
behaving as closed pipe of length 3 55. For a plane wave intensity ( energy crossing per unit
V area per unit time) is constant at all points.

So new fundamental frequency n' = 4,/3
V
For plane wavefront 7

original fundamental frequency n = 4, n' = 3n
1 2

So option (b) and (d) are correct
I11 = I22
λ
51. x = 4
1 2
V 340
But for a spherical wave, intensity at a distance r

λ = n = 340 = 1 m
from a point source of power P ( energy transmitted
1 per unit time ) is given by

x = 4 m = 25 cm


For other resonance position

x = ]2n - 1g 4
s
λ



so x = 25, 75, 125 cm...
P 1
I = or I ? 2

Tube length is only 120 cm so get resonance 4πr 2 r
minimum length of water = 120 - 75 = 45 1

" For a line source I ? r

Distance between two successes nodes
λ 100 P
= 2 = 2 = 50
Because I = 2πrL

52. By the concept of doppler effect 56. n ? v/,


Apparent frequency (no. of waves )
C+u
To increase fundamental resonance frequency of
striking the wall υ = C υ tube.

Frequency of reflected wave (wall
Velocity of sound in hydrogen can be increased.
C+u
Length of tube can be decreased.
become source) υ'' = C–u υ

For tube closed from one end fundamental frequency
C C (C–u) is v/4,

Apparent wavelength υ'' = υ (C + u)

For tube open from both ends fundamental frequency

So option (a), (b), (c) are correct is v/2,
53. High pitch means apparent frequency is increased in
reflected wave so there is relative motion between
girl and reflection and it is of approaching nature so
(c) and (d) is correct.
Wave Motion 13.27
57. Total energy
vt = 10 cm
= 12 µω 2 A 2 = 2π 2 fn2 µA 2 = 2π 2 n 2 f12 µA 2

as ]vt - 4g < ] x = 7 cmg < vt

y = 4 ]7 - 10g + 1 = 4 cm = 0.25 cm

( f1 = Fundamental frequency) 1 1

fn = Frequency of nth harmonic δy
64. Transverse velocity =
TE δt
KE
< >=< PE >= 2

At t = 0.015 s, vt = 15 cm
58. V = A cos ]ωt - kxg As for x = 13 cm ]vt - 4g < x < vt


` _φ P - φQ imin = π
Therefore

^φ P - φ R hmin = 2π δy v
=- 4 =- 250 cm/s
δt
V 300
59. λ = f = 25 = 12 m 65. Medium of string BC is denser. These will be π
φ A - φ B = 6m phase difference between reflected and transmitted
wave.
∆ max = 2A (when both have same velocity.
λr v µ2

= 5 # 10 5 m - 66. = v1 = µ1
λt 2

60. In the case of open organ pipe vibrating in µ1 4 λ 3


fundamental mode, end points are displacement
Now µ2 = 9 so r = 2
λt
antinode (pressure nodes) and mid point is v2 - v1
displacement node (pressure anti-node). Ar v1 + v2 Ai v -v
67. At = 2v2 = 22v 1 = 14
2
61. Let a = initial amplitude due to S1 and S2 each. v1 + v2 .Ai
I0 = k ^4a 2h, where k is a constant

 v 

After reduction of power of S1 , amplitude due to f1 = f0  
 v − v1 
S1 = 0.6 a 68.
Submarine v1

Due to superposition f0
f1

amax = a + 0.6a = 1.6a, and
Island

amin = a - 0.6a = 0.4a
v+v f"–f
Imax /Imin = ^amax /amin h = ]1.6a/0.4ag = 16

2 f" = f0 c v - v1 m, v = 1050 ; f o = 0.1E
1 o

62. Shape of the pulse at t = 0 y(cm)


f"–fo 2V1
1 fo = V–V1 = 0.1

That is a triangular pulse
–4 0 1
x(cm) v1 = 50 m/ sec

Area of the pulse
69.
= 2 6]4 # 1g + ]1 # 1g@
1 Indian 50m/s
v2 Enemy ship
marine f0
5
f ' = f0 a v - 502 k
= 2 cm 2 v+v
f " = f ' a v - v k
v + 50

2
T = (v + v ) (v + 50)
63. v = µ 10 m/s. f " = f0 c (v - v2) (v - 50) m = 1.21f0

Solution of the wave equation that gives displacement 2

of any piece of the string at any time


[21% greater then sent waves]
]Z] (x - vt)
]] + 1 for vt - 4 1 x # vt
get v2 = 50 m/ sec toward Indian submarine
] 4
[
]
y = f (x,t) = ]- (x - vt) + 1 for vt 1 x 1 vt + 1
]]
] 0 otherwise
\

Using v = 1000 cm/s, t = 0.01 s
13.28 Physics

v wrt to observer = v + v2 v - 50 75. End correction = ]0.3g d = 1 cm


70. λ' = f´ (v + v2) = f0
f0 (v - 50) 10
d = 3 cm
(v - v2) (v - 50)
vol. of tube = b π d l , = 4 b 10 l # 100cm3
v + 50 2
π 2

λ" = (v + v2) (v + 50) = f0 (v + v2) 3
4
f0 (v - v ) (v - 50)
2

(Take I = 0.99 m . 1 m )
λ' v+v 1050 + 50
= v - v2 = 1050 - 50 = 1.1 10π
= 36 lit
λ´´ 2

B B 9 2
10π
71. v = ρ & 1050 = 1000 B . 10 N/m
Moles = 36 # 22.4 moles (22.4 lt. contains 1 mole

72. ξ = ]0.1 mmg cos 80 ^ y + 1cm h cos 2π 7 ]400g t


2π 10π 10π
36 lt contain 36 # 22.4 mole)

End correction is 1 cm, so at y =- 1 cm
76.
ξ = ]0.1mmg cos 80 ]- 1cm + 1cmg

= ]0.1mmg cos ]0 g = antinode



So upper end is open at lower end

Linear momentum of (man+plank) system is

y = 99 cm conserved.
ξ = ]0.1 mmg cos 80 ]99 + 1g

150V + 50 ]V - 8g = 0

5π V = 2 m/s.
= 0.01 cos 2 = 0 & Node
f' = 330 - 6 ^ f0 h
330 + 2

tube is closed at lower end

324 ^ 0 h
= 332

So tube is open closed.
f

2π 2π 77. When the man jumps off the plank.


73. 80 = So λ = 80
λ

f' = 330 + 6 ^ f0 h

and effective length of air column = 99 + 1 = 100 cm 330 - 2


So
, =5 = λ , so five half loops will be = 328
λ 4 & 1 54 open

336 f0
formed 78. Graph given in option (a) satisfy the frequencies
determined in earlier two questions.
, = 5 b λ l
4 so second overtone. closed


74. Pex =- B dx

= ^5 # 105h # ^0.1 # 10 -3h 280 sin 80 ^ y + 1cm h


π 2π

cos 2π ]400g t

π ^ y + 1cm h
= ^125 π N/m 2h sin 280
cos 2π ]400t g
Wave Motion 13.29
79. (a) 80. Use base fundamentals of modes of vibration of a
& A sin ]ωt - kx + 0g
string.

81. (a) y = 4 sin ]5x - 4t g + 3 cos ]4t - 5x + πg
is superposition of two waves having same
ω and k . Resultant is wave with individual
particles undergoing SHM.

(b)
(b) y = 10 sin bt - 330
x l
cos 100 bt - 130 l .
x
& A sin b ωt - kx + 2 l
π


= A cos ]ωt - kxg represents superposition waves of different
frequencies.

= A cos ]kx - ωt g

(c) It is a standing wave.

(c)

(d) Similar to (b)
& A sin ]ωt - kx + πg 82. In the case of tube closed at one end.
]24 + 1g

= A sin ]kx - ωt g f= 4 v = ]2n + 1g 80


= 80, 240, 40, 560.

(d)
83. Use different cases of doppler effect in theory.
& A sin b ωt - kx + 2 l


=- A cos ]ωt - kxg


=- A cos ]kx - ωt g

Numerical Type f
= 1 - f0 = 684 - 676 = 8 Hz.
2v
1. f1 = 2l B
1 3. V =
v t
f2 = 4l
2 B
Now, f1 = f2
Vpipe 2t 1
Vair = =
2v v B 2

& 2l = 4l t
1 2
& l1 = 4l2 = 80 cm
Vair

Vpipe =
2. Given, 2
The speed of each can approaching each other (n + 1) Vpipe
fn = 2,
Vcar = 7.2 Km/hr = 2 m/s
The frequency of the horn blown by each car Vpipe 300
f1 - f0 = 2, =
f0 = 676 Hz. 2 2
The velocity of the sound in the air, V = 340 m/s.
= 105.75 Hz (If 2 = 1.41)
Each driver hears 2 sounds: (1) Sound emitted by = 106.05 Hz (If 2 = 1.414)
his/her own car (2) Sound coming from another car.
Single Option Correct
According to Doppler effect, Frequency of sound,
f0 = c m fs
v + v0
coming from car-2, heard by car-1 is, 4.
v
f1 = f0 b 340 - 2 l
340 + 2 v
f0 = f v + 5 p fs
v
& f1 = 676 b 338 l = 384 Hz
342
6
f0 = 5 fs
& The number fo beats observed by each driver
13.30 Physics

8. Imax = 1 or b A1 - A2 l = 1
f0 - fs I 16 A + A 2 16

% change = # 100
fs min 1 2

1 A1 + A2 = 4A1 – 4A2
= 5 # 100 = 20%

A1 5 I1 25
5. The diameter of the resonance column tube d = 6 3A1 = 5A2 or = ` =
cm. A2 3 I2 9
The frequency of the tuning fork f = 504 Hz. 25
9. l = 50 cm = 0.5 m, vp = 10 km/ h = 9 m/s
The speed of the sound v = 336 m/s.
& The end correction e = 0.3d = 1.8 cm v = 330 m/s
Let, the reading of the water level at first resonance
Frequency of second harmonic produced by an open
= l. flute,
In the resonating tube, the end correction occurs v 330
y = 2 # 2l = 0.5 = 660Hz
because the reflected wave goes a little above the
mouth of the resonating wave. Hence, the length of Frequency heard by the person
the resonating tube becomes the original length of 25
the resonating tube plus the end correction. v + v0 330 + 9

y' = v y = 330 # 660 . 666Hz
m V
l & + e = 4 = 4f
10. Let f1 and f2 be the frequency registered by the
336 observer in two cases.
& l + e = 4 # 504 = 0.16666 m = 16.66 cm
& l + 1.8 = 16.66 cm v+v 340 + 0 340
f1 = f v - v 0 = f 340 - 34 = 340 - 34 f
& l = 16.66 - 1.8 cm S
340 - 0
f2 = f 340 - 17

& l = 14.866 cm.
f 340 - 17 19
T The required ratio f1 = 340 - 34 = 18
6. v = n 2

11. The velocity of wave in the string is


T
& = nv 2
nv 2 T
3, m/l

& A =Y ,
So, the distance between successive nodes is
nv 2 ,

& 3 , = AY m 1 v 1 T
2 = 2 y = 2y m/l

after substituting value of n, v, ,, A and Y we get 1 8#1
= 2 # 100
n, v, ,, A and Y 5 # 10 -3

O, = 0.03mm = 0.2 m = 20 cm

12. v = gx
T1 Mg
7. Case I : v1 = n = n dx =
dt gx
T2 M a2 + g2 20 dx = # t
Case II : v2 = = n = n #0 x 0
g dt
v1 g
v2 = 2 # x1/2 dx = g t
_g 2 + a2 i
1/2

1/4
2 x = gt
b v1 l = e o
g2 + a2 1 a2
v2 . 1+ 4 2 2 20 = 10 t
g2 g
60.5 1 a2 0.5 1 a2 2 2 =t
60 = 1 + 4 2 ; 1 + 60 = 1 + 4 2
g g
13. % Change in frequency
a2 1 g g
2 = 30 or a = .5 V V
g 30 V - Vs - V + Vs
= V # 100
V - Vs
Wave Motion 13.31
2V 16. The transverse displacement y ^ x, t h of a wave on a

= V + sV # 100
s
string is given by
2 # 20 #
+ 20 100 = 12% _ 2 + bt2 + 2 ab xt i
y ^ x, t h = e
= 320 - ax

14. For closed organ pipe - (ax + bt) 2

It is transverse type y (x, t) = e
^ 2n + 1 h v
f = 4, , ^n = 0, 1, 2...h b

Speed v =
^2n + 1h v a
4, < 1250
and wave is moving along - x direction
]2n + 1g < 1250 #
4 # 0.85 - [ a x + b t] 2
340
y (x, t) = e
]2n + 1g < 12.5
17. Y = A sin ]ωt - kxg + A sin ]ωt + kxg
2n < 11.50

Y = 2A sin ωt cos kx standing wave
n < 5.25

For nodes cos kx = 0

So n = 0, 1, 2, 3....5
.x = ]2n + 1g 2
2π π


So we have 6 possibilities. λ

v 1 T 1 T ]2n + 1gλ
15. f = = = x = 4 , n = 0, 1, 2, 3, .....
2, 2, n 2, Ad
T YD, 1 yD, 18. By equation
T,

Also Y = A∆t & = & f=
A , 2, ,d 1
f = 0.04 and λ = 0.5
D,
, = 1.5m, = 0.01, d = 7.7 # 103 kg/m3 1 25
,
& V = 0.04 # 0.5 = 2

V = µ & b 2 l
y = 2.2 # 1011 N/m 2 T 25 2

by

After solving

= 0.T04 & T = 625 4 # 0.04
2 103
f = 7 # 3 T = 6.25 N
f . 178.2 Hz

1 γRT ∆f ∆, 2. V = ω A2 - y2
1. f = 4, M & f = ,
-3 VP = 2πf A 2 - y 2

(a) M = 20 # 10 f = 320 Hz ∆f = ! 4.5 Hz

VP = 2π a k A 2 - y 2
Not possible V
λ
(b) M = 20 # 10 3
f = 253 Hz ∆f = ! 3.6 Hz
-

Not possible

= 02.π5 # 0.1 (0.1) 2 - (0.05) 2
(c) M = 32 # 10 3
f = 237 Hz ∆f = ! 3.4 Hz
-

Not possible 3π
VP = 50 j m/s
(d) m = 36 # 10 3
f = 242.8 Hz ∆f = ! 3.5 Hz
-

Possible
13.32 Physics
3. As string and tube are in resource f1 = f2 tOA
n O
dy
O

f1 - n = 4 Hz
#
dt =- g #
O L M + ny
P

2
tOA = 7 M + nL - M A y
ng
A

For pulse 2. (From A " O ) M

Tension at P is ^ M + ny h g


When T increases, f1 also increases. It is given that
T ^ M + ny h g dy
beat frequency decreases to 2 Hz. υ = = =
n n dt

& n - f1 = 4 t AO L
dy
n
n = 4 + f1
#
dt = g #
O O M + ny

as f1 = f2
On simplifying as above
n = 4 + f2
` t AO = tOA
3V 3 # 340
Choice (a) is correct
f2 = 4, = # ] g = 340
4 3/4
n = 344 ^ M + ny h g
υ =
4. For resonance, n

v - u # fs = ]oddg # f0
v L

At y = 2 (Midpoint)
v
(a) v - 0.8v # f0 = 5f0
v L
(b) v - 0.8v # 2f0 = 10f0 cM + n mg
2
(c) v - 0.8v # c m = 2 f0
v f0 5 υ=
n
2
(d) v - 0.5v # c m = 3f0
v 3f0 L

At y = 2 (Midpoint)
2
u
L
fs cM + n mg
2
f0 v= is same for both
n
5. ]2n + 1g 4 = 50.7 cm + e
λ the pulses at the midpoint. Choice (b) is also correct

frequency of the pulse remains same throughout the
string. For pulse 1
]2n + 3g λ
4 = 83.9 cm + e υ0 = υ A


using n = 1 e =- 0.9 cm vO v
= A
λO λA

λ = 66.4 cm vO λ
= O
vA λA

v = fλ = 332 m/s.
^ M + µy h g

Now v= µ
6. For pulse 1 (from O to A)

Since v0 > v A
Mass
Tension at P is ^ M + ny h g where n =
λ0 > λ A
length

Choice (c) is wrong. As seen from the expression
T ^ M + ny h g - dy of v, velocity is independent of its frequency and
y = = = wavelength choice (d) is also correct.
n n dt
Wave Motion 13.33
7. V = 100 m/s 10. At open end phase of pressure wave charge by p so

Possible modes of compression returns as rarefraction. While at closed
vibration end phase of pressure wave does not change so
compression return as compression.
, = ]2n + 1g 4
λ

11. (a) The intensity of sound decreases with increases
12
λ = ] + gm
of order. The intensity of
2n 1
sound is maximum for first
2π = 2π (2n + 1) π

k = resonance.
λ 12/(2n + 1) = 6
r ]2n + 1g 50r
~ = vk = 100 ]2n + 1g 6 = 3
(b) The prongs vibrate in
vertical plane.
π 50π
if n=0 k= 6 ω= 3
5π 250π
(c) The prongs does not vibrate in amplitude of
n=1 k= 6 ω= 3
that order.

n=7 k= 2 ω = 250π

(d) Consider end correction, the length of air
8. (a) There are 5 complete loops column is slightly less than λ /4
Total number of nodes = 6
(b) ω = 628 sec 1

-
So ans are (a) and (a)
2π = 1 12. VS/A = 340 + 20 = 360 20 30
k = 62.8 m 1 =

-
λ & λ 10
A B
ω 628
vw = k = 62.8 = 10 ms 1

-

VS/B = 340 - 30 = 310 340 340
20


L = 2 = 0.25
13. Because

(c) 2A = 0.01 = maximum amplitude

VS = V0 = 20

of antinode
v 10
so as seen from passengers of train A.

(d) f = 2, = 2 # 0.25 = 20 Hz
'
f = f0
9.
14. As seen from B
Intensity

f ' min = a 340 - 20 k(800)


340 - 30

f´1 = f1 f´2 = f2

If wind blows from source to observer frequency

f max = a 340 - 20 k(1120)


' 340 - 30
= c (v + w) - u m f1
(v + w) + u
f2
f ' max - f ' min = 310

& f2 > f1

If wind blows from observer to source

= c (v - w) - u m f1
(v - w) + u
f2


& f2 > f1
13.34 Physics

1 k = 2I0 + 2I0
cos 60c
15. f ? &f= .....(1)
,1 ,1
= 3I0
( ,1 & initial length of pipe)
n=3
V k
e
of = {VT speed of tuning fork, , 2 "
V - VT ,2 19. Let speed of cars are V1 and V2

new length of pipe} .....(2)


frequency received by car f1 = c v - v m f0
v

1
(1)
v+v v+v
(2) ∆f = f2 ' - f2 = c v - v2 - v - v1 m f0

2 1
V - VT ,2
= (v + v2) (v - v1) - (v + v1) (v - v2)
V ,1 ∆f = c
(v - v1) (v - v2)
m f0
,2 V - VT
-1 = -1 2v (v - v1) f0 2 (v2 - v1) f0
,1 V ∆f = (v - v2) (v -
.
1 v2) v
, 2 - ,1 -2
# 100 = # 100 =- 0.625 2 (v2 - v1) f0 1.2
,1 320
Given = 100 f0
v

Therefore smallest value of percentage change
& v2 - v1 = 7.126
required in the length of pipe is 0.625

Answer in nearest integer is 7
16. . λ =
20. (a)
4 L , λ = 4L
Sound waves are longitudinal waves
λ =

(b)2 L , λ = 2L
Sound waves are longitudinal waves
λ =

(c)2 L , λ = 2L
String waves are transverse waves
f1 = f b v - 2 cos θ l f2 = f b v + cos θ l
v v


(d) λ=L

Beat frequency = f1 - f2 String waves are transverse waves
fvx3 cos θ 2f cos θ
= ]v - 2 cos θg]v + cos θg .
f =5

17. Frequency received by the source

p = 492 a 330
330 - 2 k = 498 Hz
v + vO
= f0 f
+2

v - vS

` Beat frequency = 498 - 492 = 6 Hz

18.

I = I1 + I2 + 2 I1 I2 cos φ
Mechanical Properties of Matter

14
14.1

Mechanical Properties
Chapter
of Matter
INTEXT EXERCISE: 1
1. Young’s modulus depends only on nature of the 8. If whole system is accelerating upwards with
material. acceleration 2m/ sec 2 , tension in the string is
5 ^ g + ah
2. Work done = ∆P $ E (considering no loss of energy
in the form of heat) Stress
Strain = y
= 12 b F l b1 l
2
A $ Y $A, 5 ]12g /0.5 # 10
-6

= 11
2 # 10
∆P $ E1 ,1 1
∆P $ E2 = , 2 = 2
= 6 # 10 4 -

3. Energy per unit volume = 2 ]Stressg # ]Straing


1
9. P $ E stored in the wire due to it’s own weight

E = 2 Y ]Straing2
1 M g L
2 2

= 6 AY
2E

& Strain = Y ρ 2 A 2 L3 g 2 ρ 2 AL2 g 2

= 6 AY = 6Y
4. Total elongation in the wire due to it’s own weight
10. Restoring force generated in AB part is 5N, TBC
MgL = 2N , TCD = 1N
]e g = 2AY ; M = dAL

2
dgL

= 2Y

2

` e ? dL

3W
5. TP = 4 + W1

T
Stress = AP

3W +
W1

= 4 S 5 ]0.6g 1
e1 = .
10 3 8 # 10 2
-

6. Material possessing higher Young’s modulus


exhibits more elasticity. 2 ]1 g 1
e2 =
10 3 8 # 10 2
-

Stress 1
Y = Strain = Slope of given graph 1 ]1.2g 1
e3 =
^10 -3h 8 # 10 2

As R has minimum slope, it exhibits more elasticity.
- ]6.2g
1
7. As stress is same in both the wires, etotal =
8 # 10 1
T1 T2
A1 = A2
= 54 ]6.2g
T1 A1 1

& T2 = A2 = 2 = 0.5
= 7.5
14.2 Physics
11. F b - Ll

= AY L 2

T ] xg = F b1 - Lx l
= 2FL
AY

de = AY b1 - 2 l dx
F x
e F
Strain = total
2 = 2AY
L

etotal = # de
0

INTEXT EXERCISE: 2
Y 5
η = ]1 + σg = 2 0.110 # 100
1. #
2

0.5Y
= 2 # 108 N/m 2
σ = η - 1
1
0.5Y - η 7. C = B

= η
]dv/vg " Fractiοnal change in vοlume
3. Shearing strain = θ ]= tan θg

= dP
" Change in pressure
` C means fractional change in volume

= ∆,x per unit change in pressure.

B =- b AV/V l
5 - ∆P
= 2 10 8.
#

-1
10
=+ c 100 # 10 N # 100 m
5
= 2 # 10 4
]0.01g

-

4. Let φ is angle of shear.



= 1011 N/m 2 = 1 # 1012 dyne/cm 2

Then Lφ = rθ
∆V
9. Bulk Strain = V ; V = x3
]100g φ = ]0.4g # 30c
x - ] x - ∆xg3
& 3

= 3
x
φ = 12 # 10 2
-
&
x ]a ∆x << xg
= 3∆ x

= 3 ]0.01g

= 0.12c

5. Free fluids can’t sustain longitudinal and shear - ∆P


10. Bulk modulus = ]∆V/V g
stress.

- dP +^103h]9.8g]200g

]0.1g
B =] = # 100
dv/vg
6. ; dP = ρgh

= 2 # 105 N/m 2

= 19.6 # 108 N/m 2

INTEXT EXERCISE: 3
1. Surface tension refers to work done by external 3. Liquid surface doesn’t wet the solid if contact angle
agent in increasing the surface area slowly by one is greater than 90c as cohesive forces dominate the
unit. Thereby this process is nearly isothermal. adhesive forces.

2. Water raise to the height of 12 cm only at the top


contact angle adjusts accordingly.
Mechanical Properties of Matter 14.3
4. 2T, = mg 2T
10. Pin = Pout + R

2T, = ^10 -3h]9.8g = P0 + dgh + 2RT



4
98 # 10 4 = 0.98 =
-
11. n 3 πr3 = R3
7 cm
2 ^70 # 10 -3h
& , = 14
R = n1/3 r

]S $ Ag f - ]S $ Agi = n $ ^4πr 2h S - S ^4πR 2h


4 3 4 3
5. 3 πR = 125 3 πr

& R = 5r
= ^4πr 2 n - 4πR 2h S

Work done = T ^S2 - S1 h 12. If PP = PQ


= ^75 dyne/cm h^125 ]4πg r 2 - 4πR 2 h
Then water does’t flow outside.
2T

= 74 erg. P0 - r + ρgh = P0

6. Liquid in the capillary raises if contact angle is 2T =



& r ρgh
acute.
2 ]70g

& -2 = h ]980g
7. If water droplets combine to form a larger drop, 2 # 10
energy is released. 10
2

& 14 =h ; h = 7.14 cm
1
8. h ? g
13.

` Capillary rise on moon is 6h

4T = c 4T m
9.r1 3 r
2
r2
r1 = 3
P0 - r + ρg ]3.6g = P0 - r
2T 2T
4π 4π
V1 = 3 r13 & V2 = 3 r23 1 2

d = 8 # 10 3 m

-
V1 : V2 = r13 : r23


= r13 : 27r13 14. Surface of the liquid is concave if cohesive forces
are greater than adhesive forces.

= 1 : 27
INTEXT EXERCISE: 4

Vt = qη r 2 ^σ - ρh g
2 dv
1. 4. Velocity gradient = dx

Vt ? r 2

= 0.05 -m3 /s

As radius of the lead sphere increases by 2 times 10
terminal velocity increases by 4 times.
= 50 s -1
dv πρr 4
2. F =- ηA dx
5. Rate of flow = 8η,
F = ^20 poiseh^10cm 2hb 1cm0/.1sec . l ` Rate of flow increases, if radius of the glass tube

= 2000 dyne increases.

dv = m/s 6. Velocity of the body increases initially and finally


3. dy m
becomes constant.

=s 1 -
14.4 Physics
2
7. Vt ? r 12. V ? r 2

? r ba m l
m V
r ?r
2

` is constant.
r2
mg
8. V ? ηr
13. When body attains terminal velocity.
9. Curve C represents velocity profile of the body as a
function of distance. FB + FV = mg
10. Rain drops finally fall with some terminal velocity,
FV = mg - c d d2 g m
which is proportional to square of radius of the body. M

&
1
11. After travelling large distance h , particle attains
d
terminal speed.
= mg d1 - d2 n
1

` Final velocity is almost independent of h

EXERCISE - 1
1. d = 4mm 1

For same load rα
10 2 ∆l

Y = 9 # 10 N/m
F ∆l ∆l
F = ∆l 4. A = Y l if Y & l are constant
A Y l
= π ^2 # 10 -3h # 9 # 109 # 100
1 ∆l
2

F = AY l & F \ A & F' = 4F
= π # 4 # 10 -6 # 9 # 10 7

5. Area = 1cm2

= 360 π N

∆l = 1.1 l - l
2. ∆lwater = LW
L ∆ air = La Y = 2 # 1011

W

F = ∆l
A Y l
ρ
;W - ρ ρW EL W <1 - ρW FL
W
F = AY b l l = 2 # 106 N
WL 0.1l

La = YA Lw =
0
=
0

YA YA
La ρW ρ0 La Y A ]∆Lg
LW = <1 - ρ0 F & ρW = La - LW 6. L = F ; ∆L = 2π ]R - rg

YA ]R - rg
F/A
3. ∆l/l = Y

& r =F
2
F = Yπr
∆l l 3W
7. TP = 4 + W1
Fl # 1 = 2
& Y π ∆l r
& Y & l are same for all then
T
Stress = AP

3W +
W1

= 4 s


Mechanical Properties of Matter 14.5
8. g

& , = 2

11. Tension in both the strings

10KN
T = 2 = 5000 N

T,
F YA ∆, = AY

From graph, ∆, = Slope L
^5 # 10 h^1500 # 10 3h 3 -

As tan θ 2 > tan θ1

^103h^10 -6h 103 # 106
=
A2 > A1 (since Y & L are same. )

= 75 # 10 -4 m

i.e. Crosssectional area of B is greater than cross
sectional area of A
= 7.5mm (Elongation in the wire)

9. A, = A',' 7.5mm =

` Pulley comes down by 2 3.75 mm
A, = A' ]2,g
12. Volume of wire is
A
A' = 2
V = L # rr2 = 1 # r ^10 -3h2 = r # 10 -6 m3
&

r

& r' =
2
Area of square cross-section
∆ , ,

& Strain = , = , = 1; = ^2 # 10 -3h2 4 # 10 -6 m2

Stress Volume r # 10 -6 r
Y = Strain & Y = stress
Length of new wire = Area = 4 # 10 -6 = 4 m
Elastic energy per unit volume = 2 ]Stressg]Straing
1
Initially extension is

V = 2 ]Y g
1
x = AY = Y c # -6 m
FL F 1

r 10
Y =2 ` Y = ^r # 10 -6h x
& F


` Option (a) is wrong

Finally extension is
10. Considering mass of the wire is FL' F L'
negligible.
x' = A'Y = Y . A'

T = mg = ^r # 10 -6h x.
r/ 4 r2

or x' -6 = 16 x
4 # 10
T,
Y = A∆, 13. Extension in the first case is
Ya∆, FL wL

& T = ,
l = AY = AY

When the mass is made to rotate along circular path

Extension in the second part is
in horizontal plane.
T' = m,ω 2
w ^L/2h w ^L/2h wL

= m, ]4πg2
l' = AY + AY = AY

Y A ]∆,g'

It is clear that l' = l
T' = ,

Note: In the above problem,
T' = ]∆,g ' even if lengths of wire are
T ∆, unequal on two sides of the
, ]4πg2 pully, the elongation will still be 1.
g =4 ; ∆,' = 4cm & ∆, = 1 cm
14.6 Physics
14. V = 1/2K ]2g2 3
& B = 200 # 10 # 10 # 1000 = 2 # 10
9

V' = 1/2K ]10g2


∆v
23. Bulk strain = v

then V' = 25V
∆v = ∆L
r v = L3
& v 3 L
1
15. r1 = 2
2 ∆v ∆v =

& v = 3 # 0.02 & 0.06
PE ^per unit volumeh = 2Y b A l
1 F 2 v

Dp
PE \ 1/A 2
24. B = v; K = 1/B;
Dv
2
PE1 A2 4 # 10 -5
PE2 = A12 = 16: 1
K= = 4 # 10 -10 m 2 /N
5
10
16. Total energy store in the wire
1

B= N/m 2
= 12 ]Stressg]Straing]Volumeg r # 10 -10
5 2

∆p = 100 # 1 # 10 N/m ; v = 100 c.c.
= 12 F b ∆, l
A $ , $ A, Dp

Dv = v = 0.4 cc
B
= 12 F ∆,
25. Volume elasticity is possessed by all three types of
= 12 ]200g^10 -3h = 0.1 J matter owing to intermolecular interactions.

- dp
17. P $ E = 2 b A l $ b ∆,, l $ ] A,g
1 F B =]
dv/vg
26.

- ]0.155g # 105
= 12 F2 ]∆,gb ∆,, = AY
F l
2

- ]1/10g

& B =
A Y
M g ,
2 2
= 1.55 # 105 N/m 2
= 2 AY
dp
18. Elastic forces are conservative when loading and 27. B =
dv/v
unloading curves coincides even when they are not dp v
linear as in case of rubber. i.e. elastic forces can be dv = B
conservative or non-conservative.
v ]α∆T g = B
Pv

&
19. U = 2 σε = 2 ]Yεg ε
1 1
P

& ∆T = Bα
1 2
& U = 2 Yε is similar to x = ky 2
210g 210 # 10 3
-
which is a parabola passing through origin and 28. vf = 3 = 2
symmetric about X-axis 3500 kg/m 35 # 10

x 0.02 # 10
-2

= 6 # 10 5 m3 = 60 cc -
20. F = ηA h = 0.4 # 1011 # 1 # 0.005 # 1
4
= 4 # 10 N ∆p
B = ∆v/v ; v = v f + ∆v
5
DV DP 1 # 10 v f + ∆v
21. # -7
V = B = 1.25 # 1011 = 8 10
& ∆v = ∆Bp ; ∆v is magnitude of
22. Depth = 200 m
change in volume.

∆V = 0.1 = -3 vf 6 # 1011 =
V 100 10
& a + ∆v = 30
2 # 1010
Density = 1 # 103
vf

& ∆v = 29

g = 10

& ∆v - 2cc
∆p hgρ

B = ∆v/v = ∆v/v
` vinitial = 62 cc.
Mechanical Properties of Matter 14.7
- dv P m - dv dρ 43. When charge is given to a soap bubble (whether
29. v = B & v= P & v = ρ positive or negative), these charges experience
dρ P repulsive forces due to the other charges. Hence they
& ρ = B tend to move out. Hence the size of bubble increases.
ρP
& dρ = K 44. Contact angle is independent of orientation of the
tube.
30. Surface tension is a property based on intermolecular
force, at critical temperature intermolecular force is 4T 4 # 30
45. Pexcess = r = 0.4 = 300 Dyne/cm 2
zero, hence surface tension is zero.
31. Cohesive force > adhesive force 46. As total surface Area of the decreases, surface
energy decreases. Thereby energy is liberated in this

i is obtuse. process.
32. f = 2πrT = 2π # 5 # 75 = 75 0π dyne 2T
47. p = pa + hρg + r = 1.07 # 105 N/m 2
33. f = 2 Tl
48. In the satellite, geff becomes zero but the surface
2 # 10 2 = 2 # T # 0.10

-
tension still prevails. Hence the water will experience

T = 0.1 N/m only surface Tension forc which will push it fully
outward.
34. The property fo surface tension try to decrease the
surface area of the liquid. 49. Water will rise to a height more than h when
downward force ^mgeff h becomes lesser than mg .
35. mg = 2rr1 T + 2rr2 T
so in a lift accelerating downwards, geff is ^ g - a0 h .

7.48 # 9.8 # 10 -3 = 2r ^17 # 10 -2h T
Hence capillary rise is more

T = 70 # 10 -3 N/m

On the poles geff is even more than g . hence the
37. f = 2 # 2πrT = 2 # πdT = 6πT capillary will even drop.

38. We know that surface energy 50. Insects use the surface tension force to keep floating.

Us = T # Area 51. For an acute angle of contact, liquid rises.

Here, as 2 films are formed because of ring, so 2T cos θ
52. h =ρrg

Us = T # 2 # 02 m 2
= 0.2 J
π

as θ = 2 , capilary hise is zero.
39. Potential energy of the molecule on the surface of
liquid is more than P $ E of molecule inside the H 2m = 2 1.80.8 = 89
#
53. ρ V mix = m m
liquid as inner molecule experience cohesive forces mix
0.8 + 1
from all sides. Thereby, it’s binding energy is more
and potential energy less. 2T
m = Rρg

40. W = T∆A 2T # 9

s = 0.1 # 8 # 980
2 # 10 4 = ]60 - 30g # 10 4 T # 2

- -
98 # 4 # 5 =

T= 9 217.77 Dyne/cm
2 1
T = 30 # 2 = 30 = 3.3 # 10 2 N/m

-
2T
54. P - r = pa
4 4
41. 8 # 3 ρR13 = 3 πR3 2T 2 # 0.07
P = Pa + r = 105 +
-3
0.14 # 10

R1 = R/2
= 1.01 # 105 N/m 2
W = T∆A = T 68 # 4π ]R/2g2 - 4πR 2@ = 4πR T
2
h
55. l = cos 45c
2
E T∆ A R
42. E1 = T∆A1 = = 16
9
1
2
l = 50 2
2 2 R 2
14.8 Physics
56. 2πrT = W 59. v ? r 2
-3 -4

2 # 3.14 # 2 # 10 T = 6.28 # 10 v1 r12 1
-2
v = r2 = 4

T = 5 # 10 N/m
-4
5 # 10 =
v1 = 1.25 # 10 4 m/ sec .
-
57. 4
60. v = 9η r 2 . ^ρ0 - ρ w hg
2

61. Velocity increases till FB + FV = mg



FB = Bouyancy force

FV = Viscous force
62. There will not be any viscous force so velocity will
mg = B + Fv mg = B + Fv
keep on increasing.

F is force equivalent to twice the effective weight of
the body in the first case. P mv P 1
63. P1 = m1 v1 , m \ r3, v ? r5 then P1 = 32
2 2 2 2

In both cases viscous force magnitude is same when
64. Buoyant force + Viscous force = mg
the bodies attain terminal speed.
yg + 6πηrv< = xg

` V = 10 cm/s
^ x - yhg
r ^6πηh
58. v = 5 # 10 4 m/s
-
& v< =
x-y

` v< \ r
2
v = 9η r 2 ρg

2 r ^ρ - ρ L hg
2

-5 -4
65. v< = 9 $ η
5 # 9 # 18 # 10 # 10 = # -12
r2 =
9 10
2 ^3 # 10 h ^10 - 10 h ]9.8g
2 900 10
# # -4 2 4 3

& 2gh = 9 $ -6
9.8 # 10
r = 3 # 10 6 m

-


` h b 1650 m

EXERCISE - 2

1. A Y 1 2 # 1011 # =
L = A S YS lB = 2 # 10 11 2 2
k1 k2 B B 1 # 10
r1
3. r2 = b
l1

Keq = K1 + K2 l2 = a
Y 2A YA YA
l = 1l + 2l ^3mg h l1
∆l1 = A2 Y2

Y=
Y1 + Y2 ^2mg h l2
2 ∆l2 = A2 Y2
2. , B = 2m ,S = L ∆l 3l 3 a 3a
∆l1 = 2l A1 Y # A2 Y2 = 2 2 = 2
2 2 1 1 b c 2b c
AB = 2cm 2
AS = 1 cm 2
4. F = 106 # A

∆, B = ∆, S
Density = 3 # 103

F ,B F ,S A # 106 = A # L # 3 # 103 # 10

AB YB = AS YS
A Y F , 106 = 100 =
L = A S YS = A YS

L= 34 m
B B S S 3 # 10 4 3
Mechanical Properties of Matter 14.9

L ] L - xg
= F
5. T = 20 N

T ] xg dx
2 9 20
π # 10 = πr 2 de = AY
] L - xgdx
-4

& r = 10 m

= F
L AY
∆l = Fl
6. AY
l etotal = # de
l

∆l \ A L


= L FAY # ] L - xg dx

So, Ans is (c) 0
2

= L FAY L2 = 2FAY
L
F
7. A = 7 # 107
2
↑a = 1.5 m/s
200 kg
e
Strain = L
F - a # 7 # 10 7


= 2F
AY

F - 2000 g = 2000 # a
11. Fmax # σA
^7 # 10 7h A = 2000 ^a + g h

2000 ]10 + 1.5g mg # σA



A= 7
7 # 10

& ρ A L g # σA
A = 3.28 # 10 4 m 2

-
σ

& L # ρg
8.
AY 4AY
12. K = l , K' = l/2 = 8K

1 2
U = 2 # 8K # ∆l
2 1 # # 2 & U = 16 J
2 K ∆l
F ]dxg r2 - r1 13. A = 1 mm 2
de = A Y ; r = r1 + 2 .x

L = 10 cm

= F 2dx
πr Y
∆l = 2 cm
x=L

etotal = # de
m = 5g
x=0

L Y = 5 # 108


= πFY # dx
c r1 + m
r2 - r1 2 ∆l 2
F = YA l = 5 # 108 # 10 6 # 10

-
2 x
0

= 100 N
c - r m
FL -1 -1
πY ^r2 - r1 h r2

=
1
14. Elastic P.E.=K.E. of missile
FL 1 # 1

= πY rr 2
1 2 2 stress # strain # vol. = 2 mv
10. 1 # F # ∆l # = 1
2 A l Al 2 mv 2

F∆l 100 # 2 # 19 2 = 2000


-

Tension at a distance of x from the end of which v2 = m =
5/1000 5
force is applied
v 2 = 400

L ] L - xg m
T ] xg = b M lF

v = 20 m/s.
14.10 Physics
F = x Bρ
15. A ηh B - ]n - 1gρ0

=
500 6 x
(Using binomial approximation)
-4 = 2 # 10
4 # 10 2
-
4 # 16 # 10
-2
21. The small portion of film is
5 # 10 approximately a straight
& x= 32 m = 0.156 cm
part. Balancing forces on it.
1
16. 46.4 # 10 6 atm = B
-

1 P
B = -6 & B = ∆v/v
46.4 # 10

∆v ∆p -6
& v = B = 46.4 # 10
17. Isothermal electricity for Isothermal

` pv = cons
F denotes tension. T denotes surface tension.
pdv + vdp = 0 T # 2 ]dl g is the surface tension force because 2

p layers are formed.
v
dp =- dv
So, 2 F sin ]dθg = T # 62 # R ]2dθg@

we get; ^sin ]dθg . dθ. for small dθh
dp dv
p =- v

dp p
so F = T # 2 R
dv =- v
22. Clearly the surface tension force on
- ∆p - ∆p
B = ∆v & B = ∆v v & B = p Hemisphere Fs = ]2T g . ]2rrg

v
18. p = 72 cm of hg

v = 1 lit. = 1000 cm3

Process = isothermal

∆v = 100 cm3 N
& FS = 2 # 500 m # 2 # 3.14 # 5 m
∆v 1000 - 900 =
. 30, 000 N . 3000 Kg. wt.
Strain = v = 1000 0.1
Stress = B strain 6a B = p@ 23. wapp = mg - fB + 2T ]l + t g = 8 # 10 3 # 10
-

72 # 3

= 100 13.6 # 10 # 9.8 # 0.1 # # -3 # 2 # 10 -3
- 0.1 15 102
# 103 # 10
-2

= 9596.16 N m
+ 2 # 72 # 10 3 ^0.1 + 2 # 10 -3h = 79.4 # 10 3 N

- -

∆v hρg ∆ρ hρg
19. v = B & ρ = B 4 4
24. n # 3 πr3 = 3 πR3 .....(i)
ρ 2 gh
∆ρ = B "a volumes are equal ,

]n - 1gP0
c a - dV = m ∆A =- 64πR 2 - n.4πr 2@
dρ dρ
20. ρ =
and
B V ρ
ρ ]n - 1gP0
where W = ]∆Ag # T
& dρ = B
& ρ final = ρ + dρ
=- 4π 6n 2/3 r 2 - n.r 2@ # T

]n - 1gρ0
= 4πr 2 T.n 2/3 6n1/3 - 1@

= ρ c1 + m
B

Now R 2 = n 2/3 .r 2;
Mechanical Properties of Matter 14.11
25. Energy released = ]DAg # v !v = surface tension +
and PA = Patm + dgh


Let us say n no. of small drops coalesced. 2T

& Pinside bubble = P + dgh + r
4 4

& n. 3 πa3 = 3 πb3 29.

& n =bal
1 /3 b 3

& b = a.n
2 2
∆A = 4πb - n.4πa
4σ 4σ

{this is -ve, hence energy is released}
PA = P0 + r ; PB = P0 + R

= 4πa 2 ^n 2/3 - nh
" P0 = atmospheric pressure ,



Clearly PA > PB; so air will flow from A to B

& U = 4πa 2 T ^n - n 2/3h

Ultimately bubble A collapses and B becomes
bigger in size.
= 4πa 2 T ;b ba l - b ba l E
3 2


This U converts to K.E. 30.

1 4 3 2= 2 b2 b b - a l
R = 4 cm

Hence ρ.
2 3 πb V 4 πa T 2 a
a

r = 3 cm
6T b 1 - 1 l
& V=
Pr = r ; PR = R !a outside is vacuum +
t a b 4σ 4σ

26. In the shown diagram

The two bubbles are coalescing; so conserving the

Pc = PB no. the moles.
2T 2T
Pfinal # 3 π ]r'g3

P0 - r + ρgh = P0 - r 4 4 4
1 2
Pr . 3 πr3 PR . 3 πR3
T + T = T

Here, we may not know in advance which tube will
rise above the other, but let say the liquid leel is 4σ

Putting Pfinal = r' we get
higher in thinner tube.
ρghr1 r2 r' = r 2 + R 2

so 2T c r - r m =- ρgh
1 1
2 ^r2 - r1 h
&T =
2 1
= 3 2 + 4 2 = 5 cm.

as r2 > r1 ; so we assumed correctly.
31.
P1 P2
r/2
27. Before r After

4 4
By equating volume : 3 πR3 = 8 # 3 πr3


get r = R/2
Lets say, initially, the pressure due to air inside the
4σ bubble is Pair

Now pressure difference in A = R
4π 4σ

and that in B = R/2 = 2 # pressure
& Pair - P1 = r .....(i)


difference in A
Finally, the radius becomes half; so volume becomes
1
28. 8 th and hence pressure becomes 8Pair .


So, 8Pair - P2 = r/2 .....(ii)


Solving (i) and (ii)
2T 24σ
Pinside bubble - PA = r
get P2 = 8P1 + r
14.12 Physics
32. = patm + 2000 + 2800
Fs Fs

Pressure required to blow bubble

= 4800 dyne/cm 2

By balancing forces
0.5 # 2
37. Velocity gradient =
T # ]2 l g # ]cos θg = dx l h g
-2
2.5 # 10
Also, F = 2ηA dz = 2 # η # ]0.5g
dv 0.5
2T cos θ -2

we get h = xdg 1.25 # 10

33. When the excess pressure at the hole becomes equal


to the pressure of water height ;then only water will
& η = 2.5 # 10 2 kg - sec /m 2

-
start coming out of the holes : [atm pressure on both
sides is same].
38. Gravitational force is constant with respect to time.
2σ 2σ

& ρhg = r & h = ρrg
` Graph 'P' represent gravitational force viscous
force initially increases and finally becomes constant
N
2 # 70 # 10 3 # m
-
^a Fv = 6πηrVt h
= kg = 0.28 m
1000 3 # b 2 l # 10 3 # 10
0.1 -

` Graph 'Q' represents viscous force net force
m
decreases and finally becomes zero when body
34. Since the contact angle in both cases remains the
attains terminal speed.
same.

` Graph 'R' represents net force.

40. r = 20nm

ρ s = 2000 kg/m3

η = 1 # 10 3 N - s/m 2
-

cos
Fs θ = Mg & T # 2πR cos θ = Mg .....(i)
ρl = 1000 kg/m3

2r # ^ρs - ρl hg

after doubling the radius 2
v = = 0.87
T # 2π ]2Rg cos θ = M'g .....(ii) 9η
41. Viscous force leads to generation of very high

= M' = 2m temperature.
2T 42. F = mg
35. p - r = Pa .....(1)
1

2T 4
2 # 10 5 v = 3 πr3 ρg
-
p - r + hρg = Pa .....(2)
2

2T 2T 4 # ^1.5 # 10 -3h # 10 # 3.14


3
p - r = p - r + hρg v =
3 # 2 # 10 5
2 2 -

2T 2T

hρg = r - r
= 7 m/s.
2 1

2 # 75 # 10 3 ;
- E
-
2 2 43. Relative to liquid, the velocity of sphere is 2v0

h = - -
3
10 # 9.8 0.5 # 10 3 1 # 10 3 (upwards)
300 =
= 3 3.1 # 10 2 m ` Viscous force on sphere = 6πηr ^2v0 h
-
10 9.8
#
2T
(downwards)
36. p = patm + h pg + r
= 12 πηr ^2v0 h

2 # 70
= patm + 2 # 1 # 1000 + 0.05
(downwards)
Mechanical Properties of Matter 14.13

EXERCISE - 3

Numerical Type t0 A~ 2 cos x



Strain, x =
1. Ma = Fnet = Mg - B - 6phrv AY

Since strain must be independent of x,

at v = vT, a = 0

⇒ Mg - B = 6 phrv
\ Y = Y0 cos x = Y0 sin (x + p/2)

4 3
r /2
t 0 A~ 2 t0 r
rr =t - Gg
t

Elongation = # AY0
dx =
Y0
~2
2
3 2 0

⇒ vT = = 2m/ sec 2#4 r
6rrh
= # = r # 10 -11
4 # 10 2
2. F 3F \ a = 1, b = p/2, c = p
r
x \a+b+c=1+ + p = 5.71

Tension in the rod at a distance x from its right edge 2
x
is T = F d 3 - 2 n
5. Q S F
L L S
T 2F Q

\ Net extension in the rod =
YA
#
dx =
YA
L E
0
T TQ TS F
3. Stress =
A
1

E= # Stress # Strain # Volume
In equilibrium condition,
2
3L
1 T Olcu OlSteel TS d n

= > # Vcu + # VSteelH 2 9FL
2 A lcu lSteel
dQ = =
2A ( 2Y ) 8AY
Tl
F = TS - TQ

a Ol =
AY
1 T 2 Vcu VSteel
F(6L) = TS(2L)
` = # 2>
E + H
2 A Ycu YSteel

TS = 3F
& E = 0.25 J

TQ = 2F
4. 

6.

x dx
d d
2 2
t0 sin x

dT = (rAdx)w2x = x Adx.w2x 

From force balance,
= r0Aw2sin x dx dv d

hA = tA g

\ T(x) = r0Aw2cos x dx 2
dv dg

` =
dx 2h
14.14 Physics
7. When the tube is placed vertically in water, water
9.
rises through height h given by

2T cos i

h= T0 T0
rdg
m m

Upward force = 2pr × T cos q
T = 2T0 = 2mg

Work done by this force in raising water column Fv
through height h is given by FB


ΔW = (2pr T cos q) h = (2prh cos q) T
rhdg
= (2prh cos q) f p = rr 2 h 2 dg 4m
2 cos i
From the free body diagram of the sphere,

However, the increase in potential energy ΔEp of
4 3 vrr3
the raised water column = mg
h 4 m = vd
rr n & m =
2 3 3
4 3

where m is the mass of the raised column of water.
and FB = t rr g
3

a m = pr2hd

Fv = 4mg - 2mg - FB
hg rr 2 h 2 dg
So, ΔEp = (pr hd) e
2 o=
⇒ Fv = 2mg - FB
2 2
rr 2 h 2 dg 2vrr3 4trr3 g 2rr 3 g

Further, ΔW - ΔEp =
⇒ 6rhrv = g- = (v - 2t)
2 3 3 3
The part (ΔW - ΔEp) is used in doing work against r2 g
viscous forces and frictional forces between water
= v= (v - 2p)
9h
and glass surface and it appears as heat. So heat
10. Consider an element of disc at a radius r and having
3 W - 3 EP rr 2 h 2 dg
released = = a width dr. Linear velocity at this radius = wr. Shear
J 2J du
stress x = n
2Tl dy
8.
53o 37o 

Liquid h
o
53o 37 r
dr
2Tl
Assuming the gap h to be small so that the velocity
distribution may be assumed linear.

2(2T l ) cos 530 = Kx V ~r

x = n# =n
12Tl h h
=x
Viscous force dF = t × 2prdr
5K

Torque dT = dF × r = t × 2pr2dr
n~r 2
2rn~r3 dr

or dT = # 2rr dr =
h h
d/2 2rn~r3 dr nrd 4 ~

Total torque T = #
0 h
=
4h

thus, x = 4
Mechanical Properties of Matter 14.15
11. Consider an element AB of length dl . Let T be L + DL + 2r = 5.22
the tension and a be the area of cross-section of the
wire. DL = 5.22 - L - 2r

= 5.22 - 5 - 2 ]0.1g

= 0.02 m


Let T be the tension in the wire at mean position
during oscillations. Then,

T/A
Y =
DL/L
YA DL Y ^rr h DL
2

& T = L = L


Mass of the element is ^2 # 1011h]rg^0.5 # 10 -3h2 # 0.02

or T =
dm = ]Volumeg^Densityh = ]adlg t
5


= 200r
The component of T , i.e. bT sin 2 + T sin 2 l
i i

provides required centripetal force to the element
Also, at mean position
considered.
mv2
T - mg = R
i

` FC = 2T sin 2
(here, R = 5.22 - r ) = 5.22 - 0.01 = 5.12m
]dmgR ~2 = 2T b i l
2 ]8rgv2
` ]200rg - ]8rg]9.8g = 5.12


ba i is small, sin i b i l
2 2
]adlg t R~2

& v2 = 5 # 20.48
T =
` N =5
i

dl
i = R 13. If L is the maximum length of the wire that can be
suspended. Then F = mg , where m = tLA (where,

` T a R2 t~2 .....(i) A = cross-sectional area)


Let DR e the increase in radius of the ring

Dl D ]2rRg DR
Longitudinal strain = l = 2rR = R

T/a TR
Y = & DR = aY
DR/R
DR = aY ^aR2 t~2h
R

From Eq. (i) mg tLA

` Maximum stress = A = g $ A = tLg
tR 3 ~ 2
Given, breaking stress

` DR = Y

& x =1
= σb
12. Let DL be the extension of wire when the sphere is
= 7.8 # 109 Nm 2 -

at mean position. Then,



` ρLg = 7.8 # 109
1

& L = 7.8 # 109 # 7800 # 10


or L = 105 m


& k =5
14.16 Physics
14. Consider an element as shown in the figure ]2000g ^2.12 # 10 -3h

= 1ms -1
^1.06 # 10 3h^4 # 10 -3h
=


Corresponding flow rate is

Q = AV = ^rr2h v

Q = π ^2 # 10 -3h # 1
2

= 4π # 10 -6 3 s -1
m


& x =4
Force xAtg

Stress in the element Area = A = xtg Dv 2 - 0
17. Velocity gradient = = = 2ms -1 m -1
Dy 1 - 0

Now, elastic potential energy stored in the wire is

= 2 ]Stressg]Straing]Volumeg
1
dU

1 ^xtgh
2 2 2
1 t g A
dU =2$ Y Adx = 2 $ Y x2 dx


Total elastic potential energy
2 2

Viscous force,
1 t g A #0
L

=2$ Y 2
x dx
F = hA Dv = ^0.01 # 10 -1h ]2g ]2g = 0.004 N
2 2 3 Dy
t g AL

= 6Y
So, to keep the plate moving with constant velocity,
a force of 4 # 10 -3 N has to be applied. Thus, a = 4
15. Energy per unit volume
18. Velocity gradient
= 2 ]Stressg ]Straing
1


Dv -1 0.5 - 0
= = = 40 ms -1 m
Dy 1.25 # 10 -2
u = V = 2 ]Stressg b Bulk modulus l
U 1 Stress

F = 2 ;hA Dy E
Dv

u = 2 ]Stressg2 ^Compressibilityh
1

or

& 1 = 2 # h # 0.5 # 40
= 2 ^tghh ^Compressibilityh
1 2 1


& h = 40 kg - sm -2

= 2 ]1000 # 9.8 # 1000g2 ^5 # 10 -10h


1
or h = 0.25 # 10 -1 = 25 # 10 -3 kgm -2



= 2.4 # 10 4 Jm -3 = 24 # 103 Jm -2
& b =5

= 3 ]8g # 103 Jm -3 1
19. cos 60c = R & R = 2 cm

` x =8
16. D = 2r = 2 # 2 # 10 -3 m = 4 # 10 -3 m

For laminar flow with maximum velocity, Reynold’s
number is

N R = 2000

NR =
tVD
h
4T 4 ]0.05g

&
NR h
V = tD Dp = R = 0.02 = 10Pa


` b =5
Mechanical Properties of Matter 14.17
20. The forces acting on the plate are
= 148.92 # 10 N -4

(i) mg ^. h

BF = :
10 # 0.2 # 1.54 # -6D #
(ii) Force due to surface tension ]FTg^. h
2 10 1000 # 9.8
(iii) Buoyant force ]BFg^- h

= 15.092 # 10 -3 N


` Apparent weight


w + FT - BF = ]80.36 + 14.892 - 15.092g


# 10 -3


= 80.16 # 10 -3 N

80.16 # 10 -3
w = mg = ^8.2 # 10 h # 9.8 -3
= kg - wt
9.8

= 8.1796 # 10 -3 kg - wt

= 80.36 # 10 3 N -

wapp = 8.1796 # 10 -3 kg - wt

The water is in contact along the length
= ]2.0449g # 4 # 10 -3 kg - wt
L = 2 ]10 + 0.2g = 20.4 cm




` FT = ^20.4 # 10 -2h^7.3 # 10 -2h
& a = 4

EXERCISE - 4
1. 2. lc = 1 m


ls = 2m
1m 2m
C S →F

2 2 Dl = 1cm

AS = 16 cm , AB = 10 cm
Yc = 1.2 # 1910
Ys = 2 # 1010
6 6

YS = 2 # 10 , B = 1 # 10
FlS FlC

∆lB = ∆lS
DlS = DlC =
AS AYC
FB lB FS lS FB # 20 FS # 30 ` DlS + DlC = 1 # 10 -2

AB YB = AS YS & 6 =
10 # 10 16 # 2 # 106
FlS FlC
15
-2

& FB = 32 FS AYS + AYC = 1 # 19

F = 1 # 10 2
-

2FB + FS = 4000 g & b 32 + 1 l FS = 4 # 10 4


30 A 1
+
2
1.2 # 1010 2 # 1010
4
32 # 4 # 19

F3 = 62 -2 10
F = 10 # 1.2 # 10
A
F 4 8 2
1 + 1.2
σ S = AS = 31 # 10 N/m

S F = 1.2 # 8
A 2.2 10
1.2 2
∆lS = 2.2 # 108 #
10
2 # 10

∆lS = 0.545 . 0.55

∆lC = 0.45 cm
14.18 Physics
3. Area = A A
7. YB = 11 # 1011 200 cm

New length
YS = 22 # 1011
l' = 2 ^ l + x h
B
2 2

Fl 200 cm
∆lB = AY
2l c1 + 2 m - 2l
2
x C
Dl l - 2l 2l

strain = l = 2l & = 4 # 9.8 # 2


π ^0.35 # 10 -3h # 11 # 1011
2l = 2

x2

=
2l2
= 1.85 # 10 4 m = 0.185 mm
-

4. 2T cos q = W F l
∆lC = A Y


W
T = 2 cos θ :cos θ = x D

4 # 9.8 # 2
π 0.35 # 10 -3h # 22 # 1011
^
l = 2

F = W wl

stress A 2 cos θA = 2Ax
= 0.092 mm

= 2 l ]Dlg2
1 YA ∆lB + ∆lC = 0.185 + 0.092 mm = 0.277 mm
5. Work
-4
8. AB = 0.2 # 19
= 2 l ]a ltg2
1 YA

AS = 0.1 # 10 4

-

1
= 2 YAl a2 t2

= 2 YA a t ]l a tg
1

6.


Area = 10cm 2

F1 + F2 = mg
Y = 8 # 10 2 t/cm 2

F1 F2
A B B C AB = AS .....(1)
5t 5t 2 t 2t

Fx = F2 ]2 - xg .....(2)
= 5tA Y60 + 2tA Y10 + t A120
# # #

Total extension Y f2 AB x
AS = F2 ]2 - xg
]300 + 200 + 120gt

= AY 2A
x = A +SA = 66.6 cm
]300 + 200 + 120g
B S


= 2 9. YB = 1 # 1011
10 # 8 # 10
62 YS = 2 # 1011

= 2 cm
8 # 10
F F

= 0.775 cm Strain = A 1Y = A 2Y .....(1)
B B S S

F1 x = F2 ]2 - xg .....(2)


Equation (1) and (2)
A Y 2 2YS AS
AB YB = x - 1 & x= A Y + YS AS
S S B B

= 100 cm
Mechanical Properties of Matter 14.19
10. After the portion A is punctured’ the thread has 2 15.
options as shown in the figures.

or
Let (a) and (b) coalesce to form (c)

By mole conservation:
Pa .a3 + Pb .b3 = Pc .c3 .....(i)

Clearly, due to surface tension, the soap film wants
to minimize the surface area which is happening in 4γ

Also Pa = P0 + a .....(ii)
option (ii)

Pb = P0 + b .....(iii)

Hence the thread will become concave towards A

11. Pc = P0 + c .....(iv)


Putting there values:

b P0 + 4γ l a3 + c P0 + m b3 = b P0 + 4γ l c3


a b c


& P0 a3 + b3 - c3 + 4γ a 2 + b 2 - c 2 = 0

The FBD of disc is shown in the figure. The net
c3 - ^b3 + a3h = 4π and c 2 - ^a 2 + b 2h = 4π
upward surface tension force. 3v s

also
= Fs cos θ = ]T # 2πr g cos θ

Putting there values:

so, Fs cos θ + W = mg = Wdisc
P0 b 4π l + 4T b 4π l = 0
- 3v -S

12.

& 3P0 V + 4ST = 0
16. Consider a very small (differential) disc element at a
distance y from the upper base.

Let r and r + dr denote the radii of upper and
PA ahs to be equal to PB . lower surfaces, respectively and A and A + dA be
the corresponding areas. Let w denotes the weight
PA = P0 + ρgh .....(i) of the body above the elemental disc and dw that of

Now PC - P0 + tgh a soap bubble has 2 times. the elemental disc. Normal compressive stress at the
top surface of the whole body is

and PC = PB a same air is filled.
P
4v σ= .....(i)

& P0 + r = P0 + tgh .....(ii) πr 02
tghr
get v= 4
13. When the capillary rise is 'h' that means
the force of surface tension ] F g is
supporting the height 'h' of liquid level.

Now if the whole capillary is taken out
Now, consider the normal compressive stress acting
the liquid tries to come out due to gravity over both the surfaces of the element. We can write.
from the bottom point. P+w P + w + dw
A = A + dA = σ = constant

But force of surface tension 'F' now
becomes 2F in the upward direction.
PA + PdA + wA + wdA = PA + wA + Adw
Hence 2F can support a maximum of
'2h' height even if l is very high. So 'h' will be dA ]P + wg = Adw

2h if l > h & will be h + l only if l is lesser than dA A = 1

& dw = P+ w v .....(ii)
h.
14.20 Physics

The increment in area between upper and lower F = 2TR
surfaces fo the differential disc is
FL FR
dA = π ]r + drg2 - πr 2 - 2πrdr
Y =
A DL
=
A DR

The increment in weight is FL FR
Y = =
A DL A DR
dw = πr ρ ^dy h
2

DR = AY = ]2TRg AY
FR R

&

Using these in Eq. (ii)
DR = AY = ]2TRg AY
FR R
2πrdr = 1
σ
πr 2 ρdy
2TR2
ρ DR = AY
dr
2 r = σ dy
18. The pressure of the water changes linearly with the
1 ρ y
2 # r
r dr = σ # dy increase in height. At the bottom of the meniscus
r0 0 it is equal to the external atmospheric pressure p0
r ρ and at the top it is equal to p0 - tgh . The average

& 2 ln r = σy tgh
0
pressure exerted on the wall is pav = p0 - 2 . The

From Eq. (i), force corresponding to this value, for an aquarium
with side walls of length L is F1 = Lhpav
r ρ
2 ln r y
_ P/πr02 i
=
or F1 = Lh c p0 - m
0 tgh

2 2
r πr ρ 0

& ln r = 2P y
0

a 2 k
r = r0 e πr0 ρy/2P

17. F is the tension in the wire



Consider the horizontal forces acting ‘ON’ the
volume of water enclosed by the dashed lines
in the figure. F1 is towards right. ( F1 is by the
wall). External air pushes it to the left with a force
F2 = p0 LH and due to surface tension along a line

Force due to tension in the wire, on the element is of length L , a force F3 = LT acts towards right due
to remaining water.
F1 = 2F sin b 2 l .
i


The volume itself is at rest.

Force on the element due to surface tension of soap
film is
` F1 + F3 - F2 = 0
F2 = 2 ]T R ig -
Lh c p0 - m + LT - p0 Lh = 0
tgh

2

a The wire is in contact with the soap film along
upper length Ri and along lower length Ri both tgh2 L
2 = LT
of the element AB .

For Equilibrium 2T
F2 = F1
& h = tg
2 ]T R ig = 2 F sin b 2 l
i

or h =
2 # 0.073
1000 # 9.8 = 0.00386m = 4mm
2 ]T R ig = 2 F b 2 l
i

ba i is small, sin i b i l
2 2
Mechanical Properties of Matter 14.21
19. d = 2R 23. For hemispherical shape-


For water-glass i = 0c
For flat surface--
θ =0 Tangent on
Tangent θ = 90°
liquid surface
on liquid
surface θ = 90°

Solid
surface

Solid surface


` d = 2R
24. Balancing the force:
d

& R =2
T.4a cos 120c + l18 a2 g = a2 h18 g


Excess pressure is Dp = T : R + R D
1 1 T.2a = a2 18 g ]l - hg

1 2


Here, for one surface, R1 = R l
]l - hg = a2tTg
h


For other surface, r2 =3
25. Viscous force = mg sin6g 39
T 2T

` Dp = R = d
h A t = mg sin6g 39
v
V = Ad
or ha2 t = a318 g sin6g 39 Þ
v ttg sin ia
V h= v

& d = A
26. When the ball is just released, the net force on ball is
2TA
Weff = ^= mg - buoyant forceh .

` Dp = V


` Force, F = A $ ∆p
The terminal velocity 'v f ' of the ball is attained
2 when net force on the ball is zero.
= 2TV A
& Viscous force 6 4g 12 hrVf = Weff


20. Figure shows one of thelegs of the mosquito 2
landing upon the water surface. Therefore,
When the ball acquires 3 rd of its maximum
T.2 4g 12 a # 8 = W = weight of the mosquito.
velocity Vr
2

The viscous force is = 3 Weff
21. The force exerted by film on wire or thread depends
only on the nature of material of the film and not on 2 1

Hence, net force is Weff - 3 Weff. = 3 Weff.
its surface area. Hence, the radius of circle formed
a
by elastic thread does not change.
& Required acceleration is = 3
r
22. As weight of liquid in capillary is 27. The force F1 causes extension in rod. F2 causes
balanced by surface tension, then compression in left half of rod and an equal extension
T # 2 4g 12 r = 4g 12 r2 h118 g
h1
(for in right half of rod. Hence F2 does not effectively
uniform r radius tube) change length of the rod.
F1
2T

h1 = rtgs
F2


but weight of liquid in tapered tube is more than C

uniform tube of radius r , then in order to balance


r

h < h1

2T

h < rtg
14.22 Physics
33. Surface tension depends on nature of tube and liquid.
# F $ ds
28. Work done by F =
2T cos θ
h = ρrg

= # F dx
,
` Capillary rise depends on inner radius of the tube.

= # YAx
L dx 34. Surface area of the bigger drop is less than sum of
0
surface areas of two smaller drops.
= YA ,2


Energy released = T ]∆S $ E g
2L

P $ E = 2 Y ]Straing2
1

Change in
2T cos θ
2 35. h = ρrg

= 12 YA2 ,

if h = 0, T may be zero

Work done by F = change in P $ E
contact angle may be 90c

heat is not produced during elongation. 36. Angle of contact depends on nature of solid and
liquid in contact.
29.
37. When drop splits into smaller drops, total surface
area increases.

` To increase the surface energy. Energy need to be
absorbed.
38. Let PQ is part 1, QR part 2 and RS part 3.


For equilibrium,

mg 4mg
]StressgB = 3 & ]StressgA = 3S
F2 + F4 = F1 + F3

SB A
F2 = F1 + F3 - F4

A will break, if S A < 4SB; rA < 2rB

= 4500 + 45000 - 1300

` (a), (b), (c) are correct.

= 36500 kg

Breaking stress is independent of length.

For the sake of simplicity, the force of 36500 kg
30. Restoring force is not generated when the rope is (acting at Q ) may be split up into two forces of
falling vertically under it’s own weight. 4500 kg and 32000 kg . The force of 45000 kg.
∆, L acting at R may be split into two forces 32000 kg
31. Strain = L = L = 1 and 13000 kg . Now, it can be seen that the part PQ
of the member is subjected to a tension of 4500 kg ,
Stress
Y = Strain & Y = Stress part QR is subjected to a compression of 32000 kg
and part RS is subjected to a tension of 13000 kg.
LA = L'A' ; L' = 2L
F'L F'L
∆l = Y = 1 1 - 2 2 + 3 3 G
A' = A/2 ; radius is 1 FL
&
Now,
A1 A2 A3

not halved
1

=
P $ E per unit volume = 2 ]Y g]1 g 2.1 # 106
1

: 4500 # 120 - 32000 # 60

& Y = 2 ^ P $ E per unit volumeh
6.25 25
# 90 D
32. As cohesive force is more than adhesive force,
+ 13000
12.5 cm
contact angle is obtuse.

= 0.049 cm = 0.49 mm
` Shape of the meniscus will be convex upwards &
liquid descend downward.
Mechanical Properties of Matter 14.23
39. 42. W ]h + xg = 1/2 kx 2

100 ]0.99 + xg = 2 # 2 # 106 # x2


1


Excess pressure is always present on concave side.

So, interface is concave towards small bubble.
10 4 x2 - x - 0.99 = 0

4T 100 # ]100x - 1g + 0.99 ]100 # - 1g = 0



p1 = p0 + r
1
4 T 1
p2 = p0 + r
x = 100 m = 1 cm
2


Excess pressure P xY 10 -2 # 2 # 1010
43. v = A = L = 4 = 5 # 107 N/m2

= p1 - p2 = 4T c r - r m
1 1
44. K1 = 106, K2 = 2 # 106
1 2


This excess pressure for the interface is also equal to 2 # 106 # 106 2 # 6

Keq = = 3 10
4T 3 # 106
R 2 # 106 100

~= 3 # 600 = 3
4T c r - r m = R
1 1 4T

`
1 2 45. Total weight = 1000 + w
rr ]0.002g]0.004g
R = r 1-2r = 0.004 - 0.002 = 0.004m 1000 + w
2 1 weight on each rod = 4
40. ^QQ' h T # 2 = F
1000 + w
stress = = 9 # 106
4 # 4 # 10 -4

& w = 14400 - 1000 = 13400 N
1340
No. of persons are = 50 = 26
du
46. F = ηA dy


As u = C1 y + C2


at y = 0, u = 0 hence C2 = 0
F 16 # 10 3
-

2 ]0.1g

` QQ' = 2T =
du =
dy C1

= 0.08 m = 8cm
F = ηA C1

` QM = 4cm
C1 = 200
2 2

` OM = 5 - 4 = 3cm
47. u = c1 y + c2
Required distance = ]5 + 3g cm or

c1 = 200, c2 = 0
]5 - 3g cm
u = 200 # 2 # 10 2 = 4 m/ sec .

-


i.e. 8 cm or 2 cm

AY 4 # 10 -4 # 2 # 1010
41. K = l = 4 = 2 # 106

K

~= m = 100
14.24 Physics
48. y = 0, u = 0, C3 = 0 55. 7Mstyrofoam + mAg + Fsurface tension = Buoyant force
y = 2 cm, u - 2 m/ sec
; 1 # l # l 3 # L # ρ sf + mE g + :2L + 2 $ l D

2 = C1 r # 10 4 + C2 2 # 10 2
- -
.....(1) 2 2 2

T = ;1 # l # l 3 E # Lρ g
du l 3 1 l
dy = 2C1 y + 2
2 2 -2#2#2 2 W

du
F = η A dy


at y = 2 cm, F = 2N

2 = 10 2 # 1 # 72 # 2 # 10 C1 + C2A
- -2

4 # 10 4 C1 + 10 2 C2 = 2 .....(2)
- -

-4 -2

4 # 10 C1 + 2 # 10 C2 = 2 .....(1)
l2 3 3 3 l2
16 LρW - ]2L + l g g
T
4 L ρ sf + m =

on solving


C2 = 0 & C1 = 5000 l2 3 3 l2 3
m = 16 L ρW - 4 Lρ sf
du F 2
- ]2L + l g g
49. dy = hA = 10 -2 # 1 = 200 T

du
50. dy = 2C1 y + C2
2
= 4 : 34 ρW - ρ sf D
Ll 3


du
at y = 0, dy = C2 = 0
- ]2L + l g g
T
53. Fnet (by water) = Buoyant force

= ;1 # l # t E L # tW g
3

56. Loss in P $ E stored in the wire
2 2
Mg
= 12 A . L, ] LAg
3

= 4 Ll2 tW g

; p0 + ρW g c H - l 3 mE + 7 p0 + ρgH @

Mg,
= 2
2
54. pav = 2

& mg = k,
= p0 + ρW g c H -
l 3m


4
mg

` K = l

ω
Heat generated = . ext - ∆P $ E
0

mg,

= 2



On slant face,

Fnet = ρ av # Area = ; p0 + ρ w g c H -
l 3 mE
Ll
4
Mechanical Properties of Matter 14.25
57.
` Tensile Stress

F F cos z F

= AN' = = A cos2 z .....(ii)
A/ cos z

Shearing stress is maximum when


Tangential force is Ft = F sin z z = 45c [from Eq. (i)]


Area of section considered is
Tensile stress is maximum when

[from Eq. (ii)]


A' = L c cos z m =
L L2 A z = 0c
=
cos z cos z
58. For A and B , the rod is in equilibrium and hence

Shearing stress
internal restoring force developed per unit area
F F sin z F across any cross-section is same and thus stress is

= At' = = A sin z cos z .....(i)
A/ cos z uniform. But in C and D , the case is opposite.

Normal force on plane section considered is

FN = F cos z

& 3 l2 = ]2gb 2 l
Numerical Type 3l 1 2

1
stress
1. Y = strain = 20 # 1010 3l 1

& 3 l2 = 2
1
1

Energy density = 2 stress # strain 3l

& 3 l2 = 2 2
= 2 ]straing2 Y = 2 ^5 # 10 -4h2 # 20 # 1010
1 1
0.04

& 3 l2 = 2
kJ
= 25 # 10 2 # 10 = 25
0.04
m3
& 3 l2 = 2
Ans. 25


& 3 l2 = 0.02 m
2. Given that a uniform metallic wire is elongated by
0.04 m when subjected to a linear force F. 3. 
0.3 m
10kg
F
T
= v ..........(1)
3l A

According to Hooke’s law, F = Y.A. l , where Y
T = m~ 2 , ..........(2)

is Young’s modulus, A is area of cross section of the
wire, and 3 l is elongation in the wire. Solving

~ = 4 rad/s
F

& 3 l = Y.A. l Single Option Correct

2gr2 ^t1 - t2 h
F.l

&3l =
Y.rr2 4. vt = ,
9h
l

&3l \ 2
Where t1 = the density of water and t 2 = the density
r
of air.

Later, length and diameter of wire is doubled and
subjected to the same force. 5. Y -Young modulus, K -Bulk modulus, h -modulus
& 3 l2 = b l2 l b r1 l
3l l r
2 2
of rigidity and v -poisson’s ratio. We know that
Y = 3K ]1 - 2vg

1 1 2
14.26 Physics
mg
& v = 2 b1 - 3K l
1 Y 3 dr
... (i) r =
3a k
Further Y = 2h ]1 + vg
Weight Density # Volume
Y 9. Stress = Area = Area

& v = 2h - 1 .. (ii)

If dimensions increase by a factor of the stress
From Eq. (i) and Eq. (ii), 2 b1 - 3K l = 2h - 1
1 Y Y becomes 9 times

Y Y P

& 1 - 3K = h - 2 10. K = ∆V
V
Y Y

& 3K = 3 - h ∆V P

` V = K
Y 3h - Y

& 3k = h
Coefficient of volume expansion = 3a
P
hY ` 3αT = K

& 3K = 3h - Y
P

` T = 3αK
hY

& K = 9h - 3Y . dhg
11. T = 4

Hence, the correct answer is (d).
d " diameter
du 1 stress
6. = stress # y h " rise of water
dv 2
1 F2
= g " acceleration due to gravity
2 A2 y
du 1 ∆T ∆g
\ 4 T = ∆dd + ∆hh + g = 10..25
01 + 0.01 + 0.01
1.25 1.45
dv d
du
d n
b 1.5%
dv 1 d 24 1
= 4 = 12. Here a = 1.1 # 10 -5 K -1
du 4
d n
d1
dv 2 Y = 2 # 1011 Nm -2
d1
= (4) 1/4
and DQ = 100cC
d2
d1 DL
= 2 : 1
Now Strain = L = aDQ
d2
7.
` Pressure = Stress = Y # strain


= 2 # 1011 # ^aDQh
V

Fundamental frequency =
4 ,/2
= 2 # 1011 # 1.1 # 10 -5 # 100
V
=
2,
= 2 # 1011 # 1.1 # 10 -5 # 100
1 y
=
= 2.2 # 108 Pa
2, P
{ 5 kHz Hence, option (c) is correct
dv dp
8. v = r
13. sin i = R
k
mg/A
=
The bubble will detach if-
k

Buoyant force $ Surface tension force
Mechanical Properties of Matter 14.27

^t whb 4 rR3l g $ ]Tg ]2rrg sin i


18. W = T∆A
3
= 0.03 _2 # 4π # ^5 2 - 3 2h 10 -4 i
2t w R 4 g

Solving r = 3T = 24π ]16g # 10 6
-


No option matches with the correct answer (Bonus)
= 0.384π # 10 3 Joule
-

F/S F/S
14. Y = = b 0.4 π mJ
Dl aDT
l
4 4

& F = SYaDT 19. 2. 3 πr3 = 3 πR3

R = 21/3 r
15. 2TL = mg
2
S $ E = T4πR
mg 1.5 # 10 2
-
T = 2L =
2 # 30 # 10 2
-
2

= 4πT2 3 r 2
1.5 =

= 600 0.025 N/m
8

= 2 3 πTr 2
16. When radius is decrease by dr
Y Ax

decrease in surface energy = Heat required for 20. F= l
vaporisation
Y ]3Ag x
F2 = ] g = 9F
]4rrdrg # T # 2 = 4rr2 dr t
and
l/3
2T
r
& = tL
22. Water fills the tube entirely in gravity less condition.
17. Vρ g = 6πηrv + vρl g

Vg ^ρ - ρl h = 6πηrv

Vg ^ρ - ρl h = 6πη'rv'

^t - t l 'h

^t - t lh
V' h' = # Vh

^t - t l 'h Vh

^t - t lh
V' h' = #
h'

]7.8 - 1.2g 10 # 8.5 # 10 -4



=
]7.8 - 1g
#
13.2

v' = 6.25 # 10 -4 cm/s


14.28 Physics

5.
1. Using geometry R = cos bi + 2 l
b a
e


4S
Pgas = Pa + r

PVc = constant [adiabatic process]

c Pa1 + r mb 4 rr13 l = c Pa + r mb 3 rr23 l


4S 5/ 3
4S 4 5 /3
3 2
1 2
JK 4S NO
r15 KKK Pa + r2 OOO
2

5 = K
2S r2 KK P + 4S OOO

Using Pressure method P0 - Rc + htg = P0 a r1
1
L P
P1 - c T c = constant
& h = R tg = btg cos ^i + a/2h
2S 2S
c Pa + r m T25/3 = c Pa + r m T15/3
4S 1 - 5/ 3
4 S 1/ 5/ 3
e 2 1
2 1

bFl
JK 4S N -2 / 3

A T2 5/3 KKK Pa + r OOO


2. Y = .....(i) d n =K 1 O
1

Dl1 T1 4S OO
KK P + r O
L a 2
L 2
P
JK 4S NO
bFl T 5/2 KK Pa + r OO
4A d 2 n = KK 2 O
2


Y= .....(ii) T1 4S OO
Dl2 KP + r O
2L K a 1
L 1
P
Dl
(d) is correct.
1 = 2
Dl2 2T cos i1
6 When T1 is in contact with water then h = rtg
4T = 7.5cm < 8cm.
3. Pressure inside tube = P = P0 + r
But in option (B) height is insufficient.
When T2 is in contact with water
2T cos i2
then h = rtg = 3.75 cm 5cm
Volume of water in the meniscus depends upon the
angle of contact.
7. The maximum stress is called the breaking strength

` P2 < P1 (Since r2 > r1 ) (stress) or tensile strength.

Hence pressure on side 1 will be greater than side 2.
The materials of the wire which break as soon as
So air from end 1 flows towards end 2. stress is increased beyond elastic limit are called
4. Pushing force =< p > ]Areag
brittle. While the materials of the wire, which have a
good plastic range are called ductile.
^p0h + ^p0 + tghh
= c m ]2Rhg
2
= 2p0 Rh + t g h2 R

Pulling force = ]Tg]2Rg


Net force = 2t 0 Rh + tgh2 R - 2TR
Mechanical Properties of Matter 14.29
plastic region
v = density of fluid

Strain
t = density of object

P VP ^ρ1 - ρ 2 h η1 η
^ 2 1h

VQ = η2 = η1
Q ρ - σ 2


a v2 - t1 = t2 - v1
proportional Stress breaking
limit strength 9.
F
elastic
limit
R
r

8. v1 4 rR3 + T = t1 4 rR3 g
3 3 R
& ^v1 - t1h 3 rR g = T
4
3


Due to surface tension, vertical force on drop

r T2rr2

= Fv = T2rr sin i = T2rr R = R

10. Equating forces on the drop:


T2rr2 4
R = t 3 rr3 g
4 4

v2 3 rR3 g + T = t2 3 gpR3 g

(Assume drop as a complete sphere)
& ^v2 - t2h 3 rR3 g = T
4

R = c 32Tr m c 3 0.11 325 10 m
2 -8 1 / 4
# # #
tg = 2 # 10 # 10

= 14.25 # 10 -4 m = 1.425 # 10 -3 m

11. Surface energy of the drop


U = TA

= 0.11 # 4π ^1.4 # 10 -3h



σ1 - ρ1 = ρ 2 - σ 2 2


σ 2 - ρ1 = ρ 2 - σ1
= 2.7 # 10 6 J
-


v1 - t2 = t1 - v2
4 4 12.

v2 3 rR3 g = t1 3 rR3 g + vrh2 Rv p


VP σ 2 - ρ1

VQ = σ1 - ρ 2

2 ^ρ - σ h g
V< = 9 r 2
η
Pressure at the bottom of disc = pressure due to
14.30 Physics
surface tension 4T 4 # 0.04
17. PA = P0 + rA & PA = 8 + 0.02
1 1
tgh = T e
+ o
PA = 16 N/m2
R1 R2

R1 >>> R2 4T 4 # 0.04

PB = P0 + rB = 8 + 0.04
1 1 h
PB = 12 N/m2

So <<< and R2 =
R1 R2 2
1
for bubble A, PV = nRT
G = T >0 + h H
1 1
` tgh = T =
+
R1 R2 ]16g 4 r ]0.02g3 = n A RT
.....(1)
2 3
2T
h2 =

for bubble B,
tg

2T 2 # 0.07 14 ]12gb 4 r ]0.04g3l = nB RT


.....(2)

h= = = 3
tg
10 # 10 3 4
10 # 100

dividing equation (1) and (2)

h = 14 mm = 3.741
nA 1 nB
dV 3da nB = 6 ; nA = 6
13. = a
V

Ans: 6
dP - V ^tghh tgh
B =- V = = a
dV dV 3da
18. Terminal velocity v< = 9h ^t s - t Lh
2r2 g
1 # 5000 # 103 # 10 # 1
70 # 109 =
3 # da
and viscous force F = 6rhv<
6
da = Da = # 10 -2 m = 2.38 mm
Viscous force is the only dissipative force. Hence,
32
2 r ^t - v h g
2

^6rhrv<h ]v<g = 6rhrv<2


14. VT = dQ
9 h dt = Fv< =
^VT hP ^rP h2 hQ ]8 - 0.8g
= 6rhr ( 2 ^ - h2
2
r2 g
_ r i h P ]8 - 16g
=
^VT hQ

Q
2
9 h ts t L

2 7.2
= 27h ^t s - t Lh r5 = dt \ r5
8rg2 dQ

= 22 $ $
2
3 6.4

=3
force
15. Surface Tension c = length

2< + 2F = c # a 2 # 2
2 kq2 kq2

a2 2a
1

a = ]Some constantg d n
2 3
q
so N = 3
c

k yA/l yA
16. ~ n = m = m = lm

^n # 109h # ^4.9 # 10 -7h



& 1 # 0.1 = 140


& n=4
Thermal Expansion & Calorimetry 15.1

Thermal Expansion
& Calorimetry
INTEXT EXERCISE: 1
1. ∆T = 1cC = 1cF 9. ∆V = V0 γ∆T
= ]30g]45g]10g^10 -6h^51 # 10 -6h]100g
C = 9 ]F - 32g
5

2.

& 9C = 5F - 160 ; let C = F = x = 68 85 cm3 = 6.9 # 10 5 m3
.
-


& 4x =- 160 10. ∆V = V0 _ γ L - γ glass i ∆T
x =- 40 = ]12g`68 # 10 - γ glass j]25g
-4 -5
& 167 # 10
3. As
αG > α H 66.8 # 10
-5
= 68 # 10 -5 - γ glass

for same DT, DLG > DLH γ glass = 1.2 # 10 5
-


Therefore, strip is bent downwards. α glass = 4 # 10 6 /Cc
-

4. Hole always expands with plate to accommodate d' = d0 ^1 - γ∆T h


11.
increased intermolecular distance between the
= ^2.7 # 103h^1 - 72 # 10 -6 # 300h
molecules.
= 2 7 # 103 - 58.32
.
5. Both the spheres undergo expansion similarly, since 3 3

.
- 2 64 # 10 kg/m
intermolecular distance among the molecules on the
surfaces of both the spheres increase equally. 12. ∆d = d γ ∆ T
6. d' = d0 ]1 + α∆T g
& 0.12 = ]999.84g γ ]4g

= ]1.2g^1 + 19 # 10 -6 # 200h
-2

3 # 10

& 999.84 =γ

= ]1.2g^1 + 38 # 10 -4h
∆V = ]1Lg 999.84 # 4
-2
3 # 10
-6

= 1.2 + 45 $ 66 # 10

- 1.205cm
= 1.2 # 10 7 m3

∆V 13. ∆, = 10 ^2.4 # 10 -5h]75g + ]90g^1.2 # 10 -5h]75g


7. ∆, = A
V γ∆T = 0 90 mm
.
= 0A
14. ∆,1 + ∆, 2 = 1.2 cm
]0.1g^cm3h^1.8 # 10 -4h # ]30g

,1 α1 ∆T + , 2 α 2 ∆T = 1.2 cm
^12 # 10 -3h^10 -2h cm 2
=

` ∆T = 470cC
= 4.5 cm

= 45 mm 15. A0 492 ^1 + 18 # 10 -5 ∆T h = A0 ]500g
8. DR = RaDT 61 + 2 # 10 -5 # ∆T @


= 6.37 # 106 # 24 # 10 -6 # 0.5 8
1 + 16 # 10 5 ∆T = 1 + 492
-


= 76.44 m 1
2 # 10 5 ∆T = 492
-


wire will be at a distance of 76.44 m from surface of
the earth. ∆T - 101c & Tf = 121cC
15.2 Physics

INTEXT EXERCISE: 2

1. In the case of thermally insulated system, heat lost 8. 50 ]0.11g]100 - T g = ]75g]1 g]T - 0g
by body at higher temperature equal to heat gained
by body at lower temperature.
& T = 6.8cC

2. S1 = 352, m1 = 4 m2 9. 160 ]S g]305 - 32g = 90 ]910g]12g + ]160g


DQ1 m1 S1 DT
]4190g]12g
= = 12
DQ2 m2 S2 DT
& S = 470 J/kg/K
Q
DQ2 = 12 Lv Heat received during vapourisation
10.
Lf =
4 ]910g]T - 10g = 16 # 10
4 Heat received during melting
3.
7
3

= 2
4 # 10

& T - 10 = 91

= 3.5

∆Q = mSw ]25 - 0g + mL + m Sice ^0 - ]- 10gh



= 44c
11.

& T = 54cC
12. ^40gm h]2090g]15g + 40 ]33.5g^10 4h + ]40g
1 ]900g]80g + ]4186g
4. 1 80 = ]500g t
2
= m ]4186g]15 - 8g

& ]36 + 334.88g # 103 = ]500g t

` mwater = 546gm

& t = 740 sec .
13. 400gm ]560g]400 - 100g
1 b1 2
l
5. 2 2 mV = ms∆T
= ]60g]4186g 70 + m ^22.6 # 105h
= ]128g]∆T g
2
44

& 4 m = 22 g

& ∆T = 3.78 K 14. ]920g]448g]100 - T g = ^50g h^33.5 # 10 4h
6. 2000 = m ]0.18g 20 + ]74g]1 g]20g

+ ]142g]4186g T
7. mgh = mS∆T

` T = 24cC
500

& ∆T = 4186 = 0.12cC

EXERCISE - 1

1. On heating the expansion will take place along all 5. For any solid body,
dimensions hence both the distances will increase.
-5 -6
I = # dmr2
2. 6 # 10 = 1 # 12 # 10 # OT

` I ? r2 & I = Kr 2
6 # 10 5 =
-
OT & OT = 5cC
12 # 10 6
-

r Increases with increase in temperature.
3. In bimetallic strips the two metals have different
thermal expansion coefficient. Hence on heating I' = K ]σ'g2
it bents towards the metal with lower thermal
= K r 2 ]1 + α∆T g2
expansion coefficient.
I' = I ]1 + 2α∆T g
4. Given ∆,1 = ∆, 2
,1 αs
= K r 2 ]1 + α∆T g2
or ,1 α a t = , 2 α B t ` =
I' = I ]1 + 2α∆T g
,2 α a

, α
or , +1 , = α +s α
(Using binomial approximation)
1 2 a s
Thermal Expansion & Calorimetry 15.3

6.
5
∆C = 9 ∆F 13. d' = d0 ^1 - γ∆T h

]∆TcC g = 5c
= ]13.6g^1 - 18 # 10 -5 # ]200gh

` 9 F

= 13.6 ]1 - 0.036g
∆, = ,α∆TcC

= 13.6 - 0.4896

= ]1 g^10 -6hb 59c F l
= 13.13 gm/cc

= 59 # 10 -6 m 14. Mass at 80c = _Vglass dliquid iat 80cC
7. ,' = , 0 ]1 + α∆T g = V0 ^1 + 24 # 10 -6 # 80h d0 ^1 - 500 # 10 -6 # 80h

= 5m + 5 ]1.2g # 10 -5 # ]15g = V0 d0 ^1 - 476 # 80 # 10 -6h


= 5m + 9 # 10 4 - = ^50 gm h^1 - 476 # 80 # 10 -6h


= 5.0009m = 48 gm
8. ∆d = d α ∆T 15. F = 2 # 1010 # 0.8 # 10 4 # 10 5 # 10
- -


Diameter of the ring need to be increased by
= 160 N
-3 16. F = YA α ∆T
5 # 10 m

= ]1mg^12 # 10 -6h]T - 10cC g
-3 ` Compressive force is independent of length.

` 5 # 10
17. As uniform rod is fixed at one end, with increase
12 # 10 = ]T - 10g
5 -3

& in temperature, rod undergoes desired expansion on
5 ]250g = - another side of rod. Thereby stress doesn’t develop

& 3 T 10 in the rod.

& T = 426.8cC 18.
∆, = F =
, AY α∆T
1
9. Loss (or) gain per second = 2 α ∆T
Considering mass of wire to be negligible, restoring
force “F” in the wire “mg”, where m is mass of the

Clock runs fast at lower temperature owing to
block.
decreased pendulum length.
mg

5 = 2 α ]T - 20cC g^1 day h AY = α∆T
1 `

mg = π ^10 -3h # 1011 # ^10 -5h # 10
2

&

Clocks runs slow at higher temperature owing to
increased pendulum length.
ks m = 3.14 kg

10 = 2 α ]35c - T g (1 day)
1 ,75 ]1 - α∆T g
19. Longitudinal strain =
, 75
35 - T
=- α∆T

& 2 = T - 20

& 2T - 40 = 35 - T
=- 12 # 10 6 # 50 -


& 3T = 75
=- 6 # 10 4 -


& T = 25cC 1
20. Heat received = 2 mV 2 = ms∆T
10. On heating volume of substance increases while V2
mass of the substance remains the same. Hence the
& ∆T = 25
density will decrease
= ] 400
2 30g # ]4.2g


11. Water attains maximum density of 4cC

= 1.58 cC
12. DV, > DVe

(“s” need to be considered in J/kg)

F = yAaDT

γ > 3α
15.4 Physics
21. Given, 0.1 m3 of water at 80°C is mixed with 0.3 m3 27. 13.3 # 10
5
= ]0.5g]4186g]50g + ]0.5g]22.6g
of water at 60°C.

# 105 + ]0.5g]2010g]T - 100g

The density of the water = 1000 Kg/m3.
1 # 105 = 2 ]2010g]T - 100g
1

The mass of the first liquid m1 = 0.1 # 1000 = 100
Kg. t - 100 - 100

The mass of the second liquid m2 = 0.3 # 1000 = 300
& T = 200cC
Kg.
28. ∆Q = m sice ^0 - ]- 8gh + mL + m swater ]14 - 0g

Note: Density variation is considered negligible.

= 8 ]2050g]8 g + 8 ]33400g + 8 ]4186g]14g

According to the law of mixtures, the final
m s T +m s T
= 780 Kcal
temperature of the mixture is, Tf = 1m1 s1 + m2 s2 2
]1.8g]4186g]80c - 10cg = m 3.34 # 105
1 1 2 2

& The specific heat of the liquid S = 1 cal/g/°C. 29.
]100g # 1 # 80 + ]300g # 1 # 60
+ m ]4186g]10g
T
& f= 400
Heat lost by Tea = Heat gained by ice

& Tf = 65°C. 22. Thermal capacity per unit
volume = ρs ]1.8g]7 g]41860g = m ]3.75g # 105
ρ1 s1 2 3 5.166
ρ 2 s2 = 3 $ 4 = 1 : 2 m = 3.75 = 1.3776

mcθ
- 1.4 kg
23. mcθ = m1 L & m1 = L
30. Let x gm of water is converted into ice.
24. Heat released by steam = heat gained by water and
calorimetry x ]335g = ]35g]2.09g]14g
mL + m # S ]100 - 80g


& x = 3.05 gm
= ]1.1 + 0.02g # S # ]80 - 15g


` amount of ice present = 38 gm

m # 540 + 20 m = 1.12 # 65

m = 0.130 kg 31. Heat lost by (water + container) system equal to heat
gained by ice.
25. Heat removed in cooling water from 25cC to 0cC
` ]0.36g s ]22 - 15g + ]0.18g]4190g]22 - 15g


= 100 # 1 # 25 = 2500 cal

Heat removed in converting water into ice at 0cC
= 24 # 10 3 # 3.3 # 105 + 24 -


= 100 # 80 = 8000 cal
# 10 -3 # 4190

Heat removed in cooling ice from 32. Heat lost by water = heat gained by alcohol

]0.3g]4190g]20 - 5g = ]0.2g] Lg + ]0.2g s ]20g


C
0c to - 15cC = 100 # 0.5 # 10 = 500 cal.

Total heat removed in1hr 50min
.....(1)

= 2500 + 8000 + 500 = 11000 cal
]0.5g]4190g]20 - 10g = ]0.2g] Lg + ]0.2g s ]20g


Heat removed per minute
.....(2)
= 11000
110 = 100 cal/ min
26. 700 kJ = ]0.7g]4186g]30g
& 4190 ]0.5g = s


+ m ^22.6 # 105h
& s = 2100 J/kg/ c C
& m ^22.6 # 10 h = 7 # 10 - ]4186g]21g
5 5
33. Total amount of ice won’t be converted into water.
5
6.13 # 10
` Final equilibrium temperature is 0c C.

& m =
22.6 # 105

= 0.271 kg
Water left = 429 gm
Thermal Expansion & Calorimetry 15.5

34. mL + ms ]100 - 80g = ^10 h]1.12g^ sw h]80c - 15cg


3 3C 3F
35. 100c = 180c
m ]560g = ]1.12g]65g # 10
3

& 25 #
100c 180c = 3 F & 3 F = 45cF
]1.12g^10 h
3
9c

& m = 560 # 65 36. ∆T = 1cC = 1cK = 5 F

= 0.13 kg 5c
∆ T = 1c F = 9 C


` ∆Hc = ∆HK > ∆HF

EXERCISE - 2

1. Coefficient of linear expansion of brass is greater


than that of steel. ` Brass radius decreases faster 10. ∆, = # Expansion in dx =
when temperature is decreased.
# 7^α0 + α1 xhdxA∆T
2. ,1 ^1 + α1 ∆T h + , 2 ^1 + α 2 ∆T h = , 2

, f = ,1 + , 2 + ^,1 α1 + , 2 α 2 h∆T

= 100 # ^1.76 # 10-5 + 1.2 # 10-6 xh
0

, f = ^,1 + , 2 hc1 +
,1 α1 + , 2 α 2
m = 100 ;^1.76 # 10 -5h x + ^1.2 # 10 -6hb x lE
2 2
,1 + , 2 ∆T 2 0
-5 6 # 10 5 =
- = 3.76 mm
3. ∆L = 6 # 10 = Lαθ & θ = -6 5cC
1 # 12 # 10 T2

11. a d, = α,0 dT ` ∆, = # d, = # ^aT - bT 2h , 0 dT


4. Expansions of a metal is same as photographic T1
enlargement. & d1 will increase by 0.3%
= , 0 : 2a _T 2 - T1 i - b3 _T 2 - T1 iD
2 2 3 3

5. Strain t = , =- α∆θ =- 12 # 10 6 # ]75 - 25g
∆, -

= : 32 aT12 - 73b T13D , 0



=- 6 # 10 -4
12. For simple pendulum T = 2π
6. , = ,0 ]1 + α∆T g & , - , 0 = , 0 α∆T
, ∆T 1 ∆, 1
, α ∆T 104 - 100 4 α g & T ? ,1/2 & T = 2 , = 2 α∆θ
2
& ,0 α A ∆T = 106 - 100 = 6 & α A = 3

0 B B

Assuming clock gives correct time at temperature
7. Clearance = R' - R but 2πR' = 2πR ^1 + α s ∆T h
θ0 & 24 # 3600 = 2 α ^θ 0 - 20 h & 24 # 3600
6 1 6


& R' - R = Rα s ∆T
= 2 α ^40 - θ0 h & θ0 = 30cC
1

= ]6400g^1.2 # 10 -5h]30g = 2.3 km
& α = 1.4 # 10 5 cC
-1

-
' '
8. x = , -,A B
13.

= , A ^1 + α A ∆T h - , B ^1 + α B ∆T h


= ,A - ,B 20°C


& ,A αA = ,B αB
9. For rod A ∆, = ,0 α A ]100g
40°C
2, 0 α B ]100g
∆, =

For rod B 2
,0 ^1 + 20αC h = ,1 ^1 + 20α S h & ,1 = c 1 + 20αC m , 0
1 + 20α

For rod C 2∆, = xα A ]100g + ^3, 0 - x hα B ]100g
S

5 4
& x = 3 ,0 & 3,0 - x = 3 ,0
15.6 Physics
14. w1 = Mg - FB 21. As same thermal stress is developed in the rods,

1 + γ OT 1
w2 = Mg - FB < 1 + γmOT F
y1 a1 Di = a Di
2 2
,

mg
= - FB 71 + ^ γ m - γ , hOT A
&
1
y1 a1 = a 2
2

Since γ m < γ , y1 a2

& y2 = a1

So w2 > w1

= 3: 2
15. Let at temperature θ , volume increases by 2% then
according to question 22. ,' = , 0 ]1 + α∆T g
100 = 98 61 + 3.3 # 10 ]θ - 4g@
= 1 ^1 + 10 -4 ]20gh




& θ = 60.4 + 4 = 64.4 cC

= 1.002 m
16. Pressure at the bottom in both arms will be equal
,' is supposed to be the length of the rod when
ρ0 ρ0
c m c m
,1 - , 2 temperature is increased by 20cC .
1 + γt1 .,1 = 1 + γt2 , 2 & γ = , 2 t1 - ,1 t2
Stress = Y b ∆,, l
γ - γ 2 + 3α1
17. γ r = γ1 + 3α1 = γ 2 + 3α 2 & α 2 = 1
= 1011 b 10..002
001m l
3

m
18. ∆L = 3 Lthermal - 3 Lcontact force

= 108 ρa
FL ]rod - 1g

= 0 & α1 Lθ = AY
1 23. Rate of cooling of water = Rate of cooling of alcohol
1 2 (250 + 10) # 1 # (5) (200s + 10) # 5

& 130 = 67


& Specific heat of alcohol
α 2 Lθ = AY ]rod - 2g & Y1 α1 = Y2 α 2
FL
2 s = 0.62

19. Thermal stress developed in the rod, when rod is 24. Amount of energy utilised in climbing
F
fixed between two walls = mgh = 0.28 # 10 # 4.2
A
0.28 # 10 # 4.2 =
D, h = 1.96 # 10 2 m
-
=y & 60 # 10
,
= yaDi
= 1.96 cm

y1 a1 = y2 a 2 25. Entire KE gets converted into heat.

y1 a2 ∆KE = ms∆θ & 10 # 10 # 10 = 2 # 4200 # ∆θ



& y2 = a1
& ∆θ = 0.12cC
6 26. Water has high specific heat and due to this it

= =3
2 absorber more heat in rise of temperature.
20. Work done = 2 A $ , $ ] A,g
1 F ∆, 27. When water is cooled to form ice, the energy is
released as heat so mass of water decreases.
1 b F l2 ] A,g 28. For minimum value of m , the final temperature of

= 24 A $ the mixture must be 0c C.

10 c -6 m ] A,g
1 98 2 1

2 ]9.8g 10 10
∆ω =
` 20 # 2 # 10 + 20 # 80 = m540 + m.1.100
1700 85
m
` = 640 = 32 gm
^ρA,h s ∆T = 3 ∆ω
4
1
∆T = 20 cC
Thermal Expansion & Calorimetry 15.7
29. Heat released by 5 kg of water when its temperature 35. HA = ]6 cal/sg # ]6 - 2gs
falls from 20cC to 0cC is HB = ]6 cal/sg # ]6.5 - 4gs
Q1 = ms∆θ = ]5 g^10 h]20 - 0g = 10 cal.
3 5
HA 4 8

` HB = 2.5 = 5

when 2 kg ice at - 20cC comes to temperature of
36. Let x = percentage of water solidified
0cC , it takes an energy
Q2 = ms∆θ = ]2g]500g]20g = 0.2 # 10 cal
5
then heat lost = Heat gained


The remaining heat & x # 3.36 # 105 = ]100 - xg # 21 # 105

100
Q = Q1 - Q2 = 0.8 # 105 cal will melt mass m of
& x = 1.16 = 86.2%

the ice, where 37. Let θ = temperature on X-scale corresponding to


50cC on Y-scale
0.8 # 105 =

m= 3 1 kg 375°C -30°C
80 # 10

So, the temperature of the mixture will be 0cC
θ

mass of water in it is 5 + 1 = 6 kg and mass of ice X 50°C
Y
2 - 1 = 1 kg.
V
30. V # 4.5 # 103 # 0.1 # T = 2 # 0.9 # 103 # 80 –125°C –70°C
9 # 800 X - (- 125) 50 - (- 70)
T = 2 # 4.5 # 11 = 80cC

& 375 - (- 125) = - 30 - (- 70)
31. M = Mass of hallstone falling.
38. For centrirade and arenheit scale
m = Mass of hallstone melting
F - 32 C-0 100
212 - 32 = 100 - 0 & C = 180 # (140 - 32)
As Mgh = mL
m gh 10 # 103 1 39. Slope of line
So M = L = 33 # 103 = 33
=3 C 100 - 0 100 5
3 F = 212 - 32 - 180 = 9
32. Heat lost = Heat gained
40. If we take two fixed points as tripe point of water
msteam # 540 = 1100 # 1 # ]80 - 15g and 0 K. Then

+ 20 # 1 # ]80 - 15g T -0 T -0
X200 = Y450 & 450TX = 200 TY & 9TX = 4TY
33. A part of liquid will evaporate immediately sucking
latent heat from the bulk of liquid. Hence a part of X - LFP
41. UFP - LFP = constant (for all temperature scales)
liquid will freeze.
where
34. ∆Qvap = ∆Q freezing

LFP " lower fixed point
m. ^ ηL h = M ] L g & M = ηm
UFP " Upper fixed point
L = latent heat of freezing X - ^- 5h C-0 60 + 5

95 - ^- 5h - 0 & 95 + 5
= 100
m = mass of vapour
C
M = mass of freezed
= 100 & C = 65cC


` Fraction of water which freezed
M ηM η
= m + M = m + ηm = 1 + η
15.8 Physics

EXERCISE - 3

Numerical Type 3 TL
1.
Dl =
AY
m
420 J m1g ice at 0o C
But Dl = LaΔq
5o C Omg L
1 g water at 0o C
\ = LaΔq
ice o
2 sin 37 AY
ΔQ1 = m × 2100 × 5
⇒ Dm = aΔq × A × Y × 2 sin 37° /g
1 2 # 10 -5 # 10 # 10 -6 # 5 # 1011 # 2 # 3
ΔQ2 = kg × 3.36 × 105 j/kg = 336 J = = 12
100° 5 # 10
\ ΔQ1 + ΔQ2 = 420 J \ ΔQ1 = 420 - 336 = 84 J 1 1 # 0.5 1
84 168 24 9. ΔE = # # (1 - (- 0.5)) 2 # d1 - n
\m= # 1000 g = = = 8g 2 1 + 0.5 4
2100 # 5 21 3 0.5 3 4. 5
m = # 1.5 2 # = J
2. m (T - 10) s = s (50 - T) ⇒ T = 20°C 3 4 16
3 4.5
1 × 1000 × ΔT =
3. Heat used for evaporation = 900 kJ 16 # 2

mass evaporated = 0.2 kg 9
ΔE = # 10 -3 °C
64
900 # 103

` Lv = j/kg 10. Δd = daΔT ≈ 6 × 10-4 = 1 × 12 × 10-6 × ΔT
0.2
mT 4 Q 15
4. Q = # mSdT =
5
& m =
5
=3
ΔT = 50o C ⇒ Tf = 70°C
21 × 570 × 50 = m × 540 × 4200 + m × 4200 ×
l 3T 1 3l 1
5. T = 2r g & = = a3i
T 2 l 2
(100 - 70)
T 10 # 8.64 # 10 4 21 # 4500 # 50 1
&3 T = (a 3 i) = e o m = = kg = 250 gm
2 2 540 4200 4200 30
# + # 4
= (7 × 10-7) (20) = 6 s
11. Temp. is increased by Δq then
6. l0A0 = V + V0

Δ l = laΔq

V’ = V(1 + bt)

A = A0 (1 + 2at) 3l

&3 i =
la

l0A = V’ + V0
3l

l0A0 (1 + 2at) = V(1+ bt) + V0
E1 = (rAl)SΔq = rAl S
la
V (b - 2a) V (b - 2a) 3l

V0 = = = 4 × 10-5 m3
when stretched, Stress = Y
2at 2a l
1 3l 3l Y (3 l) 2 A
7. Increase in length of dx = aΔT(dx)
E2 = dY nd n # Al =
2 l l 2l
x x
=d n (dx) (100) = dx E1 tAlS 3 l # 2l 2tSl
5000 50
So, = = = 100
Total thermal expansion E2 2
l # Y (3 l) A a (3 l) Y
1
x x2 1 1

= # dx = e o = m = 1 cm 12. By consewrvation of heat energy
0
50 100 0 100
MA SA ΔTA = mw sw Δ Tw
8. Change in tension = DT mw Sw OTw (60) (4200) (10)

mA = = = 600 kg.

\ 2 DT sin 37° = Dmg S A OTA 140 # 30
3 mg

DT =
2 sin 37 o
Thermal Expansion & Calorimetry 15.9

EXERCISE - 4

L
2
2L 3 L 11. With increase in temperature, interatomic distance
1. DC 2 = L22 - 41 & 0 = 2L2 3 L2 - 1 4 1 increases between all adjacent atoms. Therefore, to
accommodate increased interatomic separation, all
0 = 2L2 ^α 2 L2 θ h - 1 41 1
2L (α L θ)

& values a, b, x and , increase.


& a1 = 4α 2 12. * Gas has infinite number of specific heats.

2. At 30c true length is given by


* If α = 0 , volume remains constant thereby
specific heat is uniquely defined.

= SR ^1 + α zinc ∆T h
13. Heat is lost by body at higher temperature and gained

= 100 ^1 + 26 # 10 -6 # 30h by body at 10ω er temperature. So equilibrium

= 100.078 cm temperature is greater than lower temperature and
lesser than higher temperature. (If both the bodies

At 0c , True length is given by in same state). Otherwise, equilibrium temperature
SR ^1 + α zinc ∆T h
may be equal to lower (or) higher temperature.
100.078

_1 + α glass ∆T i ^1 + 8 # 10 -6 # 30h
= =
Option (b) includes option (d).


= 100.054 cm 14. Water attains maximum density of 4cC . Therefore
volume increases when temperature of water either
3. Tensile Stress increases (or) decreases from 4c C.
15. Length
= ^Ysteel hb ∆,, l = YS ^αb - α S h∆T

, = , 0 ]1 + α∆T g = , 0 ]1 + 20αg
= 200 # 109 ^0.8 # 10 -5h]200g


Area
A = A0 ^1 + β∆T h = 6, 02 ]1 + 40αg
8 -2

= 3.2 # 10 Nm
= 0.32 GNm -2

Volume

∆V V ^ γ - 3αh t V = V0 ^1 + γ∆T h = , 03 ]1 + 3α∆T g = , 03 ]1 + 60αg


4. , = A = 0 A
0 0
Density
5. When rods are connected end to end, to ensure ρ ρ0
length of the system doesn’t change, amount of ρ = 1 + γ0∆T = 1 + 60α
pressure need to be applied
16. Required heat Available heat
F Y1 Y2 ^a1 + a 2 h T 10 g ice ]0cC g
5 g steam ]100cC g
=
A Y1 + Y2
. 800 cal . 2700 cal


=
3 # 10 20
3 # 10 5 ]100g
- 10 g water ]0cC g
5 g water ]100cC g
10
4 10
#
. 1000 cal
10 g water ]100cg
7 2

= 2.25 # 10 N/m

8. All dimensions increase on heating.
So available heat is more than required heat therefore
9. Solid body undergoes expansion along all final temperature will be 100cC
dimensions under free expansion conditions.

Mass of vapour condensed
` All mentioned quantities increase.
= 800 5401000 = 10
+
3 g
t
10. R = ; t is thickness of each rod.
α B - α C ∆T
Totl mass of water

` R is inversely proportional to ∆T and α B - αC .


= 10 + 10

40
3 = 3 = 13 3 g
1


Total mass of steam
= 5 - 10

5
3 = 3 =13g
2
15.10 Physics
17. Entire amount of ice will not be converted into x x
water.
& y1 d a1 T - n = y2 d a 2 T + n
L L
18. In 40 min. temperature of water has come down by y1 x y2 x
y1 a1 T - = y2 a 2 T +
40cC . Therefore rate L L
x y1 a1 - y2 a 2
=f pT
mS∆T = 0.60 # 4200 # 40 =
P= t 40 # 60 42.0 W
&
L y1 + y2
19. Sample of ice has been receiving heat at constant
y1 a1 - y2 a 2
rate P from water. Its temperature has increased by F = A y1 f a1 T - $Tp
y1 + y2
30cC in time 60 min.
y2 ^a1 + a 2 h
= Ay1 T f p
mi si ∆Ti

Therefore = 60 min

P y1 + y2
]60 # 60sg # ]42W g
y1 y2 ^a1 + a 2 h AT
^2100 J/kg h . ]30cC g
&m= = 2.4 kg

=
20. Thermal equilibrium reaches after 60 min. Ice y1 + y2
conversion takes place for 20 min. during this time F
water at 0cC continues to give heat at rate P . new length of rod P = ^ L + La1 T he1 -
o
Ay1
m # L f = P # ]20 # 60sg F

= L ^1 + a1 T he1 - o
Ay1
= 42 20 560 kg = 0.15 kg
# #
m
&
F
= L e1 + a1 T - o
3.3 # 10
21-23. As steam has comparatively large amount of heat

Ay1
to provide in the form of latent heat we check what
F
amount of heat is required by the water and ice to go new length of rod Q = ^ L + La 2 T he1 -
o
up to 100°C, that is Ay2
^mi L + mi Sw 3 T h + mw .Sw . 3 T 30. Buoyant force
Mg
[(
= 200×80) + (200×1×100)] + (200×1×45) = Mg = constant = Vsub ρ , # g Vsub = ρ g
,

= 45, 000 cal.


volume of displace fluid = constant
` density of fluid must be constant.
That is given by m mass of steam, then
(b) ^Vsolid - Vsub h = constant
ms .L = 45, 000
M
45, 000 500
ms = 540 = 6 = 83.3 gm & eVsolid - solid o = constant
ρliquid
Therefore 83.3 gm steam converts into water of Mγ∆T
100°C. v # 3α # ∆T = d
d
Total water = 200 + 200 + 83.3 = 483.3gm & γ = 3α ρ
Steam left = 16.7gm. (c) Ahin dliquid = A ^hin + hout hρ solid
24-26 Let junction moves a distance “x’ forward after = M ^mass of solid h
raising the temperature by T.
M - = M - M
DL1 = ^ L + La1 T h - ] L + xg
hout = Aρ hin Aρ Adliquid
solid solid

= constant

= La1 T - x
Lx Lx M (1 + 3α∆T) M (1 + γ∆T) M M
A (1 + 2α∆T)ρ - A (1 + 2α∆T) d = Aρ - Ad
DL2 = ^ L + L a 2 T h - ]1 - xg d
γ = 2ga + α ρ

= 2a 2 T + x

As total system is at rest, restoring force developed (d) dAhin g = Buoyant force = constant = Mg

A0 ]1 + 2 α ∆T ghin 1 + γ∆T = constant


in both the rods is same. Thereby, stress developed d

in both the rods is same ( a both have same area of
hin = A d ^1 + ^ γ - 2α h ∆T h
cross section) M
D,1 D, 2
F 0
= y1 = y2 γ = 2α
A L L
Thermal Expansion & Calorimetry 15.11

Numerical Type 5.
20cm
1. m = 2000 gm/ min
o
0C
Heat required by water/min = mSTT

= ]2000g # 4.2 # 40

21cm
= 336000 J
o
4C
dm 336000

` dt = g/ min
8 # 103
mg = A(80) t 0 C g 0

= 42 gm/ min


mg = A(79) t 4 C g 0

2. 1i1 + 2i 2 = (1 + 2) 60 t 80
& t 4°C = 79 = 1.01.

0° C
i1 + 2i 2 = 180 ….(1)
6. As the temperature of rod rises, Extension in the rod,

0 # i1 + 1 # i 2 + 2 # i3 = (1 + 2) 30 DL = L a DT
F/A
Young’s modulus of the rod, Y =
i1 + 2i3 = 90 ….(2) DL/L
F#L L#F
Y= =

2 # i1 + 0 # i 2 + 1 # i3 = (2 + 1) 60 ADL ALaDT
7. Final temperature of calorimeter and its contents is
2i1 + i3 = 180 ….(3) given, T0 = 75cC

and i1 + i3 + i3 = (1 + 1 + 1) i ….(4) & 100 # 0.1 # ]75 - T g + 100 # 0.1 ]75 - 30g

+ 170 # 1 # ]75 - 30g = 0

from (1) + (2) + (3),
& 75 - T + 45 + 765 = 0

3i1 + 3i 2 + 3i3 = 450 & i1 + i 2 + i3 = 150
& T = 885cC
8.

from (4) equation 150 = 3i & i = 50cC
Single Option Correct
3. Increase in volume TV = cV0 TT
c = 3a

So TV = ]3ag V0 TT

Total surface area = 6a 2, where a is side length

,
So, 24 = 6a 2 & a = 2m

Time period T = 2π g
Volume V0 = ]2g3 = 8m3
= 2π g0 ]1 + α∆θg
,


TV = ^3 # 5 # 10 -4h]8 g # 10 = 120 # 10 -3 m3

T = T0 :1 + 2 α∆θD
1
= 1.2 # 105 cm3

2 # 86400
N = T
4. When the rod, which is lying freely, is heated, its
=c
2 # 86400 mb
1 + 2 α∆θ l

1
length will increase naturally and there won’t be any T0
thermal stress . When we try to restrict the change in

= N b1 + α∆2
θl
the length of the rod due to change in temperature,
thermal stress develops. ∆N = N - N0


On heating, the rod length will increase.
= 12 α∆θN0 & ∆N ? ∆θ

& θ0 = 25cC

` A and R are true but R is not the correct
Putting θ0 , we get α = 1.85 # 10 5 /cC

-
explanation of A.
15.12 Physics

9. ∆, =
12. , α ∆T

F F/A

and Y = ∆,/,

0.10 # 1.1 # 10 # 100 =
-5 A # 0.10
2 # 1011

So, F = AYαt
F -5 11

` A = Pressure = 1.1 # 10 # 100 # 2 # 10
Thermal stress b A l = Yαt.
F
8

= 2.2 # 10 Pa
13. ∆V = v0 ]3αg ∆T
10. L & S

L
3 = L ?3 T
= 34 π ]10g3 # 3 # 23 # 10 -6 # 100

F = ∆L ∆V = 28.9 cc
A L Y
14. Let time taken in boiling the water by the heater is t

F = α∆T YS
sec. Then

11. Q = # mcdT
Q = ms∆T
4
T3 n 836 1000 # 30 # 4.2

= # 0.1 # 32 # d 3 dT 4.2 t = 1000 # 30 & t = 836
20
400

. 0.002 k J
= 150 sec .

1. ^ Pheater - Pcooler h # t = ms∆T 420 = m S ∆θ + ]1 g # 10 3 # L



-

& ^3 # 103 - P h # 3 # 3600 = 120 # 4.2


420 = m S ]5 g + 3.36 # 10 2


# 103 # 20 420 - 336 = m ]2100g #


& P = 2067 1
m = 125 # 1000 = 8 gm.
2. Net heat given/sec = 1000 - 160
5. 5(s)(50) + 5L = C(30) …(i)

= 840 J/S
80(s)(20) = C(30) …(ii)

if it takes time t then
840t = 2000 # 4.2 # ]77 - 27g

\ from (i) and (ii)


L

t = 500 sec = 8 min 20 sec S = 270°C
F = ∆L
\ 270.00
3.A y L
mg 6. / ∆Q = 0
A = y # ]α∆θg

Heat lost by steam to convert into 0cC water
Ayα (∆θ) πr 2 yα (∆θ)
HL = 0.05 # 540 + 0.05 # 100 # 1
m = g = g
= 27 + 5 = 32 kcal
π (10 3) 2 # 1011 # 10 5 # 10
- -

= 10
Heat required by ice to change into 0cC water
=π.
3 1

Hg = 0.45 # 2 # 20 + 0.45 # 80 = 4.5 + 36.00
4. S = 2100 J kg 1 cC
- -1
= 40.5 kcal
5 -1

L = 3.36 # 10 J kg
Thus, final temperature of mixture is 0cC = 273 K
Thermal Expansion & Calorimetry 15.13
7. When the temperature is increased, volume of
the cube will increase while density of liquid
will decrease. The depth upto which the cube is
submerged in the liquid remains the same, hence the
upthrust will not change.
F = F'

` Vi ρ1 g = V'i ρ' L g (Vi = volume immersed)

` ^ Ahi h^ρ L h^ g h = ^1 + 2α S ∆T h^ Ahi hc 1 + γ L ∆T m g


ρ

L
 Heat Transfer 16.1

Chapter 16 Heat Transfer


INTEXT EXERCISE: 1
1. Under steady state, across any cross section KCu A ]T - 0g KBr ]100 - T g
6. L = L ; KCu = 4KBr
inward heat flow equals to outward heat
flow, therefore, temperature doesn’t change & 4T = 100 - T
w.r.t time but different at different points of & 5T = 100
the body. & T - 20cC

2. 7.

Kcu A ^∆T1 h KBr A ^∆T2 h



& =
, ,
; Kx = 2 K y
∆T1 KBr
∆T2 = KCu

as Kcu > KBr ,
Thermal resistance " R = KA
∆T1 < ∆T2
R AB = RBD = RDA = 2RBC = 2RCE = R

` Temperature at the interface is greater if
left end is at 100c C.
dQ &
A
3. dt ? L

` at L = 100 cm, r = 2 cm, maximum heat is 60c - TB T - 10c
conducted. R = B2R/3

60 - TB = 2 ^TB - 10c h
3
c m
dQ
&
dt 1 T
4. = T2 = 32 120 - 2TB = 3TB - 30
c m
dQ 1 &
dt 2

& 5TB = 150
K1 2

` K2 = 3 (as both the vessels
& TB = 30cC

are identical) TC = TD = 2 ^TB + TE h
1

5.
= 20cC

8.


At junction A, I1 + I2 = I3
K A A ]100 - T g K A ]T - 25g
2x = B x ; K A = 2K B
a all are identical rods,

& 100 - T = T - 25 90 - T + 90 - T = T - 0

& 2T = 7.5
` 180 = 3T

& T = 37.5c C

& T = 60c
16.2 Physics

9. Temperature gradient = ∆∆Tx 11. As there is no heat flow through AB ,


TB = TA = 20cC
= 125 c - 25c
50 cm
90 - 20 = 20 - 0
= 2c C/cm BD BC
dQ kA ]100c - 0cC g BD = 70 = 7
10.dt = L
& BC 20 2
]92g^10 -3h]100g

= K A ]∆T g2
c m = 2 2
1 dQ
12.
]Cal/secg
dt 2 L2


Heat transfered in one minute K 4A ^ ∆ T h

= 14 1 1 1
1


= ]9.2g]60g 2 L1
= 2c m

Let m amount ice is melted in one minute dQ

dt
` m ^8 # 10 4h^Cal/kgh = ]9.2g]60g
1


` Ice melts at double rate.

& m = 6.9 # 10 3 kg
-

INTEXT EXERCISE: 2
1. λ m T = Constant 5. λ1 T1 = λ 2 T2

As temperature is reduced by half, λ m T1 λ2 350 7
T2 = λ1 = 510 = 17 = 0.69
becomes doubled.
2. From Newton’s law of cooling, rate of 2 3
dθ = 4σ4πr θ0 ^θ - θ h
cooling is proportional to temperature 6. dt 4π 0
ρ 3 r3 s
difference.
σθ 3

= ρ r 0s ^θ - θ 0 h
b dθ l > b dθ l > b dθ l
dt 1 dt 2 dt 3

` T1 < T2 < T3 b dθ l 1
dt ? r
3. Emissive power refers to energy emitted per
as rQ = 3rP & b dt l = 3 b dt l
dθ dθ
unit area per second. P Q
3
b dθ l = ρ r s0 ^θ - θ 0 h
4σθ
u = σaT 4 8.dt
E1 T14 6 4
b 2 l = 16 dθ 1
4

` E2 = 4 = 4 = 3 81 dt ? ρs
T2 9

- 3 : 16 9. Net heat transfer ]ug = σA _T24 - T14 i
4. ]rate of emissiongSun = ]14000g^4πr 2h ]u/Ag = 5.67 # 10 -8

# ^300 4 - 200 4h

= ]1400g]4πg^1.5 # 1011h
2


= 5.67 # 65
= ]4πg]1400gb 32 l # 10 22
2



= 368.6 Joules
u = σ A T4
10. λ1 T1 = λ 2 T2
5.67 # 10 -8 # 4π # ^7 # 108h # T 4 = ]4πg]1400g
4800 T

b 9 l 10 22
1
T2 = 3600
4

` T = 5801 K
= 34
 Heat Transfer 16.3

EXERCISE - 1

]100 - 20g
-12 ]209g 9 # 10
1.
i1 =
-4
3 10
#
100 - 20 ]385g # 3

i2 = -12 9 10 w
3 # 10
3

At cross section C , under steady state conditions.
iT = i1 + i2 = 1.42 # 10 w
100 - TC TC - 0
= 17 iCu i2 385
8cm cm i Al = i1 = 209

& TC = 68cC

m ? A, c
dQ
2. L as the rods are made up d d
dt 9. R1 = K A R2 = K A
1 2

of same material.
` (c) is correct option. 1 1 1 th
KA ]∆T g
Req = R1 + R2 + ........ upto n
3. L = 6000 J/s.
n R1 + R2
]200g]0.75g $ ]∆T g Req = 2R1 + 2R2 = 2 d R1 R2 n
1 n n
= ]6000g

& 1m
2c K A m # K A
∆T = 40cC d d
2 ^ R1 R2 h
&
d
Keq ]nAg
1
n ^ R1 + R2 h
2
4. If the rods are connected in series,
Req = & =
dc1 + 1 m
nA K K2
1

Keff = d K1 + K2 + K3 n L
1 b l
,1 ,2 ,3 1 K1 K2
+

& Keq = 2


1 b 1 1 1 lb 1 l ,
10. Thermal resistance R = KA
Keff = 1 + 2 + 3 3
for same temperature difference, thermal current
1 6 + 3 + 2 b1 l 1

& Keff = 6 3 αR

18 i R , , , AK

` Keff = 11 = 1.6 units & i1 = R2 = K 2A / K 1A = ,1 A1 K1

2 1 2 2 1 1 2 2 2

^dQ/dt h
2
A L
= A1 $ L2 = 14 $ 32 = 83 = 12 # 1 2 # 11 = 1/8

^dQ/dt h
5. 2
2 1
K1 K2
11.
6. 36c K A = 2KB = 2 K 0°C 70°C 100°C


Heat current is same

b 36 - T l K A A = b T - 0 l KB A

iH =
70 - 0 = 100 - 70 R K
& R2 = 3/7 & K1 = 3/7
d d R1 R2 1 2

]36 - T g2 K = T K

72 0°C K2 T K1 100°C

T = 3 = 24

∆T = temp diff = 36 - 24 = 12 T - 0 100 - T

Heat current is same = R = R1
2

100 - T R1 = K2 =
7-8.
& T R2 K1 7/3

& 300 - 3T - 7T & T = 30cC
16.4 Physics
12. 25 cm from cold end is at 25cC (since temperature
Where L specific heat condensation
gradient is 1cC/cm ) m
T = ] total g
dm/dt

` Rod should be touched at this point to ensure no
heat transfer.
= 12 sec .
20. As the slabs are in contact and system is in thermal
13. a a ∆t kA = a ∆t kB
∆Q ∆Q
equilibrium,
K A A ^100 - 70h KB A ^70 - 35h c
dQ
m =c
dQ
m

` 30 = 70 dt A dt B
K K 1 21. & Reff = 2R
& A = 2B & KA = 2
K
B

Leff L1 L2 '
& R eff = R/2
14. Keff = K1 + K2
^dQ/dt h1
2L L L = R2/R2 = 14
^dQ/dt h2

&
Keff = 2K + 3K
2L 5L 20/4 = 1
^dQ/dt h

& Keff = 6k
& 4

c m = 20 Cal/sec.
12K dQ

& Keff = 5 = 2.4K
&
dt 2
15. As heat is not flowing through AB,
` 20 Cal. of heat flow in 1 minute.

TB = TA 22. ]80 - T g + ]80 - T g = T - 20


90 - 20 = 20 - 0
& 180c = 3T
BD BC

& T = 60c
BD = 70 = 7

& BC 20 2 23. u = σAeT 4

16.
= ^5.67 # 10 -8h^10 -4h]0.32g^3 # 103h


= 146.9 W
100 - P = P - 20

At P, 30 10 24. u ?T
4


& 100 - P = 3P - 60
T2 = 600K, T1 = 300K

& P = 40c

` E2 = 16 E1
17. In series connection, heat current flow should be
same. 25. ustar = 10 4 usun

K∆T1 = 2K∆T2 = 3K∆T3 Astar ]2000g4 = 10 ^ Asun h]6000g
4

&
1 1
∆T1 : ∆T2 : ∆T3 = 1 : 2 : 3 Astar 4 4

& Asun = 3 # 10

= 6 : 3: 2
∆T total = 55cC rstar 2 2

& rSun = 3 # 10 = 900

` ∆T2 = 15cC
18. ∆T Should be same in both the cases. 26. u ? A T4

^10 - ]- 10gh = ]43 - T g u2 A2 T24


u1 =
A1 T14

` T = 23cC
dQ KA ]∆T g = 14 b 600 l 1 81
4
19.
400 = 4 16
dt = ,
dm = ]0 . 02 g^6 ^10 2hh]100g 81
u2 = 64 E
dt L ]0.2g
 Heat Transfer 16.5

27. b - dt l = mS 7T - T S A
dT σeA 4 T1
b1 l
4 1/4

& T2 = 16

Rate of temperature fall will be maximum when T1 1

7T 4 - T S4A has mass value i.e. T has max value. T2 = 2

eA 6 4 u1 A1 T14
c - dT m
df = σmS 500 - 300 4@ put all values & 35.
u2 = (for Black bodies, e = 1 )
max A2 T24
1 1
]1/4g4
Get answer.
= 64 $

28. From Stefan’s law


=4

1cal # 4.2J = 36. u ? A T4


5.67 # 10 8 # 1 # T 4
-
& cal
s - m2
Power Uhalf A1
29. Area = Intensity
` Utotal = A2
2

Power absorbed by the foil = Intensity at foil #Area + 2

= 2πR 2πR
4 4πR
P0 σA0 T
2 #A =d nA

& P=
4πd 4πd 2
= 34


Now P' =
σA0 (2T) 4 # A
= 4P 37. Rate of loss of heat α ] Ag
4π (2d) 2

as temperature is same for both the bodies.
4σaθ03
mS ] g
dθ ∆θ
30. =
dt U1 A1 r ]4πg
2
4πr 2
U2 = A2 = 6a 2 = 4π 2/3
msolid > mhollow b l ]6 g r 2
3

` Hollow sphere cools faster. 4π 3 = 3

as 3 r a
4
U1 T
b 300 l
4
U1
b 4π l $ 1
1 1/3
31. U2 = T = 1200 = 1 : 256
` U2 = 3
4
2 2

Assuming area is equal.
= ]π/6g1/3
UP A1 T14 8 2 ]400g4 38. Rate of loss of heat = eσA _T - T0 i
4 4
32.
2 2 ]800g4
UQ = 4 =
A2 T2

= ]0.4g^5.67 # 10 -8h

= 16
16 = 1 : 1
]200g # 10 4 #

-
4
33. u = σAθ
6]800g4 - ]300g4@


ms dt = σAθ 4

= 182 J/s.
T2 t

# dθ =- U2
ms σA # dt 39.
T1
θ4 0
U1 = 16
1 1
t = σA 3 d T 3 - T 3 n
ms 1 T2

2 1
i.e. T1 = 2 as U ? T4
1 1

= Cd T 3 - T 3 n
` T2 = 800 K = 527cC
2 1

u1 αA1 T14 e1 40. u = σAeT 4


34. u2 = σA T 4 e
= ^5.67 # 10 -8h # ^20 # 10 -4h # 12 # ]2000g4
2 2 2

T 4


= 12 12 12 $ 14 ^ A = ]2πrg ,h
4
T2
= ]5.67g # ]16g # 10
1 = d T1 n
2 8 T2
= 912 W
16.6 Physics
41. U ? AT
4 Q 4
47. t = σAT = same but Tred < Tgreen
U A T4 1
U2 = 2 24 = 4 $ 2 4 = 4
as λ red Tred = λ green Tgreen (see VIBGYOR)
1 A1 T1

` U2 = 4 ]450g

& Area of red star is greater

= 1800W dθ =- ^θ - θ h
48. dt K 0

dθ A
42. dt ? m
θ t

# dθ # dt
θ - θ0 =- K
]dθ/dt g A m
]dθ/dt g = m1 $ A2 θi 0
2 1 2
θ-θ
r 2 r 3
& ln θ - θ0 = - kt

= d r1 n $ d r2 n i 0
2 1
r2 49. λ1 T1 = λ 2 T2 = λ3 T3

= r
1
4 1 1
= b 13 l
1 /3


& T1 : T2 : T3 = 3 : 4 : 5

43. ms dt = σA _T - T0 i; s = 1400 J/kg/cC


dθ 4 4
= 20 : 15 : 12

44. For small temperature difference, Stefan’s law can Q ? A T4


written as
Qa : Qb : Qc = 2 2 ]20g4 : 4 2 ]15g4 : 6 2 ]12g4
∆u = 6eσA ]T + ∆T g4 - T 4@

` Qb is maximum.
∆u = eσAt ;:1 + ∆TT D - 1E
-4
4

or
50. TA = 4 TB
∆T
4

or ∆u = eσAT # T λ B - λ A = 30 µm

or ∆u \ ∆T
& 3λ A = 30µm

Hence Newton’s law of colling is a special case of
& λ A = 10µm
stefan’s law.
λ B = 40µm
Rate of cooling, y = ^T - T0 hk
&
45. a
(from newton’s law of colling. 51. λ m T = ]0.289g Cm - k


T0: surrounding temperature & ]289.8g # 10 # T = ]289g # 10 5
-9

-


k: + ve constant.

& T = 10 4

& graph is straight line with +ve slope.

u ] g -8
A = 5.67 # 10 # 10
16
46. Newton’s law of cooling

= 5.67 # 108
∆θ = 7θ - θ A
∆t k 0
52. λ1 < λ3 < λ 2
θ = surrounding’s temperature

` T1 > T3 > T2
80 - 60 = 8 80 + 60 - B

& t k 2 30 .....(i)

60 - 50 = 8 60 + 50 - B

and t k 2 30 .....(ii)


& t = 48 sec
 Heat Transfer 16.7

EXERCISE - 2

Req = R1 + R2 + R3
1. where
, ,
R1 = ] g , R2 = kA , R3 k
,
2k A b lA
2
OT 700 - 100

IH = R = R + R 100 - 0 = 100 - T1 = 100 - T2 = T2 - 0
eq 1 2
Req R1 R1 + R2 R3
0.24 0.02

where Req = R1 + R2 = 0.9 # 400 + 0.15 # 400
5. Req = 2Rg + Rair
OQ OQ OmL

IH = dt = Ot = Ot 2 ]3mmg ]3mmg

= k A + k A
Om = I H g air
Ot L where L = 540 cal/gm; ∆t = 3600 sec . 6mm
R = k .A
0 - ]- θg θKA dΩ
g

^ y/KAh
2. iH = = y = dt ∆T
iA Req R
dQ dm ρ.A dy i = ∆T = R
dt = L dt = L dt
B
R
eq

6mm
KAθ = dy Kg .A
ρAL dt
2 ]3mmg c 3mm m
y
=
Kg A + Kair A
4 3600

# ydy = # cK
ρL m dt
θ 1
2 0
Kg 31

= 2K + K =
air g 656
c
y m = kθ ] t g3600 2Kg .Kair
2 4

2 2 ρL 0
6. Assuming rate of heat transfer to be constant.
4 # 10 # θ # ]3600 - 0g s ^m1 + w h∆T = Q1
& 2 # 516 - 4? =
-3
1
0.9 # 80

where s = specific heat of water = 1 g/cc
1 12 # 0.9 # 80 =

&θ= 2#
4 # 3600 # 10
-3 30cC s ^m2 + 2 h∆T = Q2


` θ =- 30cC
w = water equivalent of calorimeter

Q1 ]50 + wgs 50 + w
3.

& Q2 = ]100 + wgs = 100 + w
Q1 t1 10 50 + w

but Q2 = t2 = 18 = 100 + w & 50 = 4w
Q1 100 - 0 50
t1 = iH1 = 2R = R
& w = 12.5 g

100 200 Q
iH = R/2 = R = t 2
2

Q1 = Q2 = 10 cal.
7.
50 # ]2g = 200 #
R R t2
1
t2 = 2 min .

It is a parallel combination of two rod
R1 = Thermal resistance of inner cylinder
4. ,
R1 =
K1 rR2
16.8 Physics
R2 = Thermal resistance of outer cylinder. –4°C

ICE X 3K
, ,
R2 =
K2 ^rR - rR h K2 ^rR - rR h
2 2 = 2 2 0°C
2 1
WATER (10–X) K
1 1 1
R = R + R +4°C
eq 1 2
2
1 = Keq 4πR = K1 πR
2
K2 3πR 2 K A (θ + 25) K A (25 - θ)
Req 14. (i) Qo = 1 t = 2 t
L L + L 1 2

–25°C Q +25°C
4Keq = K1 + 3K2
t1=2 t2=3
K1 + 3K2 KA (θ + 25) KA (25 - θ)
& 2 = 3
Keq = 4
K1 K2

8. Let θ = junction temperature



Net heat current at junction is zero K A (θ + 25) K A (25 - θ)
(ii) Qo = 1 t
= 2 t
3k ]100 - θg + k ]0 - θg + 2k ]50 - θg = 0
1 2

2A (θ + 25) 3A (25 - θ)

200 & 2 = 3 & θ = 0c C
& θ = 3 cC
t ? _ x2 - x1 i
2 2
9. a 100 - 0 #
15. Fig A: 20 = e L L o 4 .....(i)

For x1 = 0, x2 = 1 cm +
kA kA
7 ? ^1 2 - 0 2h
Fig A: 20 = Q = ]100 - θg : L + L D t
kA kA
.....(ii)

For x1 = 1 cm, x2 = 2cm
4/2

Equation (i) ' (ii) 1 = 2/5 t = 1 min.
t ? ^2 2 - 1 2h & t = 3 & t = 21 hrs
7 1

: TA - TB TA - TB k1 .k2
T -T T -T R 16. Q = L = L L & k3 = k1 + k2
10. H = 1 R 2 and 2H = 1 R' 2 & R' = 2
k3 A k1 A + k2 A
T2 - T1 KA (T2 - T1) KA (T2 - T1)

(where R & R' are thermal resistances). 17. x 4x = 3x =: x
Df
+
KA 2K.A
1 1

R= L 3L & R' = kA k'A
kA + kA 3L + 3L
Then f = 1/3
7k K A ] AT g2
c m = 2 2

& k' = 3 ( k' = contd. of ADB wire). dQ
18. L2
dt 2
K 4A ^∆T1 h
11. Tp = 50cC; TQ = 45cC = 14 1 11

` Heat will flow from P to Q 2 L1
= 2c m
dQ
1 KA 1

12. R = KA KA = 4L ; R' = KA KA = 34 R dt 1
L/2 + L/2 3L/4 + L/4
` Ice melts at double rate.

∆T 19. As there is heat flow through vertical rod,



HI = R = 1.2W
A B A C R1 R3
C R2 = R4
B

∆T 4 ∆T 4 K2 K4

HII = 3R/4 = 3 R = 3 # 1.2 = 1.6W
& K1 = K3

: KA (4 - 0) 3KA (0 + 4) K1 K4
13. Q = 10 - x = x & x = 7.5 m
& K2 K3 = 1
 Heat Transfer 16.9
dQ dT d T 2 4
20.
dt
= kA dx & d d1 n = d T2 n

1 1

dT dT dT d2 T 2
` kA dx = KB dx = KC dx
& d n =d 2n
A B C d1 T1
< b dx l (from graph)
dT dT dT

as dx < dx
A B C
25. H = σAT 4

& K A > KB > KC
3σAT0 ]∆T g
4

T1 - TB T -T T -T C = ms
21. R = 3B R/2 2 + 3B R/2 3

` H ? r2
T1 - TB = 3 ^2TB - T2 - T3 h
2

1
C ? r

& 3T1 - 3TB = 4TB - 2T2 - 2T3
26. t = 1400 W/m 2

& 3T1 + 2T2 + 2T3 = 7TB

Let power of sun = P
3T + 2T + 2T3 4

& TB = 1 7 2
P = σeAT Intensity at earth surface
P =
22.
I= 1400
4πd 2

P = 1400 # 4π ^1.5 # 1011h


2

1400 # 4π ^1.5 # 1011h = s ]1 g 4πR32 T 4


2

1400 ^1.5 # 1011h


2
T==
G = 5800 K
σ ^7 # 108h
2
θ1 - θ θ - θ2
1

θ1 - θ 2 = ∆θ R = R2
dr 27.
# K4πr
2 # dr
2
R1 R1
K4πr

∆θ/2 = ∆θ
R = 12 cm
1 ;1 - 1E 1 c1 - 1 m
4πK1 R1 R 4πK1 R1 R2 P = 450W = σeA ]500g4

2R R P' = σe ] A/4g 16 ]500g4

& R = R +1 R2

= 4σeA ]500g4 = 4 # 450W = 1800 W
1 2

23. For sphere kA b - dr l = P


dT
28. Rate of cooling
r2
= ms b - dt l = 4σAt03 ∆T
T
dr.P - dr.P dθ
& k # (- dT) =
# A & k # dT = # 2

4πr
4σA # T03 ]50 - 20g = 10
0 rr1

= 4Pπ a 2r r 1 k & t = ^r2 - r1 h = kT


r -r 2
&
kT
& P 4πR
1 2

and 4σA # T03 ]35 - 20g = ms b - dt l = ms b 0602 l


dθ .
U1 kT14
24. I1 = 2 =
4πd1 d12
ms = 0.2 # b 15 3010 l = 1500 J/cC
60 #

&
K T24
I2 =
d22 29. For a grey body
4
kT 2 α + r + t = 1 if ga = 0.4, r = 0.6
I2 = 2
d 2

then t = 1 - 0.4 - 0.6 = 0
T14 T24

` 2 =
The body is opaque
d1 d22
16.10 Physics

ln ]2g = βt 7a θi - θ0 = 2 _θ f - θ0 iA
3
b dθ l = rρs0 ^θ - θ0 h
3σθ
&
30. dt
t = β ]ln 2g
1
dθ 1
&
` dt ? ρs
37. Intensity in first case
` Sphere D shows fastest rate of cooling.
P1 σAT14
ms 1 1
31. t = 3σ A d T 3 - T 3 n I1 =
4πR12
=
4πd12
2 1
9 # 10 # ]4g ; E

Intensity in second case
3
1 - 1
3 ]5.67g # 10 # ^4πr 2h ]300g ]1000g3
= -8 3
P2 σAT 24
I2 = =

35
= hrs 16 min. 4πR 22 4πd 22
4 4
σAT1 σAT 2 d2 T2 2
2 & d =cT m
32. From graph, x is loosing temperature at faster rate
Given I1 = I2 & 2 =
than y 4πd1 4πd 2 1 1

m0

` Ex > E y 38. m0 T = T'
2

As good emitters are good absorbers

& T ' = 2T
ax > a y
P' ? ^T ' h & P' = 16P
dθ A 2πr, ,
33. dt ? m ? 4 πr3 ρ ? r 2
& P' T ' = 32 PT
3
b dθ l r 2 39. According to Wien’s law
dt 1
= 22 = 4
b l
d θ r1
λ m ? 1/T & vm ? T
dt 2
]dθ/dt g1 k
As the temperature of body increases, frequency
b dθ l = = 4
dt 2 4 corresponding to maximum energy in radiation
34. Ratio of emissive power and absoptivity is constant. ^vm h increases. Also area under the curve
100 = E # Eν dν ? T4
1 0.2

& E = 20ω/m
2
40. e1 T14 = e2 T24
T1 4
d n = 16
35. Radiation emitted by non black body is less

& T2
than radiation emitted by blackbody at the same
temperature. T1

& T2 = 2
dθ =- ^θ - θ h
36. dt β 0

= 2 ^a λ1 T1 = λ 2 T2 h
λ2

λ1
& θ - θ0 =- β dt

` λ 2 - λ1 = 1 micron
θ f - θ0

& ln θ - θ =- βt
` λ1 = 1 micron & λ 2 = 2 micron.
i 0

EXERCISE - 3

Numerical Type 2. Using stefan’s law,


Energy radiated, 3 ET = veT 4

1. Heat radiated (at same temperature) \ A
3 T = vCT5
E (a e = CT)

⇒ Q \ 4pR2 and Q’ \ (4pR2 + 2p R2) 3 ET T15
Q'
1

6r R 2 = 5
= = 1.5 3 ET T2
Q 4rR 2
2


Here, pR2 is extra surface area of plane surface of
3 ET = 5W
2

one of the hemispheres.


 Heat Transfer 16.11
T2 - T1 KA ]800 - 100g
= dt _ Lvaρ i
dm
3. iPA = x
l1 l2
f pf p KA ]800 - 0cC g
= dt ^ Lcon h
dm
K1 A K2 A iPB = 1-x
700 = 7 ]800g

K1 (tan 60°) = K2 (tan 45°)
` x ]1 - xg
4. If q be the temperature of B, then
& 1 - x = 8x
2kA (i - 100) (k/2) A (i - 0)
& x = 1/9; k =9
+ = 200
l l
Substituting the given values, we get q = 880°C 9.
kA (880 - 0)
` Q = mL; # t = 440 # 80
2l
80 # 1 # 2 # 440
t= = 800 s d
100 # 10 # 10 -4 # 880 r ] xg = 20 ]1 + αxg
5. T 1 T2
dx
T3 T4 dR = KA
Q Q Q x=L

Rtotal = # dR
x=0
L
1 3 4 2
= # 4 dx

]1 + αxg2
2
Q = v (T14 - T34) W/m 2 = v (T34 - T 44) W/m 2
0
K π d0

= v (T 44 - T 24) W/m 2

= 4 2 b 1 + αL + 1 l
-1



3Q = v (T - T ) = Q0 W/m4 4 2 πK d0
1 2
Q 1 4 b αL l

=
h=
Q0
=
3 π K d02 1 + αL

KA ^0 - ]- 10cgh dQ
= σAe _T - T0 i
= dm ] g
4 4
10.
6. 10cm dt L dt

as T0 = 0K ,
KA ]10g ] g ^a m = ρAxh

& 0.1 = ρA dx
dt L - dT
ms dt = σ Ae T 4

Where x is thickness of ice at time t .
ρ b 3 πr3 l S dt = σ ^4πr 2h e T 4
4 - dT

&
dx = KA ]100g = K ]100g - dT e 4
dt ρAL ρL
& dt = 3ρσ
Sr T
Net rate of heat loss = σAe _T - T0 i
4 4 100 t
7. dT
# =- 3ρσrse # dt
210 = σAe ^500 4 - 300 4h 200
T4 0

& 3 c ] g3 - ] g3 m = ρrs

In the case of black body, 1 1 1 3σe

700 = σA ^500 - 300 h
4 4 100 200

24 $ 106 = ρrs ] t g
21 7 1 3σe
70 = e
&


& e = ]0.3g
&
7ρrs -
t = 72σe 10 6
8.

` x =7
16.12 Physics
11. Thermal resistance from (radius “a” to radius “2a”) , 0.2 = 8 # 10 2
]1/4g^10 -3h
14. R1 = K 1A =
2A 1

= # dr

2 , 10 2
A
K 4πr R2 = K 2A = K
2 2


= 4π1K b - 1 1l
2a + a Rtotal = R1 + R2
1
R1 = 8πKa
= 10 2 c8 + 2 m
1


K
R2 ]2a to 3ag = 24πKa
1

Rate of heat transmission


100 - T = T - 0 ]360g]80g
R2 R1
= 3600
300 - 3T = T 100
Rtotal = 8

& T = 75c
1 8

(Temperature of Interface) 1 = 100
8+ K
i2 K2 A2 ,1 2
12. i1 = K1 A1 , 2 8

& 100 = 64 + K
2

= ]2g]2gb 12 l = 2
8

& 36 = K
i2 = ^i1 h2
2

&
K2 = 9 = 2 ]0.111g
2

= 8 cal/ sec .
&


` P =1
` K =2
13. At 50cC , temperature is constant
15. From Wein’s displacement law,

i.e. rate of emission = rate of absorption
` σAe ]273 + 50g4 = σAe ]293g4 + 60W
λ m T = constant
60W = 4σAe ]293g4 ]30g
]2000g λ m = ]3000g λ1


`

when heater is removed and temperature of the body

is 30cC
`
'
λ = 3m
rate of heat loss = σAe ^303 4 - 293 4h

& K =2

= 4σAe ]293g3 ]10g

using binomial approximation.


= 13 ]60W g

= 20W ; z = 5

EXERCISE - 4

36 = b T 8100 l kA + b T 8 4 l kA
1. - -


K = 0.5 cal/cC/cm
A = 12 cm 2

 Heat Transfer 16.13

6. KA a- dx k = Q & ` T j A a- dx k = Q
2. dT α :
dT :

T2 : 2

# dT = Q # dx

& - T αA .....(i)
T1 0

T : x

# dT = Q # dx

& - T αA .....(ii)
Incoming heat current at C = Outgoing Heat current T1 0
from C : :

,n c T1 m = d αA n L and ,n a T1 k = d αA n x
T Q T Q

&
2T-T T -T 2
R c = C
2R T x/L
T = T1 c T2 m
3T =TC ^ 2 + 1 h
&
1

5R
3T 7. For the cube, net resistance = 6
Tc =d n
2 +1 (Where R = thermal resistance of each side)
100 - 0
dx dx H = 5R/6
3. dRH = KA = KA (1 + αx)
0
For side A

= KA1 α ,n (1 + α, 0)
0 H = 100 - θ A
3 R & θ A = 60cC

OR

Check dimensionally 8. Heat current flow rate is uniform everywhere.

= σA _T - T0 i =- σ4πr 2 _T0 - T i
4 4 4 4
4. Newton’s law of cooling implies that rate of 9.
cooling is proportional to temperature difference
if the temperature difference between body and d (mL) dv 2 dr
= dt = ρL dt = ρL4πr dt
surrounding is small.
- σ4πr 2 _T0 - T i = σL4πr 2 dr
4 4

& dt

Then,

& Radius decreases with time.
a- dθ k = tan φ 2 α ^θ 2 - θ0 h and

dt 2 10. For same power of radiation

a ddtθ k = tan φ1 α ^θ1 - θ0 h && tan φ2 = c θ2 - θ 0 m


tan φ θ -θ PA = PB = PC & e A σATA4 = eB σA TB4 = eC σATC4

1 1 1 0

5. From newton’s law of cooling. & λ A TA = λ B TB = λC TC b e A: eB: eC = 1: 12 : 14 l




& TA . TC = TB or eA TA eC TC = eB TB
σA _T - T0 i = ms b - dt l
4 4 dT


& λ A .λC = λ B
& σ.4grr 8^T0 + ∆T h - T B = ρ 34 πr3 c a- dT
dt k
2 4 4
0
11. Heat current through every cross-section is equal.

& c Prc m^T - T0 h =- dt


3
12σT dT ∆T2
;K A b dT lE = ACD
0

dx AB x
t T

& ^T - T0 hdt =- dT & k # dt =- # dT ∆T1 A


^T - T0 h
K
& = ACD
0 T1 ∆T2 AB

12σT 3 ∆T1 < ∆T2


& = T0 + ^T1 - T0 he
-kt
`
T where K = rρc 0
16.14 Physics
12. Rnet = R1 + R2 + R3 dQ KA ^T1 ' - T2 ' h

& = 2L
dt

= b 20..5125
cm + 1cm + 25cm lb 1 l
1.5 1 137 dQ = dU


= b 20 + 32 + 25 l 137
1 # -2
10
=- n b 52R l ^dT1 ' h

= 137 = n b 52R l_dT 2 i]a ∆W = 0g


1 -2 '

3 $ 137 # 10

= 13 # 10 -2 k - s/J _T1' - T 2' i KA _T1 - T 2 i


' '

n b 2 lb 2 ld n =

5R - 1 d
2L
dt
∆T
iQ = 1
aT1' - T 2' k
d _T1 - T 2 i
-2
3 10
# ' ' t
- 2KA

= 900W
& # ^T1 - T2 h
= # 5LRn dt
^T1 - T2 h t=0

(Heat dissipated through walls)
T1' - T 2' - 2KA

` Power of electric heater = 900W
& ln T - T = 5LRn $ t
1 2

T1 - T 0 = ^T1 - T2 he 5LRn
-2KA
13.
&
' ' t

2L ^ 1

`
dQ
= KA T - T2 he - µt
dt
16. Let T is temperature of the water at time "t" .

Rate of heat lost by water

iQ = K ]2πr,g dr
dT dQ
_iQ i =
dt
KA ^T - T0 h
r2 500cC
iQ dr =
2πK, # r # dT
= b
r1 0

Rate of heat received by body
iQ r2

& 2πK, ln r1 = 500
=H
2πK, ]500g KA ^T - T0 h
ln ^r0 /r1 h

& iQ =

` net rate of heat loss = b -H
iQ ]∆t g = mL
KA ^T - T0 h

- dT
C dt = -H
mL ln ^r2 /r1 h
b

& ∆t = 1000πK, T t

# dT # dt
C KA ^T - T h =-
14. At junction “D”. 0 -H
T0
b 0


Total heat current entering should be equal to total
heat current leaving the junction “D”. bc :ln KA ^T - T h - D+ =-

& KA b 0 H t
T0
2 + 2 + 4 + 3 = 1 + 1 + iDE
& ln bb KA ^T - T0 hl - H l - ln ]- H g = e
-KAt

` iDE = 9 J/s. b
bc

H - b ^T - T0 h
15. at t = 0 KA
-KAt
H = e bc

KA ^T - T h = _
H 1 - e bc i
-KAt
b 0

& T = T0 + KA _1 - e bc i
bH -KAt

KA ^T1 ' - T2 ' h
iQ = 2L at time "t"
 Heat Transfer 16.15
'
dT 2 2 4
= 2 b 32R ld
dQ - dT n IπR - σ4πR T
17. 1
dt = 4π
dt dt ρ 3 R3 $ S
dT '

= 2 b 52R ld 2 n 3 ] I - 4σ + 4g
dt
= 4RS ρ
= 5R c 2 m
'
dT
= 3R d
- dT1 n
KA d
T11 - T 2' n
20. Bad absorber is a bad emitter and good reflector.
, dt dt
Bad reflactor is a good emitter.
- _dT1 - dT2 i = 3R, _T1 - T 2 i _ ' i
' ' KA ' '
+ 5KA
'
R, T1 - T2 21.

- d _T1 - T2 i = _T1 - T2 i R, b 15
8 l
' ' ' ' - KA

Pemitte = σeAT 4
since T1 + T2
d _T1 - T2 i - KA
T1' - T2' ' ' t

# 8
= R, # 15

& dt Pabsorb = σeATs4
So P1 = P2 at t = 0
T1 - T2
T1' - T2' 0

Cooling rate b - dt l = mS 7T - T S A
dT σeA 4 4

T1' - T2' -8KAT

& T1 - T2 = e
15R,


since MH < MS, so cooling rate will be different
= KA b dr
- dT l since cooling rate is not same so both will not have
18. iQ
same temp at any instant t ^except t = 0 h

iQ = k ^4πr 2hb dr l
- dT 22. P1 = P2 TA λ A = TB λ B

σ A ATA4 = σ eB ATB4
e TA λ = TB ]λ + 1g

TA
b 0.81 l = 3 1
1 /4
TB = 0.01 λ = 2 µm

T 5802
TB = 3A = 3 = 1.34 K


λ B = λ + 1 = 1.5µm

R2 T2 23.
# dr = # T dT ^a k = a0 Tr h
R1
a0 4πr3 T 1


Let the diameter of the sun be D and its distance
iQ _T 2 - T12 i
c 2 m =- 2
- 1 R2 from the earth be R .

& 4πa0 T 2r R1 2
D
R = θ
PQ - 1
& 4πa d R 2 + R 2 n = c 1
1 T 2 - T22 m

the radiation emitted by the surface of the sun per
0 2 1 2
unit time is
4πa0 _T1 - T2 i_ R1 R2 i
2 2 2 2
4π b D l
2
4 2 4

& iQ =
_ R22 - R12 i 2 σT = πD σT
At distance R , this radiation falls on an area 4πR 2

19. Rate at which energy is incident on the sphere
in unit time. The radiation receive at the earth’s

= I ^πR 2h surface per unit time per unit area is, therefore

Rate at which thermal energy is emitted
πD 2 σT 4 = σT 4 b D l2

= σAT 4 s = 2 4 R
4πR
Net rate of absorption = IπR 2 - σAT 4


Thus s \ T 4 and s \ i2
dT

` ms dt = IπR 2 - σAT 4
16.16 Physics
C 31. Net rate of heat emission
24. Tλ = Constant vm =
= σA _T - T0 i
m max 4 4
T

vmax = Constant

` It depends on temperature of the black body,
T1 T2 T 2T surface area and temperature of surroundings.
v1 = v2 v2 = T2 .v1 = T .v1 = 2 v1
1
dQ KA ∆T
E = σe A T 4
32. dt = x
E2
As surface area of body sphere and cube is same.
E α T4
E1 = ]2g = 16
4
Energy lost by both sphere and hollow cube are
initially at the same rate.
4
25. For body A P = σAT
dQ dT dT
P = = m1 s dt = m2 s dt
100 = σ # 300 4 dt sphere cube
A
Vsphere > Vcube for same surface area.

For body B

A = ^σT 4h d = ]1 - 0.5 - 0.3g # 6σ # 300 4@


P
` m1 > m2


= 0.2 # 100 = 20 W/m 2 dT dT

& dt < dt
sphere cube

26. For surface areas to be same


33. Rate of heat loss = σA _T - T0 i
4 4

a2 2
Ssphere = Scube & 4πR 6a & R

as T & T0 are same for both spheres

= 23π ]> 1g
dQ
? r2

Volume ratio dt
4 3 dQ 4 dQ
Vsphere πR
i.e. = 23
V = 3
3 = 2aR = 6
π (2 1)
dt A dt B
cube a
ms b dt l = σA _T - T0 i

` (Mass of water in sphere) > (Mass of water dT 4 4


in cube)

Energy host by radiation depends on the surface - dT σA _ 4 - 4 i
dt = ms T T0
area. Hence initial rate of energy loss by the two
area equal. But mass of water inside the sphere is - dT 1
greater, hence it will cool slowly. dt ? r
27. For same rate of heat transfer the body having higher
conductivity will have lower temperature difference.
dT
2 2/3 dt
dT
= dt _a rA = 2 2/3 rB i
A B
If cylinder with higher conductivity is connected
= σA _T - T0 i
with hot reservoir first then the function temperature dQ 4 4
34.
Tb, will be closer to hot reservoir temperature. dt
- ms dT _ 4 - 4i
29. In steady state,
& dt = σA T T0

* Temperature doesn’t change with time. dT = - σA _T 4 - T 4 i

& dt ms 0


* Different parts are at different temperature, which
m1 s1 = m2 s2
doesn’t change w.r.t. time.

* There will be no net absorption of heat.
i.e. heat capacities are equal for both bodies

30. u ? T4 ρ1 v1 s1 = ρ1 v2 s2

as TP = 2TQ 1

as v1 = v2 , s ? ρ
uP = 16 uQ
 Heat Transfer 16.17

iQ = KA b dr l
- dT
& ∆T = 14cC
35.
T3 = ]- 3 + 14gcC = 11cC

iQ = K ]2πr,gb dr l
- dT

&
3Q 3Q
R0 T0 39.
iQ dr 3t layer1 3t layer2

& 2πKA, # r =- # dT
0.06 # A # 5 = K2 # A # 14
R1 T1
&
1.5 # 10 2 1.4 # 10 2
- -

iQ ln d R0 n = 2πKA, ^T1 - T0 h
R

&
& K2 = 0.02 W/mK
1

θ - θ s = ^θ0 - θ s he
2πKA, ^T1 - T0 h
-kt
40. We have
ln ^ R0 /R1 h

& iQ =

Where θ0 = Initial temperature of body = 40cC

θ = temperature of body after time t
2πK, ^T1 - T0 hdr
r T

# = ]2πk,g #
ln ^ R0 /R1 hr
- dT
Since body cools from 40 to 38 in 10 min, we have
38 - 30 = ]40 - 30ge
R1 T1
-k10
.....(1)
^T1 - T0 h
ln c R m = T1 - T
r
ln ^ 0 1 h

Let after 10 min, the body temp. be θ
R /R 1
θ - 30 ]38 - 30g e
-k10
.....(2)
^T1 - T0 h ln ^r/R1 h
]1 g
ln ^ R0 /R1 h

& T = T1 -
]2g gives

36. Net rate of absorption = IπR 2 - σAT 4 8 10
θ - 30 = 8 , θ - 30 = 6.4 & θ = 36.4cC
dT = IπR 2 - σA T 4

` ms dt
41. Temperature decreases exponentially.
2 2 4
dT = IπR 4πσ43πR T
-
dt 42. during heating process from 38 to 40 in 10 min . The
ρ 3 R $S body will lose heat in the surrounding which will be
exactly equal to the heat lost when it is cooled from
dT 3 ^ I - 4σ T 4h
= 40 to 38 in 10 min , which is equal to m s ∆θ = 4 J
dt 4RS ρ
T t
` Total heat required = 8 J
# dT 3 #
4 = 4RS ρ dt
I - 4σT ∆Q 1 # 4200 # 2
T0 0
44. t = 20 J/sec

at maximum attainable temperature
K ^TS - TLh & 20 = K ^40h & K = 400
dθ =+ 2 1
45.
b dT l = 0 dt
dt
40 t

` I = 4σT 4 dθ
# =+ K # dt
^20 - θh
T = b 4σ l
I 1 /4 60c 0

&
& ,n a 40 k =- 400 t & t = 277s
20 1

∆Q ∆Q
37. In steady state ∆t ∆t
layer1 layer4

0.06 # A # ]30 - 25g 0.10 # A # ∆T


*

& =
1.5 # 10 2 3.5 # 10 2
- -


& ∆T = 7cC

T3 = ]- 10 + 7gcC =- 3cC

∆Q ∆Q
38. ∆t ∆t
layer1 layer3

0.06 # A # 5 = 0.04 # A # ∆T

&
1.5 # 10 2 2.8 # 10 2
- -
16.18 Physics
46. (a) Initially more heat will enter through section A R R 2R
due to temperature difference and no heat will
So B E ≡B E≡
2R/ 3

flow through section B because initially there B E


R/ 2 R/ 2 R
is no temperature difference.

For (b)
(b) At steady state rate of heat flow c m is same
dQ

dt R 2R/ 3 R/ 2
F≡ A
13R/ 6
F
for all sections.
A B E


(c) At steady state For (c)
dQ dT dT 1 c dQ m
Total heat current form A to F,
dt = kA dx or dx = kA dt
100 - 0 = 600
dT
I = 13 13R
dx is inversely proportional to area of cross- 6 R

Let temperature of B be TB then
section. Hence is maximum at B and minimum
at A 100 - TB 600

I= R = 13R & TB = 700
13 cC

(d) At steady state heat accumulation = 0

For (d)
dT
As heat current is inversely proportional to heat
So dt = 0 for any section.
resistance.
47. For (a)
So heat current in BD
C
= ` R + 2R j I = 3 I
R R 2R 2
, , R R R/ 2

R y = K A = 2K A = 2 A B
R
E F
Y X
R/ 2 R/ 2 TB - TD 2 2 a 600 k
D
& R/2 = 3 I = 3 13R
No heat current flows through rod CD
700 200 500

& TD = 13 - 13 = 13 cC

Numerical Type 6.5 4 5


Keq ]120g K ]120g 4K ]120g

& = +
1.
6.5
21
4 2.5
& K = 4K
eq

& Keq = 21 K = b1 + 21 l K
26 5
K 2K
(1) (2) ` a = 21

TQ
b1 l Single Option Correct
Tt = R TT
l

R : Thermal resistivity 2. TT a R a k ,
L 4
` R1 = K 1A = ] 1 g
1TT 1 2 l 16 K k
1 K 120 TT2 = k1 # l2 = k1 # 8
2.5 20 16 K
R2 =

]2K g]120g & 80 = k # 8 & k1 = 8K

1
Now, Req of this series combination

Req = R1 + R2

Leq 4 5
Keq ] Ag K ]120g 4K ]120g

So, = +
 Heat Transfer 16.19
3.
2
]DTgPQ = I b 3R l = 75 # 3R = 45°C
5 R 5
R2
 6. In the steady state, the energy flux is
R1 KDT ]103 - 102g
= 0.1 # 1 = 90Wm -2
Dx
1 dQ1 dQ2 dQ2
7. dt = dt + dt

Heat flow rate will be same through both the rods

i -i i-i

` 1R = R 2
1 2

R2 i1 - R2 i = R1 i - R1 i2
R2 i1 + R1 i2

i= R1 + R2

4.
50 - 40 b 50 + 40 - 20 l
300 = b 2
300 = b b 2 - 20 l
40 - T 40 + T

100 0.92 ]100 - T g 0.26 ]T - 0g 0.12 ]T - 0g


`T = 3
& 46 = 13 + 12
5. Assume R be the thermal resistance of uniform rod
& T = 40cC
0.92 # 4 ]100 - 40g
of length L. dQ1
dt = 40

= 4.8 cal/s

8. The temperature goes on decreasing with time (non-


linearly) The rate of decrease will be more initially
DT 120 75 which is depicted in the second graph.
Heat current : I = R = 8R/5 = R
eq

1. From given data:


Or, ∆, = 0.78 mm

RPQ 1 2. Rate of radiation energy lost by the sphere


RRS = 2

= Rate of radiation energy incident on it
400 - T & σ # 4πr 6T 4 - ]300g4@ = 912 # πr
2 2

so, T - 10 = 2


& T = 140 cC
& T = 11 # 10 2 . 330 K

As a function of x,
L L 3L
T ] xg = 10 + 130x 3. R1 = kA + 2kA = 2kA


& ∆T ] xg = T ] xg - 10 = 130x
1 1 + 1 = 3kA
R2 =
b L l b L l
L

Extension in a small element of length dx is kA 2kA
d, = α∆T ] xg dx = 130 α xdx
L
R2 = 3kA

& Net extension
1 ∆Q1 = ∆Q2
∆, = 130 a # xdx
∆T ∆T
0 R1 t1 = R2 t2

= 130 -5
2 # 1.2 # 10 # 1
&
R
t2 = R2 t1 = 2 sec .
1
16.20 Physics
4. Option (c) is correct

(d) Power received by one eye of observer


hc
=e o # No
m
No = Number of photons entering into eye per
σA ]2T g4 + σA ]3T g4 = σ2A ]T'g4
second

16T 4 + 81T 4 = 2 ]T'g4 & 3.189375 # 10 -8

97T 4 = 2 ]T'g4 6.63 # 10 -34 # 3 # 108


= # No
]T'g 4 = 97 T 4
1740 # 10 -9
2 & No = 2.79 # 1011
T' = b 2 l T
97 1 /4
Option (d) is correct
`
8. * Heat radiated by body remains unchanged even
5. Temperature of sun would be maximum out of the after change in room temperature.
given three

* Energy lost by the radiation depends upon the

as λ m T = constant surface area.
λ m for Sun is minimum.
* From Wien’s law λ m T = constant
6. Heating of glass bulb is by radiation. 9. (a) is correct.
7. A = 64 mm2, T = 2500 K ( A = surface area of
[ ` (b) is wrong, as flow will be same in (A) and (E)
filament, T = temperature of filament, d is distance is steady state]
of bulb from observer, Re = radius of pupil of eye)


d = 100 m


Re = 3 mm

(a) P = vAeT 4

& P = 5.67 # 10 -8 # 64 # 10 -6 # 1 # ]2500g4




^e = 1 black body h

Let width is x

Option (a) is wrong A A = 4l ] xg
= 4A IA = I
(b) Power reaching to the eye AB = l ] xg
=A lC = lB = lD = 4l
AC = ]2l g] xg = 2A
lE = l
# ^rRe2 h
P

=
4rd 2
AD = l ] xg
=A
AE = 4l ] xg
= 4A
# r # ]3 # 10 -3g2
141.75

=
4r # ]100g 2
2K ]4Ag
GA = l = 8KA
l = 8G
= 3.189375 # 10 -8 W
3KA = 34 G

Option (b) is correct GB = 4l
(c) m m T = b
4K 3 ]2Ag
GC = 4l = 2KA
l = 2G
m m # 2500 = 2.0 # 10 -3
6K ]4Ag
& m m = 1.16 # 10 -6 GE = l = 24lKA = 24G

]5K g A
= 1160 nm GD = 4l = 5KA 5
4l = 4 G
 Heat Transfer 16.21
5KA 3KA 2KA K
GBCD^eqh = 4l + 4l + l 700 = d
n ]msg DT
A
= b 54 + 34 + 2 l G = 4G 700 # 5 # 10 -2 50 25

` DT = = =
(c) ]∆T gA = 8G
i 10 # 4200-3 6 3


DT = 8.33
]∆T gBCD = 4G = 4G
i i
12. In steady state, heat current in both material is same
]∆T gE = 4G
i
K ]300 - 200g A K ]200 - 100g4A
]∆T gE < ]∆T gA < ]∆T gBCD 1 L = 2 L

(c) is correct K1

& K2 = 4
iB = 4 G ]∆T gBCD
3

(d)
13. Power radiated P = eσAT 4
iC = 2G ]∆T gBCD
At 487c C; P1 = eσA ]760g4
iD = 4 G ]∆T gBCD & iC = iB + iD
5 .....(i)

P P

(d) is correct.
Given, log 2 p1 = 1 & P0 = 21
0

10. Heat radiated = evAT 4


At 2767c C; P2 = eσA ]3040g4

P
Reading = log 2 d P2 n
KT
= 4

`
0
dT
eσA ]3040g4 # 2

- mS dt = KT 4
= log 2 d n
eσA ]760g4


100
dT

- mS # = Kt1
200
T4
= log 2 ^4 4 # 2h = 9

t1 = K ; E= 1 ;
K1 2003 E ^T1 h]500nmg = T2 ]1500nmg
1 1 1 7
3 - 14.
1 100 2003

t2 = K ; 3 - E= 1 ;
K1 2003 E
1 1 1 63 T1 = 3T2

1 50 2003
E A = σ $ 4π ]6cmg2 ^T1 h
4
t2
t1 = 9
dQ EB = σ $ 4π ]18cmg2 ^T1 h
4
= veA ^T 4 - T04 h

11.
dt
EA
b 1 l # ]3 g4 = 9
dQ DT 4 2
= ev ^T0 + DT h - T04 = >e1 + o - 1H
4 =
T0 EB 3
Adt
DT
= vT04 >e1 + 4 o - 1H DQ 300
e o =
e
T0 15. = mLV .....(1)
DT 10x - mx
dQ ,

= veT03 $ 4DT ...(i) KA


Adt
10 x

constant for newton’s law of cooling

]K g =

4vAeT 03
ms 100 cC
0 cC mx

from eq. (i) 400 cC
dQ
= evT03 $ 4DT
Adt
16.22 Physics
DQ 400

and e o = = mL f .....(2)
Dt mx
KA

300 mx Lv

dividing # =
10x - mx 400 Lf

3 m 540

4 ]10 - mg
& =
80

m = 9 ]19 - mg

10m = 90


& m =9
Kinetic theory of Gases 17.1

Kinetic theory of gases


INTEXT EXERCISE: 1

1. Ideal gases can never be liquified due to absence of


& 5 ]4g + 2 ]2g = ]7 g] M g
intermolecular interations.
24

& M = 7
2. Average molecular K $ E of a gas depends upon
temperature and degrees of freedom. 8. Umixture = U1 + U2
3. In general, average K $ E of a molecule means
Considering no loss of energy
3
average translational K $ E , which is 2 KT for all
` Umixture = 100 + 200
kinds of molecules irrespective of atomicity.

= 300 J

` both molecules have same avg kinetic energy
(translatory) as temperature is constant. 9. Utotal = UHe + UH2

^ Avg K $ E htrans = 32 KT
4.
= 32 ]1.4g]Rg]260g + 32 ]0.7g 260R
^ Avg K $ E hrot = 2 KT
2

= 32 ]2.1g]8.314g]260g
^ Avg K $ E htrans
= 32
^ Avg K $ E hrot
= 6.8 kJ
PV = 3 ]nM g V rms2 ; M is molecular wt. of He
1
5 10.
5. Mean K $ E per one mole of diatomic gas = 2 RT
P = 3 b 6.023 l^2.04gm h]200g2 $ c 10 m
2
1 2
1^ 5 ]∆T g
6. -3
h^ 4h 8
2 30 # 10 10 = 2 R
60
= 1.13 kpa
& ∆T = R
MHelium = 6.023 # 10 23 # 3.40 # 10 27 = 2.04gm
-

7. n1 M1 + n2 M2 = nM

INTEXT EXERCISE: 2

^1.3 # 105h^7 # 10 -3h


= 1.644
1. n = ]8.3g]273g m m
21 + 42 = 1.644
= 2.4 # 10 23
2m1 + m2 = 6.576 gm .....(1)
2. As volume and temperature are constant, to raise the
pressure no. of moles need to be increased. m1 + m2 = 5 gm .....(2)
35kg m1 = 1.5
ni = 28gm = 1.25 # 103 `
^ntotal hRT
Pf
n f = P ni = 32 ^1.25 # 103h
4. P V =
i
b 28 4 8 l] g] g

= 1.875 # 103 28 + 2 + 4 8.314 400

= 5
8.3 # 10
= ]5 g]4g # 10 -3
n f - ni = 625 moles


mass of oxygen added = ]625g]32g


= 2 # 10 2 m3 -


= 20 kg 40gm
ρ = = 2 # 103 kg/m3
2 # 10 2
5 -2 -
2 # 10 # 2 # 10
3. n = ]8.3g]293g
= 2gm/litre
17.2 Physics
5. Let temperature is doubled on Kelvin scale.
Vmean = u b 2 10
+ + 11 l = 7.664u
3
' 3R ]2T g
V = m
r . m. s . = 2 Vr.m.s.
` Vr.m.s. is higher than Vmean by u

If temperature is doubled on Celsius scale, 9. As mouth of the tube is open, pressure remains same
3R ]2C + 273g
and volume remains same.
'
V r . m. s . =
m 3n
n1 T1 = n2 T2 ; n2 = 4 1
2C + 273 < 2 ]c + 273g
T2 = 3 ]333g = 444k
4

` Increase in Vr.m.s. is more in first case.

= 171cC
6. PV = nRT
2g

as V and n are same, P ? T
10. ^105h V =
M1 R ]298g

^1.5 # 105h V = d M + M n R ]298g


2g 3g

` P' = 2P
1 2
7. To increase the average speed 2 times, temperature 2g 3g
need to be raised 4 times. M1 + M2

& 2g = 1.5

As volume is constant, pressure also increases by 4 M1
times.
3g 2g
2 2 + 10 2 + 11 2 = 225
& M2 = 0.5 M1
8. Vr.m.s. = u 3 u 3
M1 1

= 8.664u
& M2 = 3

EXERCISE - 1
1. As temperature increases, velocity of molecules 8KT
increases. Thereby number collisions per unit time 9. Vav = πm , as T = constant ` Vav = constant
increases. 10. Pvav = MVvav , as the average momentum of an ideal
2. During collision, molecules transfer momentum to gas is zero
the wall.
` option (d) is correct.
3. In the absence of intermolecular forces, degree
11. Real gas behaves as an ideal gas at low pressure and
of randomness of molecules increases. Thereby
high temperature
molecules transfer more momentum to the walls
during collision. Hence, pressure increases. Pm =
12.
ρ nRT slope of T1 > slope of T2
4. In the case of an ideal gas, intermolecular interactions

` T1 > T2
are considered to be absent. Therefore potential
energy is zero. Thereby, molecules have only kinetic 3RT
energy. 13. vr.m.s. = m
5. Total kinetic energy is conserved in collisions of v1 m2
v = m
molecules. 2 1

6. With decreased volume, number of collisions per 14. At a given temperature, vr.m.s. is inversely
unit time increases without change in velocity of proportional to square root of molecular weight.
molecules. Therefore pressure increases. ` r.m.s. velocity of hydrogen molecules is greater
7. With increased temperature, velocity of the than oxygen molecules.
molecules increases. Therefore number of collisions 3RT
per unit time increases, leading to increased rate of 15. vr.m.s. =m
momentum transfered per unit time per unit area.
In isothermal process, T is constant. Therefore,
1 vrms will remain unchanged.
8. For isothermal, PV = constant, P α V . As area
decreases, number of collision per unit area 16. vr.m.s. of gas molecules on the moon surface is greater
increases. than escape velocity from the moon’s surface.
Kinetic theory of Gases 17.3
17. At absolute zero temperature, molecules possess 26. One molecule has some single value of speed which
non-zero “low point energy” (which can be is equal to most probable speed and average speed
explained using quantum mechanics concepts) due of the gas
to vibration.
8RT
27. VAV = πM0
18. Internal energy related to translatory motion
8R # 2T =

For nitrogen VAV = 2

= 32 nRT πM0 /2
1 3 28. PV = nRT
2 nm v r2.m.s. = 2 nRT

` Temperature remains same for all ideal gas
` vr.m.s. is primarily proportional to square root of 2 2 2 2
temperature, independent of pressure. v1 + v2 + v3 + v4
29. Using Vrms = 4
2RT 3RT 8RT
19. vm.s. = m , vr.m.s. = m vavg = πM and substituting the corresponding velocity we get
Vrms is greatest in situation b.

` vr.m.s. > vavg > vms
3RT
1 30. Vrms = M
20. vr.m.s. ?
m

i.e. Vrms \ = T

` vH2 > vN2 > vO2

When temperature is increased form 120K to 480
21. At a given temperature, K (i.e. four times), the root mean square speed will
1 become 4 or 2 times i.e. 2v.
vavg ?
m 3
CH 31. The average translational KE = 2 kT which
4
CHe = 1 = 2 is directly proportional to T, while rms speed

& CH2 = 2CHe molecules is given by

22. R.m.s. velocity is independent of pressure- 3RT


vrms = M i.e., vrms ? T
(espicially when it is not mentioned that how change
in pressure is affecting temperature). When temperature of gas is increased from 300 K
to 600 K (i.e. 2 times), the average translational
12 + 22 + 32 + 42 + 52 KE will increase to 2 times and rms speed to 2 or
23. vr.m.s. = 5 1.414 times.

= 11 km/s
` Average translational KE
-21
= 2 # 6.21 # 10 J
= 1 2 35 4 5
+ + + +
vavg KE = 2 # 6.21 # 10
-21
J
-21

= 3 km/s = 12.42 # 10 J
and vrms = ]1.414g]484g m/s.
v 11
vr.m.s. = 3 = 684 m/s.
avg

TA TB 3 2 + 4 2 + 12 2
24. m A = 4 mB 32. vr.m.s. = v
3
169
T m
=v 3
TA = 4 mA
B B

= v 56 - 7.5 v
v TA mB
vA = TB $ m A 33. At absolute zero temperature, translatory kinetic
B
energy becomes zero;

= 4=2
34. H2 is adiatomic molecule and He is monoatomic
3RT = 3R # 273 molecule. At the same temperature, H2 molecule
25. 32 28 has more mean kinetic energy (translatory plus
273 # 32 = rotational); but both posses equal mean translational
T= 28 426.3 k
kinetic energy.
17.4 Physics
35. Average translational kinetic energy of an ideal gas
U = UO2 + U Ar cU = m
f
38.
3
molecule is 2 kT which depends on temperature 2 nRT

only. Therefore, if temperature is same, translational


= 52 ]2g RT + 32 ]4g RT
kinetic energy of O2 and N2 both will be equal.
RT
= 11
36. At a given temperature, mean kinetic energy of all 39. PV = nRT
molecules is same.
nRT

or P = V
37. Average kinetic energy per molecule per degree of
1
or P ?T
freedom = 2 kT . Since both the gases are diatomic

If V and n are same. Therefore, if T is doubled,
and at same temperature (300 K), both will have the pressure also becomes two times i.e. 2P
same number of rotational degree of freedom i.e.
two. Therefore, both the gases will have the same 40. Total translational KE
average rotational kinetic energy per molecule
= 32 nRT = 32 PV

b = 2 # 1 kT or kT l
2
Thus ratio will be 1 : 1

EXERCISE - 2

1. Change in momentum 9. If gas molecules undergo inelastic collisions with


walls, energy is lost during collision. Thereby
= 2mv cos ]45cg

momentum transfered to the wall per unit are
= 2 # 3.32 # 10 -27 # 103 # 1
decreases. So pressure decreases. As the velocity
2 of molecules decreases, temperature of the gas also

-24
= 4.7 # 10 kg ms -1 decreases.
3RT
2. At any temperature, average momentum of gas 10. Vr.m.s. =
molecules is zero. M
TA TB TC TD

(Considering container is at rest) 2
4m
= m =2 3m
=
2m
3. ∆P = P f - Pi
TA TB 4TC TD

` ∆Pmax = 2P m = m = 3m
=
2m
4. With increased temperature, velocity of the gas
molecules increases. Thereby momentum transfered 4TC TD
TA = TB = =
per collision increases, leading to greater pressure. 3 2
5. After the collision among gas molecules, total K $ E .
& TD > TA = TB > TC
of molecules conserved. K $ E of one molecule may
increase and another molecule may decrease. # dN V2
11. ^Vr.m.s. h =
2
6. If mass of molecule is reduced to half and speed is
doubled. Then momentum of the molecules doesn’t # dN
change. Therefore, pressure exerted by gas on the
walls of container doesn’t. V0
3N0 2 2
7. Mean free path refers to average distance travelled # e o V $ V dv
0
V03
by particle between two consecutive collisions. It is = N0
inversely proportional to pressure.
mV 2 ^V0 h5
8. Vrms =
3KT
& T = 3Krms = 33 5 = 53 V02
m V0
2
Therefore T ? mV rms 3

Vr.m.s. = 5 V0

Hence option (c) is correct
Kinetic theory of Gases 17.5
13. T3 > T1 = T2 > T4 = T5 320 = T2

& 32 2

` VP at 3 VP at 1 = VP at 2 > VP at 4 = VP at 5

& T2 = 20K

as Vmost probable ? T
1 + 2 + ... + N 20. ^Vr.m.s. hHe = 57 ^Vr.m.s. hH2
14. Vavg = N
T1 5 T2
N +1
= 2 4 =7 2
T1 25
Vr.m.s. = c 1 + 2 + ... + N m
2 2 2 1/2

& 2 = 49 T2
N
] N 1 ]2N + 1g l1/2
g
=b T1 = 49 ]273g
+ 50

6
&
Vr.m.s.
Vavg = ] N + 1g6 ]2g
2N + 1 T
21. Vr.m.s. ?
M
f1 n1 + f2 n2 + f3 n3
as T is halved and M is doubles Vr.m.s. becomes
15. feq = n1 + n2 + n3 half i.e. 150 m/s.
(5n) (3) + (n) (5) + (5n) (6) 3PV

= 22. Given, E = 2
5n + n + 5n

= 32 nRT
50

= 11

Which refers to average translational K $ E of n
16. moles of gas at temperature T .
23. Total K $ E is not same for all kinds of ideal gas
at a given temperature, even for gases having equal
number of moles only translational K $ E is same
for all kinds of gases, of equal moles but not equal
mass.

24. u = c mN
f
2 KT
` Number of molecules having most probable
Where u is internal energy of the gas and N is the
speed is higher than number of molecules travelling total number of molecules.
with r.m.s. speed. Most probable speed means that
2u
most number ofmolecules in a given sample are
` fKT refers to N
travelling at this speed. It is not maximum speed of
molecules in a given sample. T
25. Vavg ? m
17. At a given temperature.
` Compared to Helium molecule, Nitrozen
_Vmp iC > _Vmp iB > _Vmp iA
molecule hits the wall with smaller average speed.

_Vmp i ? 1 26. As temperature increases vibration of the molecules


m becomes more effective. There degree of freedom
mC < mB < m A increases, leading to increased internal energy.

` C " He, B " Ne, A " O2 kx
27. Pgas = A
^Vr.m.s. hO2 2 1 PV = nRT
^Vr.m.s. hH2
18. = 32 = 4
kx ] L + xg =
A nRT
^Vr.m.s. hH2 = 2 km/ sec .

& A $

&
19. If ^vr.m.s. hO2 = ^Vr.m.s. hH2 Rotational kinetic energy = 2 b nRT l
2
T1 T2
M1 = M2 = ]kxg] L + xg
17.6 Physics
28. Heat capacity at constant volume
= 5PV

= 124.5 J/k
= 5000 J
m 33. Both ( H 2 and O 2 ) gases are diatomic
PV = M R T
m PV
` At a given temperature, both the molecules

& M = RT possess same average kinetic energy.
ms = 124.5; S is specific heat. 34. m = 1 kg, ρ = 4 kg/m
3


& CV = M s 1
V = 4 m3
.5 RT
= ]124.5g = 124PV PV = n R T

`
m/M
4 1

& n R T = 8 # 10 # 4
]124.5g R ]675K g

= 5 -3
= 2 # 10 4
4.2 # 10 # 80 # 10

= 2.5 R 5

Total energy of the gas = 2 n R T

= 52R = 5 # 10 4 J

` f =5 35. K$E ?T

29. Internal energy of H 2 at T


` T2 = 2T1

= 2 ]293g
Ui = n b 2 l T
5R

= 586 K = 313cC

Internal energy of H atoms at
36. Momentum transfered during collisions with the
U f = ]2ngb 2 l T
3R wall depends upon relative velocity of molecules
and walls of container, which doesn’t change due to

= 3n RT motion of the container.
∆U = U f - Ui
Therefore pressure remains same.
37. PV = nRT

= nRT
2
30. Degrees of freedom associated with translational and
= 32 b 32 nRT l
Rotational motion of a linear polyatomic molecule 3
are 3 and 2
` E = 2 n R T, which is total

transilational kinetic

` Ratio of corresponding K $ E is 3/2.
energy of molecules.
f 38. In the given process,
31. Mean energy per molecule = 2 KT as each degree

Volume and temperature are constant
of freedom contributes b 12 KT l energy. (Law of
P1 P2 107 n

` n1 = n2 & = 1
equipartition of energy). 2.5 # 106 n2
m
32. PV = nRT
& u = m1
2
3

For helium, Total K $ E = 2 nRT
& m2 = 2.5 kg ^a m1 = 10 kg h

= 32 PV
i.e. 7.5 kg gas is removed.

= 1500 J 39. For given amount of gas,
5 T ? PV

for N2 ,
Total K $ E = 2 n' RT'
` T2 = 4T, ^` P & V are doubled) h

= 52 2PV

= 4 ]300g

]a 2PV = n'RT'g

= 1200 K ]927cC g
Kinetic theory of Gases 17.7
mgas
` v decreases and density increases, for
40. ρ = V
graph (2),
mgas
PV = M K T; ( M is mass of one ρ is constant and T is increasing
molecule)

` v increases and density decreases.
PM = gas m
& KT V For graph (3),
4500
41. 4.5 kg of water = 18 moles v is constant & density is constant.
= 250 moles. P0
44. P =
1 +c 0 m
V 2

If intermolecular forces vahishes, it behaves as Ideal V
gas. P0

at V = V0 Pi =
^105h V = ]250g R ]275g 1 &

` 1+ 4
V = 5.6 m3
4P0
= 5
nRT
42. P ? V Pf Vf Pi Vi
Tf - Ti = nR - nR

as n is double after mixing the gases, pressure of the
mixture is 2P . 1 c 8P0 V0 P0 V0 m

= nR
43. For graph (1), 5 - 2
11P V
T is constant and P is increasing
= 100R 0

EXERCISE - 3

Numerical Type P0 P0 P0 V0
4. P = ; when V = V0, P = &T=
n1 RT n2 R (2T) n1 V 3 2 2R
1.
2V
=
V
& n2 =
4 1 +e
o
V0

\ Translational kinetic energy
3RT
2. vrms =
M 3 3R P0 V0 3P0 V0

= RT = =
2 2 2R 4
cRT

vsound = 5. Momentum change/collision = 2mvx
M
vrms 3 Also, v 2 = v x2 + v 2y + v z2


` v =
sound c v2
& v x2 =
(sin ce v x2 = v 2y = v z2)
1
Equation of the graph : d n = T2 3
v
V v
` x=
⇒ T2V = Constant (process equation) 3
3
⇒ PV3/2 = Constant ⇒ c =
2 m
3. n = 7v - v - 10
2
vx
dn
For most probable velocity, =0
dv v
( a n is maximum at this velocity.)
\ Momentum change/collision = 2m
3
7
Let there by n collisions with unit area of wall.

7 - 2v = 0 ⇒ v = = 3.5 m/s v
2 P
` = 2m #n
3
P 3
n
` =
2mv
17.8 Physics
6. Pressure, P = nRT/V T1 + T2
PL (2n/2V) RT T
` =
2
& =
PR (n/V) RT
After mixing, rms speed of helium
7. Finally pressure, temperature are equal in all 3 parts. 3RT 3R (T1 + T2) 400 + 32000
v1' = = =

Moles in A initially and finally will be same. M M 2 # 4 # 10 -3
'
PA (L/3) v
& 1 = 1500 2 m/s
P'Ax
= ....(i)
For oxygen,
RT RT'

Moles of B remain same. 3R (T1 + T2) 4000 + 32000
v 2' =
=
A B 2M 2 # 32 # 10 -3
P’ P’ P’
T’ T’ T’ v 2' = 750 m/s

10. Let x moles of the gas dissociate at 1000 K.

Number of moles of diatomic gas milecule = 1 - x

Number of moles of monatomic gas molecules = 2×x
x L - 2x x
2PA (L/3) P'A (L - 2x)
Energy of diatomic molecules
= ....(ii)
= Energy of monatomic molecule
RT RT'

Dividing (i) by (ii), we get 5 3
(
& 1 - x) RT = 2x # RT
1 x 2 2
= & x = L/4
⇒ x = 5/11
2 L - 2x
8. Let m gram be the mass of neon. Then the mass of
Now, new number of moles = (1 - x) + 2x
argon is (28-m) g.
= 1 + x = (16/11)

Total number of moles of the mixture, nRT

P=
m 28 - m 28 + m V

n= + = ....(i) 300R
20 40 40
Pressure initially at 300 K = Pi =
V
PV 1 105 # 0.02
#

Now, n = = = 0.8 ....(ii) (1 + x) R # 1000
RT 8.314 # 300
Pressure finally at 1000 K = Pf =
V

From (i) and (ii),
16 R
28 + m
= # 1000 d n
= 0.8 & 28 + m = 32 & m = 4 g 11 V
40
Pf 33

or mass of argon = (28 - 4) g = 24 g Pi = 160
3RT 11. 6avg ]K $ Egrot@O2 = ^avg ]K $ E grot hN2
9. Vr.m.s =
M
3RT1
= 2 b 12 KT l

For helium, 1000 =
4 # 10 -3
` Ratio is 1

⇒ 3RT1 = 4000 12. Number of moles are constant.
3RT2 3P0

For oxygen, 1000 = 4P0 ]3V g ]2V g P ]5V g
RT + 2 RT
-3
32 # 10 = RT

⇒ 3RT2 = 32,000
By internal energy conservation, U1 + U2 = U1' + U 2'
Temperature remains same.
f
` 5P = 15P0
Ua = nRT
2
& P = 3P0
3 5 5 24 3 5 5 24
& # RT1 + # RT2 = d # + + n RT
` n =6
2 4 2 32 2 4 2 32
Kinetic theory of Gases 17.9
nRT 14. As temperature constant
13. Pi = V

1.4 -2

(by considering gas inside the bell closely behaves
nN2 = 28 = 5 # 10
as ideal gas).
0.4
nHe = 4 = 10 # 10 2
-
PV = constant

^ P0 + ρgh h]50ccg = ^ P0 h V'


ntotal = nHe + nN2

&

= 15 # 10 2 -

& b0.7 + 13.6 l 50cc = ]0.7g V'


47.6
]15g # 10 # 253 # ]1500g
-2

Pi =
5 # 10 3
-
& ]4.2g]50ccg = ]0.7g V'
]4.2g]50ccg = ]0.7g V'
5 2

= 3.75 # 10 N/m
&
Pf nf
= n
& V' = 300 cc
Pi i

` x =3
n f = 0.7nN2 + 0.6nN atom + nHe
15. Considering temperature is constant.

= ]1.3g5 # 10 2 + 10 # 10 - -2

-2 Pi Vi = Pf Vf ; y is level of mercury in

= 16.5 # 10
16.5
glass tube
Pf = 15 # 3.75 # 105
& ^ P0 h] A 5cmg = ^ P0 - ρgy h] Ag^48 - y h


= 1.1 # 3.75 # 105
Length of air column = ^48 - yh

11K = #
8 11 0.375

& K =3
EXERCISE - 4

3RT = 8RT 3.
1. ML πMR ;

( T is same on both sides as they

are separated by diathermic wall)
3π = mL
8 MR
1.4 gm R ]1800g
2.P = 28 gm $
5 # 10 3
-


= 1-1 ]1800g R 2
10
dP A = dm xω

= ]18g]8.314g^103h
MP dv 2

dP A = RT x ω ; M is molecular weight

= 1.5 # 105 N/m 2
n' = 1.3 n ( ` 30% of molecules P ,

# dP = Mω 2 # x dx

are dissociated)
& P RT
P0 0

` P' = 1.3P

= ]1.5g]1.3g # 105
& ln ^ P/P0 h = RT 2
Mω 2 , 2

= 1.95 # 105 N/m 2
mω 2 , 2

& P = P0 e 2RT
17.10 Physics
4. Considering both the gases have same number of
degrees of freedom.
Ui = n1 CV T1 + n2 CV T2

U f = ^n1 + n2 h CV T T ' + PπR 2 = P'πR 2


5P 2 2
n T +n T T ' = 4 0 πR - PπR

` T = 1 n1 + n2 2
1 2
2
5. P = P0 - AV
PV _ P0 - AV 2 i V
T = nR = nR

dT T = P0 πR 2 - PπR 2

At maximum temperature, dV = 0 .
dT 1 _ - 2 +- 2 AV 2 i P0 2

` dV = nR P0 AV T' - T # 4 πR
^ 0 - P hπR 2
T 100 = # 100
1 _ P - 3AV 2 i P

= nR 0
25P
dT P0
= P - 0P = 1 -25
P/P
dV is zero at V = 3A
0 0

8. Initially, let the no. of moles of gas in the container


c P0 - 0 m P
P
is x .
3 0

` Tmax = nR 3A
After 1 stroke, no. of mole of gas remaining
2P P b x - υ x l = x b1 - υ l
V V

= 3nR0 3A0

i.e., after n strokes, no. of moles remaining
^Vr.m.s. h' = a1/2 ^Vr.m.s. h = x b1 - Vυ l
n
6.

` Tf = a Ti
As temperature and final volume is constant
R R
x b1 - Vυ l
n

1 - ]- ag
C = CV + = CV + 1 + a Pf
P = x
∆Q = aui, nCv ∆T = anCv Ti .
Pf = P b1 - Vυ l
n

& C ]a - 1gTi = a CV Ti
&
a
Considering υ << v
& C = a - 1 CV
R a P
CV + 1 + a = a - 1 CV Pf =
b1 + υ l
n

V
R 1
& a + 1 = a - 1 CV
9. Average translational K $ E of both kinds of
a-1
& CV = a + 1 R molecules should be same.

7. Pi = P0, , Vi = b πR 2 + πR l , Ti = T0
2 1 2 1 2
2 m v A = 2 2m vB
4
Pf = P' , Vf = ^πR 2h 2 Tf = 2T0
1 ^ 2h = 1 ]2mg 2
& 2 m 3ω 2 v

P' 2πR 2 = 2T0 ω2 = 2



& 3
v2
P0 b 5πR l
2 T0
4
4

& P' 5 = P0
5P

& P' = 4 0
Kinetic theory of Gases 17.11
2 2 2 2
5 +6 +7 +8 +9
2
16. Pf ]1Lg = ^n1 + n2 hRT
10. Vr.m.s. = 100
5

= P1 V1 + P2 V2

= 100 51

= ]1atmg]1Lg + ]2g]0.5g
Vavg = 100 b 5 + 6 + 57 + 8 + 9 l
Pf = 2 atm

= 100 ]7 g
18. Avg K $ E of monoatomic molecule

Vr.m.s. is higher than Vavg by 14 m/s.

= 32 KT
11. Average translational K $ E of a molecule of any gas
is same at the same temperature.

= m43 2KT
m ; m is mass of Molecule
]Translational K $ E gmolecule = 32 KT
3mV 2
12. P0 A ]8 cmg = P' A x
= 4P

19. P = 3 ρ ]c g
1 2

P' + ρg ]52 - xg = P0
20. If mass is doubled and speed of all molecules is
halved, kinetic energy of all molecules becomes
P0 ]8 cmg = ^ P0 - ρg ]52 - xgi x cm
half. Thereby, temperature becomes half as volume
&
is constant, pressure also becomes half.
]76g]8 g = ]76g] xg 0 ]520xg x
m 1
V = ρ = 4 m3
21.
]76g]8 g = 24x + x 2
x 2 + 24x - ]76g8 = 0

PV = nRT

x = 15.4 cm 5
& U = 2 nRT, , for diatomic gas.
13. ^n1 hi + ^n2 hi = ^n1 h f + ^n2 h f

= 52 PV
2P0 V0 V0 V0

& RT0 = P d RT0 + 2RT0 n
= 52 PV
4P0

= 52 # 8 # 10 4 # 14
& 3 =P

( a both connected containers are
= 5 # 10 4 # 14

at the same pressure)

= 5 # 10 4 J
^4P0 /3 h^V0 h

^n2 h f = 2RT 22. PV = nRT
0

2P0 V0 P nR

= 3RT
& T = V
0

By considering 'n' is constant, slope of P vs T
14. ∆p = 2m V cos 45c
graph should be inversely proportional V .

= 2 mV

As V is reduced to half, slope of the curve should be

Momentum transfered greater than the earlier one.

per second = 10 23 2 mV
Ptransfered
pressure = Area

15. At constant temperature, kinetic energy of molecules


doesn’t change.
17.12 Physics
23. 26. For an ideal gas molecules, Average velocity and
momentum is zero in any finite time interval. (note:
Average speed is not zero).
27. 2 m ^Vr.m.s. h = 2 KT
1 2 3

Vr.m.s. depends upon mass of the molecule.


Therefore, option (a) is correct. Temperature is a
macroscopic quantity. It is not defined for a single
particle. ] K $ E gavg ? T ` Option (c) is correct.
By keeping temperature constant, pressure and
volume of monoatomic gas can be varied. ` option
(d) is correct.

= c V mV
dN a
P1 A = P2 A + mg .....(1) 28. dV 0

P0 A ]45 cmg = P1 ]45 - xgA = P2 ]45 + xg A


V0

& # dN = Va # V dV
0

(Considering temperature is constant) 0
aV
P0 ]45g P0 ]45g
& N = 20

& ]45 - xg = 45 + x + ρg ]10cmg
# ]dN gV
]76g]45g ]76g]45g
45 - x = 45 + x + ]10 cmg
Vavg = N

&
V0

& ]76g]45g c 2

2x
m = 10 cm
a
V0 # V2 dV
45 - x 2 0


9 ]76g x = 45 2 - x 2
= a V0

& 2

& x + 9 ]76g x - 45 = 0
2 2


V 3 2V
= 22 30 = 3 0
x - 3 cm V0
Vavg
P1 n1 T1 V = 32
24. P = n T , means that both container have same 0
2 2 2 V0

volume. If both the containers are joined, at # dN V


2

equilibrium, gas in both the containers are at same ^Vr.m.s. h =


2 0

temperature & pressure. # dN


P ]2V g = ^n1 + n2 hRT V0
a # V3 dV
^n1 + n2 hR
V0
P 0

& T = 2V
= a V0

^n1 + n2 hR
2
4

= n RT n RT V
1 1 2 2
= 22 $ 4 0

P1 + P2 V0
P1 P2 ^n1 + n2 h

V2
= 20

n1 P2 T1 + n2 P1 T2
P n V0

= 2T1 d1 + n2 n
& Vr.m.s. =
2
1 1
V0
P PT # a
= 2T1 d1 + P2 T1 n N = V0 V $ dV
V0

2 to V0
1 1 2 V0
2
= 2aV dV02 - 0 n
PT +P T V

= 1 22T T 2 1

1 2 0 4
25. During collision between two molecules. Total 3V0 a

= 8
kinetic energy of the molecules is conserved, with
K $ E of one molecule may increase and other may
= 34 c 0 m
aV
decrease.

2
Kinetic theory of Gases 17.13
dV 32. According K.T.G.,
29. =r
P = t ^Vr.m.s. h
dt 1 2
*
3

Consider ‘P’ is the pressure at t. ` ^Vr.m.s. h is not directly proportional to

In next small “dt” interval, “dn” number of moles of pressure.
gas is ejected. 1 3
* mV 2 = KT

nt = nt + dt + dn 2 2
` Average kinetic energy is directly proportional
PV0 ] P + dP g V0 P dV

& = + to absolute temperature.
RT RT RT

* Rate of diffusion is directly proportional to

“t + dt”, Remaining gas immediately occupies total
volume of the container. average speed of molecules.
33. All given statements are basic assumptions of KTG

& dP V0 =- P dV 34. At 0K1
dP - dV
* Pressure is zero.

& =
P V0
* Volume is zero.
P t
dP -1
* Kinetic energy of the gas is zero.

& # P
=
V0
# r dt
P0 0 35. At a given temperature, molecules of both the
- rt gases have same average, translatory kinetic energy

& ln P/P0 =
V0 b 3 KT l .
2
- rt
P = P0 e
But average total kinetic energy of hydrogen
V0
molecules is higher than average total kinetic
In addition, energy of Helium at degree of freedom are more for
PV = nRT Hydrogen.

When number of moles becomes half, pressure also 1 dV
36. γ = V dT
becomes half.

at constant pressure,
P0 -rt
= P0 e V nRT
2 V = P
0

rt

& ln 2 = dV = nR
V0 dT P
]ln 2g V0 1 nR 1

& t =
` γ =V $ P =T
r
30. Assumptions of Kinetic Theory of Gas.
As 'T' is given to be same for all gases, γ is same.


* All molecules of a gas are identical.
Average translational K $ E per molecule

* molecules are like rigid balls.
= 32 KT

* collisions are elastic and duration of molecular
mean free path is inversely proportional to pressure.
colliion is negligible.
37. ]∆Pgper molecule = 2mV

It doesn’assume all gases are identical.

= 2 ^6 # 10 -27h # 103
31. According to KTG,

= 12 # 10 24 kg m/ sec . -

molecules move randomly and doesn’t exert any
forces except during collisions.
number of collisions per unit area per seocnd

= ^103h^10 26h
17.14 Physics
38. * Ideal gas doesn’t get liquified at absolute zero PA V = n A R T
temperature.
PB V = nB RT ; nB = 8n A

* Actual gas behaviour deviates from ideal gas
due to pressure of intermolecular interactions
` PB = 8PA
and finite size of molecules. 41. * Mean K $ E per mole for diatomic gas - 5/2 RT

* Mean K $ E per molecule of a gas -c m KT,


f

* Kinetic energy of Ideal gas is zero at absolute
2
zero temperature. where f is no. of degree of freedom.

1
* Mean translation K $ E per mole of an ideal gas-
39. P = 3 ρ v r2.m.s. 3
2 RT .
3KT
vr.m.s. = m
* Mean K $ E per molecule of polyatomic gas-
7
P = V N
Ntotal
RT 2 KT (Including vibration)
avg
42. A gas sample is enclosed in a container. As

= NKT; N is number of molecules temperature is contiuously increasing,
* Internal energy of the gas increases.

per unit volume.
* Average momentum of gas molecules is zero and
40. 6]K $ E gavg@A = 6]K $ E gavg@B remains constant.
* Number of molecules moving with most

As both the gases are diatomic in nature and are at probable speed decreases.
the same temperature.
Vavg
_^V h i _^V h i
1 2 1 2 8RT

* V 3π ; remains constant.
2 m A A r . m. s . = 2 m B B r . m. s .
=
r . m. s .

^VA hr.m.s.
= 14
^VB hr.m.s.

&

& ]vrmsg2 =
Numerical Type
2 #
200.
3
1. Given,
& ]vrmsg2 =
400
m/s

The initial temperature of the gas T1 = 27°C = 300 K, 3
pressure of the gas P1 = 1 atm, and rms speed
]Vrmsg1 = 200 m/s.
2. PV = nRT

P1 V1 = nR 250

The final temperature of the gas T2 = 127°C = 400 K,
P2 ^2V1 h = 4 R # 2000
pressure of the gas P2 = 2 atm, and rms speed 5n

]Vrmsg2 = x m/s.
3
Divide
3RT

rms speed of gas molecules is, vrms = M . P1 4 # 250
2P2 = 5 # 2000
v
& rms \ T
]vrmsg2
P1 1
T2 P2 = 5

&
]vrmsg1
= T1
]vrmsg2
P2
400 2 P1 = 5

&
]vrmsg1
= 300 = 3

& ]vrmsg2 =
2 ]v g
rms 1
3
Kinetic theory of Gases 17.15
Single Option Correct
3 p = 2 mv cos θ
3
Total change in momentum = F = 2 mvcos θ × n per
3. Average translational K.E./molecule = 2 kT
second
4. PV = nRT F

Pressure = A
100 # 103 # 2000 # 10 -6 2 mn v cos θ
n=
25 #
= A
3 300
n = 80 # 10
-3 2 # 3.32 # 10 -27 # 10 23 # 103 # cos 45°

=
2 # 10 -4
n1 + n2 = 0.08

Also, n1 # 2 + n2 # 32 = 0.76
= 2.34 # 103 N/m 2

_0.08 - n2 i 2 + n2 ]32g = 0.76
9. ni Ti = n f Tf as pressure and volume are constant
& n2 = 0.02

nf T 290 105 # 30
& n1 = 0.06
& n = Ti = 300 and ni = ] g N A

i f R 300
n 3
So, n1 = 1

29
2 n f = 30R # 10 4 # N A

5. Both translational kinetic energy & rotational kinetic
energy obey Maxwell’s distribution independent of n f - ni =- 2.5 # 10 25

each other. Mean free Path
10. Average time between collision = Vrms
The translational KE and the rotational KE are
1 CV
both dependent on the degrees of freedom. Since, t = ;t =
πd 2 N/V T
for diatomic molecule, the translational degree of 3RT
freedom are 3 and the rotational only 2, the average M
energy in translational and rotational are different. M
Therefore, option (d) is correct. e where C = 2 = constant o
πd N 3R
c1 R c2 R
n1 C p + n2 C p n1 c - 1 + n2 c - 1 V2
6. cmixture = n C + n C =
1 2 1 2

& T ?
1 v 2 v n1 R n2 R t2
c1 - 1 + c2 - 1
1 2


For adiabatic
on rearranging we get
TV γ
-1
=k
n1 + n2 n1 n2 5 3 2
c mix - 1 = c1 - 1 + c2 - 1 ; c mix - 1 = 1/3 + 2/3 V 2
2 V γ 1 = k
-
t
5 17 5
c mix - 1 = 9 + 3 = 12 & c mixure = 12 = 1 + 12 Vγ 1 =
-
k

& c mix = 1.42 t2 γ+1
t ?V 2
3RT γ+1
7. vrms = m
so, q = 2
]v rmsg 40 n1 T1 + n2 T2 + n3 T3
]v rmsgAr
He = 4 = 10 = 3.16 11. t = n1 + n2 + n3
8. mH = 3.32 # 10 -27 kg
1 2
12. 2 Mv = CV .∆T
n/s = 10 23


wall area = 2cm2 1 2 R
2 Mv = γ - 1 .∆T

M.v 2 ^ γ - 1 h
∆T = 2R

^ γ - 1 h Mv 2

= 2R

17.16 Physics

ρ RT Cmix γ mix MHe


1. P1 = 1M .....(i) d nd n
1 CHe = γ He Mmix
ρ RT
P2 = 2M .....(ii)
2
= 32 # 53 # 34 = 6
5

by (i) and (ii)
Vrms He MH2 1
ρ1 8
Vrms H2 = MHe = 2
ρ2 = 9
3RT Numerical Type
vRms mHe m Ar 40 =
2. v He == mHe = 10 . 3.16 6. Assuming temperature remains constant at 300 K
Rms Ar 3RT 4
m Ar

From P1 V1 = P2 V2
3. At low pressure and high temperature inter molecular
forces become ineffective. So a real gas behaves like V0 V0
an ideal gas. P1 e o P'1 e - AX o
2 2
4. n1 >> ^n1 - n2 h = Dn
T
=
T
n1 RT n2 RT
p1 = p2 =
NA NA P’2
F = ^n1 - n2 h kB TS = DnkB TS (a)
DnkB TS x
V=
b
Force balance & Pressure # Area P’1

= Total number of molecules # bv
^ P'1 - P' 2 h A = mg

DnkB TS = ,n1 Sbv
RS V0 VWW
& n1 Bv, = DnkB T (b) SS = V0 G e oW
SS 1 2P P
2 WWW
2
^n1 vdt h S S
SS - WW A = mg
Total number of molecules/sec = SS V0 V0 W
dt SS + Ax WWW
DnkB TvS 2 2
T X
= n1 vS =
1 1
nRT < F = mg
bv,
-
4-x 4+x
Dn kB T
=d nf pS (c) 4+x-4+x
, b ]0 1g]8.3g=
. G = mg
16 - x 2
2x
3e o=1
5. U = nCV1 T + nCV2 T

16 - x 2

= 1 # 52 RT + 1 # 32 RT = 4RT
6x = 16 - x 2


& 2CVmix T = 4RT
x 2 + 6x - 16 = 0


Average energy per mole

= 2RT & CVmix = 2R
x=2


distance = 4 + 2 = 6 m
Thermodynamics 18.1

Thermodynamics
INTEXT EXERCISE: 1
the molecules of perfect gas are considered to be
1. W = # Pdv negligible.

` It depends on the thermodynamic process i.e. the
manner in which volume is changed. 6. Charge in internal energy

2. (i) Isochoric process (as volume constant)


= ∆Q + ∆Wext
` W =0
= mL - P0 ^V2 - V1 h

(ii) Isobaric process (as pressure is constant) 7. ∆u = nCv ∆T, is valid for all processes.
2V
8. ∆u = ∆Q + Wext
` W = # P dv
V
or ∆Q = Wgas + ∆u

= P1 ]2V - V g = P1 v

` u f - 30 = ]- 20g + 8
n ^CP h + n ^CP h2
3. ^CP hmixture = 1 n1 + n2 u f = 30 - 12
1 2

= 5 R/2 2 7R/2
= 18 J
+


9. PV = P1 2V

= 3R
P
1 ^Cv h1 + 1 ^Cv h2

` P1 = 2
4, ^Cv hmixture
= PA ]2V gγ
= 1+1 γ
PV
= 3 R/2 2 5 R/2
+

P

& PA = γ
2

= 2R P1 ]P/2g γ-1
PA = ^ P/2 r h = 2
_C p imixture
^Cv hmixture
` γ = 10. Condensation process occurs at constant temperature.
Therefore kinetic energy of the system remains
= 32R
R = 2
3
constant during condensation process but potential
5. The internal energy of a perfect gas is totally in energy decreases owing to decreased intermolecular
the form of kinetic energy, as interactions among interactions.

INTEXT EXERCISE: 2
Wext =+ 1572 J
mc
dp
= VP
-
1.
dv Isothermal
constant c as V = ρ m
P = m
-γ p 3.
c m
dp ργ
= V
dv adiabatic
P1 P
= 2γ
Wby gas = nRT ln ^v2 /v1 h
&
2. ρ1γ ρ2
P2 ρ2 γ

= 1 ]Rg]273g ln b 11 .2 l
& P1 = d n
22.4 ρ1

=- ]273g]Rg]ln 2g ρ2 P γ
1


& d n d 2n

=- 1572 J ρ1 = P1
18.2 Physics
4. ∆Q = Wby gas + ∆u V2 γ - 1

& d n =4

& Wby gas = 100J - n Cv ∆T V1

n CP ∆T = 100 J V2

& V1 = 16
Wby gas = n ^CP - Cv h∆T

Temperature reduced by 4 times.

= n R ∆T
8. PA v γ = p ]2V gγ

= 72 ]100g

& PA = p 2 γ

= 28.57 J
PB = P
5. Wby gas = # P dv
PC V = P ]2vg

If 'p' is constant

& PC = 2P

= P ^V2 - V1 h
PA : PB : PC = P2 γ : P : 2P
= Pc 0 -
α T 2 α T12 m


= 2 2 :1: 2
P P
= α _4T0 - T0 i
2 2

P1 V1 αV 2
9. T1 = nR = nR1 = 300 K

= 3αT02
2 2
P2 V2 αV2 4αV
6. WAOC > WABC T2 = nR = nR = nR1 = 1200 K

& ∆U AOC = ∆U ABC
T2 - T1 = 900 K

` ∆Q AOC > ∆Q ABC
7. In adiabatic process, 10. as P ? T, slope of the graph is equal to b nR
v
l

T Vγ
-1
= Constant V3 = V4, V1 = V2
T V3 < V2
T1 V1γ = 41 V2γ - 1
-1

&

INTEXT EXERCISE: 3

V ? T 2 /3 & V = K T 2 /3 V = K b TP l
1/3
1. As 3.
nR
P = K T1/3 ^a ρv = nRT h
V = K b nR l
1/3

& V
Wgas = # P dv V 4/3 = K ]nRg1/3
T2

` V is constant

= # nR 1 /3 2 - 1 /3
K T $ K 3 T .dT
i.e. given process is isochoric.
T1
4. A " B is Isobaric process.

= 32 nR ^T2 - T1 h

` Temperature increases in this process.

= 32 ]8.314g]30g U
` B > UA


= ]8.3g 20 = 166.2 J 5. Wby gas = ∆Q - ∆u
T2
- dP
B =]
dV/V g = # nc dT - n CV ∆T
2.

T1
V n P n dP + P n .dv = 0
-1
T2

T - γ-1^ 0
= a # dT R ηT - T h
dP = - P
0 dT

& dv nv T1

R ]n - 1g
= a ln ^η h - γ -
B =-b -nvP $ v l = P/n
1 T0
Thermodynamics 18.3
T 10
4
6. η = 1 - T1 Q = 15
2

T
& Q = 5 # 10 4 J
7. η = 1 - T1
2
10. In a cyclic process,
300

= 1 - 1000
Qnet = Wnet + Unet

= 0.7
Q1 + Q2 + Q3 + Q4 = W1 + W2
W =
Q 0.7

+ W3 + W4

& W = ]3000K Calg]0.7g

^a Unet = 0 h

= 2.1 # 106 Cal

& 5960 - 5585 - 2980 + 3645 = 2200 - 825

= 8.4 # 106 J
400
- 1100 + ω 4
9.
η = 1 - 500

& W4 = 765 J

= 1/5

EXERCISE - 1

1. ∆U = same is both processs 10. As f = 5



Qacd - Wacb = Qadb - Wadb nfRdT

dU = nCv dT = 2

200 - 80 = 144 - Wadb Wadb = 24 J
fR
2. ∆U = Qacb - Wacb = 200 - 80 = 120 J Cv = 2
5R

∆U = Qba - Wba, - 120 = Qba + 52, Qba =- 172 J C ` v= 2
3. Ub - Ua = 120 Ub = 120 + 40 = 160 J 11. For an ideal gas, heat capacity at constant pressure
is greater than heat capacity at constant volume as
4. in db. Wdb = 0 Ub - Ud = Qdb more heat energy is required to do positive work

160 - 88 = Qdb Qdb = 72 J against external pressure.
12. Gas has different specific heat for different processes
5. If temperature is doubled, pressure will also be
ρRT
` gas has infinite number of specific heats.
doubled as P = M
w 13. ∆U > 0
& 100% increase
and ∆W > 0
6. Internal energy and volume depend upon states.
` C > Cv
7. Work done = Area enclosed by cycle 14. As compare to gas solid expand very less.
= ^2P0 h^3V0 h = 6P0 V0
` CP is slightly greater then Cv .

8. As C p - Cv = R 15. P ?V
P = kV

for above equation, we can say that both C p and Cv
-1
increase by same amount.
PV = Constant
Q
C = cCV + 1 - γ m
R
9. S = m∆T

= c 2 + 1 - ]- 1g m
For changing state 5R R



t = const or ∆T = 0

= 3R

` s = 3 (Infinite)
18.4 Physics
16. ∆u = 3∆W γ 1

PV = C, P2 α .....(ii)
3nR ]∆T g
V 2γ

& ncv ∆T = 1-γ
From (i) and (ii)
Cv
C = Cv + 1 - γ d a - n
R R
P1 > P2

1 γ = 3
24. As W.D. by gas in isothermal is more as compare to
4C
= 3v adiabatic process

= 34 52R = 103R = 3.33


17. dQ = dW + dU

dQ = PdV + dU

dQ = nRdT + dU

;dU = E
2dU + nfRdt

dQ = f dU
2

` ∆W2 < ∆W1
dU = 1
dQ a 2
f + 1k
25. Adiabatic process

∆Q = 0
dU = 5
dQ 7
For any process

18. At constant volume


∆U = nCV ∆T

∆Q = nCv ∆T
Hence, option (c) is correct
VdP ]- PdV g
6300 = nCv 150K 26. B = dV =- dV (for isothermal process)

At constant pressure, B=P
∆Q = nCP ∆T dP
27. Slope =- γ dV
8800 = n CP ]150g As slope of A > slope of B

When temperature is increased by 300 K,
` γ of A > γ of B
∆u = nCv ]300g = 12600 J
or A " Helium
19. ∆Q = ∆W ]T = constantg B " Hydrogen

if heat is released then W =- ve 28. Gas undergoes cyclic process which includes
20. In isothermal expansion isothermal, adiabatic & isobaric process in sequence.


T = constant ∆U = 0 W = ∆Q 29. As ∆U = nR∆T For closed path

Option (d) is correct.
∆T = 0 ` ∆U = 0
21. As Volume decreases
P
30. T α P or T = constant

` pressure of the gas in the cylinder increases
P nR
22. For adiabatic
As T = V = constant
T Vγ

-1
=C ^ γ > 1h .....(i)
or V = constant

For isothermal T = const .....(ii)
` W=0

From (i) and (ii)
31. ∆U = 0 ` T = constant

T2 < T1
23. For isothermal
or PV = constant or P - V curve is a rectangular
hyperbola.
1

PV = C. or P1 α V .....(i)
1
Clearly, option (b) is correct

For adiabatic
Thermodynamics 18.5
V nR 1
∆U3 =- ve
32. T = P P ? slope

` ∆U1 > ∆U2 > ∆U3

or Pα
1 ` P2 < P1
slope
As volume increases, ∆W =+ ve
39. A " B, Pressure is constant
33. W.D. = π # Pressure Radius # volume Radius (area
of ellipse)
` V vs. T Graph should be a straight line

passing through origin.
W = πa ka k = π ^ P2 - P1 h^V2 - V1 h
P2 –P1 V2 –V1
2 2 4
B " C isothermal i.e. T is constant.
34. AB " isothermal

` V vs T graph is straight line parallel to

PA VA = PB VB .....(i)

y-axis, on which V is plotted.

BC " Adiabatic

C"D Volume is constant
PB VBγ = PC VCγ .....(ii)


D "A Temperature is constant.

CD " Isothermal
40. ∆u can be zero when ∆T = 0 , or in cyclic process,

PC VC = PD VD .....(iii)
(or) in the process, where ∆U =- W .

DA " Adiabatic

(b) option might be representing isothermal process.
PD VDγ = PA VAγ .....(iv)

T1 - T2 W T

From (i), (ii), (iii) and (iv) 41. η = c 1 m
T1 - Q & Q = T1 - T2 W
VB VA

VC = VD = (600600
- 300) # 800 = 1600J
T 273 273
35. P P P 42. K = T -2 T = 303 - 273 = 30 = 9
1 2

43. In a refrigerator, the heat dissipated in the


T T T atmosphere is more then that taken from the cooling
density
increases
density
decreases
chamber, therefore the room is heated if the door of
a refrigerator is kept open.
ρRT
For equation: c a P = M m
T 400 1 W 1 W
w 44. η = 1 - T2 = 1 - 500 = 5 ` η= Q & 5 = Q
1

At constant temperature ρ ? P
T 300 1
45. η max = 1 - T2 = 1 - 400 = 4 = 25%

For 1 graph: st
ρ ? P at constant temperature. 1

ρRT
So 26% efficiency is impossible.
ca P =

For 2nd graph: Mw m
1 46. Heat cannot flow itself from a lower temperature to

At constant P , ρ?T
a body of higher temperature. This corresponds to

For 3rd graph L : density ρ = constant second law of thermodynamics.

36. Volume can’t be negative. 47. T1 = 627 + 273 = 900K


At constant pressure Q1 = 3 # 106 cal

V ? T or V α ]t + 273g
T2 = 27 + 273 = 300 K
37. Here V = aT + b where a, b > 0 W T1 - T2

` =
nRT nR b b Q T1

So P = aT + b = a + b/T but T 1 T so P2 > P1
2 1
2
38. ∆U1 =+ ve
& W = # 3 # 106 Cal
3

∆U2 = 0
= 2 # 106 Cal
18.6 Physics

EXERCISE - 2

1. p1 = 20 # 105 N/m 2 ;
-6
v1 = 1500 # 10 m
3 7. P - V indicator diagram for a given sample of
5 2 -6 3
monoatomic ideal gas is shown in figure. The
p2 = 30 # 10 N/m ; v2 = 3000 # 10 m average molar specific heat capacity of the system
p1 v1 p2 v2 for the process ABCD is : (R is a universal gas
T1 = nR and T2 = nR
constant).
dU = nCv dT = n. 32 R. ^T2 - T1 h (C p) p0 V0

Q AB =- R
(p v2 –p1 v1)
= n. 32 R. 2 nR
(C ) p V
QBC = v R 0 0

= 32 ^ p2 v2 - p1 v1 h = 9000 Joule
4p V
Q = ∆U + W & + Q = ∆U + P0 ^V2 - V1 h
QCD = C p R0 0

2.

Average molar specific heat capacity
& Q = ∆U + P0 a ρ - ρ k
1 1

2 1 Q AB + QBC + QCD ; 3C p + Cv E

= =
& ∆U = Q + P0 a ρ - ρ k
1 1 η (TD - TA) 4

1 2

3. ∆Q =- 30, ∆W =- 18 R
8. c = CV + 1 - N .....(1)
T2 V
-3
∆u = ∆Q - ∆W = constant

=- 30 - ]- 18g
PV = nRT
2 2 -3
U f - 60 =- 12 J
P V V = constant
2 -1

P V = constant

& U f = 48 J -1/2

PV = constant
4. ^∆Q hAB = ]∆ugAB + ]∆W gAB

N =- 1/2

& 150 J = 40 J + ]∆ugAB 5R 2R 19R
C= 2 + 3 = 6

& ]∆ugAB = 110 J
9. When water is heated from 0c C to 4c C , its volume
]∆ugnet is zero in a cyclic process.
decreases.

` ∆u AB + ∆uBC + ∆uCA = 0 P
` ∆ V is negative
110 + 50 + ∆UCA = 0
Hence C p - Cv < 0 & C p < Cv

& ∆uCA =- 160 J 10. ]P = constantg
5. For an ideal gas ∆Q nCP ∆T CP 5
∆W = nR∆T = R = 2
CP - CV = R
11. Process1 " 2 and Process3 " 4 are isochoric process.

if CP - CV = 1.09 R
W12 = 0

or p A > pB TA < TB
W34 = 0
W23 = n R ^T3 - T2 h

Then gas will be real. Thus pressure is high and
temperature is low for real gas.
6. CP = 3.5 R (At STP)
= 3R ]2400 - 800g = 4800 R


As temperature increases, vibrational degree W41 = nR ^T1 - T4 h
of freedom becomes 2 at higher temperature as
= 3R ]400 - 1200g =- 2400 R
molecular vibration becomes effective.
Wtotal = ]4800 - 2400g R = 2400 R
9

CP = 2 R = 4.5 R

= 20 kJ
Thermodynamics 18.7
12. Since elasticity of balloon is negligible ` pressure 16. ∆Q1 - ∆Q2 = n ^CP h1 ∆T - n ^CP h2 ∆T
inside baloon - pressure outside balloon Patm

= n 7^CP h1 - ^CP h2A ∆T

` W = Patm ∆V
Vin = 10 litre
= n b 72R - 52R l ∆T

Vfin Vin Tfinal


= nR ∆T
Tin = T & Vfinal = d T n litre
V
in fin in
∆u1 - ∆u2 = n 7^Cv h1 - ^Cv h2A ∆T
Tfinal

& w = Patm Vin d T - 1 n
= nR ∆T
in


` ∆Q1 - ∆Q2 = ∆u1 - ∆u2
& 10 # 10 b 290 l = 200 J
5 -2 58
∆Q2 > ∆Q1 & ∆u2 + ∆W2 > ∆u1 + ∆W1
13. A is free to move, therefore, heat will be supplied at
∆v2 > ∆v1
constant pressure

` dQ A = nCP dTA 17. As initial and final state are same


B is held fixed, therefore heat will be supplied at
` TI = TF As Vrms, Pav and K av
constant volume
Depends on temperature

` dQB = nCV dTB
` all are equal

But dQ A = dQB
18. B " A

` nCP dTA = nCV dTB

∆Q = 0
Cp
` dTB = d C n dTA = γ ^dTA h

0 =- 30 + ∆UBA
V
dTA = 30K = ]1.4g]30 K g
∆UBA = 30 J

` dTB = 42 K
` ∆U AB =- ∆UBA =- 30 J
ρ 19. XY Adiabatic compression
14. Ideal gas equation P = M .RT
For state A YZ
Isothermal Expansion
ρ ZX
Compression at constant pressure
P0 = M0 R T0
For state B 20. Given: P ? T3 .....(i)

ρ 3
In adiabatic process
3P0 = M R2T0 & ρ = 2 ρ0
T γ P1

= constant
15. WA " B + WB " C + WC " A = 12 $ 12 # 105
1

p(1 γ)/γ
5 3 -
1.5 # 10 + 0 + WC " A = 25 # 10 J
^ γ /γ - 1 h
WC " A =- 125 kJ T ? P .....(ii)

U A " B + UB " C + UC " A = 0


Comparing equations (i) and (ii), we get


& UC " A = 1 # 105 J γ

` γ-1 = 3

= 100 kJ 3γ - 3 = γ
QC " A = WC " A + UC " A 2γ = 3

=- 125 kJ + 100 kJ CP 3
CP = γ = 2

=- 25 kJ
18.8 Physics
21. For adiabatic process n RT n RT n RT

= AV - 2A V = 2A V .....(1)
γ γ
P V 1 = PA V 2

In Chamber B "
V γ
PA = P c V1 m .....(1) 1.5∆P
2

^ PB hi - ^ PB h f =

For isothermal process nB RT nB RT nB RT
V - 2V = 2V .....(2)
P V1 = PB V2

From (1) and (2)
V
PB = P V1 .....(2) NA 1 2
2 NB = 1.5 = 3

From (1) and (2)
m A /M 2

PA < PB [For expansion V2 > V1 ]
or mB /M = 3
mA 2

and by PV = nRT TA < TB
or mB = 3
22. TV γ
-1
=C
or 3m A = 2mB

26. WAB = ^2V0 - V0 hP0 = P0 V0



γ-1 = n
6

n = 13

[Isoboric process]
7 5 V
4# 2R+2# 2 R 19 WBC = nR (2T0) ,n 2V0 = 2P0 V0 .,n2


γ= 5 3 = 13 0
4# 2R+2# 2R
[Isothermal process]
23. Q AB = ∆U AB + WAB 3
Q P0 V0 + P0 V0 5
` WAB = 2 2P V .,n2 = 4,n2

WAB = 0 BC 0 0

f f
∆U AB = 2 nR∆T & 2 ]∆PV g 27. PV γ = C; ,nP + γ,nV = ,nC

& ,nP =- γ,nV + ,nC & y = mx + c
∆U AB = 52 ]∆PV g - [2.10 - 2.38]

& m =- γ =- (1.30 - 1.10) =- 1.4
Q AB = 2.5 P0 V0

` The gas is diatomic

Process BC
nR (T1 - T2)
QBC = ∆ UBC + WBC 28. Wadiabatic = γ-1 = 6R

QBC = 0 + 2P0 V0 ,n2 = 1.4 P0 V0 1 # R (T1 - T2)



& 5/3 - 1 = 6R & T2 = ]T - 4g K
Qnet = Q AB + QBC = 3.9 P0 V0
29. ∆UCA = nCv ^TA - TC h
24. There is no heat transfer in adiabatic compression.
= 1 # 3 R # ]300 - 450g
In isothermal process


2
V

Q = W = P1 V1 ln V2
= ]- 225 Rg
1

∆U AB = nCv ^TB - TA h
= 400 # 12 ln 14 =- 6654 J


= 1 # 32 R # ]600 - 300g =+ 450 R
25. Process is isothermal. Therefore, T = constant
b P ? 1 l volume is increasing therefore, pressure ∆UBC = nCv ^TC - TB h
V
will decrease.
= 1 # 32 R # ]450 - 600g

In chamber A "
=- 225 R
∆P = ^ PA hi - ^ PA h f ∆U ABCA = ∆UCA + ∆U AB + ∆UBC = 0
Thermodynamics 18.9
30. Work done in process 1 - 3 is greater than that in 38. U = a + bPV = a + bnRT
proces 1 - 2 . While change in internal energy is
same for both processes ` Q2 > Q1
& ∆U = bnR∆T = nCv ∆T

31. In free expansion, work done is zero. C


& v = bR & C p = bR + R
C p bR + R b + 1
∆Q = ∆u + ∆W
& γ = C = bR = b
v

= ∆u
39. P 2 V = Constant

As the container is fully insulated.
b Tv l V = constant
2

∆Q = 0 Therefore, ∆u = 0
T 2 V 1 = constant
-

i.e. no change in temperature

` P becomes half. T12 T22
V1 = V2
32. P PV n = constant

as V2 = 2V1 & T2 = 2 T1

∆Q = n bCv + 1 - x l ∆T
R
n1 40.
n2
V
= n b 32R + R l
2 ∆T
PV n = constant, n2 > n1


= 2nR∆T

As n increases, work done by gas decreases, for
same change in volume.
= 2RT0


` work done by gas is minimum when n is 41. As PV = nRT m = ρV = constant
maximum. 1

or ρα V and Pα ρ
33.

A " B T = constant, pressure increases or volume
decreases

B " C Volume is constant, V = constant

C"C P is decreases or volume increases
5T = constant?
2

So, final pressure is maximum for PV = constant.
D " A Volume is constant .
clearly option (b) is constant.
1 k
34. Pα & P= & PV 2 = k
V2 V2 42. L " M P = constant

& PV.V = k & nRTV = k & TV = k1
Here, option (b) is constant.

Since temperature increases therefore volume
decreases or work done by the gas is negative. 43. As volume increases

` Work done continuously increases
35. PT11 = Constant & PV = nRT
∆V ∆T ∆V 12 44. As W = P∆V ∆V = same is both process
V & ? T12 & V = 12 T & γ v = V∆T = T
36. ∆U = 2P0 ∆V & W = P0 ∆V
As PB > PA ` ∆W2 > W1


So Q = W + ∆U = 3P0 ∆V = 3P0 V0 45. Correct graph is shown in option (a)
37. V ? T & V ? ]PV g
4 4

Process 1-2 adiabatic process,
& 4 V3 = constant & PV3/4 = constant
P
Process 2-3 Isochoric process,
R R
Process 3-1 Isothermal process

` C = Cv + 1 - x = 3R + 1 - 3/4 = 3R + 4R = 7R
18.10 Physics
46. Work done by gas = Area under P-V diagram
` P ]vsg T graph is a straight line passing
π (4–3)(4–2) π (2–1)(3–2.5) through origin.

= 2 + 2 C " A, Temperature is constant.
2 .5 π 5 π Option (a) is correct
= 2 = 4 atm L `

W =- ` 4 j atm L
5π ∆P nR
53. ∆T = V

(Work done by gas is negative as cycle is As volume is constant, P ]vsg T is a straight line.
anticlockwise)
As V is different for both the vessels, slope is
different.
47. Here, t = V tan 45c + T0 & T = V + T0
54. Comparing process (1) and (2)
nRT

and P= V

Expansion in the process (1)
PV = is done at higher pressure.

` R V + T0 (Since, n = 1 )

` ]P - Rg V = RT0
` W1 > W3

Therefore, Graph will be rectangular


Comparing process (1) and
(2)
hyperbola.

` W1 > W2
1
48. W = 2 P0 V0, TA = T0, TB = 2T0, TC = 4T0
` W1 > W2 > W3
Heat supplied 55. From the given graph, pressure is constant.
= AB + QBC = CV T0 + CP 2T0 = 13
Q
13
2 RT0 = 2 P0 V0

` ∆Q = ∆CP ∆T, ∆W = nR∆T

` Efficiency ∆Q CP γ
γ∆W = R = γ - 1
1
P0 V0
# 100 `a 13
2 P0 V0 = 2 RT0 j
13 1
= 132
56.
Work done = 2 P0 V0
2 4P0 V0
As the cycle is anti cyclic, work done.
1 #
= 13 100 = 7.7 %
57. A " B, Temperature is constant as pressure
49. As process BC is isochoric. increases, volume decreases, i.e. work done by

VB > VA, VB = VC, VD > VC gas is negative.

` option (d) is correct
C " D, volume increases
50. Q1 = ∆U + W1; Q2 = ∆U + W2
` workdone by gas is positive.

Ratio of specific heats B " C, Temperature is increasing.

C1 a ∆T k a ∆U + k
∆Q1 ∆W1
` Internal energy increases.
= == ∆ T ∆T 1 1 ^a W > W h
a 2k a ∆U + k
C2 ∆Q ∆W2 2 1
58. Efficiency of the cycle
∆T ∆ T ∆ T
work done
51. WOBC = WODA
= heat supplied

But WOBC is negative and WODA is positive. 2P0 V0

=Q +Q
Therefore, Wnet = 0 B"C A"B

52. A " B, pressure is constant as V is


heat is released during C " D & D " A.

QB " C = ^3P0 V0 h + n 2 ∆T
proportional to T . 3R


` P vs T graph is a straight line parallel to

= 3P0 V0 + 32 ^6P0 V0 - 3P0 V0 h
axis on which T is plotted.

= 15
2 P0 V0
B " C, volume is constant.
Thermodynamics 18.11

Q A " B = 0 + 2 ^3P0 V0 - P0 V0 h
3
& Q1 = 4.8 # 10 J 4

η1 = 1 - T2 , ^W1 h = η1 Q1

= 3P0 V0 T
62.
2P V 1
η = 21 0 0 T
2 P0 V0 η 2 = 1 - T3 , W2 = ^η 2 h Q2
2
4
= 21 η Q

as W1 = W2 & η1 = Q2
59. As the pv graph is anti-cyclic, area of ABCD is work 2 1

done by gas. T -T T T
& T1 - T2 $ T2 = T2

2 3 1 1

∆Qnet = ∆Wtotal + ∆vtotal = 0

& T1 - T2 = T2 - T3

` ∆Qnet = ∆Wtotal

=+ ve, from graph T1 + T3

& T2 = 2

` heat is absorbed.

i.e. II and III are correct.
= 11000
2 = 550 K
T
60. η = 1 - T2 63. From the derivation of efficiency of carnot cycle,
1
efficiency is independent of atomicity of the gas.
300
0.4 = 1 - T
T1 - T2
300 = h =
T 0.6 T1


&
300
T = 0.6 = 500 K W 684 - 342
=
Q 684
300
T' = 0.4 = 750 K W 1

& =
T' - T = 250 K 1000 2
Q2 T2
& W = 500 J
61. Q1 = T1
` area enclosed by PV diagram in both the cases is
Q1 400 500 J.

& 4 = 500
6 # 10

EXERCISE - 3

Numerical Type 3. Process A → B :


1. Q = 7 J; 3 Q = 3 U + W 3 1/2 3 1 /2 1
#
WAB = P dV = #2
T dV =
2
#
T # RT -1/2 dT
3
5 & WAB = 50R = 50 # 8.3 = 415 J

& 7 = nCv 3 T + PdV = n R 3 T + nR 3 T
2
7 1 1 /2

7= (nR 3 T) & nR 3 T = 2 J
Process B → C : U = V
2 2
3 1
2. OU AB + OUBC + OUCD + OUDE = OUif RT = V1/2 & 3PV1/2 = 1
2 2

0 + (- WBC) + 0 + (- WDE) = OUif 1
P
` =
3 V

- 40 J + (- (- 25)) = OU-if 1600
1

- 40 + 25 = OU-if

Now, WBC = # P dV = # dV
3 V
100
2 2 2

OUif =- 15 J
= V = [40 - 10] = # 30 = 20 J
3 3 3

(For path AB : Isothermal process QAB= WAB = 50 J)
Total work, W = 415 + 20 = 435 J
18.12 Physics
nRT 2 c
4. PT = Constant & = Constant 9. 3 Q AB = nC p 3 T = nR 3 T
V c-1
dV 2nRT c c

& =
= [3P0 V0 - P0 V0] = 2PV0 #
dT Cons tan t c-1 c-1
B B
Cons tan t

W= #
P dV =
T
#dV
56 = 2P. V0 #
c
A A c-1
100
Constant 2nRT Q
3 AC = 3 U + 3 W

&W= # T
.
Constant
dT
300 nR 1

= 3 T + # 3V0 [P0 + 4P0]
P T c-1 2

& PA = TB
B A [16P0 V0 - P0 V0] 15P0 V0
1 TB +

& = c-1 2
3 TA
c+1
300 & 360 = 15P0 V0 >
H

& TB = = 100 & W = 2nR (100 - 300) 2 ( c - 1)
3
W
& AB =- 400nR
(ii) ' (i), we get

5. For isochoric process : dQ = nCv dT 360 15 (c + 1)


=
56 4 c

For isobaric process :

12g = 7g + 7

dW = nRdT = n(Cp - Cv) dT
7 2
c= = 1+
dQ Cv 1 5 f

` = = =4
⇒f=5
dW C p - Cv 1.25 - 1
10. Temperature is constant,
6. Wnet = WAB + WBC + WCD + WDA
nR (TB - TC) dV Adx
= 9 # 10 4 + + P (V2 - V1) + 0
&3 E = 0, dW = nRT = nRT
r-1 V AL/2
PC VC dW nRT dx
a TB = TA = 300 K and TC =
& =
nR dt L/2 dt
100 # 8 (300 - 250)
Q = ΔE + W
& Wnet = 9 # 10 4 + + 105 (1 - 2) + 0
5
-1 dQ dW 1 3T 2nRT dx
3 = & K# # = d n
= 5 × 104 joule dt dt 900 L L dt
7. Wgas + Wspring + Watm = 0 dx K # 27 415.5 # 27
& = =
dt 900 # 2nRT 900 # 2 # 0.5 # 8.31 # 300
1

Wgas - # 25 # 103 # (0.2) 2 - 105 # 0.05 # 0.2 = 0 1
2
= m/s = 5 mm/s
200

Wgas = 500 + 1000 = 1.5 kJ V2

8. Since walls are adiabatic, heat lost to the surrounding 11. W = # pdV
V1
is zero. Movable saperator is highly conducting
V2


⇒ Temperature remains same in both the chambers
= # b av + b l dv
at all time V1


⇒ Separator doesn’t move or work done by gas = 0
= a ln ^V2 /V1 h + b ^V2 - V1 h
nC
` v dT = dQ
= 10 ln 2 + ]2.1g]1 g
3
⇒ dQ = (2 # n) d R n (2T0 - T0)

= 6.93 + 2.1 = 9
2

⇒ dQ = 15 kJ
Thermodynamics 18.13

PV = nR _T0 + α v i T
2
12. 17. η1 = 1 - 8

P = Rc m
T0 2

V +αv η2 = 1 - T
dp

at minimum value dv = 0
as η1 = η 2

- T0 T 2
R d + αn = 0 1- 8 = 1- 7
V2
T0 T = 2
8 T & T = 4k
V = α

` Pmin = R _ αT0 + T0 α i 18. ∆u = n Cv ∆T


= αR 2 T0 4 W =- nR ∆T

` N =4
∆Q = n C p ∆T
R
13. C = Cv + 1 - x ∆u Cv 5
∆Q = C p = 7
= 3R + -R1
= 2R
` N =7


` N =2 19. 1 " 2, isochoric process
-b
14. PV = Constant W12 = 0
R
2 " 3, Isobaric process
C = Cv + 1 + b
R - 5R W23 = nR ^T3 - T2 h

asC = 0,
1+b = 2
-2
= 3R ]1600g = 4800R

& 1 + b = 5

3 " 4, Isochoric process

` - 5b =7

` W34 = 0
700
15. Maximum Efficiency = 1 - 2100
4 " 1, Isoblaric compression,


=2
` W41 = 3R ]- 800g
3
^2 5 h
=- 2400 R
Fraction = 2 = 53
3

` Wnet = 0 + 9800R + 0 - 2400R

` N =3
T 1
= 2400 R
16. n1 = 1 - T1 = 6
2

= 2.4R kJ = 20 kJ
T1 5
T2 = 6 N =

` 5 4
T1 - 65

& T2 = 32 # 56


& 3T1 = ]15g]65g


& T1 = ]65g 5

T2 = ]65g6


` T1 = 5
18.14 Physics
20. P Wnet = W021 - W043
P2 3 4
= ^ P0 - P1 h^V2 - V1 h
1
P0 0

2
1 2
- ^ P0 - P2 h^V4 - V3 h
P1 1


V 2
V1 V2 3
10

= 103 -

As Dle 021 & Dle 043 similar triangles, 4
3
P0 - P1 P2 - P0
= ^103 h J
4
=
V2 - V1 V4 - V3

` N =4
V4 - V3 = ^V2 - V1 h
1

&
2

EXERCISE - 4

1. (1) C ]∆T g = ∆W + ∆u 4. WAOC > WABC


U AOC = U ABC
= 12 ^4V0 h^9P0 h + ]1 g Cv ∆T

` Q AOC > Q ABC
= ]18g R27
∆ T + 3R
2 ∆T dQ dw du
5. dt = dt + dt
= b 32 + 32 l R du

as dT = 0, dt = 0
= 13
6 R
dQ 2 dW = mg dx
& dt = i R & dt dt
2. If T is doubled, by
= mg v


Keeping volume
` i 2 R = mg v
constant, pressure get i2 R
doubled
& V = mg


Tension in the string = P0 A 6. A " B, Adiabatic process

3P P nR ^T2 - T1 h
3. p = 2 0 - 2V0 V WAB =
1-γ
0
PV - 3RT0
T = nR
= 4
1 d 3P0 P0 2 3RT

n WBC = 2 0
= nR 2 V - 2V0 V
dT R ^- T0 h

at Tmax , dV = 0 WCD = 1 - 5/3
3P0 P0 3RT
nR < 2 - 2V0 ]2V gF
1
= 20

&
3 -V = WDA = R ^- T0 h

& 2 V0 0
5
3V Wnet = 4 RT0 = 1.25 RT0

& V = 20

T = c 0 $ 0 m nR
3P 3V 1

4 2
9P0 V0

= 8nR
Thermodynamics 18.15
V2 12. Both parts x and y of the chamber are thermally
7. Wby gas = # P dV insulated.
V1
5
V
for part 'x' , c =

= nRT ln d V2 n 5
3
5
1
V 3 -1 2V 3 - 1

= RT ln ]ng T $d n = T' d n
3 3
8. p ]5V g = p'V1 ' T

& T = 2/3
10p ]v g = p' V2 2
7

At equilibrium, pressure should be same in both the
For part y , c =
5
chambers. 7 7
2V 5 - 1 V 5 -1
V2 = 2V1 Td n = T" d n
3 3
V1 + V2 = 6V
T = T ]2 g3/5
"

&
V1 = 2V1 , V2 = 4V

Considering total chamber as thermodynamic
9. Pi = 1 # 105 N/m 2
system,
Vi = 2.4 # 10 3 m3
-
DWext = Du - DQ
= ]Dugsystem
Ti = 300 K


Pf Vf Tf 3R 5R "
= T ;
kx
Pf = Pi + A
= ]1 g ^T' - T h + 2 ^T - T h
Pi Vi i 2 2

2 # 10 # ]3.2g # 10 Tf 3R 1
e 2/3 - 1 o + 5RT ^2 2/5 - 1 h
5 -3

=
^1 # 10 h # ^2.4 # 10 h
& 5 -3 = 300 2 2


& Tf = 800 K
= RT

= ]8.314g T J
10. From the given graph,
13. Case-I
Uρ = constant,
ρ is constant, as T ]vsg V graph is straight line


as U = n Cv T , Tρ = constant passing through origin.
p = ρ RT/M
W1 = P V0

i.e. p is constant and volume decreases as density 2V0
increases.
W2 = # P dV

` (d) is correct. V0
2V0
11. ∆Q1 = Wby gas + ∆u T

= nR # V dV

= mg ]∆xg + p0 ]∆V g + nCv ∆T V0
2V0


= ]80gb 15 l + ^105h^60 # 10 -4hb 15 l
= nR V0 T0 # dV
2
V
+ n Cv ]70g
V0


nRT PV

= nR V0 T0 21V = 2 0 = 2 0
∆Q2 = Wby gas + ∆u 0


= 0 + n ^Cv h]- 70g
W1
` W2 = 2 : 1
∆Q1 - ∆Q2 = 16 + 120

= 136 J
18.16 Physics
14. Pi ]0.83g = ]1 g]8.3g]100g nfR 3 T
22. 3 Q = 3 U + 3 W & 25 = 2 +0
Pi = 103 N/m 2 1 # f # 25 # 2

25 = 2#3
Wby gas = # b Pi + kxA lA dx f = 3 (monoatomic)
0. 1


Where x is further compression 23. CP > CV

and CP - CV = 2
= 103 ^10 -2h^10 -1h + 10 2 b x l
2 0. 1


2 0

` option (a) and (b) is correct

= 1 + 0.5
P2

= 1.5 J 24. Equation of process & ρ = constant = C .....(1)
P nR
15. di = heat supplied to the gas. Equation of State ρ = M T

d~ = work done by system From 1 and 2

du = dQ - d~
PT = constant & (c) is false, (d) is true.

For isobaric process, du = dQ - d~ ρ P
As ρ -changes to 2 & P changes to

For adiabatic process, du =- d~ 2
From equation (1) & (a) is false

^a dQ = 0 h
Hence T changes to 2 T & (b) is true)

For Isothermal process, du = 0

If gas expands, ]dugisobaric > 0 25. For equilibrium of piston
` ]dug is least under adiabatic process.

PS = Kx0
16. W = PdV then W =- ve Kx
P = S0


As pressure and volume both decreases

For piston Wall = KE2 - KE1

` temperature of system decreases
1 1
Wgas - 2 kx 2 = 2 mv 2

17. 3 U = 3 Q - 3 W is same in both methods as it is a
state function. 1 1
Wgas = 2 kx 2 + 2 mv 2 = positive

18. In cyclic process 3 U1 + 3 U2 = 0 ∆Unet = 0
∆Q = 0
Q
3 -3 W = 0 Q
3 =3 U + W
19. Specific heat of a substance can be finite, infinite,
∆U =- W = negative
zero and negative.

As internal energy of gas decreases ` temperature
20. From information, the process may be adibatic or of gas decreases.
isothermal.
26. For any process 3 U = nCV 3 T
21. ∆U = 0

U = constant
For isothermal ∆T = 0


nCV T = constant
or U = constant


As Q2 and N2 are diatomic, so there temp are equal
∆Q = 0 (For adiabatic process)

but if different from He


` ∆U + W = 0
For adiabatic PV γ = constant
∆U =- W

For O2 , N2 value of γ is same ` pressure of O2 ,
N2 remain same but different from He .
Thermodynamics 18.17
27. V, T, P 2P1 T2 2P

Left compartment
PV m1 R
T = M

Right compartment

2P2V m2 .R
T = M
So By first law
m2 4m1
Q = ∆U + W
M = M
f

n1 + n2 = 5
Q = 2 nR∆T + W Q AB > Q AC

n2 = 4n1 31. U f - Ui = dU = Q - W = 50 - 20 = 30 Cal.

5n1 = 5
Qibf = 36 cal, dU = 30 cal.

n1 = 1
Therefore Wibf = Qibf - dU = 6 cal.

n2 = 4
For path ‘fi’ ∆U =- 30 cal

Right compartment has 4 mole and left has 1 mole
W =- 13 cal.
PV P´V´
Q =- 30 - 13 =- 43 cal.

Let p' be the final pressure for left T = T for
2T # 2V = P´ (3V - V´)
Negative Q means heat has been rejected by the gas
right T T

Again as U f - Ui = 30 cal and Ui = 10 cal

4P'V' = 3P'V - P'V'
Then U f = 40 cal.

5P'V' = 3P'V
Along path ‘ibf’ W = 6 cal.
3

V' = 5 V
again as Wbf = 0

for volume of right
therefore Wib = 6 cal
3V 12V
Ub - Ui = 22 - 10 = 12 cal.

3V - V' = 3V - 5 = 5
Hence Q = Wib + ^Ub - Ui h
= 18 cal.
P0 V0
28. (a) TA = nR and 32.
P /2.2V P0 V0
TB = 0 nR 0 = nR

` TA = TB


(b) As TA = TB, ∆U = 0


(c) workdone = area under the line AB with V

(a)
# ^2V0 - V0 h = 40 0
P0 + P0 /2 3P V
2

(d) As ∆U = 0 and W > 0 & ∆Q > 0

n1 CV + n2 CV
29. CV = 1

n1 + n2
2

CP
(c)

and CPeq = CV eq + R; γ= C
eq

Veq

30. From equation PV = nRT



PC < PB, VC = VB and TB > TC

WAB > WAC ^TB - TA h > TC - TA
18.18 Physics
33. For anticlockwise, work is negative i.e., work is 40. Slope of isothermal process
done on the gas, hence option (a) and (c) are correct. ∆P P
m1 = ∆V =- V

Slope of adiabatic process

∆P γP C
m2 = ∆V =- V =- γm1 = CP m1
v

41. Work done = Area of ABC with V-axis

= 0 ^2V0 - V0 h + 0 = P0 V0 = nRT0 = RT0


34. There is a decrease in volume during melting of
P
an ice slab at 273K. Therefore, negative work is
done by ice-water system on the atmosphere or
Change in internal energy = nCV ∆T
positive work is done on the ice-water system by the
atmosphere. Hence option (b) is correct. Secondly = 1 # 32 R # ^4T0 - T0 h = 92 RT0

heat is absorbed during melting (i.e. dQ is positive )
and as we have seen, work done by ice-water system
` heat absorbed
is negative (dW is negative). Therefore, from first
law of thermodynamics = 92 RT0 + RT0 = 11
2 RT0

dU = dQ - dW 42. CP - CV = R; M ]0.20 - 0.15g

= 2 ] M = molar massg

Change in internal energy of ice-water system, dU


will be positive or internal energy will increase.
2
35. For insulated chambers
& M = 0.05 = 40 g

n1 + n2 = n'1 + n2 ' CP 0.2 4 2
CV = γ = 0.15 = 3 = 1 + f

(final pressures become equal)
PV + 2P.2V = P 5P
& f = degrees of freedom
RT RT RT [3V] & P' = 3

For left chamber 43. A
T
P0
5P 3V

PV = P'V' = 3 V' & V' = 5 P (V,P)


For right chamber P0 B
5P 12V 2
4PV = P'V' = 3 V' & 5 V
V0 V 2V0(Let) V0 2V0
36. Fig. A: WA =+ ve Fig B: WB =+ ve

Let V0 = initial volume = 2V0 = final volume

Fig C: WC =+ ve Fig D: WD =- ve

` In process Fig (D), heat is released.
V = volume of any state

37. Q = ∆U + W and 3 U = nCV 3 T P - P /2 P - P /2


then 20V -0V = 2V -0V

0 0

∆U can be zero if ∆T is zero or Q - W is zero.
3P P
38. At constant volume, work done by gas is zero. & nRT = PV = V ; 2 0 - 2V0 V E
0

39. For any process 3 U = nCV 3 T


` T-V curve is a parabola with vertex above.

In adiabatic process

Hence temperature first increases then decreases.

Q = 3 U + W = 0 & 3 U =- W

For any process

C = n 3 T + n 3 T = n 3 T = n 3 T + n a 3T k
3U W Q 3U P 3V


For Q = 0, C = 0 (adiabatic process)
Thermodynamics 18.19
44. PB V γ
= PA V γ
5
B A
& n1 = 3 n2
PB
PA = ]32g
7 /5

&
Substituting in (1)
PB
b 5 Rl
7

& PA = 2 CV

5 n CV = n C & 5 2
CV = 3 = b 3 l
2

TB PB VB 27 3 2 1 2 V
2R
2
2
TA = PA VA = 25 = 2
1

TB = 4 ]300g = 1200 K
Hence, Gas B is diatomic and gas A is monoatomic

48. since n1 = 3 n2 Therefore M = 3 a M k



B " C, is isobaric process 5 125 5 60
A B

` T ?V

(From experiment 1: WA = 125 gm & WB = 60 gm )
VC TC
VB = TB & TC = 2TB
& 5MB = 4M A

= 2400 K
The above relation holds for the pair–Gas A : Ar and
Gas B : O2

C " D is adiabatic process
125
49. No. of molecules in ‘A’ = nN A = 40 N A = 3.125 N A
TD PD VD VC r VD VC r - 1
TC = PC VC = d n $ VC = d n
VD VD 125
(Since n = 40 for Ar )

= ]16g0.4
50. Internal energy at any temperature T

= 13
= nCV T
TD = ]2400g]0.33g = 792 K

= b 125 lb 3R l]300g
40 2
46. For adiabatic,
3R
3P - γp [ a CV for mono atomic gas = 2 ]
PV r = constant ; 3V = V

& Ui = 2812.5 cal

For Isothermal,
51. Let initial temperature and volume be T0 and V0 .
3P -P
PV = constant3V = V ; Since the process is adiabatic, the final temperature
5
and volume is TV γ 1 = T0 V0γ 1 [ γ = 3 for mono
- -

For same change in P by ∆P , ∆V is smaller in the
case of adiabatic process. atomic gas]
2
V 3
T = T0 c V /08 m = 4T0

` ∆V2 < ∆V1

`
0
3 V1

` 3 V2 = γ
` percentage increase in temperature of gas is
47. Heat given: 3 Q = n1 CV 3 T " for gas A ∆T # 3T
1 T0 100 = T 0 # 100 = 300%
0

7 for gas B -3 Q = n2 CV 3 T
dP nRT
52. Adiabatic Bulk modulus B =- V dV = γP = γ V
2


( a For same heat given, temperature rises by same
value for both the gases.) B T V T V /8 1
a B i = V0 # T = V0 # 40T = 32
f 0 0 0

& n1 CV = n2 CV .....(1)
53. For adiabatic process dQ = 0
1 2

Also, ^3 PB hV = n2 R 3 T and ^3 PA h V = n1 R 3 T

dW =-

` dU + dW = 0 or dU 1

& n1 CV = n2 CV .....(1)
1 2

Also, ^3 PB hV = n2 R 3 T and ^3 PA h V = n1 R 3 T

n1 3 PA 2.5 5

& n2 = 3 PB = 1.5 = 3
18.20 Physics
54. In free expansion, temperature of the gas remains
(c) dQ = nCdT = nCV dT + PdV
constant, therefore
& n ^CV + 2R hdT = nCV dT + PdV
p0 v0 = p.3v0 where v0 = initial volume.
dV
p ` 2nRdT = PdV ` dT =+ ve
p = 30
Hence with increase in temperature volume
55. For adiabatic compression, initial conditions are increases and vice versa.
p0
3 and 3v0 . Final volume and pressure are v0 and
` dQ = dU + dW =+ ve
3 2/3 p0

(d) dQ = nC dT = nCV dT + PdV
3 ^ 0 h = 3 p0 ^ 0 h
p0
3v γ 2/3
v γ & 3γ
-1
= 3 2/3
or n ^Cv - 2R hdT = nCv dT + PdV
2 5

or γ-1 = 3 & γ= 3
or - 2nRdT = PdV a dV
dT =- ve

i.e. gas is monoatomic
` with increase in volume temperature
56. KEavg ? T
decreases.
Applying TV γ
-1
Also dQ = n ^Cv - 2R hdT
= K for adiabatic process-
γ-1 γ-1
TV 1 1 = T2 V 2
For expantion dT =- ve but CV < 2R for
T2 V γ -1 3v 5/3 - 1
c 1m =c 0m = 3 2/ 3 monoatomic gas. Therefore dQ =+ ve with
T1 = V2 v0
57. (a) If 2
P = 2V , from an ideal gas equation increase in temperature dV =- ve

PV = nRT we get 58. (a) PV = nRT

P = ]nRT g v = ]constantg V , P α V
1 1 1
2V3 = nRT

` with increase in volume T = constant i.e. isothermal process

(i) Temperature increases implies dU =+ ve 1


As v decreases or V increases
(ii) dW =+ ve ` ∆W = positive
Hence dQ = dU + dW =+ ve and ∆Q = ∆U + ∆W > 0 ]∆U = 0g
2

(b) If PV = constant, from a ideal gas equation

(b) ∆Q = 0 and V = increases
PV = nRT we get VT = K (constant)
PV ∆W = positive
Now dQ = dU + PdV = nCV dT - T dT

(c) PV = nRT V ? T ]P = constantg
:a dV =- V
T dT
D
As volume increases, T also increases
PV

= nCV dT - T dT
i.e. ∆U > 0
= n ^Cv - R hdT

and ∆W > 0 so ∆Q > 0
` with increase in volume dT =- ve

(d) For cyclic process ∆U = 0
and since Cv > R for monoatomic gas. Hence
dQ =- ve ∆W < 0 (anticlockwise)

with increases in temperature dV =- ve ∆Q = ∆U + ∆W < 0


Thermodynamics 18.21

Numerical Type
Tc = 1818.859

Tc = 1819K
1. Heat rejected = mL f + mSTT
Single Option Correct
= ]50 # 540g + 50 ]1 g]100 - 20g
WD
4. h = Q
= 31000 Cal H
T
= 31 # 103 Cal & WD = QH d1 - TL n

H

= 5 # 10 b1 - 1000 l
2. Given, 3400


The pressure of a gas depends on its volume as kV3 . = 3000 kcal


The initial temperature of the gas Ti = 100°C. 5. The process A to B is an isothermal process.

The final temperature of the gas Tf = 300°C.
Work done by gas in isothermal process is,

WAB = P1 V1 ln : V 1 D = P1 V1 ln ]2g

According to the ideal gas equation, PV = nRT. 2V

1


Substituting P = kV , we get kV = nRT 3 4

B to C is isochoric process, where the volume

Differentiating on both sides we can write
remains constant, hence the work done is zero.

4kV dV = nRdT
3

WBC = 0.
nRdT

Again substituting P = kV3, we get PdV = 4
Process C to A is an adiabatic process.

The work done in any process is given by
Work done by gas in adiabatic process is,
nRdT nR

W= # PdV = # 4 = 4 3 T.
P
P1 V1 - 41 # 2V1

WCA = 1-c
200
W
& = 4 # nR = 50 nR
P1 V1 :1 - 2 D
1
3. PVc = constant W
& CA = 1-c
TVc - 1 = constant

P1 V1
V1 W
2 ^1 - ch
2/5
& CA =

300 (V1) 1. 4 - 1
= TB e o
16

So, the net work done in the cyclic process is,
TB = 300 # 28/5


Wnet = WAB + WBC + WCA

Now for BC process
& Wnet = P1 V1 ln ]2g + 0 +
P1 V1


BC 2 ^1 - ch
P
& Wnet = RT <ln ]2g - ^c - 1h F !a P1 V1 = RT +
1
B C
2
A
6. P1 = 1atm, T1 = 273K
V1 V1 V1 V
16 8

P1 V1c = P2 V2c
VB VC
=
P2 = P1 :V1 D
TB TC V c

2
VC TB
= 1atm b 3 l
TC = v = 2 # 300 # 28/5
1 1. 4
B

18.22 Physics
P1 V1 - P2 V2 Pv r = constant or TV r - 1 = constant

now work done c-1 = 88.7J
& 300.Vb 3 - 1 l = ]2V gb 3 - 1 l .Tf
5 5

Closest ans is 90.5 J
300 300
7. DQ = DU + DW; DU = DQ – DW & Tf = = 1/3
2 2/3 4
(DU)ACB = (DU)ADB; 60 – 30 = DQ – 10 300
= 1.58 = 189.87 K

∴ DQ = 40 J f

(b) U = 2 n R 3 t
8. m = 15 g, T = 27°C = 300 K 3
U =- 2 # 2 # 8.3 # 10 =- 2.7 kJ
15
T′ = 4 n = 14 R
12. C = Cv = 1 - n

DQ = DU = n CV DT R
C - Cv = 1 - n
= 18 b 2 R l^ 4 - 1 h 300
15 5
R
1 - n = C - C
V
15 5

= 18 # 2 # 8.30 # 3 # 300
R
n = 1- C-C
v
= R = 10004 J ≈ 10 kJ
C p - Cv

= 1- C-C
9. For Carnot engine A, QA = b1 - T2 l
W T v
1 1
C - Cv - C p + Cv
For Carnot engine B, Q B = b1 - T3 l
W T
= C-C v
2 2
C - Cp
As per equation, WA = WB
= C-C
v

Q1 b1 - T2 l = Q2 b1 - T3 l
T T 13. Molar specifie heat

1 2

= ^molar weight h # ^specific heat h



T2 b1 - T1 l = b1 - T2 l ca m
T1 T2 T3 Q1 T1

Q2 = T2 2a = 28b
T1 T3
T2 - 1 = 1 - T2
& a = 14b
T1 + T3 600 + 400 3P

T2 = 2 = 2 = 500K 14. Maximum temperature is observed at P = 2 0 and

10. The efficiency of all the engines is same 3V


V = 20

i.e., e1 = e2 = e3
It can be obtained by differentiation of PV term.
T T T 9P0 V0
& 1 - T2 = 1 - T.3 = 1 - T4
1 2 3
` Tmax = 4nR
T22 = T1T3 ...(i)
dQ
T32 = T2T4 ...(ii) 15. (i) ∆s1 = # T
T2T3 = T1T4 ...(iii) 150 150 200

# dT # dT # dT

= ms T + ms T + ms
= T1 b T
T1 T4 l T
T
& 2
2 (Using (i) and (iii)) 100 100 150
2

⇒ T2 = (T12T4)1/3 = ln b 150
l b 200 l
100 + ln 150
Similarly, T3 = (T1T42)1/3 (Using (ii) and (iii))
= ln b 32 l + ln 3
4

11. (a) As the gas is adiabatic
Thermodynamics 18.23
∆s1 = ln 2 f
16. ∆U = 2 nR∆T
112.5 125
dQ dQ dQ
For cyclic process ∆U = 0

(ii) ∆s2 = # T = # T + # T + .....
100 112.5

For process CA
= ln b 112.5 l b 125 l
∆U = 1 # 2 R ]- 200g
100 + ln 112.5 + ..... 5

= ln b 89 l + ln b 10
l b 16 l
9 + ln 15

=- 500 R
p0 v0
17. η =
= ln b 16
l
8 = ln 2
f f
2 (p0 v0 ) + 2 (2p0 ) v0 + 2p0 v0

=
1
3 + 3 + 2 = 15.4%
2

1. Given P-V diagram ∆W = 0


(S) Process-4 is isothermal ]T = constantg
For process (1),

Q 3 1 = nC P 3 T
5
As P = constant and V increases 3. γ = 3 & monoatomic gas
so T will increase
From first law of thermodynamics
So 3 Q1 > 0
H = W + ∆U
For process (2) 3 Q2 = nCV 3 T
W = Pi ∆V
V = constant, P ., So T .

= 700 J
For process (3), 3 Q3 = nC P 3 T < 0
For process (4), 3 Q4 = nCV 3 T ∆U = nCv ∆T
As 3 T > 0 .
= 32 _ Pf Vf - Pi Vi i =- 900
8 J

So 3 Q4 > 0
So, H = W + ∆U = 588 J
1
4. Slope of adiabatic process at a given state (P,V,T)
is more than the slope of isothermal process. The
P 4 2 corresponding P-V graph for the two process is as
shown in figure.
3
V P
P3 C
2. (P) Process-1 is adiabatic ^Q = 0 h
P1

(Q) Process-2 is isobaric A
B
∆w = 6P0 V0 .

(R) Process-3 is isochoric V
V1 V2
18.24 Physics
In the graph, AB is isothermal and Bc is adiabatic Pf Vf Tf
Pi Vi = T
WAB = positive (as volume is i

increasing) Tf Pi Vi
Pf = T Vf
i
and WBC = negative (as volume is
decreasing)
From option (a),

WBc > WAB , as area under P-V graph gives the 3T P V 3


Pf = T 1 $ 21V 1 = 2 P1
1 1
work done.
kx 3
P1 + A = 2 P1
Hence
kx = P1
WAB + WBC = W < 0 A 2
From the graph itself, it is clear that P3 > P1 1
Energy stored = 2 kx 2

Hence, the correct option is (a)
= 12 b kx l ] g

A $ xA

Note: At point B , slope of adiabatic (process BC) is
= 12 c 1 m $ ]∆V g
greater than the slope of isothermal (process AB) P


2
RT0
= 12 21 $ ^V1 h ^a ∆V = V2 - V1 h
5. Wcycle = P0V0 = P
2

Q1 " 2 nC p ]T2 - T1g 5 5
nC v ]T3 - T2g = 14 P1 V1
= = -3 = 3
Q2 " 3

Q1 " 2 nC p ]T2 - T1g
` Option (a) is correct.
nC v ]T4 - T3g
= =2
Q2 " 3

Change in internal energy of gas = nCv ∆T
P
(2T0)

= 32 ]nR∆T g
(T0)
= 32 ^nR 2T1 h
2P0
1 2

P0
4 3
= 3P1 V1
(T0)
(T0/2)

` Option (b) is correct.
1 2
V0 2V0 Wby gas = P1 ∆V + 2 kx

6. Process II is isothermal expansion


= P1 ^3V1 - V1 h + 12 b kx
A ] A - xg
l

= 2P1 V1 + 12 c 1 m^2V1 h

& Heat is positive P


3

Process IV is isothermal compression

= 73 P1 V1

& heat is negative
∆Q = Wby gas + ∆u
7. Wby gas = # P dV

= 73 P1 V1 + 32 nR ^3T1 h
x


# c P1 + kyA mA dy
0
= 73 P1 V1 + 92 P1 V1
2

= P1 Ax + kx
2 = 41
6 P1 V1

= ^ P1 ∆V h + 2k x 2
` (a), (b) & (c) are correct.
Thermodynamics 18.25
8. 13. The process must be adiabatic, which is used in
laplace correction over Newton’s law for sound
speed in air.

14. Let final temperature of gases is T


Heat rejected by gas in lower compartment

^nCV ∆T h = 2 # 32 R ]700 - T g
f

a U = 2 nRT, where f, n, R are constants.

Also temperature T is same at A & B.


Heat received by gas in above compartment

` U A = UB _nC p ∆T i = 2 # 7 R ]T - 400g
2
4V

Also ∆WAB = nRT0 ,n V 0 = nRT0 ,n4 = P0 V0 ,n4
Equating above
0

P0 V0 = PB AV0 2100 - 3T = 7T - 2800


P0
& T = 490 K

As, A " B is isothermal process, PB =
4
P0 15. ∆W1 + ∆U1 = ∆Q1

B to C, is Isobaric pC =
4
9. PV c = C ∆W2 + ∆U2 = ∆Q2
TV
& c-1
=C ∆Q1 + ∆Q2 = 0
= TR b 20 l
V c-1
T
& 0V c-1
7 ]T - 400g = 5 ]700 - T g
0

R 2R 2R

CV = c - 1
6300
R
& T = 12 = 525 K

& 2R = c - 1


& c-1 = 2
1
So ∆W1 + ∆W2 = 2.R. ]525 - 400g

+ 2R ]525 - 700g
3

&c= 2

T

& TR = 2 c - 1 = 2
=+ 250R - 350R
0

10. Q = 3 U1 + 3 U2

=- 100R
3 1 = CV 3 T1 = 2R ]TL - T0g
U
3 2 = CV 3 T2 = 2R ]TR - T0g
P
U 16.

TL = 3 2 T0, TR = 2 T0 P1
Q = 2R 63 2 - 1@T0 + 2R ^ 2 - 1 h T0

Q = 4RT0 62 2 - 1@
P1/4

& RT = 4 62 2 - 1@
Q

0
P2
11. ∆u = ∆Q - P∆V


i.e. process must be isobaric V
V1 4V1 32V1


` (b) is correct P1
^4V1 h = P2 ^32v1 h
5 5
3 3

4
12. Isochoric process
18.26 Physics
P1 1 P1
5
Qib = 100 + 50 = 150 J
d n =
3


P2 =
4 8 128 Qibf = 300 + 150 = 450 J
P1 Qbf
P1 V1 - ^32V1 h
450–150
So, the required ratio Q = 150 = 2
P1 V1 - P2 V2 128 ib

Wadi = =
c-1 5
-1 19. (i) TU = TQ - TW
3 105 ^10 -3 - 10 -6h
<"^10 -3 # 2250h, -
F kJ
P1 V1 a 4 k
3 3
10
9
= (2.25 – 0.0999) kJ

= = PV
2 8 1 1
= (2.1501) kJ
3

4V1 (ii) T U = nCV T T



Wiso = P1 V1 ln f
p = 2P1 V1 ln 2 5
V1 = 2 nRTT

= 2 ]0.2g]8 g]1500 - 500gJ
Wiso 2P1 V1 ln 2 16 5

= 9 = ln 2 = f ln 2
Wadio P V 9 = 4 kJ
8 1 1

16 (iii) P1 V2c = P2 V2c


f= = 1.7778 . 1.78
& 2 b 3 l = P2 b 24 l
9 1 5/3 1 5/ 3

17. W = ]DP gavg # 4rR 2 a P2 = 64 kPa



TU = nCv TT = 2 ^ P2 V2 - P1 V1 h
3
dP

- $ 4r R 2 a
= 2 b64 # 24 - 2 # 3 l kJ
2 3 1 1

{for small change ]DP gavg < P > arithmetic mean}

=3 kJ
P cP0
PV
= c
= c & dP =- c dV =- 4r R 2 a (iv) TU = nCv TT
V V
cP0 7
2 2
n 2 RTT


W= # 4r R a # 4r R a
2V 7
3 cP0 = 9 TQ

W=
4rR 2 a # 4rR 2 a
2 # 4rR 3 = 7 kJ
3c
W = _ 4rRP0 # a 2 i

2 20. For adiabatic process ] A " Bg

` x - 2.05 PA V Ac = PB V Bc
105 # ]0.8g3 = 3 # 105 ^VB h3
5 5

18. P

& VB = 0.8 # b 3 l = 0.4


a f 1 0. 6

Work done in process A " B

PA VA - PB VB
WAB =
c-1
b
i 105 # 0.8 - 3 # 105 # 0.4
200 J & WAB =
5
100 J
3 -1
V & WAB =- 60kJ & | WAB | = 60kJ

wibf = 150 J
Work done in process B " C (Isothermal process)
wiaf = 200 J V V
WBC = nRT,n VC = PB VB ,n VC

B B
Qiaf = 500 J so Uiaf = 300 J
0.8
& WBC = 3 # 10 # 0.4,n 0.4
5

So U f = 400 J
& WBC = 84 kJ

Uib = 100 J
Thermodynamics 18.27

Work done in process C " A is zero (isochoric)

So total work done in the process A " B " C

WABC = WAB + WBC + WCA = – 60 + 84 + 0

WABC = 24 kJ

So correct options are (b,c,d)

21. A " B & V . P const & T . U . ^ p h, ] r g, ] t g



B " C & dω . 0
P.T.

C " D & V -& T -
du &+ ve
dω =+ ve

D " A & dw &- ve
dq &- ve
du = 0

You might also like